Копия Digital SAT English Workbook t.me SAT DIGITAL DSAT

Download as pdf or txt
Download as pdf or txt
You are on page 1of 442

1

Table of Contents

♟️General Test-Taking Strategies♟️...................................................................................... 5


I. SAT English Section Layout (Modules 1 & 2).....................................................................6
Categories & Question Types📣................................................................................................... 6
III. Understanding Answer Choices🧐.....................................................................................7
IV. General English Module Strategies🏆.............................................................................. 8
V. Preliminary Skill: Pronouns🤓..............................................................................................9
VI. Craft and Structure Questions..........................................................................................13
i. Words In Context🗣️............................................................................................................. 13
Strategies..............................................................................................................................13
Exercises............................................................................................................................... 15
Extra Practice.......................................................................................................................26
Answers - Extra Practice................................................................................................. 31
ii. Function of the Underlined Sentence ✌️......................................................................37
Strategies..............................................................................................................................37
Exercises............................................................................................................................... 38
Extra Practice.......................................................................................................................46
Answers - Extra Practice................................................................................................. 50
iii. Main Purpose of the Text 🤟........................................................................................... 55
Preliminary Exercise..........................................................................................................56
Strategies..............................................................................................................................57
Examples.............................................................................................................................. 58
Answers - Extra Practice................................................................................................. 65
iv. Comparative Text 👯..........................................................................................................70
Strategies..............................................................................................................................70
Examples.............................................................................................................................. 72
Extra Practice.......................................................................................................................81
Answers - Extra Practice................................................................................................. 89
VII. Information and Ideas........................................................................................................94
i. Main Idea of Text 💡.............................................................................................................94
Strategies..............................................................................................................................94

www.strategictestprep.com ♟️ 1-833-300-PREP (7737) ♟️[email protected] 2


Exercises............................................................................................................................... 95
Examples.............................................................................................................................. 97
Extra Practice.................................................................................................................... 103
ii. According to the Text 📖.................................................................................................114
Strategies........................................................................................................................... 114
Practice Questions.......................................................................................................... 115
Answers - Extra Practice.............................................................................................. 119
iii. Statement that Best Supports 👍 or Undermines👎 a Claim.............................124
Strategies........................................................................................................................... 124
Exercises.............................................................................................................................124
Extra Practice.................................................................................................................... 135
Answers - Extra Practice.............................................................................................. 143
iv. Graph Questions 📈📊................................................................................................... 151
Strategies........................................................................................................................... 151
Examples............................................................................................................................152
Extra Practice.................................................................................................................... 160
Answers - Extra Practice.............................................................................................. 164
v. Logically Completes the Text 🧠..................................................................................167
Strategies........................................................................................................................... 167
Exercises.............................................................................................................................168
Extra Practice.................................................................................................................... 176
Answers - Extra Practice.............................................................................................. 190
VIII. Expression of Ideas.........................................................................................................211
i. Transition Questions 🏃💨.............................................................................................. 211
Strategies........................................................................................................................... 212
Examples............................................................................................................................213
Extra Practice.................................................................................................................... 216
Extra Practice Answers................................................................................................. 223
Bonus Practice Questions.............................................................................................224
Bonus Practice Answers............................................................................................... 240
ii. Note-Taking Questions ✍️............................................................................................................. 256
Strategies........................................................................................................................... 256

www.strategictestprep.com ♟️ 1-833-300-PREP (7737) ♟️[email protected] 3


Examples............................................................................................................................257
Extra Practice.................................................................................................................... 260
Answers - Extra Practice.............................................................................................. 290
IX. Standard English Convention Questions 👩‍🏫........................................................................308
Lesson #1: Complete or Incomplete Sentence?...........................................................308
Lesson #2: Know Your Verb Tenses.................................................................................313
Lesson #3: Subject/Verb Agreement.............................................................................. 315
Lesson #4: Parallelism......................................................................................................... 320
Lesson #5: Comma Placement.......................................................................................... 326
Lesson #6: Lead-Ins and Their Subjects........................................................................ 334
Lesson #7: Punctuation........................................................................................................339
Lesson #8: Punctuation with a Transition......................................................................345
Lesson #9: Apostrophes......................................................................................................349
Lesson #10: Additional Grammar Concepts................................................................. 352
Standard English Convention Practice Questions.......................................................355
Standard English Convention Answers..........................................................................396

www.strategictestprep.com ♟️ 1-833-300-PREP (7737) ♟️[email protected] 4


♟️General Test-Taking Strategies♟️
Pick and Stick Majority Rules Leave a Trail 1-Minute Rule Flag ?s to come
back to

Pick a letter when Cross off Cross off Spend roughly 1 If you’re not 100%
running out of “oddballs” and answers you’ve minute max on confident in your
time and stick choose the eliminated to any given answer, flag it to
with it for all answer choice leave a trail in question. come back to later
remaining that has the most the event you with your extra
questions. in common with need to go back time!
the other answer to the question
choices! later.

✨How can you make the most out of this workbook? ✨


👉For walkthrough videos, strategies, quizzes, and extra resources that support
each lesson in this workbook, please sign up for our English Self-Paced Course!

👉For individualized support, please check out our Information Pack to learn
more about our private tutoring options and Request a Consultation!

www.strategictestprep.com ♟️ 1-833-300-PREP (7737) ♟️[email protected] 5


I. SAT English Section Layout (Modules 1 & 2)
It is important to understand the layout of the English Sections, as well as the types of
questions involved. 💪
Module 1 - 27 Questions, 32 minutes
Module 2* - 27 Questions, 32 minutes
*This module may be harder or easier depending on how you did on Module 1.

You should aim to get less than 11 questions wrong in Module 1. This will ensure you
get the harder 2nd module, which will allow you to pick up more points.

If you get the easier 2nd module, the highest score you can get is less than a 600.

Categories & Question Types 📣


1. Craft & Structure (28% of section, 13-15 questions)
● Words in Context
● Function of the Underlined Sentence
● Main Purpose of Text
● Text 1/Text 2
2. Information & Ideas (26% of section, 12-14 questions)
● Main Idea of Text
● Statement/Quotation that Best Supports a Claim/Hypothesis
● Graph Questions
● Logically Completes the Text
3. Expression of Ideas (20% of section, 8-12 questions)
● Transitions
● Note-Taking Questions
4. Standard English Conventions (26% of section, 11-15 questions)
● All your Grammar Questions

www.strategictestprep.com ♟️ 1-833-300-PREP (7737) ♟️[email protected] 6


III. Understanding Answer Choices 🧐
What does the SAT English section do ❓
The SAT evaluates your skill in linking specific textual elements to broader concepts.
Although the right answer will always find its basis in the text, the crux of the exam
lies in your ability to establish this link.

How do I pinpoint a correct answer ❓


Generally, the correct answers are not verbatim reproductions of the text. Be wary if an
option does just that. More often than not, the correct choice will be a paraphrased
version of an idea previously discussed in the passage. So, your goal is to find
synonyms in the answer to the text! It's about recognizing the same concept, just
expressed differently.

What are the common types of wrong answers ❓


👉Off-topic
👉Too broad (e.g., the passage discusses one scientist while the answer
refers to scientists)

👉Too extreme (e.g., they include words such as never, always, or


completely)

👉Half-right, half-wrong (e.g., right words, false statement)


👉Could be true but not enough information
👉True for the passage as a whole, but not for the specific lines in
question

👉Factually true but not stated in the passage

www.strategictestprep.com ♟️ 1-833-300-PREP (7737) ♟️[email protected] 7


Do you often pick the wrong one when narrowed down to two
When this happens, typically the incorrect answer will fall into either the "could be

true but not enough information" or the "half-right, half-wrong" category. In such
cases, you must be willing to read very carefully in order to determine which answer
the passage truly supports.

Reading carefully is key. Because the passages are short, cutting corners can get
you into big trouble (most of the time).

IV. General English Module Strategies 🏆


There are things you can do for all questions on the English modules:

✅ Always read the question first: this will save you time because you will
read with purpose and think about the question as you’re reading

✅ While reading, sum up each sentence in your own words: this will help
you understand the passage on a deeper level and what it is saying

✅ Look for keywords/phrases: these are words that


● Indicate points, goals, and conclusions - this gives you the argument

● Describe (adjectives) - this gives you a sense of tone

● Transitions - denotes a change in argument, which is important

✅ Look for “unusual” punctuation: if you see dashes, semicolons, or


colons, the right answer is probably nearby.

✅ Use process of elimination: there will likely be two irrelevant answers.


Once you have it down to two and you are unsure, go back into the passage
again and start checking them out.

✅ Start the module at #15 (or wherever you are strongest): You do NOT
need to go in order! Strategize and start at the best place for you to
maximize your points and make efficient use of your time.

www.strategictestprep.com ♟️ 1-833-300-PREP (7737) ♟️[email protected] 8


V. Preliminary Skill: Pronouns 🤓
Often in the texts you will read on the DSAT, the text will switch from specific phrasing
to more general language by using pronouns (it, they, this, that, them, he etc).

The key is knowing what the pronoun is connected to (what it is referring to). If you
cannot follow the pronouns in the text and where they are coming from, you will get
confused!

Let’s look at an example:

Pronoun Example #1

...Crowdsourcing is a wonderful tool, but (1) it still fails in a very particular way, which
is that any evaluation is swayed by the evaluations that have come before (2) it. A
barbershop with a one-star rating on Yelp as (3) its first review is subsequently more
likely to accrue more negative reviews- and that same barbershop, were (4) it to
receive a four-star rating on Yelp as (5) its first review, would be more likely to accrue
more subsequent positive reviews.

Can you figure out what each it refers to in the passage?

(1) it :_________________________________

(2) it. ______________________________

(3) its _______________________________

(4) it: _______________________________

(5) its: _______________________________

www.strategictestprep.com ♟️ 1-833-300-PREP (7737) ♟️[email protected] 9


Pronoun Example #2 (when it refers to an abstract noun)

In a step toward creating robots capable of spontaneous learning, a new approach has
expanded training data sets for robots that work in cluttered environments. Developed
by Dmitry Berenson and Peter Lozano, doctoral students at the University of Michigan,
(1) it could cut learning time for new materials and environments down to a few hours
rather than a week or two.

What does this “it” refer to in the passage?

(1) it :_________________________________

Pronoun Example #3 (when it/its do not refer to the same noun)

The distance between the Earth and the sun varies gradually over the course of the
year because of the elliptical nature of Earth's orbit. At (1) its closest approach, known
as perihelion, the Earth is about 3 million miles closer to the sun than at (2) its farthest
point, called aphelion. As a result, sunlight on Earth is about 7% more intense at
perihelion than (3) it is at aphelion.

Can you figure out what each it refers to in the passage?

(1) it :_________________________________

(2) it. ______________________________

(3) its _______________________________

Pronoun Example #4 (Singular pronoun referents vs. plural pronoun referents)

Many Artificial Intelligence (AI) systems involving speech recognition, computer vision,
and medical imaging make use of neural networks- computing systems inspired by the
architecture of the human brain. In neuroscience, (1) they are often used to model the
same kinds of tasks that the brain performs, in hopes that the models could lead to
new insights in understanding how the brain itself performs those tasks.

www.strategictestprep.com ♟️ 1-833-300-PREP (7737) ♟️[email protected] 10


Can you figure out what they refers to in the passage?

(1) they :_________________________________

Since the pronoun is a “they,” it is plural, and must refer to a plural noun.

Pronoun Example #5 (Compression Noun)

The evolution of our communications system from a broadcast model to a networked


one has added a new dimension to the mix. The Internet has made us all less
dependent on professional journalists and editors for information about the wider
world, allowing us to seek out information directly via online search or to receive it
from friends through social media. But (1) this enhanced convenience comes with a
considerable risk: that we will be exposed to what we want to know at the expense of
what we need to know.

What does “this enhanced convenience” refer to?

(1) this enhanced convenience: ________________________________________________

Compression noun: refers not to a single thing but rather to an entire idea

Pronoun Example #6 (Tricky Compression Noun)

While humpback dolphins look quite similar to other dolphins, their genetics tells a
different story. Researchers collected 235 tissue samples and 180 skulls throughout
the animals' distribution, representing the biggest dataset assembled to date for the
animals. The team analyzed mitochondrial and nuclear DNA from the tissue, which
revealed significant variations. Although the line between species, sub-species and
populations is a blurry one, in this case, the researchers are confident that the
humpback dolphin is distinct enough to warrant the "species" title. The mitochondrial
DNA turned up genetic signatures distinct enough to signal a separate species, and
likewise, differences in the dolphins' skulls supported (1) this divergence. Although

www.strategictestprep.com ♟️ 1-833-300-PREP (7737) ♟️[email protected] 11


the nuclear DNA provided a slightly more confounding picture, it still clearly showed
differences between the species.

What does “this divergence” refer to?

(1) this divergence: ________________________________________________

Pronoun Example #7 (Former vs. Latter)

The former refers back to the noun or phrase mentioned first, and the latter refers
back to the noun or phrase mentioned second.

In the nineteenth century, both Thomas Edison and Nikola Tesla were well-known
scientists, but (1) the former quickly came to be regarded as one of the greatest
American inventors, while (2) the latter fell into obscurity.

Can you figure out what each refers to in the passage?

(1) the former :_________________________________

(2) the latter: _________________________________

Pronoun Example #8 (Former vs. Latter)

All bodies in the solar system are heated by sunlight. They rid themselves of this heat
in two ways: by emitting infrared radiation and by shedding matter. In long- lived
bodies such as Earth, (1) the former process prevails; for others, such as comets, (2)
the latter dominates.

Can you figure out what each refers to in the passage?

(1) the former :_________________________________

(2) the latter: _________________________________

www.strategictestprep.com ♟️ 1-833-300-PREP (7737) ♟️[email protected] 12


VI. Craft and Structure Questions
i. Words In Context 🗣️
Words in context questions make up a large portion of the test, so it is important to
give yourself the best chance possible to get these correct!

Strategies
Strategy #1: Look for Synonyms or the Definition of the Word

They WILL give a synonym to the word they want you to put in the blank. Often, a
colon or semicolon will clue you in that the extra information that comes after it
explains more about the word they want in the blank.

Since melodic is closest in meaning to “pleasant musical quality,” the answer is C.


Notice that “pleasant musical quality” came after the colon.

Strategy #2: Determine positive/negative


You may not know the meaning of a word, but determining positive/negative tone can
help you narrow it down to the correct answer!

Strategy #3: Build Your Vocabulary Using Linguistics


www.strategictestprep.com ♟️ 1-833-300-PREP (7737) ♟️[email protected] 13
Understand that words are broken into parts: prefixes, roots, and suffixes. If you have
a general understanding of what each prefix, root or suffix means, you can figure out
what the word means.

Study Tips:
1. Study this list weekly. START NOW!

2. Break it up into chunks (i.e. a page at a time, a letter at a time)

3. Quiz yourself by covering up the meaning and example words to see if you can
recall them from memory

Please note that knowing this list gives you an edge with words in context
questions. It will greatly increase your chances of getting the questions correct.
However, this list is not all inclusive, meaning, there will be words you encounter
that do not have parts on this list.

Strategy #4: Move on Quickly


If unsure, put down your best guess and move on quickly. You should spend no more
than 30 seconds to 1 minute on these questions. Either you will know it or you won’t -
deliberating for a long time will just hurt your chances of picking up more points down
the line.

www.strategictestprep.com ♟️ 1-833-300-PREP (7737) ♟️[email protected] 14


Exercises
Synonym Exercise: Circle the best answer then check your answers on the next page!
1.Dangerous 8. Ramification 15. Which is NOT a synonym for
a. Scarry a. Lessen abominable?
b. Healthy b. Implication a. Angry
c. risky c. Trite b. Loathsome
d. infection c. Abhorrent

2. Achieve 9. Benevolence 16. A synonym for elite is select.


a. Make a. Alertness a. True
b. Take b. Offensive b. false
c. succeed c. Buffer
d. kindness

3. Effort 10. Recompense 17. Incredulous


a. Attempt a. Leisure a. Optimistic
b. Chance b. Payment b. Pessimistic
c. choice c. Praise c. Disbelieving
d. rethink d. shocked

4. Instead 11. Hasten 18. A synonym for disquietude


a. Better than a. Warn is anxiety
b. Alternatively b. Nag a. True
c. Such as c. Attach b. false
d. hurry

5. Goal 12. Reciprocal 19. Full


a. Object a. Exclusive a. Enigma
b. Objective b. Mutual b. Harbingers
c. score c. Isolated c. Replete
d. dramatic d. harangue

6. Throughout 13. Plaudits 20. Capitulate


a. End-to-end a. Negativity a. Win
b. All day b. Honor b. Succeed
c. None of the above c. Praise c. Surrender
d. hope d. defeat

7. Routine 14. Fame 21. Lamentable


a. Day a. Talisman a. Anguished
b. Quotidien b. Noxious b. Angered
c. same c. Bigot c. Yearned
d. renown d. deplorable

www.strategictestprep.com ♟️ 1-833-300-PREP (7737) ♟️[email protected] 15


Synonym Exercise Answers:
1. C
2. C
3. A
4. B
5. B
6. C
7. B
8. B
9. D
10. B
11.D
12.B
13.C
14.D
15.A
16.B
17.C
18.A
19.C
20.C
21.D

www.strategictestprep.com ♟️ 1-833-300-PREP (7737) ♟️[email protected] 16


Positive/Negative Exercise: Move each word in the word bank below to the appropriate
column, based on how it sounds. Then check your answers on the following page!

Word bank: congregation, hedonist, integrity, longevity, ostentatious, orator,


substantiate, apathy, augment, expedite, dogmatic, novel, affable, indolence, squander

www.strategictestprep.com ♟️ 1-833-300-PREP (7737) ♟️[email protected] 17


Answers to Positive/Negative Exercise:

www.strategictestprep.com ♟️ 1-833-300-PREP (7737) ♟️[email protected] 18


Pinpoint the Definition Exercise: Can you spot the definition of the word you need for
the blank? Circle the definition or synonym in the text! Then, check your answers on the
next page.

1. Former astronaut Ellen Ochoa says that although she doesn’t have a definite
idea of when it might happen, she _______ that humans will someday need to
be able to live in other environments than those found on Earth. This conjecture
informs her interest in future research missions to the moon.

2. Beginning in the 1950s, Navajo Nation legislator Annie Dodge Wauneka


continuously worked to promote public health; this _______ effort involved
traveling throughout the vast Navajo homeland and writing a medical dictionary
for speakers of Diné bizaad, the Navajo language.

3. Seminole/Muscogee director Sterlin Harjo


_______ television’s tendency to situate Native characters in the distant past: this
rejection is evident in his series Reservation Dogs, which revolves around
teenagers who dress in contemporary styles and whose dialogue is laced with
current slang.

4. Researchers have struggled to pinpoint specific causes for hiccups, which


happen when a
person’s diaphragm contracts _______ . However, neuroscientist Kimberley
Whitehead has found that these uncontrollable contractions may play an
important role in helping infants regulate their breathing.

5. In studying the use of external stimuli to reduce the itching sensation caused by
an allergic histamine response, Louise Ward and colleagues found that while
harmless applications of vibration or warming can provide a temporary
distraction, such _______ stimuli actually offer less relief than a stimulus that
seems less benign, like a mild electric shock.

Pinpoint the definition/synonym in the text answers:


www.strategictestprep.com ♟️ 1-833-300-PREP (7737) ♟️[email protected] 19
1. Former astronaut Ellen Ochoa says that although she doesn’t have a definite
idea of when it might happen, she _______ that humans will someday need to
be able to live in other environments than those found on Earth. This conjecture
informs her interest in future research missions to the moon.

2. Beginning in the 1950s, Navajo Nation legislator Annie Dodge Wauneka


continuously worked to promote public health; this _______ effort involved
traveling throughout the vast Navajo homeland and writing a medical dictionary
for speakers of Diné bizaad, the Navajo language.

3. Seminole/Muscogee director Sterlin Harjo


_______ television’s tendency to situate Native characters in the distant past:
this rejection is evident in his series Reservation Dogs, which revolves around
teenagers who dress in contemporary styles and whose dialogue is laced with
current slang.

4. Researchers have struggled to pinpoint specific causes for hiccups, which


happen when a person’s diaphragm contracts _______ . However, neuroscientist
Kimberley Whitehead has found that these uncontrollable contractions may
play an important role in helping infants regulate their breathing.

5. In studying the use of external stimuli to reduce the itching sensation caused by
an allergic histamine response, Louise Ward and colleagues found that while
harmless applications of vibration or warming can provide a temporary
distraction, such _______ stimuli actually offer less relief than a stimulus that
seems less benign, like a mild electric shock.

Linguistics Exercise: Study this list daily for a week, then take the quiz that follows
without looking at the list!
www.strategictestprep.com ♟️ 1-833-300-PREP (7737) ♟️[email protected] 20
www.strategictestprep.com ♟️ 1-833-300-PREP (7737) ♟️[email protected] 21
www.strategictestprep.com ♟️ 1-833-300-PREP (7737) ♟️[email protected] 22
www.strategictestprep.com ♟️ 1-833-300-PREP (7737) ♟️[email protected] 23
www.strategictestprep.com ♟️ 1-833-300-PREP (7737) ♟️[email protected] 24
Linguistics Quiz!

1. What does the root eu mean? _________________________________

2. What does the prefix aba- mean? _______________________________

3. What does the suffix -ious mean? _______________________________

4. What does the prefix mis- mean? _______________________________

5. What does the root gyn mean? _______________________________

6. What does the word misogynist mean? _______________________________

7. What does the root bene mean? _______________________________

8. What does the root pug mean? _______________________________

9. What does the root anthro mean? _______________________________

10.What does the root phil mean? _______________________________

11.What does the prefix a- mean? _______________________________

12.What does the root soph mean? _______________________________

13.What does the root loq mean? _______________________________

14.What does the root fid mean? _______________________________

15.What does the prefix im/in - mean? _______________________________

Now, go back to the table and check your answers! If you got any wrong, you need to
study this table more!!

Tip: Put a star next to the ones in the table you still don’t know and study those for
another week.

www.strategictestprep.com ♟️ 1-833-300-PREP (7737) ♟️[email protected] 25


Extra Practice

1. The Aegean civilization is noted for its ______ architecture, particularly its richly
detailed painted frescos and statues of seated female figures.
Which choice completes the text with the most logical and precise word or phrase?
A. minimalist B. monolithic C. stylized D. ornate

2. Psychologists have noted that humans tend to perceive certain colors, sounds, and
smells as either calming or energizing—in other words, they have a(n) _______ mood.
Which choice completes the text with the most logical and precise word or phrase?
A.merit for B.aversion to C.impact on D.opinion on

3. In a letter to the Stanford community, Marion Taylor noted that, for Stanford to
continue to lead the way in education in the twenty-first century, it must ________ the
standards of today’s universities and acknowledge the issues of our times.
Which choice completes the text with the most logical and precise word or phrase?
A.oppose B.exceed C.fulfill D.challenge

4. The development of new antibiotics is a slow and expensive process, and the rise of
antibiotic-resistant bacteria is _________the effectiveness of these drugs.
Which choice completes the text with the most logical and precise word or phrase?
A.facilitating B.undermining C.restoring D.reinforcing

5. Unlike many other crab species, coconut crabs have adapted to living a terrestrial
lifestyle, meaning their diet must consist of food sources ________.
Which choice completes the text with the most logical and precise word or phrase?
A.available on land B.available in the ocean C.other than coconuts D.inaccessible to
other predators

www.strategictestprep.com ♟️ 1-833-300-PREP (7737) ♟️[email protected] 26


6. In the early days of the automobile industry, Henry Ford revolutionized the way
vehicles were produced by introducing the assembly line method of manufacturing,
which greatly ______ production time.
Which choice completes the text with the most logical and precise word or phrase?
A. curtailed B. amplified C. disregarded D. accelerated

7. The nineteenth-century French novelist Gustave Flaubert was a master of _______,


which he used to great effect in his writing. By carefully crafting the sounds and rhythms
of his sentences, Flaubert was able to heighten the emotional resonance of his work and
draw his readers into his story.
Which choice completes the text with the most logical and precise word or phrase?
A. dialogue B. metaphor C. syntax D. alliteration

8. The success of the Apollo 11 mission in 1969 was a testament to the dedication of
the thousands of engineers and technicians who had worked on the project for years,
______ the spacecraft’s design and components to ensure its safe launch and return.
Which choice completes the text with the most logical and precise word or phrase?
A. reconfiguring B. rejecting C. reproducing D. reinterpreting

9. The brain is often described as having ______: the ability to change and adapt in
response to new experiences.
Which choice completes the text with the most logical and precise word or phrase?
A. stability B. plasticity C. susceptibility D. sensitivity

10. The term “affective computing” is used to describe the field of computer science that
focuses on ______, such as facial expressions and vocal intonations, to interpret and
respond to human emotions.
Which choice completes the text with the most logical and precise word or phrase?
A. data B. sensors C. signals D. circuits

www.strategictestprep.com ♟️ 1-833-300-PREP (7737) ♟️[email protected] 27


11. The concept of the “American Dream” has been ______ since the early 20th century,
when it was widely promoted by James Truslow Adams in his book The Epic of America.
Which choice completes the text with the most logical and precise word or phrase?
A. challenged B. debated C. invented D. popularized

12. The term “urban heat island” refers to the phenomenon of a city’s temperature being
higher than its ___________ rural areas, due to the large amounts of heat-absorbing
concrete, asphalt, and other materials used in urban areas.
Which choice completes the text with the most logical and precise word or phrase?
A. surrounding B. inferior C. malignant D. desolate

13. In the novel Frankenstein, the creature is described as having a ________appearance,


which repels those who encounter him.
Which choice completes the text with the most logical and precise word or phrase?
A. hideous B. unfamiliar C. conspicuous D. familiar

14. The ancient Greek philosopher Epicurus believed that the path to a meaningful life
was to ______ one’s desires: focus on simple pleasures, avoid unnecessary stress, and
live in moderation.
Which choice completes the text with the most logical and precise word or phrase?
A. accelerate B. satisfy C. amplify D. restrain

15. The term “sustainability” is used to describe a way of living that meets the needs of
the present without compromising the ability of future generations to meet their own
needs. In other words, sustainable practices are ______: designed to be maintained over
time without depleting resources.
Which choice completes the text with the most logical and precise word or phrase?
A. disposable B. renewable C. durable D. consumable

www.strategictestprep.com ♟️ 1-833-300-PREP (7737) ♟️[email protected] 28


16. The earliest known examples of written language are Sumerian cuneiform tablets,
which date back to around 3000 BCE. Cuneiform was a complex system of writing that
used wedge-shaped symbols to represent objects, ideas, and sounds, and it was ______:
everything from business transactions to religious texts in the Near East employed
Cuneiform for centuries.
Which choice completes the text with the most logical and precise word or phrase?
A. disregarded B. discontinued C. pervasive D. transitory

17. The term “anthropocene” was coined in 2000 to describe the current geological
epoch, which is marked by the ______ of human activities on the planet: the burning of
fossil fuels, the release of pollutants, and the destruction of natural habitats.
Which choice completes the text with the most logical and precise word or phrase?
A. limitations B. consequences C. benefits D. influences

18. The cello is a stringed instrument that has a more ______ sound compared to the
violin: its larger size and thicker strings produce a deeper and more resonant tone.

Which choice completes the text with the most logical and precise word or phrase?
A. dulcet B. brassy C. robust D. flat

19. Most historians agree that the discovery of the New World in 1492, an event now
known as the Columbian Exchange, eventually led to ______ between the Old World
and the New: the transfer of plants, animals, and diseases, as well as the spread of
cultural influences, between the two regions.
Which choice completes the text with the most logical and precise word or phrase?
A. stagnation B. interaction C. divergence D. dissemination

20. The discovery of the Higgs boson, a particle that is believed to give mass to other
particles, was a major breakthrough in particle physics, but the particle's actual
properties remain _______: scientists are still attempting to understand how it behaves.
Which choice completes the text with the most logical and precise word or phrase?
A. obvious B. unfathomable C. elusive D. unpredictable

www.strategictestprep.com ♟️ 1-833-300-PREP (7737) ♟️[email protected] 29


21. The development of the printing press in the fifteenth century changed the way
people disseminated information by making it ______: as books and other printed
materials could be mass-produced, knowledge could be spread more quickly and widely
than ever before.
Which choice completes the text with the most logical and precise word or phrase?
A. exclusive B. unavailable C. accessible D. elusive

22. The ancient Mesopotamian city of Ur was an important center of culture and
learning, but it was eventually ______: the city was destroyed and abandoned in the
early second millennium BCE.

Which choice completes the text with the most logical and precise word or phrase?
A. elevated B. reconstructed C. razed D. celebrated

23. The author’s attempt to explain the historical and cultural significance of the Agadir
Crisis is ______, as it relies heavily on outdated and largely discredited theories.
Which choice completes the text with the most logical and precise word or phrase?
A. perfunctory B. cohesive C. entertaining D. inaccurate

24. The author’s opinion of the new policy is ______, as he argues that it fails to address
the underlying causes of the problem.
Which choice completes the text with the most logical and precise word or phrase?
A. unfavorable B. unclear C. unacceptable D. unavoidable

25. The author’s critique of the current state of the economy is ______, as it fails to take
into account the positive effects of recent economic policies. To be convincing, his
argument would need to acknowledge the positive impacts of recent economic reforms.
Which choice completes the text with the most logical and precise word or phrase?
A. superficial B. contradictory C. incomplete D. unilateral

www.strategictestprep.com ♟️ 1-833-300-PREP (7737) ♟️[email protected] 30


Answers - Extra Practice

1. Choice D is the best answer because it most logically completes the text’s discussion
of the architecture of the Aegean civilization. As used in this context, “ornate” means
marked by complexity and richness of detail. The text indicates that the Aegean
civilization is best known for its architecture, which includes painted frescos and
statues of female figures. This context suggests that the architecture is characterized
by ornate detail and complexity.

2. Choice C is the best answer because it most logically completes the text’s discussion
of what psychologists have observed about colors, sounds, and smells. As used in this
context, “impact” means an effect. The text suggests that colors, sounds, and smells
have an effect on mood, suggesting that they have an impact on mood.

3. Choice D is the best answer because it most logically completes the text’s discussion
of what Stanford must do to lead in education in the twenty-first century. As used in
this context, “challenge” means confront or attempt to overcome. The text suggests
that Stanford should take steps to lead in education in the twenty-first century. The
“issues of our times” mentioned in the sentence indicate that the steps taken by
Stanford would likely involve some type of innovate approach or policy that would
push beyond the status quo and stretch the standards of today’s universities. In other
words, Stanford must challenge the standards of today’s universities and acknowledge
current issues.

4. Choice B is the best answer because it most logically completes the text’s discussion
of the rise of antibiotic-resistant bacteria. As used in this context, “undermining” means
weakening or eroding. The text indicates that the development of new antibiotics is
“slow” and “expensive,” which suggests that the drugs’ effectiveness is already limited.
The text then states that the rise of antibiotic-resistant bacteria is having an effect on
the effectiveness of these drugs. This context suggests that the rise of
antibiotic-resistant bacteria is undermining the effectiveness of these drugs.

www.strategictestprep.com ♟️ 1-833-300-PREP (7737) ♟️[email protected] 31


5. Choice A is the best answer because it most logically completes the text’s discussion
of what coconut crab diet must consist of due to the species’ having adapted to living a
terrestrial lifestyle. The text indicates that “food sources” must be available to coconut
crabs if they are going to survive. The context clearly conveys that the food sources
must be available on land, as the species’ adaptation is described as enabling them to
live a terrestrial lifestyle, rather than adapted to living a marine or aquatic lifestyle.
Therefore, “available on land” is the logical answer.

6. Choice A) curtailed is the best answer choice because it denotes that production
time decreased. This supports the notion that the way vehicles produced were
revolutionized.

7. Choice C. syntax is the best answer choice because it means the arrangement of
words and phrases to create well-formed sentences in a language, which supports
“crafting sounds and rhythms of his sentences.” A does not work because there is
nothing that denotes a conversational element to his writing. B doesn’t work because
there is nothing that denotes a comparison between two things. And choice D doesn’t
work because alliteration merely deals with the same sound at the beginning of each
word, so we are missing the rhythm piece.

8. Choice A is the best answer because to reconfigure means to put something


together in a different way. This supports the notion that they had “worked on the
project for years,” meaning that there had to have been multiple iterations in the
development of the space shuttle for it to launch successfully. Reproducing, although a
great trap answer, would be incorrect because if they were to simply reproduce the
space shuttle, there would be no improvements to it (it would be exactly the same as it
was before), which would not ensure that it is safer.

9. Choice B is the best answer because it most logically completes the text’s discussion
of being able to change and adapt in response to new experiences. As used in this

www.strategictestprep.com ♟️ 1-833-300-PREP (7737) ♟️[email protected] 32


context, “plasticity” means the quality of being easily shaped or molded. The text
indicates that plasticity is a term used to describe the brain’s ability to change and
adapt to new experiences. This context conveys that the brain’s capability to do this is
what is being referred to as plasticity.

10. Choice C is the best answer because it most logically completes the text’s
discussion of affective computing. As used in this context, “signals” means indications
or clues. The text indicates that affective computing is a field of computer science that
focuses on interpreting and responding to human emotions. This context conveys that
the indications or clues that the field focuses on are signals, such as facial expressions
and vocal intonations.

11. Choice D is the best answer because it most logically completes the text’s
discussion of the American Dream. As used in this context, “popularized” means made
widely known. The text indicates that the concept of the American Dream was
popularized by James Truslow Adams in his book The Epic of America. This context
conveys that the concept was made widely known by Adams’ book.

12. Choice A is the best answer because it most logically completes the text’s
discussion of the urban heat island phenomenon. As used in this context, “surrounding”
would help support the notion that the urban area is an island, as the rural areas
around it are like the water around an actual island.

13. Choice A is the best answer because it most logically completes the text’s
discussion of the creature’s appearance. In this context, “hideous” means extremely
ugly or unpleasant to look at, and the text indicates that the creature’s appearance
repels those who encounter him. Therefore, the creature’s appearance must be
extremely ugly or unpleasant.

14. Choice D is the best answer because it logically completes the discussion of
Epicurus’s philosophy. In this context, “restrain” means to limit or control. The text
explains that Epicurus believed that a meaningful life could be achieved by restraining

www.strategictestprep.com ♟️ 1-833-300-PREP (7737) ♟️[email protected] 33


one’s desires. This means limiting or controlling one’s desires and focusing on simple
pleasures, avoiding unnecessary stress, and living in moderation.

15. Choice C is the best answer because it most logically completes the discussion of
sustainability. As used in this context, “durable” means lasting. The text explains that
sustainable practices are designed to be maintained over time without depleting
resources. In other words, these practices are meant to last over time without using up
resources.

16. Choice C is the best answer because it most logically completes the discussion of
cuneiform. In this context, “pervasive” means widespread. The text explains that
cuneiform was a complex system of writing used to record various kinds of information
in the ancient Near East. This context suggests that cuneiform was used everywhere
for everything for centuries, meaning it was pervasive, or widespread.

17. Choice B is the best answer because it most logically completes the discussion of
the anthropocene epoch. In this context, “consequences” means results. The text
explains that the anthropocene epoch is marked by the consequences of human
activities, such as burning fossil fuels, releasing pollutants, and destroying natural
habitats. Thus, the term “anthropocene” was coined to describe the current geological
epoch, which is marked by the results of human activities on the planet.

18. Choice C is the best answer because it most logically completes the discussion of
the cello. In this context, “robust” supports a "deeper and more resonant tone." The
text explains that the cello has a different sound than the violin because it is larger and
has thicker strings, which create a deeper and more resonant tone. This context
suggests that the cello’s sound is not miniscule, brassy, or flat.

19. Choice B is the best answer because it most logically completes the discussion of
the Columbian Exchange. In this context, “interaction” means communication and
exchange. The text explains that the Columbian Exchange eventually led to the
transfer of plants, animals, and diseases, as well as the spread of cultural influences,

www.strategictestprep.com ♟️ 1-833-300-PREP (7737) ♟️[email protected] 34


between the Old World and the New World. This context suggests that the Columbian
Exchange eventually led to interaction between the two regions.

20. Choice C is the best answer because it most logically completes the discussion of
the Higgs boson. As used in this context, “elusive” means difficult to find or determine.
The text explains that the discovery of the Higgs boson was a major breakthrough in
particle physics, but scientists are still trying to understand its properties. Thus, the
particle’s actual properties remain elusive, or difficult to find or determine.

21. Choice C is the best answer because it most logically completes the discussion of
the printing press’s impact. As used in this context, “accessible” means easy to obtain
or use. The text suggests that the development of the printing press made it easier to
spread knowledge more quickly and widely than it had been before. Thus, the printing
press made knowledge “accessible.”

22. Choice C is the best answer because it most logically completes the discussion of
the ancient Mesopotamian city of Ur. In this context, “razed” means destroyed. The text
explains that the city of Ur was an important center of culture and learning, but it was
eventually destroyed and abandoned in the early second millennium BCE. Therefore,
“razed” is the logical choice to complete the sentence.

23. Choice A is the best answer because it most logically completes the text’s
discussion of the author’s attempt to explain the Agadir Crisis. As used in this context,
“perfunctory” means done without care or interest. The text states that the author’s
attempt is inaccurate, as it relies on outdated and discredited theories. This context
suggests that the author’s attempt to explain the Agadir Crisis is done without care or
interest.

24. Choice A is the best answer because it most logically completes the text’s
discussion of the author’s opinion of the new policy. As used in this context,
“unfavorable” means having a negative attitude or view. The text states that the author

www.strategictestprep.com ♟️ 1-833-300-PREP (7737) ♟️[email protected] 35


argues that the policy fails to address the underlying causes of the problem. This
context conveys the idea that the author has a negative opinion of the policy.

25. Choice C is the best answer because it most logically completes the text’s
discussion of the author’s critique of the current economy. As used in this context,
“incomplete” means not having all of the facts needed to make a judgment. The text
states that the author’s critique doesn’t take into account the positive effects of recent
economic policies. This context conveys the idea that the author’s critique is not based
on a full understanding of the facts. Therefore, “incomplete” accurately captures the
context of the text.

www.strategictestprep.com ♟️ 1-833-300-PREP (7737) ♟️[email protected] 36


ii. Function of the Underlined Sentence ✌️
Identifying a Function of the Underlined Sentence Question
There are two types of function of the sentence questions:

​ Type I: Those that can only be answered by looking at the specific wording in
the lines provided in the question. In such cases, the lines will typically contain
phrasing, punctuation, or a key transition that points to a particular answer.

​ Type II: Those that cannot be obtained by looking at the lines provided in the
question but that instead depend on contextual information.

Strategies

Strategy #1: Try to Cut Corners First

Is your question a Type I? Start by just reading the underlined sentence and looking at
the answers. Sometimes that is all you need and you can easily answer the question.

Strategy #2: Use Process of Elimination

Some answer choices require you to read more of the paragraph. Look for pronouns or
transition words in the sentence. These are clues that you may need to backtrack to get
more context.

Also, look for answer choices that point to other parts of the text and read those parts
when you’re determining if the answer is correct or not.

Strategy #3: Play Positive/Negative

One of the simplest ways to approach function questions and eliminate answer choices
quickly is to play positive/ negative. Positive passages or portions of passages typically
have positive answers, while negative passages and portions of passages typically
have negative ones.

www.strategictestprep.com ♟️ 1-833-300-PREP (7737) ♟️[email protected] 37


Although answer choices often contain function verbs that are more neutral than the
language of the passage itself, the information in the rest of the answer may be
distinctly positive or negative. Even if this strategy alone does not get you all the way
to the correct answer, it can allow you to quickly eliminate one or two choices upfront,
giving you more time to focus on smaller distinctions between the remaining answers.

Exercises

Common Function Words and Phrases Exercise: Please categorize the following
function words that you will see in function of an underlined sentence questions as
positive, negative, or neutral.

Word Bank: bolster, undermine, exemplify, convey, summarize, advocate, lament,


clarify, illustrate, discredit, identify, describe, concede

www.strategictestprep.com ♟️ 1-833-300-PREP (7737) ♟️[email protected] 38


Answers:
Positive - bolster, exemplify, advocate, clarify, illustrate
Neutral - convey, summarize, identify, describe
Negative - undermine, lament, discredit, concede

Positive/Negative Exercise: Determine if the text sounds positive, negative, or neutral.

Text #1

On what seems like a monthly basis, scientific teams announce the results of new
experiments, adding to a preponderance of evidence that we've been underestimating
animal minds, even those of us who have rated them fairly highly. New animal
behaviors and capacities are observed in the wild,often involving tool use- or at least
object manipulation- the very kinds of activity that led the distinguished zoologist
Donald R. Griffin to found the field of cognitive ethology (animal thinking) in 1978:
octopuses piling stones in front of their hideyholes, to name one recent example; or
dolphins fitting marine sponges to their beaks in order to dig for food on the seabed; or
wasps using small stones to smooth the sand around their egg chambers, concealing
them from predators.

What is the tone of this text? Circle one:

Positive

Negative

Neutral

Check on the next page to see if you’re correct!

www.strategictestprep.com ♟️ 1-833-300-PREP (7737) ♟️[email protected] 39


The tone is Neutral and/or Positive!

This is a science passage, so its tone is relatively neutral. If we look closely at the first
sentence, however, the phrase a preponderance (very large amount) of new evidence
implies that the author has a positive attitude toward the subject. This suggests that
the correct answer will be either positive or neutral and that any negative option can
be eliminated.

Now, let’s match that tone with an answer choice. Cross off any answers that do NOT
match the tone:

A) To describe ways that animals hide from predators

B) To highlight the lack of complexity in animal tools as compared to human ones

C) To provide instances of novel animal behavior in the wild

D) To emphasize the limits of animal · consciousness

Check on the next page to see if you’re correct!

www.strategictestprep.com ♟️ 1-833-300-PREP (7737) ♟️[email protected] 40


On what seems like a monthly basis, scientific teams announce the results of new
experiments, adding to a preponderance of evidence that we've been underestimating
animal minds, even those of us who have rated them fairly highly. New animal
behaviors and capacities are observed in the wild,often involving tool use- or at least
object manipulation- the very kinds of activity that led the distinguished zoologist
Donald R. Griffin to found the field of cognitive ethology (animal thinking) in 1978:
octopuses piling stones in front of their hideyholes, to name one recent example; or
dolphins fitting marine sponges to their beaks in order to dig for food on the seabed; or
wasps using small stones to smooth the sand around their egg chambers, concealing
them from predators.

A) To describe ways that animals hide from predators


B) To highlight the lack of complexity in animal tools as compared to human
ones
C) To provide instances of novel animal behavior in the wild
D) To emphasize the limits of animal · consciousness

As you can see, we crossed off B and D because they were both negative tone
answers!

Now, which one is the correct answer and why?

www.strategictestprep.com ♟️ 1-833-300-PREP (7737) ♟️[email protected] 41


Answer to Positive/Negative exercise: The point is actually found all the way back in
the second sentence: New animal behaviors and capacities are observed in the wild,
often involving tool use. In addition to the word new, the dashes in that sentence
indicate that it is important. (C) rephrases that sentence, so it is correct.

Shortcut: (C)
uses the word novel in its second meaning ("new"). Even in the absence of
any other information, that usage suggests that (C) has an above-average chance of
being correct.

Now, let’s try some example questions…

Example #1 - Function of a Sentence

Scientists have long known that color plays a role in warning animals about
danger. Only recently, however, have they begun to understand how wavelengths of
light (and thus color) appear at different depths and how various marine creatures'
eyes perceive this light and each other- far differently than humans see them. Where
waters are murky, the majority of creatures employ nonvisual forms of communication
such as smell, taste, touch, and sound. But in the clear waters of coral reefs. which
make up less than 1% of the world's oceans. light abounds. vision predominates. and
animals drape themselves in blazing color- not only to menace potential enemies but
also to evade predators, hunt for Prey, and even hide in plain sight.

Which choice best describes the function of the underlined portion in the text as a
whole?

A) It explains how reef animals use color to hide from predators.


B) It describes an unusual form of marine communication.
C) It presents a novel theory about underwater perception.
D) It emphasizes the controversy surrounding a claim about reef animals.

Is this a Type I (all the info you need is in the underlined sentence) or a Type II (you
need to read more of the text for more context) function of a sentence question?

www.strategictestprep.com ♟️ 1-833-300-PREP (7737) ♟️[email protected] 42


Example #2 - Function of a Sentence

To drivers, the color red means stop, but on a map it tells traffic engineers to leap into
action. Traffic control centers like the one on the seventh floor of Boston's City Hall-a
room cluttered with computer terminals and live video feeds of urban intersections-
represent the brain of a traffic system. The city's network of sensors. cables and signals
are the nerves connected to the rest of the body. "Most people don't think there are
eyes and ears keeping track of all this stuff," says John DeBenedictis, the center's
engineering director. But in reality, engineers literally watch our every move, making
subtle changes that relieve and redirect traffic.

Which choice best describes the function of the underlined portion in the text as a
whole?

A) To describe a problem commonly faced by traffic engineers


B) To point out some important differences between traffic control centers and the
human brain
C) To discuss the purpose of items found in traffic control centers
D) To provide examples of ways in which drivers’ actions can be monitored
remotely

Is this a Type I or a Type II function of a sentence question?

www.strategictestprep.com ♟️ 1-833-300-PREP (7737) ♟️[email protected] 43


Example #3 - Function of a Sentence

In August 2009 a consortium of European observatories reported the discovery of


COROT-7c, a second planet orbiting COROT-7. Using the data from both planets, they
were able to calculate that COROT-7b has an average density about the same as
Earth's. This means it is almost certainly a rocky planet made up of silicate rocks like
those in Earth's crust. Not that anyone would call it Earth. The planet and its star are
separated by only 1.6 million miles, 23 times less than the distance between the
parboiled planet Mercury and our Sun. Because the planet is so close to the star, it is
gravitationally locked to it in the same way the Moon is locked to Earth.

Which choice best describes the function of the underlined portion in the text as a
whole?

A) It emphasizes an important distinction between COROT -7b and Earth.


B) It suggests that COROT-7b could eventually come to resemble Earth.
C) It supports the hypothesis that Earth and COROT-7b may share a common origin.
D) It explains how Mercury came to occupy its position within the solar system.

Is this a Type I or a Type II function of a sentence question?

www.strategictestprep.com ♟️ 1-833-300-PREP (7737) ♟️[email protected] 44


Example #4 - Function of a Sentence

The following text is from Frances Hodgson Burnett's 1905 novel, A Little Princess.

If Sara had been older or less punctilious about being quite polite to people, she could
have explained herself in a very few words. But, as it was, she felt a flush rising on her
checks. Miss Minchin was a very severe and imposing person, and she seemed so
absolutely sure that Sara knew nothing whatever of French that she felt as if it would
be almost rude to correct her. The truth was that Sara could not remember the time
when she had not seemed to know French.

Which choice best states the function of the underlined sentence in the text as a
whole?
A) It gives a description of the physical appearance of one of the characters.
B) It establishes why one of the characters was not comfortable in the situation.
C) It reinforces an emotional state alluded to in the previous sentence.
D) It introduces the interaction between two characters in the following sentences.

Is this a Type I or a Type II function of a sentence question?

Answers to these 4 examples can be found on the next page.

www.strategictestprep.com ♟️ 1-833-300-PREP (7737) ♟️[email protected] 45


Answers
Example 1: B
Example 2: D
Example 3: A
Example 4: C

Extra Practice

1. Sarah always found solace in reading, losing herself in the worlds of far-off places
and adventurous characters. Tonight, the book on her lap seemed to resonate with
her more than ever, as the protagonist faced challenges eerily similar to her own life
obstacles. The evening sky outside her window was dark and filled with stars,
matching the sense of both wonder and loneliness that enveloped her.

Which choice best describes the function of the underlined sentence in the text as a
whole?

A) It provides a counterpoint to the character's emotional state.


B) It reflects the character's current situation through the book she's reading.
C) It offers a moral lesson that the character is learning.
D) It creates a detailed image of the physical setting of the scene.

2. Research led by environmental scientist Dr. Emily Watson indicates that urban
green spaces have a significant impact on mental well-being. Utilizing data from
a seven-year study involving 12 different cities, the team measured levels of
stress hormones in volunteers who spent time in both urban and natural
settings. The findings reveal that those who spent more time in green spaces
had consistently lower levels of stress hormones compared to those who
remained in urban areas.

Which choice best states the function of the underlined sentence in the overall
structure of the text?

www.strategictestprep.com ♟️ 1-833-300-PREP (7737) ♟️[email protected] 46


A) To highlight the major implications of the research
B) To offer an anecdotal account that supports the research
C) To outline the methodology employed in the research study
D) To identify limitations in the research study's scope

3. In the quaint town of Harmony, the annual strawberry picking season was a
cherished tradition for the community. The townspeople, young and old, would
grab their baskets and head to the fields where the strawberries grew in
abundant clusters. However, not all strawberry patches were created equal;
some areas of the field yielded large, juicy strawberries, while others offered
smaller, less flavorful fruits. Families spread out across the expansive field, eyes
sharp and fingers nimble, as they sought out the most bountiful spots.

Which choice best describes the function of the underlined sentence in the text as a
whole?

A) It emphasizes the competitiveness of the strawberry picking tradition.


B) It explains why the strawberry picking event is cherished by the community.
C) It provides details about the conditions of the strawberry field that affect the
activity.
D) It portrays the skill and expertise needed for successful strawberry picking.

4. (1) According to ecologist Dr. Sarah Thompson, the decline of bee populations is
having a devastating impact on pollination and, by extension, agriculture. (2) In
many regions, farmers are increasingly relying on artificial methods to pollinate
crops, as natural pollinators like bees become scarce. (3) One popular method
involves using specialized drones equipped with brushes that mimic the action
of bee wings. (4) These drones fly from flower to flower, depositing pollen and
facilitating fertilization. (5) As a result, technology is becoming a crucial
alternative in maintaining crop yields.

Which choice best describes the function of the underlined portion in the text as a
whole?

www.strategictestprep.com ♟️ 1-833-300-PREP (7737) ♟️[email protected] 47


A) It elaborates on a technological solution discussed earlier in the text.
B) It provides an example of how agriculture is unaffected by the decline in bee
populations.
C) It offers a counterargument to Dr. Thompson's claims about the decline in
pollinators.
D) It provides additional information about why bees are essential for pollination.

5. (1) Emily, a talented young musician, always felt a little out of place in her small,
quiet hometown. (2) She practiced the violin for hours every day, her notes
echoing through the house. (3) The only music store in town had a limited
selection, mostly focused on country and rock genres. (4) However, the internet
became her sanctuary; she could explore an infinite variety of compositions and
styles. (5) Emily dreamt of attending a prestigious music school in a big city,
where she could fully immerse herself in her passion.

Which choice best describes the function of sentence 5 in the text as a whole?
A) It provides an explanation for why Emily spends so much time practicing.
B) It shows that Emily feels confined by the limited musical options in her town.
C) It indicates that Emily has no appreciation for the music genres popular in her
hometown.
D) It suggests that Emily aspires to a more fulfilling musical experience beyond her
small town.

6. (1) Studies have shown that healthy eating can dramatically reduce the risk of
heart disease. (2) However, convincing people to adhere to a diet rich in fruits
and vegetables remains a significant challenge for healthcare professionals. (3)
Nutritionist Dr. Emily Green recently conducted an experiment using interactive
workshops to educate the public on healthy eating but found that long-term
dietary changes were minimal among participants.

Which choice best describes the function of the second sentence in the overall
structure of the text?

A) It describes a universally accepted solution to the problem stated in the first


sentence.

www.strategictestprep.com ♟️ 1-833-300-PREP (7737) ♟️[email protected] 48


B) It challenges the validity of the studies mentioned in the first sentence.
C) It highlights the practical issue that experts like Dr. Emily Green are trying to solve.
D) It summarizes the main findings of Dr. Emily Green's experiment.

7. (1) The popularity of electric vehicles (EVs) has been growing rapidly due to
concerns about climate change. (2) Governments worldwide are offering
incentives to consumers who switch to EVs, including tax breaks and subsidies.
(3) However, the production of lithium-ion batteries, a key component in EVs,
has been linked to environmental degradation and human rights issues. (4)
Researchers like Dr. Emily Thompson are exploring alternative, sustainable
materials for EV batteries. (5) Dr. Thompson's recent study demonstrates that a
new type of organic battery can offer similar performance with less
environmental impact.

Which choice best describes the function of the third sentence in the overall
structure of the text?
A) It outlines the primary objective of Dr. Emily Thompson's research.
B) It presents a counterargument to the increasing adoption of electric vehicles.
C) It gives a specific example of a government policy that encourages the use of
electric vehicles.
D) It summarizes the main benefits of switching to electric vehicles.

8. (1) Many plants have developed various strategies to ward off herbivores. (2)
Some species secrete bitter-tasting substances that discourage animals from
eating them. (3) The acacia tree, for instance, harbors ants that attack any
animal that tries to feed on the tree's leaves. (4) Scientists have noticed that
these ant-defended trees tend to have fewer instances of herbivory compared to
other plants in their ecosystem.

Which choice best describes the function of sentence 4 in the text as a whole?

A) It provides an example of an animal that benefits from the plant's defensive


strategy.
B) It offers further explanation on why herbivores avoid eating the acacia tree.

www.strategictestprep.com ♟️ 1-833-300-PREP (7737) ♟️[email protected] 49


C) It reveals the result of the defensive strategy utilized by the acacia tree in its
ecosystem.
D) It outlines the chemical components of the substances secreted by plants like the
acacia tree.

Answers - Extra Practice

1. Answer: B

Answer Explanation:

A) The underlined sentence does not provide a counterpoint to Sarah's emotional state;
instead, it actually resonates with her current situation, making this choice incorrect.

B) The underlined sentence reflects how the book Sarah is reading is particularly
relevant to her current life challenges. This makes it the correct answer for describing
the function of the sentence in the context of the passage.

C) While the book resonates with Sarah's situation, the sentence does not specifically
state that she is learning a moral lesson from it, making this choice incorrect.

D) The underlined sentence focuses on the book and how it relates to Sarah's life,
rather than detailing the physical setting. Sentence (3) provides details about the
physical setting, making this choice incorrect.

2. Answer: C

Answer Explanation:

A) The underlined sentence does not highlight the major implications of the research; it
is focused on explaining the methodology used in the study.

www.strategictestprep.com ♟️ 1-833-300-PREP (7737) ♟️[email protected] 50


B) The underlined sentence does not offer an anecdotal account; instead, it details the
data and methods used for the study.

C) The underlined sentence outlines the methodology of the study, including the
duration of the study and what was measured (levels of stress hormones), making this
the correct answer.

D) The underlined sentence does not identify any limitations in the research study's
scope; it explains the methodology employed.

3. Answer: C

Answer Explanation:

A) The underlined sentence doesn't focus on the competitiveness of the tradition; it


focuses on the variability of the strawberry patches in the field.

B) The underlined sentence doesn't explain why the event is cherished by the
community; it provides details about the strawberry field itself.

C) The underlined sentence provides details about the varying quality of the
strawberry patches, which would naturally affect where people choose to pick
strawberries. This choice correctly identifies the function of the sentence in the text.

D) While the text does talk about families spreading out and being keen-eyed, The
underlined sentence itself doesn't focus on the skill or expertise needed for picking
strawberries; it focuses on the qualities of the strawberry patches.

4. Answer: A

Answer Explanation:

A) The underlined portion provides specific details about the specialized drones
mentioned as a popular artificial method of pollination. It elaborates on how these
drones function, making this the correct answer.
B) The text overall does not argue that agriculture is unaffected by the decline in bee
populations. Instead, it describes alternative methods that are being used because of
the decline.

www.strategictestprep.com ♟️ 1-833-300-PREP (7737) ♟️[email protected] 51


C) The underlined portion does not offer a counterargument to Dr. Thompson's claims;
rather, it supports the idea that alternative methods are being sought due to the
decline in natural pollinators.

D) The underlined portion is not focused on why bees are essential for pollination; it
focuses on an alternative to natural pollination due to the decline of bees.

5. Answer: D

Answer Explanation:

A) While the underlined sentence might imply that attending a prestigious music
school is one reason Emily practices so much, its primary function is to express Emily's
aspirations for a larger musical life.

B) The underlined sentence does indirectly suggest that Emily is not fully satisfied with
the musical options in her town, but its main focus is on her aspirations for the future,
not her present dissatisfaction.

C) The text does not indicate that Emily has no appreciation for other genres; rather, it
points out that she dreams of a more fulfilling musical experience outside of her
hometown.

D) The underlined sentence illustrates Emily's aspiration to broaden her musical


experience by attending a prestigious music school in a big city. It encapsulates her
wish for a life more attuned to her passion, which is not fully possible in her current
environment. This is the correct answer.

6. Answer: C
Answer Explanation:

A) The second sentence does not describe a solution to the problem; rather, it identifies
an ongoing challenge, so this choice is incorrect.

B) The second sentence does not challenge the validity of the studies mentioned in the
first sentence. It acknowledges the difficulty in implementing the recommended

www.strategictestprep.com ♟️ 1-833-300-PREP (7737) ♟️[email protected] 52


healthy eating habits, so this choice is incorrect.

C) The second sentence sets up the practical challenge that healthcare professionals
face, including Dr. Emily Green. While the first sentence states that healthy eating can
reduce the risk of heart disease, the second sentence emphasizes that getting people
to actually make those healthy choices is difficult.

D) The second sentence does not summarize the main findings of Dr. Emily Green's
experiment. That information is provided in the third sentence, making this choice
incorrect.

7. Answer: B

Answer Explanation:

A) The underlined sentence does not outline Dr. Emily Thompson's primary objective; it
sets the stage for why such research may be necessary, but it doesn't specifically
mention her research objectives.

B) The underlined sentence introduces a complication or counterargument to the


growing popularity of electric vehicles mentioned in sentences 1 and 2. While the text
begins by talking about the growing adoption of EVs due to climate concerns and
government incentives, sentence 3 brings up the ethical and environmental concerns
associated with lithium-ion batteries.

C) The underlined sentence does not provide a specific example of a government


policy that encourages the use of electric vehicles. That topic is covered in sentence 2.
D) The underlined sentence does not summarize the main benefits of switching to
electric vehicles; instead, it presents an issue that complicates the narrative of electric
vehicles being an unambiguously good choice for the environment.

8. Answer: C

Answer Explanation:

www.strategictestprep.com ♟️ 1-833-300-PREP (7737) ♟️[email protected] 53


Option A is incorrect because the underlined sentence does not provide an example of
an animal benefiting from the plant's defensive strategy; it discusses the benefits to
the tree itself.

Option B is incorrect because the underlined sentence does not offer further
explanation on why herbivores avoid eating the acacia tree. It states the observed
result, not the reason.

Option C is correct because it accurately describes the function of the underlined


sentence. The sentence reveals how effective the acacia tree's defensive strategy is by
stating that these ant-defended trees have fewer instances of herbivory in their
ecosystem.

Option D is incorrect because the underlined sentence does not discuss the chemical
components of the substances secreted by the plants.

www.strategictestprep.com ♟️ 1-833-300-PREP (7737) ♟️[email protected] 54


iii. Main Purpose of the Text 🤟
Difference Between Main Idea and Main Purpose of Text

What is the difference between a main purpose of the text and a main idea of the text?
Let’s cover that first since the SAT asks both types of questions.

● The main idea of the passage is its conclusion; or in other words, its author's
position on the central content. The point. The main idea is usually stated
directly.

● The main purpose of the passage is the reason the author is writing the text.
Some reasons include:
- persuade the reader about the main idea
- to inform the reader about the topic of the passage
- to compare something
- to refute a theory
- etc.

✨Looking for additional resources to guide you in your Digital SAT prep journey? ✨
👉Practice wherever, whenever you want with the Preptly: Digital SAT Prep
App! With over 2,000 users and an average rating above 4-stars, see why other
students love using the app and why it is helping them achieve higher scores on
their SAT! 📲 Get Preptly in the App Store or in Google Play today!

👉Looking to improve your math score as well? 🤨 We have a new Self-Paced


Digital SAT Math Course! (Available in 2024)

www.strategictestprep.com ♟️ 1-833-300-PREP (7737) ♟️[email protected] 55


Preliminary Exercise

For each statement, circle if it is a Main Idea or Main Purpose.

Statement 1: It presents the study by Johnson and Thompson to critique the methods
and results reported in previous studies on the impact of music on studying.

Main Idea Main Purpose

Statement 2: Modern technology has greatly improved the healthcare system.

Main Idea Main Purpose

Statement 3: To provide a detailed analysis of the benefits and drawbacks of AI


technology in the workplace.

Main Idea Main Purpose

Statement 4: Processed foods tend to be more difficult to resist than healthful options.

Main Idea Main Purpose

Now that you see the difference between a main idea and main purpose, let’s look at
what strategies you can use from Main Purpose questions…

www.strategictestprep.com ♟️ 1-833-300-PREP (7737) ♟️[email protected] 56


Strategies

Strategy #1: Read the text once fully first.

Since you need the big picture, cutting too many corners and not reading will hurt your
chances. Keep in mind this while reading:

(1) Why is the author writing this?

Strategy #2: Pay attention to the first sentence.

You may sometimes be able to identify the likely answer based on what the first
sentence is saying, since it is usually the topic sentence.

www.strategictestprep.com ♟️ 1-833-300-PREP (7737) ♟️[email protected] 57


Examples

Example #1 - Main Purpose

Throughout the dinosaurs' time on Earth, there was an amplification of boniness


and spikiness; however, the advantage of skull frills and back plates is hardly
self-evident. The solid-domed skull of Pachycephalosaurus seems made for butting-
but for butting what? The skull would be all but useless against a predator with the
size and power of Tyrannosaurus Rex. The skulls of some Pachycephalosaurs,
moreover, were flat and thin—a bad design for contact sports—and the spikes
protruding from them were most probably blunt rather than sharp.

Tip: Use the first sentence to help you answer the question!

Which choice best states the main purpose of the text?

A) To suggest that a longstanding hypothesis about dinosaurs may be incorrect.

B) To introduce a study and raise some questions about the validity of its findings.

C) It describes a dinosaur species in order to illustrate a puzzling trend.

D) It presents an emerging mystery and discusses an attempt to solve it.

Answer Explanation on the next page!

www.strategictestprep.com ♟️ 1-833-300-PREP (7737) ♟️[email protected] 58


What do we learn from the first sentence? That the advantage of skull-frills and back
plates is hardly self- evident. In other words, the advantage is unclear. So logically, the
purpose of the passage is to discuss that lack of clarity.

Furthermore, the presence of the word however in the first sentence suggests that
what follows is the "new idea" - no further information will be introduced to contradict
it.

The only two answers that contain words consistent with the information we've
established should be present are (C) ("puzzling") and (D) ("mystery"). The passage
does not provide a possible explanation ("a hypothesis") for the phenomenon in
question, nor does it mention a study.

In terms of choosing between (C) and (D)t look at how the answer choices are worded.
If the mystery were "emerging/' the passage would indicate that it was new, but there
is nothing here to suggest that is the case. That eliminates (D). In contrast, (C) merely
refers to"a puzzling trend." The rest of the answer is consistent with the description of
Pachycephalosaurus.

www.strategictestprep.com ♟️ 1-833-300-PREP (7737) ♟️[email protected] 59


Example #2 - Main Purpose (Fiction example)

Tip: You should have read the question first. So, as you’re reading this text, keep in
mind you’re looking for the main purpose.

In our family, the women made the ink. We stayed home. We all worked—me, Gao
Ling, my aunts and girl cousins, everybody. Even the babies and Great-Granny had a
job of picking out stones from the dried millet we boiled for breakfast. We gathered
each day in the ink-making studio. According to Great-Granny, the studio began as a
grain shed that sat along the front wall of the courtyard house. Over the years, one
generation of sons added brick walls and a tile roof. Another strengthened the beams
and lengthened it by two pillars. The next tiled the floors and dug pits for storing the
ingredients. Then other descendants made a cellar for keeping the ink sticks away from
the heat and cold. "And now look," Great-Granny often bragged. "Our studio is an ink
palace."

Which choice best states the main purpose of the text?

A) To describe the process by which a character masters a difficult task

B) To convey a character's role and sense of accomplishment in her family's work

C) To explain the influence of a distinguished relative on a character and her family

D) explore the effects of a character's decision on the members of her family

www.strategictestprep.com ♟️ 1-833-300-PREP (7737) ♟️[email protected] 60


Example #2 Answer: The correct answer is B because the narrator takes great pride in
her family's business and her role within it, and the passage serves to convey that fact.

Extra Practice

1. In 1996, social psychologist Amy Cuddy was exploring the effects of body language
on self-perception. She discovered that adopting "power poses" could actually change
a person's hormonal levels and increase their feelings of confidence. Inspired by this
finding, Cuddy developed a series of workshops aimed at helping people improve their
self-esteem and performance in high-pressure situations. These workshops have since
become incredibly popular, with organizations from all over the world participating.

Which choice best states the main purpose of the text?

A) To provide an overview of the field of social psychology


B) To discuss Cuddy's discovery about "power poses"
C) To advertise the workshops developed by Cuddy
D) To report on the global reach of Cuddy's workshops

2. The following text is an excerpt from a 2005 article about Steve Jobs, co-founder of
Apple Inc. "From the creation of the first Macintosh computer to the development of
the iPod, iPhone, and iPad, Jobs has revolutionized the way we interact with
technology. His knack for combining aesthetics with functionality has often been
dubbed a 'stroke of genius.' Not just a businessman, but a visionary, Jobs believed that
technology could be both useful and beautiful."

Which choice best states the main purpose of the text?

A) To argue that Steve Jobs was the most important figure in the tech industry
B) To highlight Steve Jobs' unique ability to blend aesthetics and functionality in
technology

www.strategictestprep.com ♟️ 1-833-300-PREP (7737) ♟️[email protected] 61


C) To explain how Apple products like the iPod, iPhone, and iPad came to be
developed
D) To persuade readers to buy Apple products

3. The following text is from F. Scott Fitzgerald's 1925 novel "The Great Gatsby." In
this passage, Nick Carraway describes his feelings about the city. "The city seen from
the Queensboro Bridge is always the city seen for the first time, in its wild promise of
all the mystery and beauty in the world. Yet, when I closed the door of my small
apartment, I felt a wave of loneliness wash over me, as if the vibrant metropolis was
mocking my solitude."

Which choice best states the main purpose of the text?

A) To explain why Nick moved to the city in search of adventure


B) To describe the attraction and simultaneous detachment Nick feels towards the city
C) To showcase the architectural beauty of the city as seen from Queensboro Bridge
D) To reveal that Nick has grown tired of city life and wishes to leave

4. This is an excerpt from Dr. Alice Thompson's speech on climate change: "Today, we
stand on the precipice of a new era—an era that beckons us to action. Earth’s climate
has changed before, but never at the pace we are witnessing today. We owe it to
future generations to address the crisis head-on. I urge each of you to be the
change-makers, to fight for a more sustainable planet. Time is of the essence!"

Which choice best states the main purpose of the text?

A) To provide scientific data about the rate of climate change.


B) To emphasize the urgency of tackling climate change now.
C) To criticize those who do not believe in climate change.
D) To present various methods for reducing one's carbon footprint.

www.strategictestprep.com ♟️ 1-833-300-PREP (7737) ♟️[email protected] 62


5. The following text is from John Steinbeck's 1937 novella "Of Mice and Men." George
and Lennie are itinerant workers looking for employment during the Great Depression.
George sat by the edge of the campfire, his face worn and anxious. Lennie, seemingly
oblivious to George's unease, was looking at the stars, murmuring about rabbits. The
fire flickered, casting shadows on their faces as it battled the darkness of the open
field. The sounds of the night seemed amplified, as though nature itself was awaiting
their next move.

Which choice best states the main purpose of the text?

A) To illustrate the financial struggles that George and Lennie face


B) To describe George's plans for the future
C) To show the contrast between George's anxiety and Lennie's innocence
D) To suggest that George and Lennie have a disagreement

6. The following text is an excerpt from Jhumpa Lahiri's 2003 short story "The Third
and Final Continent." He arrived in Boston, from a far-off land, wearing a single
suitcase and a weary smile. Months of separation from his family had turned into years.
Here, he found a job, rented a small apartment, and embraced the solitude as his new
companion. But sometimes at night, he'd lie awake, listening to the symphony of the
city, wondering what he had left behind.

Which choice best states the main purpose of the text?

A) To demonstrate the narrator's excitement about moving to a new country.


B) To describe the struggles and sacrifices that come with living far from one's
homeland.
C) To convey the narrator's strong desire to go back to his home country immediately.
D) To show how easily the narrator adapted to his new environment without any
challenges.

www.strategictestprep.com ♟️ 1-833-300-PREP (7737) ♟️[email protected] 63


7. In Mary Shelley's novel "Frankenstein," Victor laments to Walton, "I should have
heeded my father's caution and never dabbled in things beyond human understanding.
Yet here I am, pursued by my own creation. Had I known the dire consequences, I
would have never embarked on this damnable quest."

Which choice best states the main purpose of the text?

A) To highlight Victor's regret for not listening to his father's advice


B) To imply that Victor still feels a sense of accomplishment about his scientific
experiment
C) To argue that Victor blames Walton for his misfortunes
D) To reveal that Victor was unaware of the scientific principles behind his experiment

8. The following text is adapted from a passage in Virginia Woolf's 1925 novel "Mrs
Dalloway." The character Clarissa is contemplating the view from her balcony.
She looked over the balcony and saw the beautiful park filled with people walking
their dogs, children playing, and couples taking leisurely strolls. Yet as she watched,
she couldn't help but think about the recent news of economic downturns and wars
that raged in distant lands. "How can one reconcile the tranquility of this moment with
the chaos of the world?" she wondered to herself.

Which choice best states the main purpose of the text?

A) To describe Clarissa's daily routine of observing people from her balcony


B) To contrast Clarissa's feeling of serenity with her thoughts on the world's unrest
C) To reveal Clarissa's disdain for people who enjoy the park while ignoring world
issues
D) To emphasize how Clarissa finds inspiration from the natural beauty of the park

www.strategictestprep.com ♟️ 1-833-300-PREP (7737) ♟️[email protected] 64


Answers - Extra Practice

1. Answer: B

Answer Explanation:

A) To provide an overview of the field of social psychology: The text doesn't aim to
provide an overview of social psychology as a whole; it focuses on a specific discovery
made by Amy Cuddy in the field.

B) To discuss Cuddy's discovery about "power poses," is accurate because the text
primarily focuses on detailing a specific discovery made by social psychologist Amy
Cuddy. It describes what the discovery was about ("power poses"), its implications (it
led to changes in self-perception and hormonal levels), and its wider impact (it led to
workshops and gained global attention).

C) To advertise the workshops developed by Cuddy: While the text mentions that
workshops were developed based on the discovery, the primary focus is not to
advertise or promote these workshops. The workshops are mentioned as a
consequence of Cuddy's discovery.

D) To report on the global reach of Cuddy's workshops: Again, while the text mentions
that the workshops have become popular worldwide, this is not the main purpose of
the text. The primary focus is on Cuddy's discovery regarding "power poses."

2. Answer: B

Explanation:

A) To argue that Steve Jobs was the most important figure in the tech industry: While
the text praises Jobs, it doesn't make an argument comparing him to other figures in
the tech industry.

B) The text focuses on praising Steve Jobs for his unique talent in blending aesthetics
with functionality. Phrases like "revolutionized the way we interact with technology"
and "technology could be both useful and beautiful" underscore this point.

www.strategictestprep.com ♟️ 1-833-300-PREP (7737) ♟️[email protected] 65


C) To explain how Apple products like the iPod, iPhone, and iPad came to be
developed: The text doesn't delve into the development process of these products; it
focuses on Jobs' ability to blend aesthetics and functionality.

D) To persuade readers to buy Apple products: There's no mention or implication in the


text about encouraging the reader to purchase Apple products; the focus is on Jobs'
skills and vision.

3. Answer: B

Explanation:

A) To explain why Nick moved to the city in search of adventure: The text does not
focus on the reasons why Nick moved to the city; rather, it emphasizes his current
emotional state.

B) The main purpose of the text is to highlight the contrast between Nick's perception
of the city as full of "mystery and beauty" and the loneliness he feels once inside his
apartment. This illustrates the complex emotions of attraction and detachment he
experiences.

C) To showcase the architectural beauty of the city as seen from Queensboro Bridge:
While the text does mention the view from the bridge, it does not emphasize the
architectural aspects. The focus is more on Nick's emotional state.

D) To reveal that Nick has grown tired of city life and wishes to leave: The text does
not indicate that Nick wishes to leave the city; it simply contrasts his feelings of
attraction and loneliness.

4. Answer: B

Explanation:

www.strategictestprep.com ♟️ 1-833-300-PREP (7737) ♟️[email protected] 66


A) To provide scientific data about the rate of climate change: The text mentions that
Earth's climate is changing quickly but does not provide specific scientific data.

B) The main purpose of the text is to emphasize the urgency of addressing the climate
crisis. Phrases such as "stand on the precipice," "beckons us to action," "owe it to future
generations," and "Time is of the essence" contribute to this sense of urgency.

C) To criticize those who do not believe in climate change: The text doesn't criticize
climate change skeptics; instead, it focuses on urging action.

D) To present various methods for reducing one's carbon footprint: While the text calls
for action, it does not outline specific methods for reducing one's carbon footprint.

5. Answer: C

Explanation:

A) To illustrate the financial struggles that George and Lennie face: While it's true that
they are itinerant workers during the Great Depression, the text doesn't directly
discuss their financial struggles.

B) To describe George's plans for the future: George's future plans are not discussed in
this excerpt.

C) The primary focus of the text is on the differing emotional states of George and
Lennie as they sit by the campfire. George is described as "worn and anxious," while
Lennie is "looking at the stars, murmuring about rabbits," demonstrating his innocence
and lack of awareness of the troubles that beset them. The text captures this
juxtaposition to show the contrast between the two characters.

D) To suggest that George and Lennie have a disagreement: There is no indication in


the text that George and Lennie are in disagreement about anything.

6. Answer: B

www.strategictestprep.com ♟️ 1-833-300-PREP (7737) ♟️[email protected] 67


Explanation:

Here's why the other options don't fit:

A) To demonstrate the narrator's excitement about moving to a new country: Although


the narrator may have reasons for moving, the text does not specifically highlight his
excitement about it.

B) The text captures the narrator's experience of moving to a new country, where he
has to live far away from his family. It touches upon his initial difficulties ("Months of
separation from his family had turned into years") and the loneliness he feels
("embraced the solitude as his new companion"). While he seems to have adjusted to
some extent, the element of longing for what he left behind is evident ("wondering
what he had left behind").

C) To convey the narrator's strong desire to go back to his home country immediately:
While the text mentions that he wonders about what he left behind, it does not
indicate a strong desire to return immediately.

D) To show how easily the narrator adapted to his new environment without any
challenges: The text clearly outlines the struggles he faces, such as years of separation
from family and embracing solitude.

7. Answer: A

Explanation:

A) The text primarily serves to express Victor's remorse about his actions, particularly

not heeding his father's warnings. He explicitly states, "I should have heeded my

father's caution," and speaks about the "dire consequences."

B) To imply that Victor still feels a sense of accomplishment about his scientific
experiment: There's no indication in the text that Victor feels any pride or
accomplishment; instead, he shows regret.

www.strategictestprep.com ♟️ 1-833-300-PREP (7737) ♟️[email protected] 68


C) To argue that Victor blames Walton for his misfortunes: Victor is speaking to
Walton about his own regrets, not laying blame on Walton.

D) To reveal that Victor was unaware of the scientific principles behind his experiment:
Victor's statement doesn't suggest that he didn't understand the science; rather, he
didn't understand the ethical or moral implications.

8. Answer: B

Explanation:

A) To describe Clarissa's daily routine of observing people from her balcony: There is
no indication that this is a daily routine for Clarissa, nor does the text focus on that
aspect.

B) The text serves primarily to juxtapose Clarissa's sense of peace as she observes the

park from her balcony with her contrasting thoughts about the world's problems. This

internal dichotomy between the tranquility of the moment and the chaos of the world

is the main focus.

C) To reveal Clarissa's disdain for people who enjoy the park while ignoring world
issues: The text does not imply that Clarissa has disdain for people enjoying the park;
her contemplation is more inward-facing.

D) To emphasize how Clarissa finds inspiration from the natural beauty of the park:
While Clarissa observes the park's beauty, the text actually focuses on her internal
conflict between her sense of peace and her awareness of worldly troubles.

www.strategictestprep.com ♟️ 1-833-300-PREP (7737) ♟️[email protected] 69


iv. Comparative Text 👯
Text 1/Text 2 Passages can have the following scenarios:
1) The authors strongly disagree
2) The authors agree on certain points, so their opinions slightly overlap*

*Most popular.

Strategies

There are five steps to answering the comparative passage questions:

Strategy #1: Follow these five steps to increase your success rate!

​ 1) Read Text 1: identify the main point.

​ 2) Read Text 2: identify the main point.*

​ 3) Write out the relationship.

​ 4) Answer the question in your own words.

​ 5) Look at the choices, and pick the one closest to your answer.

* often this main point starts with a contrast word

Strategy #2: Skip these and guess a negative tone answer


If you’re not aiming for a high/perfect score and your reading abilities are weak, or you
constantly run out of time, you should guess on these and flag them. They are time
consuming and there are easier questions you should tackle first.

Strategy #3: Look for a Contrast Statement


Often, the contrast statement highlights how the authors disagree in some way. It is
the most important sentence in the text for that reason, as typically the answer stems
from that disagreement.

www.strategictestprep.com ♟️ 1-833-300-PREP (7737) ♟️[email protected] 70


Strategy #4: Weed out positive tone answer choices
Often, the authors will disagree in some way and that is the focus of these questions.
This strategy doesn’t always work, but is sometimes useful.

Now, let’s try a few examples following the 5 steps highlighted above…

www.strategictestprep.com ♟️ 1-833-300-PREP (7737) ♟️[email protected] 71


Examples

Example #1

Text 1

Our food now travels an average of 1,500 miles before ending up on our plates. This
globalization of the food supply has serious consequences for the environment, our
health, our communities and our taste buds. Much of the food grown in the
breadbasket surrounding us must be shipped across the country to distribution centers
before it makes its way back to our supermarket shelves. Because uncounted costs of
this long-distance journey (air pollution and global warming, the
ecological costs of large-scale monoculture, the loss of family farms and local
community dollars) are not paid for at the checkout counter, many of us do not think
about them at all.

Text 2

Just how much carbon dioxide is emitted by transporting food from farm to fork? Pierre
Desrochers and Hiroko Shimizu cite a comprehensive study done by the United
Kingdom's Department of Environment, Food and Rural Affairs (DEFRA) which
reported that 82 percent of food miles were generated within the U.K. Consumer
shopping trips accounted for 48 percent and trucking for 31 percent of British food
miles. Air freight amounted to less than 1 percent of food miles. In total, food
transportation accounted for only 1.8 percent of Britain's carbon dioxide emissions.

Text 1 Main Point:

Text 2 Main Point:

Relationship:

How would Desrochers and Shimizu (Text 2) most likely describe the view
presented in Text 1? (answer this first before looking at the answer choices)
www.strategictestprep.com ♟️ 1-833-300-PREP (7737) ♟️[email protected] 72
Now, let’s look at the answer choices as the last step.

Based on the texts, how would Desrochers and Shimizu (Text 2) most likely describe
the view presented in Text 1?

A) It is strongly supported by data compiled by DEFRA.

B) It overstates the effects of transporting food on the environment.

C) It appears justified by preliminary findings but has not yet definitively proven.

D) It is highly implausible because most consumers do not consider the source of


their food.

Classify the answer choices as positive/negative.

A: _______________

B: _______________

C: ________________

D: _________________

After eliminating any positive answer choices, pick the correct answer.

Answer Explanation on the next page.

www.strategictestprep.com ♟️ 1-833-300-PREP (7737) ♟️[email protected] 73


Answer Explanation to Example #1:

(A) Positive - eliminate it.

(B) Negative - keep it.

(C) Positive - eliminate it

(D) Negative - keep it.

Now, be careful with (D). You might remember something about people not thinking
about where their food comes from, but that's in Text 1. If you go back to Text 2, you'll
find it says nothing about that idea.

Text 2 does, however, state that food transportation accounted for only 1.8 percent of
Britain's carbon dioxide emissions, which stands in contrast to the doom-and-gloom
environmental scenario presented in the first passage. Based on that statement, you
can assume that Desrochers and Shimizu would consider the claim in Text 1 to be
somewhat exaggerated, i.e.,"overstated." That makes (B) correct.

www.strategictestprep.com ♟️ 1-833-300-PREP (7737) ♟️[email protected] 74


Example #2

Text 1

In recent years, there has been an explosion of scientific research revealing precisely
how positive feelings are beneficial. We know that they motivate people to pursue
important goals and overcome obstacles, offer protective benefits against the effects of
stress, improve our social connectedness, and even ward off illness. The science of
happiness has spawned a small industry of motivational speakers and research
enterprises. Clearly, happiness is popular.

Text 2

Happiness, it turns out, has a cost when experienced too intensely. For instance, we
often are told that happiness can open up our minds to foster more creative thinking
and help us tackle problems or puzzles. This is the case when we experience moderate
levels of happiness. But according to Mark Alan Davis's 2008 analysis of
the relationship between mood and creativity, when people experience intense and
perhaps over- whelming amounts of happiness, they no longer experience the same
creativity boost. What's more, psychologist Barbara Fredrickson has found that too
much positive emotion- and too little negative emotion-makes people inflexible in the
face of new challenges.

Text 1 Main Point:

Text 2 Main Point:

Relationship:

How would Mark Alan Davis most likely respond to what “we know” in Text 1?
(answer this first before looking at the answer choices)

www.strategictestprep.com ♟️ 1-833-300-PREP (7737) ♟️[email protected] 75


Based on the texts, what would Mark Alan Davis most likely respond to what "we
know" in Text 1?

A) By emphasizing the connection between creativity and negative emotions

B) By acknowledging the benefits of positivity in moderation but cautioning against


it in excess

C) By questioning the motives of the participants in the happiness industry

D) By challenging the connection between positive feelings and personal


fulfillment

Classify the answer choices as positive/negative.

A: _______________

B: _______________

C: ________________

D: _________________

After eliminating any positive answer choices, pick the correct answer.

www.strategictestprep.com ♟️ 1-833-300-PREP (7737) ♟️[email protected] 76


Example #2 Answer: B

(A) Positive - eliminate it.

(B) Negative - keep it.

(C) Negative - keep it.

(D) Negative - keep it.

What does Text 1 state that "we know"? If you scan the passage for these words,
you'll find them at the beginning of the second sentence, which tells us that positive
feelings are associated with a host of happy outcomes (motivation to achieve goals,
overcome obstacles, protect against stress, etc.).

Next, what position does Mark Alan Davis (Text 2) hold? That when people experience
intense and perhaps overwhelming amounts of happiness, they no longer experience
the same creativity boost. In other words, too much positivity is not a good thing.

Logically, how would Davis respond to the example from Text 1? Presumably by
reiterating his position that while moderate amounts of happiness are helpful, too
much of it is not, i.e., "by acknowledging [its] benefits but cautioning against it in
excess." (B) is thus correct.

(A) is incorrect because Davis's position on negative emotions is never mentioned. (C)
is beyond the scope of what can be inferred from Davis's position.

Be careful with (D): Davis is only mentioned in relation to creativity, not "personal
fulfillment," and he does not deny the beneficial effects of happiness - he only
indicates that too much of it can have negative effects.

www.strategictestprep.com ♟️ 1-833-300-PREP (7737) ♟️[email protected] 77


Example #3

Text 1

By investigating interactions between tree species, scientists have found that trees
leverage similarities and differences in their microbial "makeup" to recognize other
trees of their own species, and that they preferentially share nutrients with them
through their mycorrhizal network- thesystemsofrootsandfungithat connect them. For
example, Douglas Fir trees growing in the same plot have been shown to share more
carbon among them than with trees of other species.

Text 2

The notion that trees send out resources to strengthen a community composed of
members of their species is unlikely because groups that cooperate would need to win
out over groups made up of competing individuals. According to plant ecologist
Kathryn Flinn, while trees can sometimes facilitate each other's growth, a forest does
not function like a single organism: it includes a vast array of species with a constantly
shifting variety of interactions, both cooperative and competitive.

Text 1 Main Point:

Text 2 Main Point:

Relationship:

What would Kathryn Flinn most likely say about the “Douglas Fir trees” in Text 1?
(answer this before looking at the answer choices):

www.strategictestprep.com ♟️ 1-833-300-PREP (7737) ♟️[email protected] 78


Ok, now let’s look at the answers…

Based on the texts, what would Kathryn Flinn most likely say about the “Douglas
Fir trees” in Text 1

A) Their mycorrhizal network is not fully understood.

B) They function as if they were a single organism.

C) They are also likely to compete among themselves for some resources.

D) The amount of carbon they share will vary according to environmental


conditions.

Answer Explanation on the next page.

www.strategictestprep.com ♟️ 1-833-300-PREP (7737) ♟️[email protected] 79


Example #3 Answer: C

What does Text 1 say about Douglas Fir trees? When they grow in the same plot, they
share more carbon among them than with trees ofother species.

Next, what is Kathryn Flinn's position (Text 2)? She asserts that while trees can
sometimes help facilitate each other's growth, relationships among organisms in forest
are constantly shifting and include both cooperation and competition.

Logically, how would Flinn respond to the observation that Douglas firs seem to
"prefer" sharing carbon with members of their own species? Given her perspective that
species both cooperate and compete, she would probably respond to an example of
cooperation by suggesting that the Douglas firs also compete at times. And that is
exactly what (C) says.

(A) and (D) are irrelevant to Flinn's discussion, and (B) is directly contradicted by her
assertion that a forest does not function like a single organism.

www.strategictestprep.com ♟️ 1-833-300-PREP (7737) ♟️[email protected] 80


Extra Practice

1.
Text 1
Scientists have long believed that supermassive black holes lie at the center of every
galaxy. However, a recent study challenges that notion. Researchers found evidence of
a galaxy without a central black hole, which has turned the astronomical community on
its head. The study suggests that there may be alternate explanations for the
characteristics of galaxies we observe.

Text 2
The research on a galaxy without a central black hole is intriguing but not definitive.
Dr. Anderson and her team have indicated that while the data suggests this galaxy
lacks a supermassive black hole, further research is needed. They note that there could
be other factors that account for the galaxy's unique characteristics, and that this
discovery should be treated as preliminary.

Based on the texts, what would the author of Text 2 most likely say about Text 1's
characterization of the discovery of the galaxy without a central black hole?

A) It is an irrefutable conclusion that challenges existing theories.


B) It is groundbreaking but should be considered as preliminary.
C) It is unremarkable since many galaxies likely exist without central black holes.
D) It is misleading because the astronomical community largely disregards it.

2.
Text 1

For years, nutritionists have encouraged people to consume more whole grains as a
way to improve heart health. The logic has been that whole grains are rich in fiber and
nutrients, which help regulate cholesterol and blood pressure levels.

Text 2

Dietician Sarah Patterson and her team have found that while whole grains are
generally beneficial for heart health, the impact may be dependent on an individual's

www.strategictestprep.com ♟️ 1-833-300-PREP (7737) ♟️[email protected] 81


gut microbiome. Some individuals with certain gut flora profiles may not experience
the same heart health benefits from consuming whole grains, according to Patterson's
research.

Based on the texts, how would Sarah Patterson and her team (Text 2) most likely
respond to the recommendations of nutritionists in Text 1?

A) By suggesting that the general advice is too simplistic and may not apply to
everyone.
B) By endorsing the conventional advice wholeheartedly and urging people to
consume even more whole grains.
C) By challenging the idea that whole grains are the sole contributor to heart health
improvement.
D) By asserting that whole grains are actually detrimental to heart health and should
be avoided.

3. Text 1
Marine biologist Sarah Johnson has been studying the effects of climate change on
coral reefs. Her research, based on years of observations and data collection, suggests
that rising ocean temperatures are leading to widespread coral bleaching and that the
decline could be irreversible.

Text 2

Climate scientist Emily Williams and her team caution that while rising temperatures
are a concern for coral reefs, Johnson's conclusions might be overly simplistic. They
argue that multiple factors, such as ocean acidification and human activity, also
contribute significantly to coral health and should not be ignored.

Based on the texts, how would Emily Williams and her team (Text 2) most likely
characterize Sarah Johnson's conclusion presented in Text 1?

Answer Choices:

A) As accurate, because both studies agree that rising ocean temperatures are the
primary cause of coral decline.

www.strategictestprep.com ♟️ 1-833-300-PREP (7737) ♟️[email protected] 82


B) As limited, because it does not consider other crucial factors affecting coral health.

C) As revolutionary, because it uncovers a previously unknown cause for coral decline.

D) As conflicting, because the two studies have opposing viewpoints on the causes of
coral decline.

4. Text 1

It's a popular belief that renewable energy sources like solar and wind can replace
fossil fuels within the next couple of decades. This view is supported by the rapid
advancements in solar panel and wind turbine technology, along with falling prices
and increasing energy efficiency.

Text 2

In a 2023 research paper, energy analyst Sarah Thompson and engineer Robert Choi
argue that the transition to renewable energy is more complicated than often
portrayed. While they acknowledge the improvements in renewable technology, they
point out that intermittency issues (like cloudy days for solar and calm days for wind),
the need for vast amounts of energy storage, and the infrastructure changes required
make a full transition away from fossil fuels more complex and likely slower than
anticipated.

Based on the texts, how would Thompson and Choi (Text 2) most likely respond to the
"popular belief" presented in Text 1?

A) By highlighting the benefits of renewable energy technology but questioning the


timeframe for full transition.

B) By discrediting renewable energy as a viable alternative to fossil fuels in the


foreseeable future.

C) By asserting that the transition to renewable energy is easier than previously


thought due to technological advancements.

D) By claiming that solar and wind technology is sufficient for immediate replacement
of fossil fuels without additional infrastructure changes.

www.strategictestprep.com ♟️ 1-833-300-PREP (7737) ♟️[email protected] 83


5. Text 1

A new study published in the journal Biotropica found evidence that invasive rats are
significantly reducing the number of seabirds on islands in the Caribbean. Researchers
conducted a series of experiments on the uninhabited islands of Anguilla and Anegada
and found that when rats were removed from the islands, the number of seabird nests
increased by about 125 percent.

Text 2

The researchers believe that rat control efforts have the potential to be a fruitful
strategy for conservation in the Caribbean, saying that if similar efforts were extended
to other islands, seabirds would benefit. They suggest any animal-control efforts
should also consider the removal of cats and other predators that harm seabirds, as
well as the restoration of natural vegetation and egg-laying sites.

Based on the texts, what is an additional recommendation Text 2 makes regarding the
conservation of seabirds that was not discussed in Text 1?
A. Experimental trials to measure the effects of predator removal
B. Reducing the number of cats on the islands
C. Protective legislation to ensure the safety of seabirds
D. Programs to educate people on the endangered seabirds.

6. Text 1

Observations from the 2030s of an ultra-cold gas cloud located in the nearby
constellation Vela suggest an entirely new class of astronomical object: a cold
molecular cloud. These clouds are areas of low dust, dense molecular gases, and very
low temperatures, and may be the birthplace of stars and planets. Such gases can
condense due to external pressure and, when radiation and gravity are taken into
account, form clouds of interstellar dust, as well as proto-planetary disks that are
thought to be the cradles of stars.

www.strategictestprep.com ♟️ 1-833-300-PREP (7737) ♟️[email protected] 84


Text 2

Recent observations of the same cold molecular cloud in Vela have detected molecular
hydrogen in quantities not seen in other such clouds. This molecular material is
important because it is the precursor to deuterium, which is thought to form the dust
that accretes to form stars and planets. Consequently, these discoveries suggest that
this particular cloud may be more conducive to forming stars and planets than any
other cloud.

Based on the texts, how would the discoveries mentioned in Text 2 most likely affect
the conclusions mentioned in Text 1?
A. They would contradict the assumption that external pressure is necessary for
condensing gasses.
B. They would suggest that external pressure alone may not be enough for the
formation of stars and planets.
C. They would confirm that radiation and gravity are necessary in the formation of stars
and planets.
D. They would indicate that the cold molecular cloud in Vela is unique in its ability to
form stars and planets.

7. Text 1

In the Arctic, the polar bear has long been an important symbol of the region’s culture
and ecology. But its survival is now threatened by the effects of climate change,
including the melting of sea ice. This is causing polar bears to travel farther for food,
leading to more encounters with humans and their communities, which can be
dangerous for both species.

Text 2

In response to the changing conditions, some Arctic communities are considering ways
to reduce conflicts between polar bears and humans. One possible solution is the use
of electric fences, which have been successful in other areas. Additionally, some
communities are advocating for the implementation of regulations that would limit
human activities in polar bear habitats.

www.strategictestprep.com ♟️ 1-833-300-PREP (7737) ♟️[email protected] 85


Based on the texts, how would the author of Text 1 likely respond to the argument of
Text 2?
A. Electric fences could hurt the polar bears and should thus be avoided.
B. The safety of polar bears is more important than the safety of humans and steps
should be taken to ensure their protection.
C. A better solution would be to feed the polar bears where they are so they will be
deterred from traveling.
D. Although a fence is feasible to help reduce conflicts between polar bears and
humans, it doesn’t address the underlying problem of climate change.

8. Text 1

The invention of the cotton gin in 1793 revolutionized the cotton industry, allowing for
the faster production of cotton from raw fibers. The device was an important factor in
the spread of slavery in the American South, as it made cotton production more
profitable and thus increased the demand for workers.

Text 2

Despite offering many benefits, the cotton gin had an unexpected consequence on the
textile industry. The machine allowed for the mass production of cotton, resulting in a
sharp drop in the price of cotton cloth and leading to a greater demand for other
fabrics, such as wool and silk.

Based on the texts, how would the author of Text 2 most likely respond to Text 1’s
characterization of the cotton gin?
A. By acknowledging the impact of the cotton gin on slavery but noting that it also had
a positive effect on the textile industry
B. By challenging the idea that the cotton gin had a major role in the spread of slavery
C. By asserting that the cotton gin also had a negative impact on the cotton industry
D. By suggesting that the cotton gin had no impact on the textile industry

www.strategictestprep.com ♟️ 1-833-300-PREP (7737) ♟️[email protected] 86


9.

Text 1

Until recently, the concrete psychological effects of fiction on individuals and society
were largely a matter of speculation. However, research in psychology is beginning to
provide answers about how fiction can expand our moral imaginations. For example, a
series of studies conducted by Keith Oatley, Maja Djikic, and Raymond Mar found that
fiction measurably improves people's ability to guess others' mental states by looking
at only their eyes. They interpreted this finding as evidence for the idea that fiction
allows people to connect with something larger than themselves.

Text 2

An empirical approach to the question of whether fiction improves empathy was taken
by David Kidd and Emanuele Castano, who conducted five experiments in which
participants read fictional excerpts and then responded to images of facial expressions.
The results showed that the participants had improved their theory of mind (ToM), or
their ability to infer the thoughts and emotions of others. As Kidd points out, however,
highly developed ToM does not always translate into more ethical behavior: the ability
to manipulate someone, for instance, also requires a heightened understanding of
other people's emotions.

Based on the texts, how would Kidd and Castano most likely respond to Oatley,
Djikic, and Mar in Text 1?

A) By acknowledging the importance of connecting with others

B) By conceding that fiction can allow people to transcend their everyday lives

www.strategictestprep.com ♟️ 1-833-300-PREP (7737) ♟️[email protected] 87


C) By pointing out that empathy can have negative as well as positive effects

D) By emphasizing that individuals with high ToM may sometimes prefer


non-fiction

10.

Text 1

On May 21, 2019, midsize black holes were detected for the first time when the
U.S.-based Laser Interferometer Gravitational-Wave Observatory (LIGO) and its
European counterpart Virgo captured a tremor from a pair of black holes merging deep
in space. Priyamvada Natarajan, an astrophysicist who has long worked on black hole
growth models, believes that black holes this size are born in nuclear star clusters,
dense collections of stars found near galactic centers. These holes sweep through the
cluster, adding gas and dust, until they settle at a single location and cease to expand.

Text 2

Imre Bartos and other researchers working on "hierarchical merger" models, in which
black holes grow by eating one another, focus on one major data point in the
LIGONirgo findings. The angular momentum, or "spin," of a black hole ranges from Oto
1. When two black holes of similar size combine, the resulting black hole usually has a
spin of around 0.7. Significantly, the two black holes involved in the merger recorded
by LIGO and Virgo had 0.69 and 0.73 respectively, suggesting that they both might
have formed in previous mergers.

Based on the texts, what would Imre Bartos most likely say about Priyamvada
Natarajan's belief in Text 1?

A) It underestimates midsize black holes' spin.

B) It misstates the time when the merger occurred.

C) It relies too heavily on data from LIGONIGO.

D) It overlooks the significance a crucial statistic.

www.strategictestprep.com ♟️ 1-833-300-PREP (7737) ♟️[email protected] 88


Answers - Extra Practice

1. Correct Answer: B) It is groundbreaking but should be considered as preliminary.

Answer Explanation:

A) The author of Text 2 would not agree that the discovery is an "irrefutable
conclusion." They explicitly mention that "further research is needed" and that the data
should be treated as "preliminary."

B) This is the correct answer. Text 2 describes the research as "intriguing" but
emphasizes the need for further research, suggesting that it should be treated as
"preliminary," aligning with Text 1's description of the discovery as something that
"turned the astronomical community on its head."

C) Text 2 does not suggest that the discovery is "unremarkable." In fact, the author
calls it "intriguing," indicating some level of significance or interest.

D) Text 2 does not indicate that the astronomical community "largely disregards" the
discovery; it describes the research as "intriguing but not definitive," suggesting that it
has caught the community's attention but should be further verified.

2.
Correct Answer: A) By suggesting that the general advice is too simplistic and may not
apply to everyone.

Answer Explanation:

A) This is the correct answer. Text 2 states that the benefits of whole grains on heart
health may depend on an individual's gut microbiome. Thus, Patterson would likely
consider the general advice to eat more whole grains too simplistic as it doesn't
consider individual variations.

B) While Text 2 does mention that whole grains are generally beneficial, it also points
out that this might not be the case for everyone due to variations in gut microbiomes.

www.strategictestprep.com ♟️ 1-833-300-PREP (7737) ♟️[email protected] 89


So, it is unlikely that Patterson would endorse the advice wholeheartedly without
reservations.

C) Text 2 does not challenge the idea that whole grains contribute to heart health. It
merely adds nuance by suggesting that the benefits might vary from person to person.

D) Text 2 actually says that whole grains are generally beneficial for heart health, so it
would not assert that they are detrimental and should be avoided.

3.
Correct Answer: B) As limited, because it does not consider other crucial factors
affecting coral health.

Answer Explanation:

A) Both studies agree that rising ocean temperatures are a concern, but Williams and
her team find Johnson's conclusion to be overly simplistic. They wouldn't characterize it
as "accurate."

B) This is the correct answer. Emily Williams and her team suggest that Johnson's
study doesn't take into account other significant factors like ocean acidification and
human activity, making it "limited."

C) Text 2 does not indicate that Sarah Johnson's findings are revolutionary or uncover a
previously unknown cause. Both teams are concerned with well-known factors like
climate change.

D) The studies do not have opposing viewpoints on the causes of coral decline; rather,
Williams and her team suggest that Johnson's view is incomplete. They wouldn't
characterize the relationship as "conflicting."

4.
Correct Answer: A) By highlighting the benefits of renewable energy technology but
questioning the timeframe for full transition.

www.strategictestprep.com ♟️ 1-833-300-PREP (7737) ♟️[email protected] 90


Answer Explanation:

A) This is the correct answer. Thompson and Choi acknowledge the advancements in
renewable energy technology but express concerns about the timeframe for a full
transition from fossil fuels, pointing out various complexities.

B) This choice is incorrect because Thompson and Choi don't discredit renewable
energy as a viable alternative; they just assert that the transition is more complicated
than popularly believed.

C) This choice is also incorrect. Thompson and Choi do not assert that the transition is
easier than previously thought; in fact, they argue the opposite.

D) This choice is not accurate because Thompson and Choi emphasize the need for
more infrastructure changes and energy storage solutions, directly countering the idea
that current technology is sufficient for immediate replacement.

5.
Choice B is the best answer because Text 2 mentions the removal of cats and other
predators, which is something not discussed in Text 1. Text 1 solely focuses on the
removal of rats from the islands.

6.
Choice D is the best answer because it reflects the most likely effect that the
discoveries mentioned in Text 2 would have on the conclusions mentioned in Text 1.
Text 2 states "these discoveries suggest that this particular cloud may be more
conducive to forming stars and planets than any other cloud," which suggests that Vela
is unique in its ability to form stars and planets.

7.
Choice C is the best answer because climate change is addressed as a problem in Text
1, which causes polar bears to encroach on human lands, but Text 2 fails to address
solutions for that proposed problem.

8.
Choice C is the best answer because it reflects how the author of Text 2 would most
likely respond to Text 1 based on the information provided. The sharp drop of price in

www.strategictestprep.com ♟️ 1-833-300-PREP (7737) ♟️[email protected] 91


cotton and demand for other materials like silk and wool suggest that the cotton gin
also impacted the cotton industry negatively.

9. Choice C is the best answer. What is the position of Oatley, Djikic, and Mar in Text 1?
Fiction improves people's ability to guess others' mental states and improves their
ability to empathize.

What is the position of Kidd and Castano in Text 2? They agree that fiction improves
people's understanding of others' mindsets but point out that these skills can be used
in negative ways.

Logically, how would Kidd and Castano respond to the position in Text 1? That position
only emphasizes the positive aspect of reading fiction, so they would presumably point
out the negative one. And that is what (C) says.

(A) is outside the scope of the passages, which focus on the effects of fictions rather
than relationships in general. (B) is incorrect because Kidd and Castano only discuss
the effects of fiction on theory of mind, not on "transcend[ing] everyday lives." (D) is
entirely off-topic; neither passage discusses or implies anything about the effects of
reading non-fiction.

10. Choice D is the best answer.

What is Priyamvada Natarajan's belief in Text 1? That black holes this size (i.e., of medium
size) come from nuclear star clusters and sweep through them gathering gas and dust.

What is Imre Bartos's position (Text 2)? Black holes grow by eating one another, a view that
is based on an analysis of black holes' spin- a specific statistic that falls within a
narrow range.

Logically, how would Bartos respond to Natarajan? Because it is not mentioned that
Natarajan takes "spin" into account, Bartos would presumably focus on the absence of
that figure, i.e., "a crucial statistic." That makes (D) correct.

(A) is
incorrect because Natarajan's model does not include spin at all. (B) is incorrect
because Bartos does mention when the merger occurred. (C) is incorrect because

www.strategictestprep.com ♟️ 1-833-300-PREP (7737) ♟️[email protected] 92


Bartos's model, not Natarajan's, is mentioned as relying on specific data from
UGO/VIGO.

www.strategictestprep.com ♟️ 1-833-300-PREP (7737) ♟️[email protected] 93


VII. Information and Ideas
i. Main Idea of Text 💡
As a reminder from page 39, the main idea is the point (argument) of the text. The
main purpose is the reason the author is writing the text (the why behind the
argument).

Strategies

Strategy #1: Look for a broad, general statement


Avoid detailed sentences. They are likely supporting idea sentences.

Strategy #2: Look for what the majority of the paragraph is about
If 90% of the paragraph talks about weather patterns in Detroit, then the main idea
probably isn’t about weather in the United States - it’s about the weather in Detroit!

Strategy #3: Know that the topic sentence can be anywhere in the paragraph on the
DSAT!
Main ideas typically come in the topic (first) sentence. However, on the Digital SAT, the
main idea is often placed in the middle or end of the paragraph. The main idea can
also be implied (not stated directly).

Strategy #4: Use Process of Elimination


Some of the answer choices will be ridiculous and completely off topic.

www.strategictestprep.com ♟️ 1-833-300-PREP (7737) ♟️[email protected] 94


Exercises

Locate the main idea in each example.

Example #1: Summer is a wonderful time to spend at Rehoboth. It is a beach with


light- colored, soft sand. The coastline goes on for a long way and many people enjoy
walking along it. Children like to play in the surf and walk along the rocks that are
visible at low tide. This is a fun beach for people of all ages.

Example #2: The United States seems to be in love with the idea of going out to eat.
Because of this, a real variety of restaurants has come about specializing in all kinds of
foods.McDonald's is the king of a subgroup of restaurants called fast-food restaurants.
Chances are, no matter where you live, there is a McDonald's restaurant near you.
There are even McDonalds in the Soviet Union. Now McDonald's is trying something
new. It is called McDonald's Express and there is a test site in Peabody, Massachusetts.
It is part of a Mobil gas station. This allows you to fill up with gas and fill up on food at
the same time. What will they think of next?

Example #3: Costs were low that year and the output high. There was a good person
for each job and the market remained firm. There were no losses from fire. All in all it
was the best year in the history of the company.

Example #4: Do you wear glasses? Make sure your glasses fit well. The earpieces
should be at eye level. Don’t try to adjust the earpieces yourself. Take your glasses for
adjustments to the place you bought them. Keep your glasses in a case when you’re
not wearing them. This will prevent scratches. Keep the lenses clean. A soft cloth is
best for cleaning.

Answers on the following page.

www.strategictestprep.com ♟️ 1-833-300-PREP (7737) ♟️[email protected] 95


Answers:
Example #1
Main idea: Summer is a wonderful time to spend at Rehoboth.
Location: First sentence (topic sentence)

Example #2
Main Idea: McDonald's is the king of fast food.
Location: Middle of paragraph (3rd sentence)

Example #3
Main Idea: All in all it was the best years in the history of the company.
Location: Last sentence
(Note that the first three sentences give details to explain why it was the best year in
the company’s history.)

Example #4
Main Idea: Implied, not stated directly in the paragraph
The main idea is that taking care of your glasses involves many steps.

www.strategictestprep.com ♟️ 1-833-300-PREP (7737) ♟️[email protected] 96


Examples

Example #1 - Main Idea

Admired primarily for her exquisite calligraphy, Otagaki Rengetsu (1791-1875)


was among Japan's most celebrated artists. She was also a writer and ceramicist, often
inscribing her poems in her own calligraphy onto clay vessels- a distinctive blending of
art forms not replicated by any other artist in Japanese history. Her work was in such
great demand during the nineteenth century that every household in Kyoto was said to
own her pottery, and today scrolls and ceramics bearing her calligraphy are sought
after by collectors.

Can you pinpoint the topic (main idea)? Circle or underline it in the text!

Now, these are answer choices that you would see on the SAT for this type of question.
Can you pinpoint which one best states the main idea of the passage?

A) Otagaki Rengetsu's artistic creations are prized for their unique qualities.

B) Inscribed clay vessels have traditionally played an important role in Japanese


art.

C) The collaboration between writers and ceramic workers produced highly


distinctive works of art in Japan.

D) Many households in Kyoto once featured scrolls produced by Otagaki Rengetsu.

Keep in mind that the correct answer may not say the main idea verbatim. The SAT will
use similar language and synonyms instead.

www.strategictestprep.com ♟️ 1-833-300-PREP (7737) ♟️[email protected] 97


Answer to Example #1: A
Answer choice A closely mirrors the topic sentence, which is located in the first
sentence of the text.

Example #2 - Main Idea (when it’s not the first sentence)

Can you pinpoint the main idea of this text? Underline it!

Sometime near the end of the Pleistocene, a band of people left northeastern
Asia, crossed the Bering land bridge when the sea level was low, entered Alaska and
became the first Americans. Since the 1930s, archaeologists have thought these
people were members of the Clovis culture. First discovered in New Mexico in the
1930s, the Clovis culture is known for its distinct stone tools, primarily fluted projectile
points. For decades, Clovis artifacts were the oldest known in the New World, dating
to 13,000 years ago. But in recent years, researchers have found more and more
evidence that people were living in North and South America before the Clovis.

Answer on the following page….

www.strategictestprep.com ♟️ 1-833-300-PREP (7737) ♟️[email protected] 98


Answer to Example #2

Sometime near the end of the Pleistocene, a band of people left northeastern
Asia, crossed the Bering land bridge when the sea level was low, entered Alaska and
became the first Americans. Since the 1930s, archaeologists have thought these
people were members of the Clovis culture. First discovered in New Mexico in the
1930s, the Clovis culture is known for its distinct stone tools, primarily fluted projectile
points. For decades, Clovis artifacts were the oldest known in the New World, dating
to 13,000 years ago. But in recent years, researchers have found more and more
evidence that people were living in North and South America before the Clovis.

Keep in mind that the main idea is not just the topic (Clovis Culture) but it answers “so
what?”

In other words, what is it about Clovis culture that is important to know? What is the
main point the author is trying to convey?

www.strategictestprep.com ♟️ 1-833-300-PREP (7737) ♟️[email protected] 99


Example #3 - Main Idea (finding synonyms)

Kente, the traditional fabric of Ghana's Asante people, has evolved into a symbol of
many meanings - political and cultural, African and American, honorary and everyday.
"What is called kente is many things," says Doran H. Ross, director of the Fowler
Museum of Cultural History, though he notes its origin is Ghana's strip-woven cloth.
But Ross says kente appears just as widely today in Western-style tailored clothing,
and in other ways that make it the most recognizably African textile.

Which Choice Best States The Main Idea Of The Text?

A) Kente can only be produced by specialized weavers.

B) Many different type of African textiles are used in modern clothing.

C) The origins of kente are somewhat controversial.

D) Kente has acquired a wide range of associations.

Answer on the following page….

www.strategictestprep.com ♟️ 1-833-300-PREP (7737) ♟️[email protected] 100


Example #3 Answer: D
Has a “wide range of associations” is a synonym for “a symbol of many meanings.”

Example #4 - Main Idea (Poetry)

I'd love to write of the beautiful,


I'd love to write of the brave,
And read the minds of others,
And note their winning ways.
I would not judge the beautiful
By the beauty of their faces,
By suppositions or the like,
Or their pretended graces.

Which choice best states the main idea of the text?

A) The speaker is drawn to people who are attractive and graceful.

B) The speaker does not feel suited to the task of judging others.

C) The speaker believes that external features are a reliable indicator of internal
ones.

D) The speaker is unconcerned with superficial appearances.

Answer on the following page….

www.strategictestprep.com ♟️ 1-833-300-PREP (7737) ♟️[email protected] 101


Example #4 Answer: D

Use synonyms again here! I “would not judge the beautiful by the beauty in their faces”
is a synonym for “the speaker is unconcerned with superficial appearances.”

Would not judge = unconcerned

Beauty in their faces = superficial appearances

All other answer choices have something wrong with them.

A) Says the opposite of what the text is saying

B) The text does not say this anywhere. If anything, it is implied that the author is
up to judging people, just not by their appearance.

C) This is not stated anywhere in the text.

www.strategictestprep.com ♟️ 1-833-300-PREP (7737) ♟️[email protected] 102


Extra Practice
1. The Maori tattoo tradition, known as Ta Moko, has deep roots dating back
centuries in New Zealand. Originating as a rite of passage and identity, it
became an integral facet of Maori culture. By the onset of the eighteenth
century, it had developed into an elaborate art form, with intricate patterns
signifying various tribal affiliations and personal stories. In contemporary times,
Ta Moko remains a revered practice in Maori society, embodying both ancestral
ties and individual journeys.

What is the main idea of the text?

A) Ta Moko serves as an emblem of both tribal affiliations and personal histories in


Maori culture.
B) The eighteenth century saw the decline of the Ta Moko tradition in New Zealand.
C) Only those with direct ancestral ties to the Maori are allowed to have Ta Moko.
D) Ta Moko is a relatively new trend that emerged in modern-day New Zealand.

2. In medieval Europe, town criers were crucial figures in communities. Clad in


distinctive attire and ringing a handbell, they announced news, proclamations,
and even market days in public squares. With the advent of newspapers, radio,
and now social media, news dissemination has largely moved to these
platforms. However, town criers haven't entirely disappeared from the modern
landscape. While their role as primary news bearers may have diminished, they
are still treasured in certain communities. Specifically, they hold ceremonial
roles in local festivals, parades, and other special events, wearing their
traditional regalia and ringing their bells. Their presence serves as a nostalgic
nod to the past, and people value the unique, personal touch they bring to
announcements and news-sharing in these contexts.

Which choice best states the main idea of the text?

A) Town criers have been completely replaced by modern technologies like social
media for disseminating news.
B) Despite the advent of modern communication methods, town criers are still valued
for their traditional role in specific contexts.
C) Most town criers prefer their ceremonial roles in festivals rather than announcing
news in public squares.

www.strategictestprep.com ♟️ 1-833-300-PREP (7737) ♟️[email protected] 103


D) Town criers have largely abandoned their traditional roles in favor of using social
media platforms to share news.

3. For decades, the idea of a "dark matter" filling the universe had been largely
theoretical. Scientists posited that this unseen material must exist to account for
the gravitational pull observed in galaxies. However, in 2022, physicist Dr. Emily
Johnson and her team reported they had indirectly detected the presence of
dark matter. Using a specialized telescope, they found evidence that suggests
dark matter interacts with regular matter in ways previously unconsidered.

Which choice best states the main idea of the text?

A) Dr. Emily Johnson and her team have found new evidence that challenges the
fundamental principles of physics.
B) Dr. Emily Johnson and her team have indirectly detected dark matter, providing
evidence that supports its existence.
C) Specialized telescopes are revolutionizing the way scientists explore the mysteries
of the universe.
D) The concept of dark matter has been debunked due to the latest findings by Dr.
Emily Johnson and her team.

4. Architects Sarah Zhang and Tim Lee have developed a co-living building that
aims to foster social interactions among residents. Features like communal
kitchens and shared garden spaces are designed to make it easier for people to
connect. After a year of residence, tenant Mary Smith reported that her social
life had markedly improved and that she felt a sense of community she had
never felt before.

Which choice best states the main idea of the text?

A) Mary Smith has always struggled with making social connections until she moved
into the co-living building.
B) Zhang and Lee's building has multiple features that are designed to be
environmentally friendly.

www.strategictestprep.com ♟️ 1-833-300-PREP (7737) ♟️[email protected] 104


C) The co-living building designed by Zhang and Lee appears to effectively foster
social interactions among its residents.
D) Shared garden spaces are the most crucial feature of the building designed by
Zhang and Lee.

5. The following text is adapted from Louisa May Alcott's 1868 novel "Little
Women." Jo, one of the four March sisters, has just received an unexpected sum
of money for a story she submitted to a newspaper.

Elated and surprised, Jo couldn't believe her eyes when she saw her name printed
beneath the story title. With the money, she could contribute to the family's expenses
and also indulge in more writing supplies. However, the greater thrill came from the
knowledge that others were reading her words. That very thought filled her with an
invigorating sense of purpose that she had never felt before.

Which choice best states the main idea of the text?

A) Jo is relieved to have extra money to spend on herself.


B) Jo feels a newfound sense of purpose from having her work published.
C) Jo is excited about the prospect of becoming a famous writer.
D) Jo is primarily interested in contributing to her family's finances.

6. The following text is an excerpt from "Harry Potter and the Sorcerer's Stone,” by
J.K. Rowling:

"After finding Harry spending multiple nights staring into the Mirror of Erised, a
magical artifact that shows the viewer their heart's deepest desires, Dumbledore
informs him that the mirror will be moved to a new location. Before leaving,
Dumbledore shares a piece of wisdom: 'The happiest man on earth would be able to
use the Mirror of Erised like a normal mirror, that is, he would look into it and see
himself exactly as he is. Does that help?'"

What is the main idea of the text?

A) Dumbledore is emphasizing that magical artifacts like the Mirror of Erised can be
misleading and should be viewed as ordinary objects.

www.strategictestprep.com ♟️ 1-833-300-PREP (7737) ♟️[email protected] 105


B) Dumbledore is assuring Harry that he is one of the happiest people alive, as he's
found friendships and a sense of belonging at Hogwarts.
C) Dumbledore is suggesting that ultimate happiness lies in being content with
oneself, rather than yearning for what one doesn't have.

D) Dumbledore views the Mirror of Erised as a tool for self-improvement, encouraging


Harry to change himself according to what he sees in it.

7. The following text is adapted from Henry James' 1880 novel Washington
Square. Mrs. Penniman is a widow who lives with her brother.

Mrs. Penniman was a tall, thin, fair, rather faded woman, with a perfectly amiable
disposition, a high standard of gentility, a taste for light literature, and a certain foolish
indirectness and obliquity of character. She had a passion for little secrets and
mysteries- a very innocent passion, for her secrets had hitherto always been as
unpractical as addled eggs. She was not absolutely veracious; but this defect was of no
great consequence, for she had never had anything to conceal.

Which choice best states the main idea of the text?

A) Mrs. Penniman is a puzzling and mysterious figure .

B) Mrs. Penniman is a passionate reader of novels .

C) Mrs. Penniman is frequently difficult to get along with.

D) Mrs. Penniman is romantic and sentimental

8. Among the thousands of species that have made their way around the world since
European exploration began in the fifteenth century, knotweed is widely regarded as
one of the most intractable. Removing it completely requires extracting the land itself;
if anything is left behind, the weed can return repeatedly, regenerating from minuscule
fragments after as long as twenty years. One study found that knotweed could regrow
from a root fragment weighing just 0.3g- about as much as a pinch of salt.

www.strategictestprep.com ♟️ 1-833-300-PREP (7737) ♟️[email protected] 106


Which choice best states the main idea of the text?

A) Knotweed was among the earliest plant species to be transported between


continents.

B) Knotweed can regenerate even when the land it grows on has been removed.

C) Knotweed is exceptionally difficult to eradicate permanently.

D) In comparison to other plants, knotweed takes much longer to reach its full size.

9. The following text appeared in Josephine Heard's 1890 work, "On receiving
Tennyson's Poems from Mrs. M. H. Dunton, of Brattleboro, Vt."

Dear Friend, since you have chosen to associate


My humble thoughts with England's poet laureate,
I trust that he will bear me pleasant company,
And soon we shall far more than mere acquaintance be.
Since childhood's days his name I have revered,
And more and more it has become to me endeared;
I blush not for the truth, I but confess,
I very wealthy feel since I his "works" possess.

Which choice best states the main idea of the text?

A) She is impressed by her friend's personal acquaintance with Tennyson.


B) She views Tennyson with respect and wishes to understand his work deeply.
C) She is embarrassed by her poor understanding of Tennyson's work.
D) She believes that her own poetry is equal to Tennyson's.

10. Navajo pawn originated in the 1870s as a bartering system that was altogether
different from traditional banking. Based on relationships of mutual trust, it evolved to
be a fully integrated part of Navajo life. By the middle of the twentieth century, it had
become a highly sophisticated and complex practice, with more than 150 active trading
posts. Today it remains a pivotal aspect of Navajo society.

Which choice best states the main idea of the text?

A) Pawn has played a significant role in modern Navajo culture.


www.strategictestprep.com ♟️ 1-833-300-PREP (7737) ♟️[email protected] 107
B) During the twentieth century, pawn was gradually replaced by other banking
options.

C) Pawn is more complex than traditional banking.

D) Pawn became popular in Navajo society because it was based on personal


relationships.

11. Although publishers and critics classified Octavia Butler's novels as science fiction-
a genre that Butler enjoyed deeply and referred to as "potentially the freest genre in
existence"- her works attracted a diverse readership, and Butler resisted being
associated exclusively with that form. Indeed, she was also the author of a number of
essays, and her book Parable of the Sower was adapted into an opera by the
mother-and-daughter team Bernice Johnson Reagon and Toshi Reagon. Combining
African-American spirituals, soul, rock and roll, and folk music, it debuted at The Public
Theater in New York City in 2015.

Which choice best describes the overall structure of the text?

A) A work is presented, and its effects are considered.

B) An interpretation of a novel is described, and an opposing view is introduced.

C) Examples of an author's writing are given, and their significance is discussed.

D) A claim is made, and supporting examples are provided to illustrate it.

www.strategictestprep.com ♟️ 1-833-300-PREP (7737) ♟️[email protected] 108


Answers - Extra Practice

1.A

Choice A is the best answer because it is supported by the passage which states, "with
intricate patterns signifying various tribal affiliations and personal stories." It
emphasizes the deep cultural significance of Ta Moko in Maori society, representing
both community and individual narratives.

Why the other answers are incorrect:

B: The text mentions that by the onset of the eighteenth century, Ta Moko "had
developed into an elaborate art form." There's no mention of its decline during this
period. Therefore, this choice is incorrect.

C: While the passage discusses the deep roots and cultural significance of Ta Moko, it
does not specify that only those with direct ancestral ties can have it. This choice
introduces a restriction not present in the passage.

D: This is directly contradicted by the text which describes Ta Moko as having "deep
roots dating back centuries in New Zealand." It's portrayed as a longstanding tradition,
not a new trend.

2. B

The text discusses how town criers, while no longer primary news bearers due to
modern technology, still hold value in contemporary society. They have not vanished;
rather, they have taken on ceremonial roles in certain social contexts like festivals and
parades. This is best captured by option B, which states that despite modern
communication methods, town criers are still valued for their traditional role.

A) This option is incorrect because the text specifies that town criers haven't entirely
disappeared; they still have a role in specific contexts like festivals and parades.

www.strategictestprep.com ♟️ 1-833-300-PREP (7737) ♟️[email protected] 109


C) This option is incorrect because the text does not discuss the preferences of town
criers. It focuses on how society values them, not how they feel about their roles.

D) This option is incorrect because the text does not mention town criers using social
media to share news. Instead, it highlights how they have adapted to still be valued in
specific settings despite technological advancements in news dissemination.

Therefore, option B is the most accurate in summarizing the main idea of the text.

3. B

The main point of the text is that Dr. Emily Johnson and her team have indirectly
detected the presence of dark matter, which supports its theoretical existence. This
corresponds most closely with choice B.

Choice A is incorrect because the text does not say that the new evidence challenges
the fundamental principles of physics. Instead, it supports the existence of dark matter.

Choice C is incorrect because, while a specialized telescope was used in the study, the
focus of the text is on the indirect detection of dark matter, not on the technology used.

Choice D is incorrect because the text indicates that the team's findings support the
concept of dark matter, rather than debunking it.

4. C

The main point of the text is that the co-living building designed by Sarah Zhang and
Tim Lee seems to be effective in fostering social interactions among its residents. This
is evidenced by tenant Mary Smith, who reported that her social life improved and she
felt a sense of community after living there for a year.

Choice A is incorrect because the text does not focus on Mary Smith's past social
struggles. Rather, it emphasizes the potential social benefits of living in the building.

www.strategictestprep.com ♟️ 1-833-300-PREP (7737) ♟️[email protected] 110


Choice B is incorrect because there is no mention of the building's features being
environmentally friendly. The text focuses on the building's success in fostering social
interactions.

Choice D is incorrect because the text does not claim that shared garden spaces are
the most crucial feature of the building. Instead, it mentions multiple features like
communal kitchens and shared garden spaces designed to foster social interactions.

5. B

The text emphasizes how Jo feels a "greater thrill" and an "invigorating sense of
purpose" from knowing others are reading her work. The main idea revolves around
the emotional and psychological impact of having her work published, making option B
the most accurate answer.

Choice A is incorrect because, although she does receive money, the text specifies that
the "greater thrill" comes from her work being read, not from the money itself.

Choice C is incorrect because the text does not mention anything about Jo being
excited about becoming a famous writer; her joy comes from the fact that her work is
being read.

Choice D is incorrect because, although contributing to the family's expenses is


mentioned, it is not the focal point of her excitement or sense of purpose.

6. C
Dumbledore's statement about "the happiest man on earth" using the Mirror of Erised
"like a normal mirror" and seeing himself "exactly as he is" serves as a nuanced lesson
about the nature of true happiness. He suggests that a truly happy person would be
content with their current circumstances and therefore wouldn't see any fantastical
desires in the mirror. The underlying message is that happiness comes from

www.strategictestprep.com ♟️ 1-833-300-PREP (7737) ♟️[email protected] 111


self-contentment and not from the perpetual quest for what one lacks, which is what
the mirror otherwise reveals.
Choice A is incorrect because Dumbledore is highlighting the unique properties of the
Mirror of Erised, contrasting it with a "normal mirror."
Choice B is incorrect as Dumbledore doesn't make any claims about Harry's current
level of happiness.

Choice D is incorrect because the mirror is portrayed more as a cautionary device


revealing what might be missing in one's life, rather than a tool for self-improvement.

Therefore, option C most accurately captures the main idea that Dumbledore is
conveying to Harry.

7. D

Throughout the passage, Mrs. Penniman is described as someone unserious and


unconcerned with everyday matters: she has a taste for light literature and a passion
for little secrets and mysteries, and is unpractical as addled eggs. These phrases
convey an image of someone who is "romantic and sentimental," making the answer
(D). (A) and (C) are both entirely unsupported by the passage - there is nothing to
suggest that Mrs. Penniman is either mysterious or difficult to get along with. (B) is
incorrect because the passage states only that Mrs. Penniman has a taste (a liking) for
light literature, which is much less extreme than a "passion".

8. C

The main focus of the passage is on the difficulty of eradicating (getting rid of)
knotweed: the first sentence states that it is intractable (immovable), and the second
indicates that the only way to remove it completely is to extract the land it sits on. That
corresponds directly to (C). (A) is incorrect because the passage does not state that
knotweed was one of the earliest to plants to travel. (B) is directly contradicted by the
passage, which states that knotweed can be removed when the land it sits on is
extracted. (D) is entirely off-topic: the figure (0.3g) mentioned in the last sentence
serves only to emphasize the minuscule amount of knotweed capable of regenerating
a new plant.

9. B

www.strategictestprep.com ♟️ 1-833-300-PREP (7737) ♟️[email protected] 112


The lines Since childhood's days [Tennyson's] name I have revered/And more and more
it has become to me endeared, and I very wealthyfeel since I his "works" possess
indicate that Heard is extremely proud to have received Tennyson's poems and has an
extremely positive attitude toward the poet. (C) is negative and can be eliminated. (A)
misstates the essential situation: Heard's friend does not personally know Tennyson;
as indicated in the title in the blurb before the passage, she has merely given a book of
his poems as a gift. (D) is incorrect because Heard refers to her humble thoughts,
indicating that she does not consider herself Tennyson's equal. Only (B) correctly
characterizes Heard's attitude of "respect" and desire to "understand [Tennyson's]
work deeply."

10.A

The key information appears in the second sentence and last sentence, where pawn is
described as a fully integrated part of Navajo life and a pivotal aspect of Navajo
society. In other words, it "plays a significant role in Navajo culture," making (A) correct.
(B) is entirely unsupported by the passage. (C) and (D) are incorrect because the
passage states only that pawn is different from traditional banking and is based on
relationships of mutual trust-it cannot be inferred that pawn is "more complex than
traditional banking" or that it "became popular...because it was based on personal
relationships."

11. D

Don't get too caught up in the seeming complexity of the answer choices. In reality, the
structure of the passage is fairly straightforward. The first sentence presents a
claim-Octavia Butler resisted being identified exclusively with science fiction novels
(i.e., that form) - and the rest of the passage serves to support it with specific examples
(she also wrote essays and had one of her books turned into an opera). The only
answer that corresponds to that organization is (D). The passage does not focus on
one specific work or novel, eliminating (A) and (B). (C) does not fit either because the
passage does not discuss the significance of either of the works mentioned.

www.strategictestprep.com ♟️ 1-833-300-PREP (7737) ♟️[email protected] 113


ii. According to the Text 📖
According to the Text questions are very straightforward. You are simply finding
specific information in the text. 🕵️‍♀️
Strategies

Strategy #1: Read the text and sum up each sentence in your own words as you’re
reading (paraphrase).

Strategy #2: Try to answer the question first before looking at the answer choices.

Strategy #3: Use process of elimination to narrow down your answer choices. Look for
answers that say the opposite of what you need or are irrelevant.

✨How can you make the most out of this workbook? ✨


👉For walkthrough videos, strategies, quizzes, and extra resources that support
each lesson in this workbook, please sign up for our English Self-Paced Course!

👉For individualized support, please check out our Information Pack to learn
more about our private tutoring options and Request a Consultation!

www.strategictestprep.com ♟️ 1-833-300-PREP (7737) ♟️[email protected] 114


Practice Questions

1. Dogs have the ability to detect subtle changes in human body odor, which may
signify illness or disease. Dr. Martha Jenkins and her team found this by exposing dogs
to different samples of human sweat and observing their reactions. The dogs
consistently sniffed longer and more intently at samples that came from individuals
with specific health conditions. To further confirm their observations, the team
conducted a controlled test where dogs were presented with sweat samples from both
healthy individuals and those with a particular ailment.

According to the text, how did Dr. Jenkins and her team ascertain the dogs' ability to
detect changes in human body odor?

A) They measured the amount of time each dog spent sniffing each sample.

B) They tracked the dogs' preferences for different individuals.

C) They noted the specific health conditions of the individuals whose samples were
presented.

D) They observed the dogs' behavior when introduced to new environments.

2. From Charles Dickens' 1859 novel A Tale of Two Cities, Lucie Manette is a character
of great compassion and resilience. Lucie, after being reunited with her father who had
been imprisoned for 18 years, becomes his pillar of strength. Despite the traumatic
separation in their early years, Lucie's undying love and patience aid in mending her
father's broken spirit. Her presence in the house brought an aura of light, healing not
just her father but everyone around her. Even in the most trying times, Lucie remained
hopeful, always believing in the goodness of people and the possibility of redemption.

Question: According to the text, what is true about Lucie Manette?

A) Lucie holds resentment for the years she lost with her father.

B) Lucie is often pessimistic about the circumstances around her.

www.strategictestprep.com ♟️ 1-833-300-PREP (7737) ♟️[email protected] 115


C) Lucie's kindness impacts only her father and no one else.

D) Lucie embodies optimism and has a positive influence on those around her.

3. The Amazon rainforest is often referred to as the "lungs of the Earth" because it
produces a significant portion of the world's oxygen. However, scientists are concerned
as large swathes of the forest are being cleared for agricultural purposes. They argue
that reforestation initiatives can help restore the balance and ensure the forest
continues its crucial role in the Earth's ecosystem.

According to the text, why are scientists concerned about the Amazon rainforest?

A) The forest is producing less oxygen than before.

B) The forest is becoming a hotspot for agricultural research.

C) New species of trees are overtaking the native ones.

D) The forest is expanding too quickly, leading to imbalance.

4. For decades, the only confirmed sightings of the elusive snow leopard in the
Himalayas were limited to the northern region of Nepal. This led many to believe that
the species was primarily confined to that specific territory. However, in a
groundbreaking revelation, biologist Dr. Anaya Verma and her team discovered
undeniable evidence of snow leopard habitats in the southern Himalayan region,
dispelling prior assumptions and underscoring the broader range of these mysterious
creatures.

Question:

Based on the text, why is Dr. Anaya Verma's finding about the snow leopard in the
southern Himalayan region deemed remarkable?

A) The finding indicates that snow leopards are more populous in the southern region
than the northern region of Nepal.

www.strategictestprep.com ♟️ 1-833-300-PREP (7737) ♟️[email protected] 116


B) The discovery suggests that the snow leopard's diet might be different in the
southern Himalayan region compared to the northern region.

C) The discovery demonstrates a wider habitat range for the snow leopard than was
previously assumed.

D) The evidence found indicates that snow leopards are moving from the northern to
the southern Himalayas due to climate change.

5. In 2022, Dr. Elena Garcia and her team at the Oceanic Research Institute began
developing a submarine capable of exploring the depths of the Mariana Trench, the
deepest part of the world's oceans. Traditional submarine designs couldn't withstand
the extreme pressures found at these depths. After extensive research, Garcia's team
discovered that by using a combination of specialized alloys and a unique hull shape,
they could create a submarine that could resist the crushing pressures and explore
deeper than ever before. Their submarine, the DeepQuest, made a successful journey
to the bottom of the trench, capturing unprecedented footage of the ocean floor.

According to the text, why were traditional submarine designs unsuitable for exploring
the Mariana Trench?

A) Because they lacked the technology to capture clear footage at those depths.

B) Because they could not withstand the extreme pressures found in the trench.

C) Because the unique hull shape of the DeepQuest is patented and could not be used
by other designs.

D) Because traditional designs were too large to navigate the narrow spaces of the
trench.

6. The following text is adapted from Charlotte Brontë's 1847 novel "Jane Eyre." Jane
is reflecting on her time spent at Thornfield Hall with Mr. Rochester.

www.strategictestprep.com ♟️ 1-833-300-PREP (7737) ♟️[email protected] 117


Jane gazed out of the window, her thoughts consumed by the memories of Thornfield.
She remembered the many times Mr. Rochester had teased her, the fiery spark in his
eyes, and the passion they shared. Each time she played their conversations back in her
mind, she felt a deep yearning and a sorrow for times long past. The warmth of the
fireplace, the dim candlelight in the evening, and the sound of Rochester’s voice were
all etched deeply in her heart. To Jane, Thornfield was not just a place; it was a chapter
of emotions, lessons, and profound connections.

According to the text, how does Jane feel about her memories of Thornfield?

A) She is indifferent and detached from her past.

B) She cherishes the moments and feels a deep connection.

C) She regrets the decisions she made while at Thornfield.

D) She is resentful of Mr. Rochester for teasing her.

7. The following text is adapted from J.M. Barrie's 1911 novel "Peter and Wendy"
(commonly known as "Peter Pan").

Peter Pan, the boy who refused to grow up, often visited the Darling children in their
nursery. He shared tales of his adventures in the magical Neverland, a place with
pirates, fairies, and mermaids. Wendy, the eldest of the Darling children, was
particularly taken with Peter's tales and often dreamt of flying away with him. Peter,
with the help of his fairy Tinker Bell, taught the children to fly and took them to
Neverland for the grandest of adventures.

According to the text, why did Wendy want to go to Neverland?

A) She wanted to meet the fairies and mermaids.

B) She was particularly enchanted by Peter's tales.

C) She wished to escape from her nursery.

D) Peter forced her to go against her will.

www.strategictestprep.com ♟️ 1-833-300-PREP (7737) ♟️[email protected] 118


Answers - Extra Practice

1. Answer: A) They measured the amount of time each dog spent sniffing each
sample.

Explanation:

A) They measured the amount of time each dog spent sniffing each sample - The
text specifically mentions that "the dogs consistently sniffed longer and more
intently at samples that came from individuals with specific health conditions."
This indicates that the main method of observation was based on the duration
and intensity of the dogs' sniffing, which aligns with option A.

B) They tracked the dogs' preferences for different individuals. - This option is incorrect
because the focus of the study was on dogs' reactions to sweat samples, not their
preferences for specific individuals.

C) They noted the specific health conditions of the individuals whose samples were
presented. - This option is misleading. While the text mentions that dogs sniffed
samples from individuals with specific health conditions, it does not state that the
researchers noted each specific condition as a primary method to ascertain the dogs'
ability.

D) They observed the dogs' behavior when introduced to new environments. - This
option is incorrect because the text does not mention any introduction of dogs to new
environments. The study's context revolves around dogs' reactions to sweat samples.

2. Answer: D) Lucie embodies optimism and has a positive influence on those


around her.

Explanation:

www.strategictestprep.com ♟️ 1-833-300-PREP (7737) ♟️[email protected] 119


A) Lucie holds resentment for the years she lost with her father. - This is not supported
by the text. The passage describes Lucie's love and patience in helping mend her
father's spirit, suggesting a deep bond without any resentment.

B) Lucie is often pessimistic about the circumstances around her. - This choice
contradicts the text, which portrays Lucie as hopeful and believing in the "goodness of
people."

C) Lucie's kindness impacts only her father and no one else. - This choice is disproven
by the statement that Lucie's presence healed "not just her father but everyone around
her."

D) Lucie embodies optimism and has a positive influence on those around her. - The
passage highlights Lucie's positive influence, describing her as someone who brings
"an aura of light" and heals those around her. It also mentions her unwavering hope
and belief in the "goodness of people and the possibility of redemption."

3. Answer: A) The forest is producing less oxygen than before.

Explanation:

A) The forest is producing less oxygen than before. - The text states that the Amazon
rainforest produces a significant portion of the world's oxygen. However, large parts of
the forest are being cleared for agriculture, implying that this deforestation would
result in less oxygen production. Thus, option A captures this implication.

B) The forest is becoming a hotspot for agricultural research. - The text mentions that
the forest is being cleared for agricultural purposes, but it doesn't suggest any research
activity.

C) New species of trees are overtaking the native ones. - The passage does not
mention anything about new species of trees or their impact on the native ones.

D) The forest is expanding too quickly, leading to imbalance. - The text actually
discusses the opposite concern: the forest is being cleared, not expanding.

www.strategictestprep.com ♟️ 1-833-300-PREP (7737) ♟️[email protected] 120


4. Answer: C) The discovery demonstrates a wider habitat range for the snow
leopard than was previously assumed.

Explanation:

A) The finding indicates that snow leopards are more populous in the southern region
than the northern region of Nepal. - The text doesn't compare the populations of snow
leopards in the two regions. It only discusses the discovery of their habitat in the
southern region.

B) The discovery suggests that the snow leopard's diet might be different in the
southern Himalayan region compared to the northern region. - The text doesn't
mention anything about the diet of snow leopards. The emphasis is on the location of
their habitat.

C) The discovery demonstrates a wider habitat range for the snow leopard than was
previously assumed. - The text states that earlier, many believed the snow leopard's
habitat was primarily confined to the northern region of Nepal. Dr. Verma's finding in
the southern Himalayan region dispels this notion, indicating a broader range for the
species. Hence, option C correctly encapsulates the significance of the discovery.

D) The evidence found indicates that snow leopards are moving from the northern to
the southern Himalayas due to climate change. - The text doesn't mention any
migration or reasons such as climate change. The discovery simply emphasizes the
broader range of the snow leopard's habitat.

5. Answer: B) Because they could not withstand the extreme pressures found in
the trench.

Explanation:

www.strategictestprep.com ♟️ 1-833-300-PREP (7737) ♟️[email protected] 121


A) Because they lacked the technology to capture clear footage at those depths. - The
text doesn't mention any issues with capturing footage. The main concern was the
pressure at the trench's depths.

B) Because they could not withstand the extreme pressures found in the trench. - The
text explicitly states that "Traditional submarine designs couldn't withstand the
extreme pressures found at these depths," pointing directly to the reason traditional
submarines were unsuitable for exploring the Mariana Trench. Hence, option B is the
correct answer.

C) Because the unique hull shape of the DeepQuest is patented and could not be used
by other designs. - While the text mentions a unique hull shape for the DeepQuest,
there is no mention of patents or the inability of other designs to use it.

D) Because traditional designs were too large to navigate the narrow spaces of the
trench. - Size or navigation isn't highlighted as a concern in the text. The main
challenge was related to the extreme pressures in the trench.

6. Answer: B) She cherishes the moments and feels a deep connection.

Explanation:

A) She is indifferent and detached from her past. – The text clearly shows Jane's deep
emotional connection to her past at Thornfield, especially with phrases like "deep
yearning" and "etched deeply in her heart."

B) She cherishes the moments and feels a deep connection. - The text mentions that
Jane felt a "deep yearning and a sorrow for times long past" and that the memories of
Thornfield and Mr. Rochester were "etched deeply in her heart." These phrases
indicate that Jane has a profound and cherished connection to her time at Thornfield.

www.strategictestprep.com ♟️ 1-833-300-PREP (7737) ♟️[email protected] 122


C) She regrets the decisions she made while at Thornfield. - The passage does not give
any indication of regret regarding Jane's decisions at Thornfield. Instead, it emphasizes
her emotional attachment to the memories.

D) She is resentful of Mr. Rochester for teasing her. - Although the text mentions Mr.
Rochester teasing Jane, it does not indicate that she harbors any resentment. The tone
of the passage suggests fond memories rather than negative ones.

7. Answer: B) She was particularly enchanted by Peter's tales.

Explanation:

A) She wanted to meet the fairies and mermaids. - While the passage mentions fairies
and mermaids as inhabitants of Neverland, it doesn't specifically say that Wendy
wanted to meet them.

B) She was particularly enchanted by Peter's tales. - The passage states that "Wendy,
the eldest of the Darling children, was particularly taken with Peter's tales," which
directly supports the claim that she was enchanted by Peter's tales of Neverland.

C) She wished to escape from her nursery. - The text does not provide any information
suggesting that Wendy wanted to leave her nursery due to any dislike or desire to
escape.

D) Peter forced her to go against her will. - There's no indication in the passage that
Peter forced Wendy or her siblings to go to Neverland. In fact, the context suggests
that it was a voluntary adventure.

www.strategictestprep.com ♟️ 1-833-300-PREP (7737) ♟️[email protected] 123


iii. Statement that Best Supports 👍 or Undermines👎 a Claim
Strategies

Strategy #1: Locate and Restate the Claim in Your Own Words
Find the argument in the passage. Restating the claim will ensure that you understand
what they are trying to argue. You should look for a keyword that means “claim” - that
is the sentence you want!

Strategy #2: Use Process of Elimination to Narrow it Down

Some answer choices will be irrelevant to the argument. One answer will likely be the
opposite of the argument.

Strategy #3: Find synonyms in the Quote that Support the Claim.

The quote should say the same thing as the claim, but in a different way.

Exercises

Exercise 1a: List as many synonyms to the word “claim” that you can think of. Check
your answers with those on the following page.

1.___________________ 6. ___________________ 11. __________________

2. ___________________ 7. ___________________ 12. __________________

3. ___________________ 8. ___________________ 13. __________________

4. ___________________ 9. ___________________ 14. __________________

5. ___________________ 10. __________________ 15. __________________

www.strategictestprep.com ♟️ 1-833-300-PREP (7737) ♟️[email protected] 124


Answer Key for Exercise 1a

​ Assert ​ Avow ​ Postulate

​ Declare ​ Contend ​ Pronounce

​ State ​ Profess ​ Purport

​ Maintain ​ Attest ​ Aver

​ Affirm ​ Proclaim ​ Swear

​ Allege ​ Argue ​ Suggest

​ Insist ​ Hold ​ Propose

Exercise 1b: Sometimes the question will ask for a statement that undermines the
argument instead, which means you will have to pick an answer opposite to the
argument so pay close attention to the question! List all synonyms you can think of for
the word “undermines” that you could see in the question.

1.___________________ 5. ___________________ 9. ___________________

2. ___________________ 6. ___________________ 10. __________________

3. ___________________ 7. ___________________ 11. __________________

4. ___________________ 8. ___________________ 12. __________________

www.strategictestprep.com ♟️ 1-833-300-PREP (7737) ♟️[email protected] 125


Answer Key for Exercise 1b

​ Sabotage ​ Devalue ​ Undercut

​ Weaken ​ Debase ​ Thwart

​ Subvert ​ Compromise ​ Diminish

​ Erode ​ Invalidate ​ Jeopardize

​ ​ ​

Exercise 2: Locate the claim in the text and underline it. Then, restate it in your own
words.

Example 1

One argument that is commonly cited to support the idea that birds lack a sense of
smell is that some birds' olfactory bulbs are relatively small. As a result, many
scientists concluded that these creatures gave up smell in favor of improved eyesight.
This notion became so pervasive that it once was repeated to avian expert Danielle
Whittaker as fact by a prominent neurobiologist.

Restate Claim:

www.strategictestprep.com ♟️ 1-833-300-PREP (7737) ♟️[email protected] 126


Example 2

As a person sleeps, the motor cortex—the part of the brain that controls
movements—replays skills that it learned during the day. In a recent experiment run by
the BrainGate consortium, researchers observed that the pattern of firing neurons sped
up during sleep, echoing findings from previous animal studies. The scientists were
also interested to note that replay took place not during REM sleep, which is when
people normally dream, but during deep, slow-wave sleep, which occurs in the first
three or four hours of the night. They theorize that replay during this period helps the
brain consolidate new information, moving it from short-term to long-term memory.

Restate Claim:

Example 3

Dinosaurs, with the exception of the ancestors of birds, disappeared during a mass
extinction 65 million years ago when an asteroid struck the Earth. Because a high
metabolic rate has generally been suggested as one of the key advantages when it
comes to surviving mass extinctions, some genetic paleontologists have proposed that
birds survived while non-avian dinosaurs did not because of the birds' increased
metabolic capacity.

Restate Claim:

www.strategictestprep.com ♟️ 1-833-300-PREP (7737) ♟️[email protected] 127


Answers to Exercise 2
Exercise 1 Claim:
As a result, many scientists concluded that these creatures gave up smell in favor of
improved eyesight.

When you sum this up in your own words, you must understand the pronoun “these
creatures.” Who are they talking about?

Summary: Since birds lack a sense of smell, they traded it for improved eyesight
instead.

Exercise 2 Claim:
They theorize that replay during this period helps the brain consolidate new
information, moving it from short-term to long-term memory.

When you sum this up in your own words, you must understand the pronoun “they” as
well as “this period.” Who are they talking about?

Summary: Scientists think that deep, slow-wave sleep in the first 3-4 hours of the
night helps the brain consolidate short-term into long-term memory.

Exercise 3 Claim:
Because a high metabolic rate has generally been suggested as one of the key
advantages when it comes to surviving mass extinctions, some genetic paleontologists
have proposed that birds survived while non-avian dinosaurs did not because of the
birds' increased metabolic capacity.

Summary: Since birds have a greater metabolism than dinosaurs, scientists


hypothesize that birds survived mass extinctions.

www.strategictestprep.com ♟️ 1-833-300-PREP (7737) ♟️[email protected] 128


Exercise 3: Use process of elimination to get the correct answer. Then, categorize the
answer choices as irrelevant, opposite, or correct.

Example 1

As a person sleeps, the motor cortex—the part of the brain that controls
movements—replays skills that it learned during the day. In a recent experiment run by
the BrainGate consortium, researchers observed that the pattern of firing neurons sped
up during sleep, echoing findings from previous animal studies. The scientists were
also interested to note that replay took place not during REM sleep, which is when
people normally dream, but during deep, slow-wave sleep, which occurs in the first
three or four hours of the night. They theorize that replay during this period helps the
brain consolidate new information, moving it from short-term to long-term memory.

Which finding, if true, would most strongly support the scientists' theory?

A) Athletes who slept for at least seven hours before a game competed at a higher
level than ones who slept for less time.

B) People whose neuron firings increased in speed during REM sleep retained new
information over a longer period of time than people whose neurons fired more
quickly early in their sleep cycle.

C) Students who slept for only a few hours before an exam showed no difference
in performance when compared to students who slept for a full night.

D) Dancers who slept for several hours shortly after learning a new routine knew it
better two weeks later than a group that stayed awake.

Irrelevant Answers:

Opposite Answers:

Correct Answers:

www.strategictestprep.com ♟️ 1-833-300-PREP (7737) ♟️[email protected] 129


Example 2 (An Undermine Example)

Dinosaurs, with the exception of the ancestors of birds, disappeared during a mass
extinction 65 million years ago when an asteroid struck the Earth. Because a high
metabolic rate has generally been suggested as one of the key advantages when it
comes to surviving mass extinctions, some genetic paleontologists have proposed that
birds survived while non-avian dinosaurs did not because of the birds' increased
metabolic capacity.

Which finding, if true, would undermine the genetic paleontologists' proposal?

A) The metabolisms of non-avian dinosaurs may have decreased over time.

B) Because some non-avian dinosaurs moved quickly, they likely had high
metabolic rates.

C) Birds have some of the highest metabolism rates of any creatures on Earth.

D) Many dinosaurs with very high metabolic rates went extinct 65 million years
ago.

Irrelevant Answers:

Opposite Answers:

Correct Answers:

www.strategictestprep.com ♟️ 1-833-300-PREP (7737) ♟️[email protected] 130


Example 3 (An Undermine Example)

One argument that is commonly cited to support the idea that birds lack a sense of
smell is that some birds' olfactory bulbs are relatively small. As a result, many
scientists concluded that these creatures gave up smell in favor of improved eyesight.
This notion became so pervasive that it once was repeated to avian expert Danielle
Whittaker as fact by a prominent neurobiologist.

Which finding, if true, would most undermine the neurobiologist's statement?

A) An analysis of avian genomes revealed the presence of proteins that bind to


odors and relay a signal to the brain.

B) The number of olfactory neurons is much smaller in birds than in most other
animals.

C) The odors that birds depend on for food and social relationships are quickly
dispersed in the wind.

D) Birds sometimes overlook strongly scented prey in favor of animals that are
more visually striking.

Irrelevant Answers:

Opposite Answers:

Correct Answers:

Answers to Exercise 3

www.strategictestprep.com ♟️ 1-833-300-PREP (7737) ♟️[email protected] 131


Example 1
Irrelevant Answers: A
Opposite Answers: B, C
Correct Answers: D

(A) No. This answer only involves an amount of sleep; it says nothing about
consolidating new skills.
(B) No, but be careful. This choice refers to "retaining new information" but incorrectly
ties it to REM sleep. The passage, in contrast, discusses a phenomenon that does NOT
occur during REM sleep.
(C) No. The passage discusses an effect that improves learning - if two groups
performed identically, that would not be the case.
(D) Yes, this fits. The passage discusses a phenomenon that happens early in the sleep
cycle, which means that it would occur in people who only slept for several hours. If
dancers who slept that amount after learning a new routine knew it better after two
weeks (i.e., longer-term) than dancers that did not sleep at all, that supports the idea
that replay early in sleep consolidates learning.

Example 2
Irrelevant Answers: A, B
Opposite Answers (support claim): C
Correct Answers (undermines claim): D

If we hold very tight to that idea and don't allow ourselves to become confused by any
of the other options, we can identify (D) as the sole choice consistent with that idea. By
definition, the extinction of creatures with high metabolic rates contradicts the claim
that having a high metabolic rate prevents animals from going extinct.

Example 3
Irrelevant Answers: none of the above
Opposite Answers (support claim): B, C, D
Correct Answers (undermines claim): A

What did the neurobiologist say? That these creatures (i.e., birds) gave up smell in
favor of improved eyesight. This is an "undermine” question, so the correct answer
must contradict that idea. Most logically, it will indicate that birds have both good
eyesight and a sense of smell. The relationship between (A) and the claim in question
www.strategictestprep.com ♟️ 1-833-300-PREP (7737) ♟️[email protected] 132
may not be immediately obvious, but consider it carefully: if avian (bird) genomes
suggest that birds are physiologically capable of perceiving odors, that would suggest
they do in fact have a sense of smell. So keep (A). The presence of an unusually small
number of olfactory neurons in birds would support rather than undermine the claim in
question, so (B) can be eliminated. (C) and (D) have the same problem. In (C), if birds
cannot rely on odors to find food, that would support the idea that they did not evolve
a sense of smell. And in (D), ignoring strong scents in favor of visual information
suggests a poor sense of smell.

LASTLY…..

Sometimes you may have to choose a quote that illustrates the claim. The claim is
easy to find in the text.

Exercise 4: Answer the following question using Strategy #3.

In "The Sublime Object of Ideology," a 1989 work by philosopher Slavoj Žižek, the
author delves into the intricacies of ideology, arguing that people often unconsciously
submit to dominant ideological narratives, even if they believe they are resisting them.

Which quotation from "The Sublime Object of Ideology" most directly illustrates
this claim?

A) "Ideology is not a dreamlike illusion that we build to escape insufferable reality; in


its basic dimension, it is a fantasy-construction which serves as a support for our
'reality' itself."

B) "The true act of discovery consists not in finding new lands but in seeing with new
eyes."

C) "Every ideology, without exception, serves the existing order by providing a


coherent, all-encompassing image of the world."

D) "The hidden presuppositions of our acts, more than our explicit reasoning, guide us
in our interactions with reality."

www.strategictestprep.com ♟️ 1-833-300-PREP (7737) ♟️[email protected] 133


Answer to Exercise 4

A) This quotation directly exemplifies Žižek's assertion that ideology is not just an
escape from reality but rather forms the very foundation of what we perceive as
reality. It underscores the idea that people might unconsciously adhere to
prevailing ideologies, even when they believe they are resisting them.

Incorrect Answer Explanations:

B) While this statement speaks to the notion of rediscovery or reevaluation, it does not
specifically address the topic of how individuals unconsciously submit to dominant
ideological narratives.

C) This statement addresses the universal characteristic of ideologies in general but


does not delve into the idea of unconscious submission to those ideologies by
individuals.

D) While this quotation touches upon the idea of hidden influences on our actions, it
doesn't make a direct link between unconscious submission and prevailing ideologies.
Please note that presuppositions do not mean ideologies.

www.strategictestprep.com ♟️ 1-833-300-PREP (7737) ♟️[email protected] 134


Extra Practice

1. Born in 1905 in Kyoto, Japan, Soseki Kuroyama was one of the pioneering
figures in modern Japanese literature. In a seminar discussion, a student argues
that Kuroyama's novels highlight the conflicts and tensions between traditional
Japanese values and the encroaching influence of Western culture.

Which evidence, if true, would most directly support the student's claim?

A) Many of Kuroyama's novels are set in urban environments where characters grapple
with their sense of identity amidst a rapidly modernizing Japan.

B) Kuroyama was deeply influenced by his travels to Europe and often used European
literary techniques in his writing.

C) Throughout his career, Kuroyama won numerous national and international awards
for his contributions to the literary world.

D) Kuroyama was known to have a personal collection of classical Japanese paintings,


highlighting his appreciation for traditional art forms.

2. In the arid landscapes of Arizona, the Agave murpheyi plant has evolved to
thrive. Botanists Rachel Turner and Hugo Mendez studied the Agave's
mechanisms for surviving long periods without water. Their research revealed
that the Agave's leaves store water in a gel-like substance, which is then slowly
released when the plant requires hydration. The botanists further postulate that
this gel acts as a protective barrier against evaporation, thereby conserving the
plant's water reserves during prolonged dry spells.

Which finding, if true, would most directly support the botanists' hypothesis?

A) Other desert plants, such as cacti, also store water in their tissues but do not
produce a gel-like substance similar to the Agave murpheyi.

B) Agave murpheyi plants that are exposed to regular water sources do not produce as
much of the gel-like substance in their leaves as those in arid conditions.

www.strategictestprep.com ♟️ 1-833-300-PREP (7737) ♟️[email protected] 135


C) The gel-like substance found in Agave murpheyi leaves is also observed in Agave
species native to more humid environments.

D) When the gel-like substance from Agave murpheyi is extracted and exposed to the
sun, it evaporates at a slower rate than pure water.

3. Dr. Amelia Rosen and her team of neuroscientists suggest that listening to
calming music can reduce stress levels and enhance cognitive performance. In a
study conducted by Rosen's team, participants were randomly divided into two
groups: one listened to instrumental classical music for ten minutes, while the
other group sat in silence for the same duration. Immediately after this, both
groups were given a series of cognitive tests, which evaluated memory,
attention, and problem-solving skills.

Which finding from Rosen's study, if true, would most strongly support her team's
claim?

A) Participants who listened to classical music had significantly better scores on the
cognitive tests than those who sat in silence.

B) The majority of participants in both groups claimed to have prior experience with
listening to classical music for relaxation.

C) Participants who sat in silence reported feeling more relaxed than those who
listened to the classical music.

D) Most participants who listened to classical music could name the pieces and
composers that were played during their session.

4. Many dietitians believe that personalized nutrition, based on an individual's


genetics, microbiome, and lifestyle factors, is the future of healthy eating. In a
recent experiment, a team of researchers developed a machine-learning
algorithm trained on thousands of personal dietary data points. The goal was to
predict optimal diets for individuals based on their unique factors. While the

www.strategictestprep.com ♟️ 1-833-300-PREP (7737) ♟️[email protected] 136


algorithm demonstrated a high degree of accuracy in its recommendations,
many dietitians assert that they won't be replaced by such technology.

Which finding, if true, would most directly support the dietitians' claim?

A) The algorithm's dietary recommendations were produced in seconds, far quicker


than any dietitian could create a customized diet plan.

B) Many participants in the study reported that they lacked the motivation to stick to
the algorithm's dietary recommendations without the personal support and
encouragement from a dietitian.

C) The machine-learning algorithm often suggested uncommon or exotic foods that


were difficult for participants to find in their local stores.

D) A significant percentage of dietitians surveyed were not familiar with the latest
advancements in the field of personalized nutrition.

5. Marine biologist Dr. Helena Ross and her team studied the migration patterns of
humpback whales in the South Pacific Ocean. By using tracking devices and
satellite imagery, the team observed that the humpback whales spent a
significant amount of time in areas rich in krill, their primary food source.
Interestingly, these krill-rich zones were located near underwater volcanic
ridges. The team hypothesized that underwater volcanic activity might play a
role in attracting or producing abundant krill populations, which in turn attracts
humpback whales.

Which finding, if true, would most strongly support the team's hypothesis?

A) Satellite imagery showed that humpback whales also frequented areas in the ocean
that were not particularly rich in krill.

B) Krill samples collected near the underwater volcanic ridges showed higher nutrient
content than those collected from other areas.

C) Humpback whales were observed to migrate to different parts of the South Pacific
Ocean during different seasons.

www.strategictestprep.com ♟️ 1-833-300-PREP (7737) ♟️[email protected] 137


D) Other marine animals, like dolphins and seals, were also observed near the volcanic
ridges, but they were not feeding on krill.

6. Megalodon sharks were massive marine predators that existed roughly 23 to


3.6 million years ago. Paleontologist Dr. Rebecca Stone, along with colleagues
Dr. Maxwell Jenkins and Dr. Clara Webb, studied the wear patterns on
Megalodon teeth to infer their diet. Their findings suggest that Megalodons
primarily fed on large marine mammals rather than bony fishes. Stone, Jenkins,
and Webb argue that this specialized diet made Megalodons more vulnerable to
changes in marine mammal populations, which may have contributed to their
eventual extinction.

Which finding, if true, would most directly support Stone, Jenkins, and Webb’s
claim?

A) Studies on other marine predators from the same era show different wear patterns
on their teeth, indicating a varied diet that included both marine mammals and bony
fishes.

B) Megalodon fossils, when found in locations known for large marine mammal
populations, often show less wear and damage on their teeth than those found in
fish-abundant regions.

C) A sharp decline in the diversity and population of large marine mammals is recorded
in the fossil record around the same time as the disappearance of Megalodons.

D) Megalodon teeth were significantly larger and sharper than those of other marine
predators, suggesting a unique adaptation for tearing apart large prey.

7. Biologists believe that a sharp decline in the population of sunbirds in a


particular tropical forest is directly related to a decrease in the number of
blooming native flowers. Sunbirds primarily feed on nectar and play a vital role
in pollinating these flowers. However, they also feed on a specific type of insect
that harms these flowers.

www.strategictestprep.com ♟️ 1-833-300-PREP (7737) ♟️[email protected] 138


Which finding, if true, would most directly support the biologists' hypothesis?

A) The specific insect that harms the native flowers and is eaten by sunbirds has seen
a significant population increase in the absence of sunbirds.

B) Sunbirds have shown an adaptive behavior of feeding on a different type of insect in


nearby forests where the specific flower-harming insect is scarce.

C) Some of the native flowers have evolved to produce less nectar, making them less
attractive to sunbirds.

D) Other pollinators, such as bees and butterflies, have also seen a decline in numbers
in the same tropical forest.

8. In the late 1990s, acclaimed musicologist Elena Rodriguez was disturbed by the
absence of Latin American composers in mainstream classical music programs
in Europe. In her interviews, Rodriguez emphasized that her mission was not
only to highlight Latin American composers but to demonstrate their significant
contributions to the evolution of global classical music.

Which finding, if true, would most directly support Rodriguez's mission?

A) Rodriguez's comprehensive study, "Sonic Tides: Latin America's Symphony," became


the definitive text for understanding the nuances of classical music from the continent,
focusing on the works of composers such as Arturo Márquez and Heitor Villa-Lobos.

B) In the concert series "Classical Fusion: Worlds Collide," Rodriguez curated


performances that juxtaposed pieces by Latin American composers like Alberto
Ginastera alongside European greats like Ludwig van Beethoven, revealing their
interconnected musical themes.

C) Elena Rodriguez was instrumental in establishing the Latin American Composers'


Institute in Paris, aimed at researching and documenting the unique styles and
historical significance of the continent's classical composers.

www.strategictestprep.com ♟️ 1-833-300-PREP (7737) ♟️[email protected] 139


D) Rodriguez collaborated with major European orchestras to introduce "Echoes from
the South," a concert series dedicated exclusively to showcasing the rich variety of
Latin American classical compositions spanning three centuries.

9. In many coastal regions, coral reefs serve as barriers that reduce the impact of
ocean waves and protect the coastline from erosion. Some scientists suggest
that healthy coral reefs with diverse marine life also play a crucial role in
strengthening the health of nearby marine ecosystems. Their hypothesis is
based on the observation that areas with flourishing coral reefs often exhibit
increased fish populations and marine biodiversity.

Which finding, if true, would most directly weaken the scientists’ claim?

A) Coastal regions without coral reefs also sometimes display a rich diversity of marine
life due to factors like freshwater inflow or sea currents.

B) In some areas with healthy coral reefs, there has been an increase in the number of
certain bird species that feed on marine life.

C) New technologies show that marine ecosystems can flourish even in areas where
coral reefs have been severely damaged or have disappeared.

D) Coral reefs have been known to provide habitats for certain species of marine
plants that aren’t typically found elsewhere.

10.During the early 20th century, historians studied the cultural influences on
Indian classical music. Some historians, such as Vinay K. Gupta, posited that
Indian classical music was largely rooted in ancient Vedic traditions, predating
most foreign influences. Conversely, other scholars, like Priya S. Menon,
contended that while the Vedic influence is substantial, Indian classical music
was also significantly shaped by interactions with Persian, Central Asian, and
other foreign musical traditions over centuries.

Which finding, if true, would most directly support Priya S. Menon’s argument?

www.strategictestprep.com ♟️ 1-833-300-PREP (7737) ♟️[email protected] 140


A) The ragas used in Indian classical music have been traced back to the chants used in
Vedic rituals.

B) Certain Indian musical instruments have structural and functional similarities to


instruments from Persia and Central Asia.

C) The ancient texts of India describe musical scales and notations that are still
prevalent in Indian classical music today.

D) Most Indian classical compositions are based on themes and stories from ancient
Indian epics.

11.In her novel 'Whispers from the Horizon', Elena Martinez explores the idea that
personal experiences, no matter how small, play a vital role in shaping one's
perspective. The protagonist, Clara, often reflects on her past, pondering,
_______.

Which quotation from 'Whispers from the Horizon' most effectively illustrates the
claim?

A) "The memories, tiny fragments of time, have painted my world in hues unknown to
others."

B) "The horizon beckons me, drawing me towards a future full of unknown


adventures."

C) "Friendship, like a delicate flower, blooms in the most unexpected places."

D) "My journey, every step and stumble, led me to this very moment, echoing the past's
whispers."

12."Hamlet" is a circa 1600 play by William Shakespeare. In the play, the character
of Hamlet grapples with the death of his father and the discovery of his uncle's
treachery. His internal turmoil and contemplation of life and death are evident
when he _______

www.strategictestprep.com ♟️ 1-833-300-PREP (7737) ♟️[email protected] 141


Which choice most effectively uses a quotation from "Hamlet" to illustrate the
claim?

A) says to Horatio, “There are more things in heaven and earth, Horatio, / Than are
dreamt of in your philosophy.”

B) proclaims during the play within a play, “The play’s the thing / Wherein I’ll catch the
conscience of the king.”

C) muses to himself, “To be, or not to be: that is the question: / Whether 'tis nobler in
the mind to suffer / The slings and arrows of outrageous fortune.”

D) warns Laertes, “Give me your pardon, sir: I have done you wrong; / But pardon ’t, as
you are a gentleman.”

13.In a literary review, a critic argues that many scholars of Victorian literature have
portrayed author Emily Brontë as a recluse who was detached from her society,
rather than focusing on the sociopolitical nuances present in her novel
"Wuthering Heights."

Which quotation from a work by a literary scholar would best illustrate the critic’s
claim?

A) “Emily Brontë's limited social interactions in her small English village have been the
subject of much intrigue and speculation.”

B) “'Wuthering Heights' is a rich tapestry of human emotions, ranging from love and
obsession to revenge and redemption.”

C) “Despite living in a remote area, Emily Brontë’s writings strongly suggest a deep
understanding of societal dynamics and complexities.”

D) “Emily Brontë is often romanticized as the quintessential solitary genius, with the
moors around her shaping the hauntingly desolate setting of 'Wuthering Heights.'”

www.strategictestprep.com ♟️ 1-833-300-PREP (7737) ♟️[email protected] 142


Answers - Extra Practice

1. A

This choice directly supports the student's claim that Kuroyama's novels explore the
tension between traditional Japanese values and the influence of Western culture.
Characters struggling with their identity in a changing landscape symbolize this
tension.

Incorrect Answer Explanations:

B) While this choice suggests that Kuroyama was influenced by Western culture, it
does not provide evidence for how his novels specifically highlight the tensions
between traditional Japanese values and Western influences.

C) This choice speaks to Kuroyama's recognition and acclaim but does not address the
thematic content of his novels or the specific claim made by the student.

D) While this choice indicates Kuroyama's personal appreciation for traditional


Japanese art, it does not provide evidence regarding the themes or content of his
novels.

2. D

This supports the botanists' hypothesis that the gel acts as a protective barrier against
evaporation. If the gel evaporates slower than pure water, it provides evidence that the
gel is aiding in water conservation.

Incorrect Answer Explanations:

A) While this offers a contrast to the Agave's adaptation, it doesn't provide direct
evidence supporting the specific role of the gel in the Agave murpheyi's water
conservation.

B) While this suggests a correlation between the gel production and water scarcity, it
doesn't directly support the hypothesis that the gel is protective against evaporation.

www.strategictestprep.com ♟️ 1-833-300-PREP (7737) ♟️[email protected] 143


C) This does not provide evidence about the gel's specific function in water
conservation during dry spells, as it merely points out that other species have a similar
gel regardless of their environment.

3. A

This finding directly supports the team's claim, as it indicates that listening to
calming music (in this case, instrumental classical music) enhanced cognitive
performance compared to sitting in silence.

Incorrect Answer Explanations:

B) While this indicates familiarity with classical music, it doesn't provide evidence for
the calming effect of the music on cognitive performance.

C) This choice contradicts Rosen's claim, as it suggests that sitting in silence might be
more relaxing than listening to calming music.

D) While this shows participants' knowledge of classical music, it doesn't directly


support the idea that listening to calming music enhances cognitive performance.

4. B

This supports the dietitians' claim that they won't be replaced by such technology.
Even if the technology can accurately recommend personalized diets, human
interaction, support, and motivation provided by dietitians are irreplaceable
components of the dietary adherence process.

Incorrect Answer Explanations:

www.strategictestprep.com ♟️ 1-833-300-PREP (7737) ♟️[email protected] 144


A) This highlights the efficiency of the algorithm but doesn't provide reasons why
dietitians wouldn't be replaced. In fact, it could suggest the opposite, as efficiency
could be seen as an advantage of the technology.

C) While this highlights a limitation of the algorithm, it doesn't specifically address


why dietitians would not be replaced. Dietitians might also suggest similar foods in
some situations.

D) This might imply that some dietitians are not updated with recent developments in
their field. However, it doesn't directly provide evidence for the irreplaceability of
dietitians in the face of advanced technology.

5. B)

If the krill near volcanic ridges has a higher nutrient content, it could suggest that the
volcanic activity contributes to a richer food source, which would attract humpback
whales. This directly supports the team's hypothesis.

Incorrect Answer Explanations:

A) This information doesn't specifically support the team's hypothesis. It only suggests
that whales have other reasons for visiting areas besides just the presence of krill.

C) This choice provides information about the migration patterns of the whales, but it
doesn't give evidence that supports the link between volcanic activity, krill abundance,
and whale presence.

D) The presence of other marine animals near volcanic ridges doesn't provide evidence
for the team's specific hypothesis about krill abundance and humpback whale
migration.

6. C

www.strategictestprep.com ♟️ 1-833-300-PREP (7737) ♟️[email protected] 145


This directly supports the claim as it offers a potential causal relationship between the
decline of Megalodons' primary food source (large marine mammals) and their
extinction.

Incorrect Answer Explanations:

A) While this indicates dietary differences between Megalodons and other marine
predators, it does not directly address the relationship between Megalodons' diet and
their extinction.

B) While this provides some insight into Megalodons' preferred feeding locations, it
does not offer direct evidence for the connection between Megalodons' diet and their
potential vulnerability to changes in marine mammal populations.

D) Although this gives context to the Megalodons' feeding habits, it does not provide
direct evidence regarding the potential reasons for their extinction.

7. A

This answer choice supports the biologists' hypothesis directly. If the population of the
harmful insect has increased in the absence of its predator (the sunbird), it suggests a
relationship between the declining sunbird population and the declining native flower
population, due to increased harm from the insects.

Incorrect Answer Explanations:

B) While this provides insight into the adaptive behavior of sunbirds, it doesn't directly
support the hypothesis related to the decline of native flowers in the particular tropical
forest in question.

C) This suggests an alternate reason for why sunbirds might be declining in number, as
opposed to supporting the notion that a decrease in sunbirds has led to a decline in
native flowers.

D) This introduces the idea that factors other than the decline in sunbirds might be
responsible for the decline in native flowers, thereby not directly supporting the
biologists' hypothesis.

www.strategictestprep.com ♟️ 1-833-300-PREP (7737) ♟️[email protected] 146


8. B

This choice directly supports Rodriguez's mission to demonstrate the significant


contributions of Latin American composers to the global classical music narrative. By
presenting pieces by Latin American composers next to renowned European
compositions, Rodriguez integrated Latin American music into the broader global
classical tradition.

Incorrect Answer Explanations:

A) While this demonstrates Rodriguez's commitment to showcasing Latin American


composers, it doesn't directly show how she incorporated their music into the broader
context of global classical music evolution.

C) This highlights Rodriguez's efforts to promote and study Latin American composers,
but it doesn't explicitly position their work within the global classical music narrative.

D) This demonstrates Rodriguez's dedication to highlighting Latin American


composers, but it doesn't show how she demonstrated their significance to the
evolution of global classical music.

9. C

The scientists’ claim revolves around the positive impact of healthy coral reefs on the
marine ecosystem's health. If marine ecosystems can still thrive without the presence
of these coral reefs, it challenges the core of their hypothesis.

Incorrect Answer Explanations:

A) While this statement indicates that there can be other factors contributing to marine
biodiversity, it doesn't directly counter the idea that healthy coral reefs also enhance
nearby marine ecosystems.

B) This statement supports the scientists’ claim since it suggests that healthy coral
reefs may indirectly boost bird populations by enhancing the health of marine
ecosystems.

www.strategictestprep.com ♟️ 1-833-300-PREP (7737) ♟️[email protected] 147


D) This choice emphasizes the uniqueness of coral reefs but doesn't directly challenge
the hypothesis that healthy coral reefs strengthen nearby marine ecosystems.

10. B

Menon's argument is that Indian classical music was shaped by interactions with
foreign musical traditions. If certain Indian instruments are similar to those from Persia
and Central Asia, it would suggest an influence from these regions, supporting
Menon's viewpoint.

Incorrect Answer Explanations:

A) This choice supports Gupta's argument that Indian classical music is mainly rooted
in ancient Vedic traditions, not Menon’s.

C) This choice again leans towards the idea that Indian classical music is rooted in
ancient traditions, supporting Gupta's viewpoint.

D) While this highlights the Indian origin of the themes, it doesn't provide evidence of
foreign musical influences, which is central to Menon's argument.

11. A

This quotation highlights the influence of personal experiences ("tiny fragments of


time") on Clara's unique perspective ("painted my world in hues unknown to others"),
perfectly aligning with the claim that small personal experiences shape one's
viewpoint.

Incorrect Answer Explanations:

B) While this quotation addresses the theme of an uncertain future and adventure, it
doesn't directly emphasize the significance of past experiences in shaping perspective.

www.strategictestprep.com ♟️ 1-833-300-PREP (7737) ♟️[email protected] 148


C) This quotation touches upon the theme of friendship and its unpredictable nature,
but it doesn't provide insight into how personal experiences affect perspective.

D) While this quotation does hint at the protagonist's past experiences ("every step
and stumble") influencing her current moment, it doesn't as explicitly state how those
experiences have colored her worldview compared to option A.

12. C

This is Hamlet's famous soliloquy in which he reflects on the nature of existence and
contemplates life and death. It exemplifies his internal turmoil and contemplation as
described in the given context.

Incorrect Answer Explanations:

A) While this quote demonstrates Hamlet's recognition of the vast unknown and
mysterious forces at play, it doesn't directly reflect his contemplation of life and death.

B) This quote is about Hamlet's plan to confirm his uncle's guilt through a play. It
doesn't specifically delve into his internal struggles with life and death.

D) In this line, Hamlet seeks Laertes' forgiveness, but it does not directly address his
internal contemplation of life and death.

13. D

This quotation emphasizes the portrayal of Emily Brontë as a "solitary genius" whose
surroundings—the moors—are predominantly highlighted in discussions of "Wuthering
Heights." This characterization aligns with the critic's argument that scholars focus on
Brontë's supposed reclusiveness rather than the sociopolitical nuances in her work.

Incorrect Answer Explanations:

A) While this statement touches on the intrigue surrounding Brontë's limited social
interactions, it does not tie this portrayal directly to the understanding or interpretation
of "Wuthering Heights."

www.strategictestprep.com ♟️ 1-833-300-PREP (7737) ♟️[email protected] 149


B) This excerpt discusses the emotional depth of "Wuthering Heights" but does not
provide insight into the scholar's perspective on Emily Brontë's relationship with
society.

C) This quotation contrasts with the critic's claim. It acknowledges that Brontë's
writing indicates an understanding of societal dynamics, suggesting a nuanced reading
of her work.

www.strategictestprep.com ♟️ 1-833-300-PREP (7737) ♟️[email protected] 150


iv. Graph Questions 📈📊
Graph questions can be very time consuming, unless you know exactly where to look.
There are ways you can cut corners to save time and still get the correct answers.

Strategies

Strategy #1: Read the question first to know what to look for
This is especially helpful on graph questions where you aren’t completing the sentence
by filling in the blank.

Strategy #2: Go directly to the last sentence or two of the paragraph


This is where you will get the claim/argument.

Strategy #3: Don’t look at the graph at all


In some instances, multiple answer choices will cite accurate information from the
graphic; however, only one option will correspond to the specific focus in the question.

Thus, it is entirely possible to interpret a graph or chart perfectly and still end up with
the wrong answer.

In such cases, ignoring the chart and focusing on the phrasing in the text portion of the
question may, paradoxically, give you a much better chance of getting the question
right.

Strategy #4: Look for inconsistencies in the graph


Be proactive not reactive.
Look at the graph before reading the answers.
Is there a data point that stands out/is different from the rest?

www.strategictestprep.com ♟️ 1-833-300-PREP (7737) ♟️[email protected] 151


Examples

Example #1 - A Straightforward Example

As part of a unit on pollination, a biology student is researching Colony Collapse


Disorder, the mysterious disappearance of millions of honeybee (Apismellifera) colonies
in the early 2000s. Although many wild bee populations have continued to decline, the
number of honeybees has actually increased on every continent. The student wants to
emphasize that in some regions, honeybee populations had not only rebounded by
2020 but also reached their highest level in 60 years.

Underline the claim in the text. What two things does the student want to
emphasize?

👉(1)
👉(2)
Make sure to pick an answer that supports both of these things.

Which region included in the graph should the student cite to support this claim?

A) Africa
B) Americas
C) Europe
D) Asia

www.strategictestprep.com ♟️ 1-833-300-PREP (7737) ♟️[email protected] 152


Example #2 - Another Straightforward Example

A student studying the presence of white-nose syndrome, a disease that has


decimated bat populations across North America since 2007, examined data about the
number of bats in a protected area. Because bats live in the dark and their cries are
inaudible to people, biologists must use acoustic detectors to record their sounds and
then analyze them to identify the species present in a given location. Observing that
the highest number of cries came from big brown bats(Eptesicus fuscus), the student
concluded that brown bat species were not affected by the disease.

Which statement best describes data from the graph that would undermine the
student's conclusion?

A) No bat species emitted more than 10 cries on average per night

B) The number of cries from little brown bats was among the lowest of any species.

C) More cries were recorded for tri-colored bats than for hoary bats.

D) Eastern red bats were more vocally active than big brown bats.

……..Did you even need the graph for this? 🤨


www.strategictestprep.com ♟️ 1-833-300-PREP (7737) ♟️[email protected] 153
Example #3 - No Graph Needed

Despite steady gains in readership and overall popularity, e-books are hardly
positioned to replace print books. Rather, the two types of media complement each
other, providing the same content in different forms. Print books and e-books each
have unique attributes and serve distinct purposes, which vary by demographic and
situational factors. Researchers Yin Zhang and Sunali Kudva used data from the
National Reading Habits Survey to examine book-format preferences in a variety of
situations. Although the pair found that e-books are firmly established as an option
due to ease of accessibility, they also concluded that print books offer greater appeal to
most readers in certain situations.

Wait, where is the graph? 🤷‍♀️ The length of the answer choices suggests that
the graph may not be important.

Let’s see if we can answer the question. If we cannot, then we will look at the graph.

Which choice best describes data from the chart that supports Zhang and Kudva's
finding?

A) A large majority of readers seeking immediate access to a book preferred the


electronic option, with less than 25% of readers opting for print.

B) The percentage of readers who choose a format based on the variety of titles
available was about 50% for both print books and e-books.

C) More than 75% of readers preferred print books for reading to a child, and more
than 60% preferred print books for sharing with other readers.

D) When concerned about quick access or reading during travel, most readers
preferred books in the same format.

www.strategictestprep.com ♟️ 1-833-300-PREP (7737) ♟️[email protected] 154


Now, to be safe, just spot check your answer with the graph to make sure that it is the
right one.

www.strategictestprep.com ♟️ 1-833-300-PREP (7737) ♟️[email protected] 155


Example #4 - No Graph Needed

MRSA- Methicillen-resistant Staphylococcus aureus- is a type of bacteria that is


frequently responsible for hospital-acquired infections and is known for being
exceptionally resistant to common antibiotics. When a group of these medications
were compared in terms of their ability to cure MRSA infections versus a variety of
other infections, researchers found that an antibiotic could demonstrate a high level of
general effectiveness but that the same medication could be significantly less effective
in combating MRSA. For example, ____.

According to the claim, these two conditions must be fulfilled:

👉(1)
👉(2)
Now, use the process of elimination to weed out answers that don’t fulfill those two
requirements.

Which choice most effectively uses data from the table to logically complete the
text?

A) Erithromycin had a general effectiveness of nearly 32%, whereas its MRSA


effectiveness was only about five points lower.

B) Whereas Mupirocin was around 17% less effective against MRSA than against
general infections, the gap for Clindamicin was only about 14%.

C) Only Vancomycin was fully effective in combatting both general and MRSA bacteria.

D) Rifampicin showed a general effectiveness rate of more than 85%, whereas it was
just over 60% effective against MRSA

www.strategictestprep.com ♟️ 1-833-300-PREP (7737) ♟️[email protected] 156


Now, just double check your answer with the graph just to be safe (optional):

www.strategictestprep.com ♟️ 1-833-300-PREP (7737) ♟️[email protected] 157


Answers to Graph Examples

1. A

The only line that fits both those criteria is the line for Africa: there is a dip between
2000 and 2010, and the highest point (20 million) corresponds to 2020.

2. B

What is the student's conclusion? That brown bat species were not affected by
[white-nose syndrome] because the highest average number of cries recorded came
from big brown bats. Note that answer choice B is the only answer that talks about
brown bats! The other answers are irrelevant.

3. C

The wording of the last sentence is key because it tells us what the researchers
concluded: most readers- that is, more than 50%- prefer print books (dark gray bar)
sometimes. Any answer that contradicts this idea or that has a different focus can be
eliminated.

(A) can be eliminated because the focus is on a situation in which readers prefer
e-books.
(B) doesn't fit because the focus is on a situation in which the numbers are roughly
equal.
(D) doesn't provide enough information to judge, although it is reasonable to assume
that most readers who were traveling or seeking quick access to a book would want
the electronic version.

(C) is a match, citing situations in which a clear majority of readers (75% and 60%
respectively) prefer print books.

4. D

The correct answer must fulfill two conditions:

​ 1) It must mention just one antibiotic.

www.strategictestprep.com ♟️ 1-833-300-PREP (7737) ♟️[email protected] 158


​ 2) It must include a statistic indicating a significant difference between general
effectiveness and effectiveness against MRSA.

On that basis:

● We can eliminate (A) because 5% indicates a small gap.

● We can eliminate (B) because it compares two different medications.

● We can eliminate (C) because the effectiveness rates of Vancomycin were the
same.
That leaves (D), which correctly cites an example of a single drug with a large
difference (>25%) in effectiveness.

www.strategictestprep.com ♟️ 1-833-300-PREP (7737) ♟️[email protected] 159


Extra Practice

1.

El Nino is a climate pattern in which water in the Pacific Ocean near the equator
becomes hotter than usual, affecting the atmosphere and weather around the world.
Although El Nino climate conditions are unpredictable, they typically occur every few
years and can change the weather in the United States, particularly in the southern
states and in California. Climatologists have found that although El Nino years do not
bring heavy rains every month, the difference in rainfall during the winter in El Nino
years can be much greater than the difference during other seasons. Los Angeles, for
example, receives around 15mm of rain on in May on average and during El Nifio years,
whereas _ _

Which choice uses data from the graph to logically complete the text?

​ A) 5mm more rain than average falls in October when El Nifio is present.

​ B) over 100 mm of rain falls in January during El Nifio years.

​ C) the amount of rainfall in August and September is lower than average during
El Nifio years.

​ D) the amount of rainfall in February during El Nifio years is 100 mm higher than
average.

www.strategictestprep.com ♟️ 1-833-300-PREP (7737) ♟️[email protected] 160


2.

Plants use photosynthesis to create energy from visible light from the sun. In addition
to visible light, however, sunlight contains ultraviolet (UV) light. Researchers at the
University of Hokkaido wondered whether it would be possible to provide plants with
additional visible light by employing a wavelength converting material (WCM) capable
of transforming UV light into red light. They created a thin-film WCM coating and
applied it to clear plastic sheets, which were placed next to Swiss chard plants. A
control group used sheets without the coating. In the summer, when sun irradiation
was strong, no significant difference was observed between the two groups. In winter,
however, the plants with the WCM films were significantly taller and contained more
biomass after 63 days as compared to the control group. The researchers concluded
that this accelerated growth was caused by the increased supply of red light provided
by the WCM films.

Which choice uses data from the graph to support the researchers' conclusion?

A) Light that is less than 400 nanometers long had a much lower level of intensity
than light that is more than 450 nanometers long.

B) Light ranging from 500 to 550 nanometers had the highest level of intensity.

C) The intensity of light waves more than 600 nanometers long that passed through
the films increased to nearly peak levels.

www.strategictestprep.com ♟️ 1-833-300-PREP (7737) ♟️[email protected] 161


D) UV absorption declined steeply in light waves that are less than 450 nanometers.

3.

Despite the sensationalist claims about the effects of video games on children's
development that regularly appear in the media, a growing body of research purports
to demonstrate that game players outperform non-gamers on a range of cognitive
measures, and some studies suggest that the skills acquired through gaming can be
transferred to real-world situations. However, these studies employ a variety of
methodologies, criteria, and types of participants. Marc Palaus and colleagues at
Oberta University in Spain conducted a review of 116 studies, aiming to better
understand the relationship between gaming and cognitive development. While they
concluded that it is possible to establish links between video games and skills
involving attention, cognitive control, and visuospatial processing, they also observed
that the lack of standardization could contribute to inconsistencies in the findings of
similar studies.

Which choice most effectively uses data from the graph to support the researchers'
observation?

A) The studies run by Takeushi relied on larger sample sizes and involved younger
participants than any of the other studies.
B) Few of the studies included participants under the age of 18.
C) Each study focused on participants in a particular age range and did not include
members that were much older or younger.

www.strategictestprep.com ♟️ 1-833-300-PREP (7737) ♟️[email protected] 162


D) The participants in the studies conducted by Erickson, Vo, and Kiihn and Gallinat
were all under the age of 30.

4.

Seals are among the few mammals other than humans that are capable of learning
new types of vocalizations. Whereas it is well established that adult harbor seals can
acquire new vocal patterns, until recently this phenomenon had never been studied in
pups. In 2021, researchers at the Max Planck Institute for Psycholinguistics and a
group of colleagues conducted a study in which they played a series of pre-recorded
sounds for a group of harbor seal pups ranging from one to three weeks old. They
found that the baby seals were able to modify their vocalization patterns: overall, the
pups lowered the fundamental frequency (FO) of their calls in response to increased
noise. In some cases, the response with highly pronounced, with the same animal
emitting vocalizations at much lower frequencies in high noise than in low noise.

Which choice most effectively uses data from the table to support the researchers'
finding?

A) Seal B emitted vocalizations of 330 Hz in low noise and 300 Hz in high noise.

B) The fundamental frequency of Seal A's vocalizations in high noise was 100 Hz
lower than in low noise.

C) Whereas the fundamental frequency of Seal A's call was 125 Hz lower in high
noise than in no noise, the frequency of Seal B's call dropped by less than 50 Hz.

www.strategictestprep.com ♟️ 1-833-300-PREP (7737) ♟️[email protected] 163


D) In both low and high noise, the fundamental frequency of Seal E's call was around
150 Hz than it was without noise.

**Go to https://satsuitequestionbank.collegeboard.org/ for additional graph practice.

Answers - Extra Practice

1.D

Although the passage is fairly long, the key information - as usual - appears at the end.
You cannot just read the last sentence, however: it includes the transition for example,
which means that you need to back up and find the idea that the sentence is
illustrating. You'll find it in the previous sentence, which states that although El Nino
does not result in consistently heavy rains throughout the year, the difference in rainfall
during the winter in El Nino years can be much greater than the difference during other
seasons. Los Angeles is then cited as an example of a city where this is the case. The
last sentence indicates that average and El Nino rainfall are the same in May (in a
season other than winter), and the word whereas before the blank indicates that the
remainder of the sentence must describe the opposite - a winter month in which the
difference between average and El Nino rainfall is large. The correct answer must
therefore include a comparison between El Niiio rainfall and average rainfall.

(B) does not include a comparison, so this answer can be eliminated immediately. Be
careful with (A): this answer does mention a month in which El Nino is associated with
more rain than average, but 5mm is a very small amount, and the correct answer must
involve an amount that is much greater. (C) does not fit because the correct answer
must involve months in which El Nino rainfall is higher, and there is also no information
about the size of the differences. (D) correctly cites an example of a winter month
(February) in which El Nino is associated with rainfall that is much higher than the
average (150 mm).

2. C

Don't be intimidated by the seeming complexity of the graph- this question is much
easier than it looks. What is the researchers' conclusion? You'll find it in the last
sentence: the accelerated growth of the plants with WCM films was caused by the
increased supply ofred light. The correct answer must support the idea that the films
increased the red-light supply.

www.strategictestprep.com ♟️ 1-833-300-PREP (7737) ♟️[email protected] 164


Shortcut: (D) is the only answer that mentions the films, so start by checking it. Indeed,
the fact that the light that passed through the films increased to "peak levels" of
intensity in the "red" area of the graph directly illustrates the idea that the films caused
the plants to receive more red light. (D) is thus correct. The other answers include
information about light wavelength, intensity, and absorption but do not explicitly
mention the effect of the films.

3. A

What is the researchers' observation? That the lack of standardization [among studies]
could contribute to inconsistencies in the findings of similar studies. The correct answer
must therefore mention a factor that varies from study to study and include a
comparison of some type. The words Each study in (C) and all in (D) indicate that these
answers are discussing factors that remain the same across studies, so both choices
can be eliminated. (B) is off-topic: even though the studies involve video games, the
relative lack of children among the participants is not the issue indicated by the
passage. Only (A) mentions an inconsistency that illustrates the researchers'
observation: the studies run by one of the participants differed noticeably from the
other studies in that they involved far more participants. In other words, it shows that
the studies were not standardized.

4. B

What is the researchers' finding? It appears in the last sentence: the seal pups changed
their vocalizations in response to the sounds, and in some cases, the same animal
might produce vocalizations with much lower fundamental frequencies in high noise
than in low noise. The correct answer must therefore focus on one animal only and
indicate a large gap in fundamental frequency between low and high noise. (C) makes
a comparison between two seals, so it can be eliminated automatically. Be careful with
(A): it does compare the frequencies in low and high noise, but the gap is fairly small
(30 Hz). (D) is likewise incorrect because the correct answer must cite a large
difference in the responses to the two noise levels, and here it is very similar. Only (B)
offers an example of a seal whose vocalizations showed a large gap (100 Hz) between
low and high noise.

www.strategictestprep.com ♟️ 1-833-300-PREP (7737) ♟️[email protected] 165


✨How can you make the most out of this workbook? ✨
👉For walkthrough videos, strategies, quizzes, and extra resources that support
each lesson in this workbook, please sign up for our English Self-Paced Course!

👉For individualized support, please check out our Information Pack to learn
more about our private tutoring options and Request a Consultation!

✨Looking for additional resources to guide you in your Digital SAT prep journey? ✨
👉Practice wherever, whenever you want with the Preptly: Digital SAT Prep
App! Join our 2,000+ users and start improving your score today!
📲 Get Preptly in the App Store or in Google Play today!
👉Looking to improve your math score as well? 🤨 We have a new Self-Paced
Digital SAT Math Course! (Available in 2024)

👉Check out our YouTube Channel, which provides free videos that teach you
how to take the Digital SAT like a pro!

👉Follow us for up-to-date tips, tricks, strategies, and more!

www.strategictestprep.com ♟️ 1-833-300-PREP (7737) ♟️[email protected] 166


v. Logically Completes the Text 🧠
Strategies

Strategy #1: Do not Cut Corners; Read the Entire Text to Start
You should annotate on the text or on your scrap paper, putting what the author is
saying into your own words.

Strategy #2: Look for a Contrast Statement

The contrast statement is very important because it denotes a shift from an old
thought to a new thought.

Typically, you will base your logical conclusion on the new thought, which comes in the
contrast statement!

Progress Check! What are some contrast words? List 5.

1. ____________________________________

2. ____________________________________

3. ____________________________________

4. ______________________________________

5. ______________________________________-

Most Logically Completes the Texts are structured the following way:
(1) Intro: What is this text about
—> (2) Original Argument: What did they used to think/argue
—> (3) New Argument: What are new findings/beliefs that go against the
old
—> (4) Your own logical conclusion based on the new argument

www.strategictestprep.com ♟️ 1-833-300-PREP (7737) ♟️[email protected] 167


Tip: Very often, the key information will appear very close to the blank, the earlier
part of the passage serving primarily to provide background for the argument.

Strategy #3: Put the New (Contrast) Argument into Your Own Words
This helps you understand the argument better so you can draw the best logical
conclusion.

Strategy #4: Use Process of Elimination to Weed out Silly Answers

Exercises

Exercise #1: Mark where you see the old idea and the new idea in the text? Then, sum
them up in your own words.

Sometime near the end of the Pleistocene, a band of people left northeastern Asia,
crossed the Bering land bridge when the sea level was low, entered
Alaska and became the first Americans. Since the 1930s, archaeologists have thought
these people were members of the Clovis culture. First discovered in New Mexico in
the 1930s, the Clovis culture is known for its distinct stone tools, primarily fluted
projectile points. For decades, Clovis artifacts were the oldest known in the New
World, dating to 13,000 years ago. But in recent years, researchers have found more
and more evidence that people were living in North and South America before the
Clovis.

Old Idea:

New Idea:

Exercise #2: Identify the old idea and the new idea, sum them up in your own words,
then eliminate answers that you find to be irrelevant and answer the question.

www.strategictestprep.com ♟️ 1-833-300-PREP (7737) ♟️[email protected] 168


Most grocery stores spray produce with water on a regular basis in order to ensure that
they maintain a wholesome, fresh-picked appearance. However, according to Martin
Lindstrom, author of Brandwashed: Tricks Companies Use to Manipulate Our Minds
and Persuade Us to Buy, not only does this liquid lack any practical purpose, but it
actually has a deleterious effect: _ _

Old Idea:

New Idea:

Which choice most logically completes the text?

A) shoppers are unlikely to purchase fruits and vegetables that appear dry and
withered.

B) moisture causes picked vegetables to spoil more quickly than they otherwise
would.

C) certain vegetables lose some of their nutrients when they are boiled.

D) produce must be watered at predictable intervals in order to appeal to


consumers

Answer and explanation on next page..

The answer to that last example was B. The key phrase occurs right before the blank:
not only does the liquid [used to water fresh produce] lack any practical purpose, but it

www.strategictestprep.com ♟️ 1-833-300-PREP (7737) ♟️[email protected] 169


also has a deleterious (negative] effect. The information that follows must logically
explain that effect.

Why the others are wrong:

A) the fact that shoppers are unlikely to select produce in poor condition is irrelevant to
this section of the passage.

(C) is incorrect because the passage focuses only on the effects of water on uncooked
vegetables.

(D) incorrectly states a positive effect of watering produce. (opposite of text)

Now, let’s try a few more…

www.strategictestprep.com ♟️ 1-833-300-PREP (7737) ♟️[email protected] 170


Exercise #3: Try these 3 practice questions below. Then, check the answers that follow.

Practice Question 1

Physicists have yet to figure out what exactly happens at the singularity of a black
hole: matter is crushed, but what becomes of it then? The event horizon, by hiding the
singularity, isolates this gap in our knowledge. All kinds of processes unknown to
science may occur at the singularity, yet they have no effect on the outside world.
Astronomers plotting the orbits of planets and stars can safely ignore the uncertainties
introduced by singularities and ____.

Old Idea:

New Idea:

Which choice most logically completes the text?

A) apply the standard laws of physics with confidence.

B) focus on gaining a deeper understanding of black holes.

C) attempt to peer behind the event horizon.

D) uncover phenomena not currently known to science.

www.strategictestprep.com ♟️ 1-833-300-PREP (7737) ♟️[email protected] 171


Practice Question 2

Although it is widely assumed that cognitive bias clouds our assessment of the people
around us, their research and that of others, a group of researchers at the Santa Fe
Institute has found that people's estimations of what their friends and family believe
are often largely correct. That's because as highly social creatures, we have become
very good at sizing up those around us- what researchers call "social sensing." It is
therefore possible _ _

Old Idea:

New Idea:

Which choice most logically completes the text?

A) to gather highly accurate information about trends by asking about individuals


about their social circles rather than their own beliefs.

B) to determine people's views on a variety of topics by analyzing the ways in


which they interact with others.

C) to discover what people truly believe about an issue by asking them to reflect on
their personal biases.

D) to develop an algorithm that reliably predicts people's preferences about a wide


range of items.

www.strategictestprep.com ♟️ 1-833-300-PREP (7737) ♟️[email protected] 172


Practice Question 3 - What if there is no contrast statement?

Exactly how Mars was formed approximately 4.5 billion years ago is a mystery,
although there are several theories. One idea is that the planet was created via a titanic
collision of rocks in space, spawning an all-encompassing magma ocean. When it
cooled, a crust with high levels of basalt was formed. Another possibility is that parts
of the first crust on Mars had a different origin, one that would primarily show large
concentrations of silica. Planetary geochemist Valerie Payre and her partners analyzed
data for the planet's southern hemisphere, the planet's oldest region. They discovered
nine locations rich in feldspar, a mineral associated with lava flows that are higher in
silica than basalt. This finding led them to conclude that __________.

Stay organized, annotating and summing up what you’ve read!

Idea #1 of How Mars Was Formed:

Idea #2 of How Mars Was Formed:

Researchers discovery:

Which choice most logically completes the text?

​ A) portions of Mars' surface were never covered by a crust.

​ B) the magma ocean formed from rocks colliding in space was not
all-encompassing.

​ C) the southern hemisphere of Mars contained more silica than was previously
believed.

​ D) the first crust on Mars did not develop until long after the planet was formed.

www.strategictestprep.com ♟️ 1-833-300-PREP (7737) ♟️[email protected] 173


Answers

1. A
2. A
3. B

Answer explanations:

1. The second-to-last sentence includes the transition yet, which indicates key
information: although singularities may involve all sorts of mysterious
processes, they have no real-world effect. Thus, scientists can ignore them. The
correct answer must restate or be consistent with this idea. (A) is correct
because if singularities don't have an effect, then scientists can treat everything
as normal and apply the usual laws of physics. (B) is unrelated to the required
idea, and (C) and (D) both contradict the idea of ignoring puzzling aspects of
black holes and maintaining the existing scientific framework.

2. This is a good example of a question in which the answer is very nearly stated in
the passage. The text makes clear that people are generally pretty accurate in
identifying the beliefs of their friends and family members, so logically it must
be possible to gather accurate information about trends by asking individuals
what the people close to them think rather than what they themselves
think-which is exactly what (A) says. (B) is incorrect because the passage
focuses on beliefs, not interactions. (C) says exactly the opposite of what is
indicated by the passage-reliable information can be obtained by asking people
about others, not themselves. (D) goes way beyond the scope of the passage,
which does not discuss the role of technology in determining people's
preferences at all.

3. This is a complicated question, so we're going to break it down into parts. Notice
that the process outlined here requires you to work backwards through the
passage. First, what is the finding? That nine regions of Mars were rich in
feldspar, which is associated with lava flows that are higher in silica than basalt.
What is the significance of this finding? It supports the theory that parts of the
first crust on Mars had a different origin. A different origin from what? From the

www.strategictestprep.com ♟️ 1-833-300-PREP (7737) ♟️[email protected] 174


all-encompassing magma ocean that was created from rocks colliding in space.
Logically, if parts of the crust were formed from something other than the
magma ocean, then the ocean could not have been all-encompassing (covering
everything). And that is what (B) says. (A) is incorrect because the passage only
discusses the origins of the first crust that did exist on Mars-there is nothing to
suggest that there were places without a crust. (C) is incorrect because the
passage indicates only that feldspar, which is found in Mars's southern
hemisphere, is higher in silica than basalt. The passage does not indicate how
much silica researchers previously believed was present. (D) does not work
either because the passage only mentions that Mars was formed around 4.5
billion years ago and the southern hemisphere is Mars's oldest region - there is
no mention of when the crust formed, and the amount of feldspar and silica in
that area is not discussed in relation to any particular timeline.

www.strategictestprep.com ♟️ 1-833-300-PREP (7737) ♟️[email protected] 175


Extra Practice

1. Several ancient Egyptian murals depict a male figure standing next to a river,
surrounded by a variety of animals. Some historians argue that the figure is the god
Osiris, given that he is traditionally associated with the Nile River. However, Dr. Emily
Thompson from the University of Cambridge argues that rivers and animals were
commonly used symbols for prosperity in ancient Egyptian art. The fact that animals
and a river are shown near the male figure, therefore, _______

Which choice most logically completes the text?

A) is not conclusive evidence that the figure is Osiris.


B) suggests that Osiris was often depicted with animals.
C) eliminates the possibility that the figure is Osiris.
D) would be difficult to account for if the figure is not Osiris.

2. Many of Jane Austen's novels focus on the intricacies of social relationships and the
challenges women face within the constraints of their societies. For example, "Pride
and Prejudice" delves into issues of class, reputation, and love, themes that remain
relevant to today's readers. However, understanding Austen's "Mansfield Park" can
require a deep knowledge of the social and economic intricacies of 19th-century
England. Consequently, _______

Which choice most logically completes the text?


A) many readers today are likely to find "Mansfield Park" less engaging than novels
like "Pride and Prejudice."
B) some of Austen's novels are more relevant to today’s readers than contemporary
works.
C) "Pride and Prejudice" is the most accessible of all Austen's novels.
D) experts in 19th-century English history tend to prefer "Mansfield Park" to her other
works.

3. The Indus Valley Civilization, one of the world's oldest civilizations, flourished
around 3300 B.C.E. in what is now Pakistan and northwest India. This civilization
mysteriously declined around 1900 B.C.E., leaving behind intricate urban centers with

www.strategictestprep.com ♟️ 1-833-300-PREP (7737) ♟️[email protected] 176


advanced sewage systems. Recent analysis comparing pottery shards from
Mohenjo-Daro, a significant city in the Indus Valley, to pottery found in a settlement in
what is now eastern Iran revealed identical manufacturing techniques, with similarities
becoming prominent only after 1950 B.C.E. Thus, researchers concluded that _______

Which choice most logically completes the text?


A) the decline of the Indus Valley Civilization was primarily due to internal conflicts.
B) some people from the Indus Valley likely migrated to eastern Iran around 1900
B.C.E. and brought their pottery techniques with them.
C) residents of eastern Iran primarily used metal utensils and did not make pottery
before 1950 B.C.E.
D) the Indus Valley Civilization likely adopted pottery manufacturing techniques from
civilizations in eastern Iran.

4. One challenge when studying the long-term effects of a new medication is the issue
of creating an appropriate placebo group. To properly gauge the medication's
effectiveness, researchers must compare the group taking the medication to a placebo
group that is otherwise similar but not taking the medication. Since researchers cannot
ethically assign patients to a placebo group if an effective treatment already exists,
they therefore _______

Which choice most logically completes the text?


A) find it easy to create a placebo group consisting of individuals who are already on
existing medications.
B) struggle to identify a group of people who can function as an appropriate placebo
group for their studies.
C) can only conduct studies on diseases or conditions for which no effective treatments
exist.
D) should select a placebo group of people who differ from the medication group in
several significant ways.

5. Adopted by multiple nations, the Blue Ocean Accord aims to regulate deep-sea
mining activities to protect marine biodiversity. However, critics point out a flaw: the
accord allows companies to keep the specifics of their environmental impact
assessments confidential. Therefore, some environmental activists express concern
that the accord may have the unintended effect of ______

www.strategictestprep.com ♟️ 1-833-300-PREP (7737) ♟️[email protected] 177


Which choice most logically completes the text?

A) reducing the operational costs for companies engaged in deep-sea mining.


B) limiting the types of technology that can be used in deep-sea mining.
C) preventing third-party experts from verifying the ecological impact of the mining
activities.
D) discouraging companies from exploring new, less harmful mining techniques.

6. The domesticated corn (Zea mays) is native to Central America but is also found in
Asia, where evidence shows that local populations were cultivating it long before
transoceanic voyages were established between the two regions. To understand how
corn was introduced in Asia, geneticist Emily Wang and her team analyzed the DNA of
various corn varieties, concluding that Asian varieties diverged from Central American
ones around 70,000 years ago. Given that historical records show the presence of
humans in Asia for much longer, the team concluded that ______

Which choice most logically completes the text?

A) the cultivation of corn in Asia likely predates its cultivation in Central America.
B) Asian populations likely acquired corn from Central American populations through
ancient sea voyages.
C) human activity likely played no role in the introduction of corn in Asia.
D) Asian corn varieties likely descend from a single Central American variety that was
domesticated, not wild.

7. In a study evaluating the problem-solving abilities of African grey parrots (Psittacus


erithacus), researchers did not control for the varying complexity of the puzzles used to
test the birds. Parrots given simple puzzles, such as pushing a button to release food,
were assessed using the same criteria as those given complex puzzles, like opening a
series of locks to get to a treat. The results of the study, therefore, ______

Which choice most logically completes the text?

A) could suggest that there are differences in problem-solving abilities among the
parrots even though such differences may not actually exist.

www.strategictestprep.com ♟️ 1-833-300-PREP (7737) ♟️[email protected] 178


B) are useful for identifying puzzles that the parrots can solve but not for identifying
puzzles they find challenging.
C) should not be taken as indicative of the problem-solving abilities of any bird species
other than Psittacus erithacus.
D) reveal more about the parrots' problem-solving abilities when engaging with
human-made objects than when solving natural problems.

8. Among many mammals, including cats and dogs, newborns show a tendency to
follow moving objects with their eyes. In a study led by researcher Dr. Emily Clark, a
team investigated whether this behavior is also exhibited by newborn reptiles, focusing
on the chameleon species Chamaeleo calyptratus. Despite chameleons being primarily
solitary animals with limited social interactions, the study found that chameleon
hatchlings displayed a pronounced tendency to track moving objects with their eyes.
This suggests that ______

Which choice most logically completes the text?

A) eye-tracking behavior in mammals is a learned trait influenced by the social


environment.
B) researchers should not automatically attribute eye-tracking behavior to socialization
or communal living.
C) the behavior of tracking moving objects with their eyes will likely diminish as
chameleons mature.
D) this tendency is a universal trait among all vertebrates, regardless of their lifestyle.

9. Compiled in the 12th century, "The Book of Healing" is a comprehensive scientific


and philosophical encyclopedia written by Persian polymath Avicenna. The text serves
as a record of knowledge in various scientific fields, such as astronomy, medicine, and
psychology, available at that time. However, some chapters in the book contain
concepts that seem to be borrowed from Greek philosophers and scientists, such as
Aristotle and Galen, rather than stemming from Persian intellectual traditions.
Therefore, some scholars have concluded that _______

Which choice most logically completes the text?

www.strategictestprep.com ♟️ 1-833-300-PREP (7737) ♟️[email protected] 179


A) "The Book of Healing" is essentially a translation of Greek scientific works into
Persian.
B) although Avicenna was a Persian scholar, he was influenced by Greek philosophers
and scientists when compiling "The Book of Healing."
C) the book should be credited to Greek philosophers as it mainly contains their ideas
and concepts.
D) Avicenna likely had limited knowledge of Persian scientific traditions when
compiling "The Book of Healing."

10. In modern astrophysics, the use of computational models has become increasingly
important for simulating scenarios that cannot be physically replicated, such as the
formation of galaxies or the collision of black holes. Astrophysicist Dr. Sara Ruiz argues
that while these models can offer valuable insights, they are often based on a set of
simplified assumptions that may not capture the full complexity of real-world
phenomena. For the results to be truly robust, it is crucial to _______

Which choice most logically completes the text?


A) ignore the limitations of computational models, as they are the best tools available
for understanding complex astrophysical events.
B) focus solely on refining the algorithms that underlie these computational models.
C) test the models against a variety of scenarios, including those that challenge the
underlying assumptions.
D) limit the use of computational models to phenomena that can be observed directly
through telescopes.
11. One of the most startling discoveries of the early 21st century was that
Inda-European languages seem not to have been spread by Anatolian farmers living in
what is now Turkey, as was commonly thought, but rather by a people called the
Yamnaya, horse-herding nomads who lived on the Eurasian steppes more than 5,000
years ago. A host of linguistic evidence suggesting this possibility was first compiled
persuasively by archaeologist David Anthony in 2007; DNA evidence later proved he
was on target, showing that ________.

Which choice most logically completes the text?

A) members of tribes from the steppes arrived in Germany sometime between


2500 and 2000 BCE.

www.strategictestprep.com ♟️ 1-833-300-PREP (7737) ♟️[email protected] 180


B) the Yamnaya were a genetic blend of three separate Eurasian populations.

C) around 5,000 years ago, the Yamnaya's genes began to appear throughout
Europe and
Asia .

D) the Yamnayans were linguistically unique in comparison to other groups from


the same period.

12. When Isaac Newton published the Principia in 1687, his laws of motion solved
numerous problems in physics; however, they also introduced a new conundrum, which
was not fully grasped until centuries after Newton and which still poses a problem for
cosmologists today. Essentially, Newton's laws work about twice as well as they are
intended: they describe the everyday world that people move through, but they also
account perfectly well for a world in which people walk backwards, clocks tick from
evening to morning, and ____.

Which choice most logically completes the text?

A) objects interact unpredictably with one another.

B) planets that are in motion remain in motion.

C) particles of different weights move at varying speeds.

D) apples rise from the ground to the branches of a tree.

13. The cheetah, known for its exceptional speed, can reach velocities of up to 58
miles per hour in short sprints. Some zoologists theorized that the evolution of the
cheetah's speed was a response to an increase in the agility of its primary prey.
However, fossil records do not show a concurrent increase in the agility of antelope
or gazelles during periods when cheetahs evolved their remarkable speed. This
suggests that _______

Which choice most logically completes the text?

A) cheetahs evolved their speed to escape from predators, not to chase prey.

www.strategictestprep.com ♟️ 1-833-300-PREP (7737) ♟️[email protected] 181


B) the evolution of the cheetah's speed was not solely in response to the agility of its
prey.

C) antelope and gazelles have always been slower than the cheetah.

D) the agility of prey is the sole factor determining a predator's speed.

14. In academic essays, authors often integrate research findings to support their
claims. Educational researcher Dr. Martin Upton argues that while many authors
seamlessly incorporate studies that back their thesis, effective academic writing
also involves addressing counterarguments. By examining research that may
contradict their stance, authors _______

Which choice most logically completes the text?

A) can make their essays lengthier and more detailed.

B) ensure that their arguments remain uncontested.

C) enhance the credibility and depth of their arguments.

D) show readers that they are well-read in their field.

15. In many top-tier universities around the world, a notable percentage of faculty
members have previously studied or worked at Ivy League institutions in the United
States. This phenomenon could be attributed to the fact that Ivy League institutions
offer a rigorous academic environment that prepares individuals for advanced research
and pedagogy, equipping them with skills that _______

Which choice most logically completes the text?

A) are universally recognized in the academic realm.

B) lead to lucrative offers from corporate sectors.

C) are specific only to the American education system.

D) discourage individuals from seeking opportunities outside academia.

www.strategictestprep.com ♟️ 1-833-300-PREP (7737) ♟️[email protected] 182


16. Many animals, including several species of amphibians, use toxins as a primary
defense mechanism against predators. In these species, brightly colored patterns on
their skin typically indicate the presence of such toxins, serving as a warning signal
to potential threats. Predators, over time, learn to associate these bright patterns
with a foul taste or harmful consequences and, therefore, tend to avoid these
animals. However, biologist Dr. Lucy Green has observed that certain non-toxic
amphibians mimic these bright patterns even though they don't produce the
harmful toxins. This phenomenon suggests that _______

Choices:

A) non-toxic amphibians have evolved to develop toxins but haven't been successful.

B) the colors on the skin of amphibians are a result of their dietary habits.

C) predators often make mistakes in distinguishing between toxic and non-toxic


amphibians.

D) these mimicking non-toxic amphibians benefit from a deceptive warning signal


without bearing the metabolic cost of producing toxins.

17. Historians often discuss the influence that Catherine de' Medici had on French
politics during her time as queen consort and regent, but few realize she was also a
patron of the arts. Catherine sponsored several art projects, ballets, and
commissioned notable works during the Renaissance era in France. Therefore,
individuals who solely attribute Catherine's significance to her political maneuvers
______.

Which choice most logically completes the text?

A) may underestimate the Renaissance era's influence on French politics.

B) might not be aware of the Medici family’s prominence in Italy.

C) risk downplaying the breadth of Catherine’s impact on French culture.

D) often focus on the controversies surrounding her reign.

www.strategictestprep.com ♟️ 1-833-300-PREP (7737) ♟️[email protected] 183


18. The role of honeybees in pollination is well-documented and essential for many
crops. However, recent studies have highlighted the importance of wild bees and
other insects in pollination. Research by Dr. Lara Westwood demonstrated that in
certain habitats, the absence of honeybees led to an increase in pollination by wild
bees. This observation implies that ______

Choices:

A) wild bees are more effective pollinators than honeybees.

B) honeybees may sometimes outcompete or interfere with the pollination habits of


wild bees.

C) the absence of wild bees would lead to a decrease in the pollination rate by
honeybees.

D) wild bees and honeybees share the exact same habitats and pollinate the same
types of flowers.

19. Ancient scrolls discovered in the secluded chambers of the Luristan caves offer
a window into the daily lives and beliefs of a civilization that once thrived in the
region. Written in Old Luri, a language that predates most known languages in the
area, these scrolls give detailed accounts of religious ceremonies and social
customs. Interestingly, some scrolls mention a celestial event that aligns with
records from other neighboring ancient civilizations. Yet, a few of these scrolls also
allude to certain rituals that share uncanny similarities with practices from
civilizations situated thousands of miles away. This raises the possibility that ______

Which choice most logically completes the text?

A) the ancient Luristan civilization had some form of contact or exchange with distant
civilizations.

B) the Luristan caves were inhabited by multiple civilizations over different periods.

C) the celestial event mentioned in the scrolls was witnessed by civilizations across the
globe.

www.strategictestprep.com ♟️ 1-833-300-PREP (7737) ♟️[email protected] 184


D) Old Luri was a language that was spoken not just in Luristan but also in distant
lands.

20. In a recent study on the reading comprehension skills of elementary school


children, researchers failed to account for the prior exposure of students to the
content of the passages they were given. Some students read familiar tales, such as
Cinderella or The Boy Who Cried Wolf, while others were given lesser-known folk
tales from various cultures. Consequently, the findings of the study _______

Which choice most logically completes the text?

A) might indicate disparities in comprehension abilities among the students when in


reality, familiarity with the content could be a confounding variable.

B) provide deep insights into the cognitive reading abilities of students exposed to
well-known tales as opposed to unfamiliar ones.

C) cannot be applied to understand the comprehension abilities of students in higher


grade levels.

D) suggest that students are better equipped to understand stories from their own
cultural background than those from other cultures.

21. In the early twentieth century, many Western musicians began incorporating
traditional African rhythms into their compositions, even though a significant
number of them had never traveled to Africa. Given that the complex nature of
these rhythms makes it unlikely for someone to spontaneously reproduce them
without prior exposure, this trend implies that _______

Which choice most logically completes the text?

A) these Western musicians learned African rhythms from intermediaries who had
direct exposure to traditional African music.

B) Western music of the early twentieth century was evolving to be more rhythmically
complex, making the inclusion of African rhythms a natural progression.

C) African rhythms were not as influential on Western music as previously believed.

www.strategictestprep.com ♟️ 1-833-300-PREP (7737) ♟️[email protected] 185


D) many of these Western musicians likely had a natural affinity for African rhythms,
leading them to reproduce these sounds intuitively.

22. Recent analyses of artworks from the ancient city of Dariel, located in
modern-day Turkey, show the prominent use of a specific shade of blue pigment,
which historians know wasn't synthesized locally until the 5th century CE.
However, comprehensive records from neighboring regions indicate Dariel's
prominence as an arts hub only started in the 6th century CE. If both the pigment
analysis and the historical records are accurate, it would suggest that _______

Which choice most logically completes the text?

A) the artists of Dariel were ahead of their time, innovating in pigment synthesis before
the rest of the region.

B) Dariel's artworks with the blue pigment were influenced or imported from
neighboring regions where the pigment was available earlier.

C) Dariel started synthesizing the blue pigment after recognizing its popularity in
neighboring cities during the 6th century CE.

D) the ancient records pertaining to Dariel’s prominence are based solely on the city's
innovations in pigment synthesis.

23. Researchers led by Marina S. Santos studied the adaptability of sea turtles to
changing ocean temperatures. They focused on the Green sea turtle (Chelonia
mydas) and investigated its behavior at various water temperatures. By comparing
the activities of the Green sea turtle in temperatures of 20° Celsius and 25° Celsius,
the researchers observed that the turtles' feeding rate increased significantly at the
higher temperature. To understand this response better, they replaced the native
digestive enzymes in some turtles with enzymes from another species of turtle that
does not exhibit increased feeding at higher temperatures. The modified Green sea
turtles showed no change in feeding rate at 25° Celsius, leading the researchers to
believe that _______

Which choice most logically completes the text?

A) Green sea turtles' native enzymes are essential for their increased feeding rate at
higher temperatures.
www.strategictestprep.com ♟️ 1-833-300-PREP (7737) ♟️[email protected] 186
B) All sea turtles increase their feeding rate at higher temperatures.

C) Enzymes from other turtle species impair the feeding ability of the Green sea turtles.

D) Green sea turtles can only feed in water temperatures above 20° Celsius.

24. Following the establishment of the Clear Waters Act, several nations
committed to reducing their marine pollution levels. This act incentivized countries
to develop and implement sustainable fishing practices. However, the Act does not
mandate the monitoring of practices once they're in place. Consequently,
environmental groups are worried that the Clear Waters Act might inadvertently
lead to ______

Which choice most logically completes the text?

A) an increase in marine species diversity due to sustainable fishing.

B) nations ignoring other sources of marine pollution aside from fishing.

C) the rise of new sustainable fishing technologies.

D) countries implementing sustainable practices without consistent adherence or


monitoring.

25. The African baobab tree (Adansonia digitata) is renowned for its longevity and
immense water storage capacity. Strikingly, certain varieties of this tree are also
found in parts of Australia. Ancient tribal records from Indigenous Australians
indicate that they had been utilizing the baobab tree for its water storage long
before maritime trade routes were established with Africa. Dr. Linda Warren and
her team examined the genetic makeup of the Australian and African baobabs and
discovered that the Australian variety diverged from its African counterparts
around 200,000 years ago. Given that the oldest confirmed human artifacts in
Australia date back to only 65,000 years ago, this information suggests that ______

Which choice most logically completes the text?

A) the baobab tree's presence in Australia is due to Indigenous Australians introducing


it from Africa.

www.strategictestprep.com ♟️ 1-833-300-PREP (7737) ♟️[email protected] 187


B) the baobab tree naturally spread to Australia without direct human involvement.

C) the African baobab varieties are more genetically pure than their Australian
counterparts.

D) Indigenous Australians have selectively bred the baobab tree to enhance its water
storage properties.

26. Numerous ancient Egyptian tomb paintings depict a male figure standing beside
a solar disk. Historically, this representation has been attributed to the god Ra, who
was associated with the sun in ancient Egyptian mythology. However, recent
research by Egyptologist Dr. Linda Fieldstone indicates that solar disks were not
solely associated with Ra, but also with other deities of the Egyptian pantheon. The
depiction of the male figure with a solar disk, thus, _______

Which choice most logically completes the text?

A) may not be an exclusive representation of the god Ra.

B) is evidence of the diminishing importance of Ra in Egyptian society.

C) confirms the dominance of Ra in ancient Egyptian art and culture.

D) is unlikely to have any significant meaning in ancient Egyptian mythology.

27. In the realm of classical music, Ludwig van Beethoven's symphonies are
cherished for their timeless appeal, often touching on universal human emotions
and experiences. His Symphony No. 9, for instance, is celebrated for its theme of
unity and brotherhood. In contrast, the operas of Richard Wagner, though
considered masterpieces, delve deep into Germanic myths and require some
knowledge of these tales for full appreciation. Therefore, _______

Which choice most logically completes the text?

A) Beethoven's Symphony No. 9 is more frequently performed worldwide than any of


Wagner's operas.

B) the narratives in Wagner's operas may pose a challenge for listeners unfamiliar with
Germanic myths.

www.strategictestprep.com ♟️ 1-833-300-PREP (7737) ♟️[email protected] 188


C) Beethoven's works are superior to Wagner's in terms of musical complexity and
innovation.

D) Wagner's operas focus exclusively on themes derived from Germanic mythology.

28. The Indus Valley Civilization, one of the world's oldest urban civilizations,
flourished around 2500 B.C.E. in what is now Pakistan and northwest India. The
civilization mysteriously declined around 1900 B.C.E. Among the various artifacts
unearthed from the region, seals with distinct geometric patterns were discovered.
When similar geometric patterns were found on pottery shards in Mesopotamia
dating back to 2000 B.C.E., researchers began to speculate on the connections
between the two regions. Given that these patterns were unique to the Indus Valley
before appearing in Mesopotamia, researchers posited that _______

Which choice most logically completes the text?

A) Mesopotamia and the Indus Valley were likely in some form of contact or trade
around 2000 B.C.E.

B) Mesopotamia borrowed these geometric patterns from other civilizations before


adapting them from the Indus Valley.

C) the Indus Valley Civilization was primarily influenced by Mesopotamian culture


during its peak.

D) geometric patterns on seals were the primary mode of communication between


ancient civilizations.

29. Studying the long-term effects of a new medical drug can be complicated due
to the vast number of external variables that might influence the results. To
understand the true impact of a drug, researchers need to compare the health
outcomes of patients who take the drug with those of patients who do not, but who
are otherwise in similar health conditions. Given the myriad health profiles of
patients and unpredictable ways they might interact with other medications,
researchers consequently _______

Which choice most logically completes the text?

A) should rely on patient self-reporting for the most accurate results.


www.strategictestprep.com ♟️ 1-833-300-PREP (7737) ♟️[email protected] 189
B) are forced to study only the immediate effects of the drug and not the long-term
consequences.

C) can select a group of patients who are markedly healthier than those taking the
drug.

D) face challenges in assembling a matching group that can act as an effective control
in their studies.

Answers - Extra Practice

1. A

The passage discusses how some historians believe that the male figure in the ancient
Egyptian murals is Osiris, due to his association with the Nile River. However, Dr. Emily
Thompson presents an alternative explanation that rivers and animals are common
symbols for prosperity in ancient Egyptian art. This suggests that the presence of
animals and a river near the figure does not necessarily mean the figure is Osiris,
making option A the most logical choice.

Why Other Choices Are Incorrect:

B) suggests that Osiris was often depicted with animals.

● The passage does not provide information to support the idea that Osiris was
frequently depicted with animals. The focus is on whether the presence of
animals and a river can definitively identify the figure as Osiris.

C) eliminates the possibility that the figure is Osiris.

● Dr. Thompson's argument does not eliminate the possibility that the figure is
Osiris; it merely suggests that there are alternative explanations for the
symbolism in the mural.

www.strategictestprep.com ♟️ 1-833-300-PREP (7737) ♟️[email protected] 190


D) would be difficult to account for if the figure is not Osiris.

● Dr. Thompson's argument actually provides an alternative explanation (that


rivers and animals symbolize prosperity), making it not difficult to account for
the scene if the figure is not Osiris.

2. A
The passage contrasts the general and timeless themes of social relationships and
challenges women face in Jane Austen's novels, exemplified by "Pride and Prejudice,"
with the more specific historical and social context required to fully understand
"Mansfield Park." This suggests that modern readers might find novels like "Pride and
Prejudice" more engaging because they don't require specialized knowledge of
19th-century England. Consequently, option A is the most logical choice to complete
the text.

Why Other Choices Are Incorrect:

B) some of Austen's novels are more relevant to today’s readers than contemporary

works.

● The passage does state that the themes in Austen's novels like "Pride and
Prejudice" are still relevant, but it does not make a comparison with
contemporary works.

C) "Pride and Prejudice" is the most accessible of all Austen's novels.

● The passage does use "Pride and Prejudice" as an example, but it does not say
that it is the most accessible among all of Austen's novels.

D) experts in 19th-century English history tend to prefer "Mansfield Park" to her other

works.

www.strategictestprep.com ♟️ 1-833-300-PREP (7737) ♟️[email protected] 191


● The passage doesn't mention the preferences of experts in 19th-century English
history; it focuses on the general accessibility of Austen’s works to today’s
readers.

3. B
The passage discusses the decline of the Indus Valley Civilization around 1900 B.C.E.
and mentions that similarities in pottery manufacturing techniques between
Mohenjo-Daro and a settlement in eastern Iran became prominent only after 1950
B.C.E. This suggests that there was likely a movement of people—and their pottery
techniques—from the Indus Valley to eastern Iran around the time of the decline, which
is directly supported by option B.

Why Other Choices Are Incorrect:

A) the decline of the Indus Valley Civilization was primarily due to internal conflicts.

● The passage doesn't discuss the reasons for the decline of the Indus Valley
Civilization.

C) residents of eastern Iran primarily used metal utensils and did not make pottery

before 1950 B.C.E.

● While it's implied that the pottery similarities appeared after 1950 B.C.E.,
there's no evidence to support the claim that residents in eastern Iran did not
make pottery before that time.

D) the Indus Valley Civilization likely adopted pottery manufacturing techniques from

civilizations in eastern Iran.

● The passage discusses similarities in pottery becoming prominent only after


1950 B.C.E., suggesting a movement from the Indus Valley to eastern Iran rather
than the adoption of techniques from eastern Iran.

www.strategictestprep.com ♟️ 1-833-300-PREP (7737) ♟️[email protected] 192


4. B
The passage outlines the ethical constraints in selecting a placebo group when an
effective treatment already exists for the condition being studied. This makes it difficult
for researchers to find a group that is otherwise similar to those taking the medication
but isn't taking the medication themselves. Therefore, option B most logically
completes the text, stating that researchers will find it difficult to identify an
appropriate placebo group under these conditions.

Why Other Choices Are Incorrect:

A) find it easy to create a placebo group consisting of individuals who are already on
existing medications.

● The passage states that it's ethically problematic to assign patients to a placebo
group if an effective treatment exists, making it unlikely that it would be "easy"
to create such a group.

C) can only conduct studies on diseases or conditions for which no effective treatments
exist.

● While it would be easier to assign a placebo group for conditions without


effective treatments, the passage does not state that researchers can only
conduct studies in such cases.

D) should select a placebo group of people who differ from the medication group in
several significant ways.

● This contradicts the requirement that the placebo group should be "otherwise
similar" to the medication group, as stated in the passage.

5. C
The passage discusses the Blue Ocean Accord, which aims to regulate deep-sea
mining to protect marine biodiversity. However, a flaw is pointed out: the accord allows
companies to keep their environmental impact assessments confidential. Given this
information, the most logical unintended effect would be that the confidentiality could
prevent independent experts from verifying the ecological impact of the mining
activities, as stated in option C.

www.strategictestprep.com ♟️ 1-833-300-PREP (7737) ♟️[email protected] 193


Why Other Choices Are Incorrect:

A) reducing the operational costs for companies engaged in deep-sea mining.

● The passage doesn't suggest that the accord has an impact on operational costs
for companies.

B) limiting the types of technology that can be used in deep-sea mining.

● The passage doesn't indicate that the accord limits the types of technology that
can be used for mining.

D) discouraging companies from exploring new, less harmful mining techniques.

● The focus of the accord is on regulating mining to protect biodiversity, not on


encouraging or discouraging particular mining techniques. Therefore, this option
doesn't logically follow from the passage.

6. C
The passage states that Asian varieties of corn diverged from Central American ones
around 70,000 years ago, but humans have been present in Asia for much longer. This
temporal discrepancy suggests that humans could not have been responsible for
introducing corn to Asia, making option C the most logical conclusion.

Why Other Choices Are Incorrect:

A) the cultivation of corn in Asia likely predates its cultivation in Central America.

● The text states that corn is native to Central America, making it unlikely that its
cultivation in Asia predates its cultivation in Central America.

B) Asian populations likely acquired corn from Central American populations through
ancient sea voyages.

www.strategictestprep.com ♟️ 1-833-300-PREP (7737) ♟️[email protected] 194


● The DNA analysis suggests that the Asian varieties diverged from the Central
American ones around 70,000 years ago, which contradicts the idea of
acquisition through ancient sea voyages.

D) Asian corn varieties likely descend from a single Central American variety that was
domesticated, not wild.

● The passage does not provide information to support the claim that the Asian
varieties descend from a single, domesticated Central American variety.

7. A
The passage indicates that the study didn't control for the varying complexity of the
puzzles when evaluating the problem-solving abilities of the parrots. This could lead to
erroneous conclusions, making it seem like parrots given simpler puzzles have better
problem-solving abilities, while those given complex puzzles appear to have worse
abilities. This means that any observed differences in problem-solving ability might be
due to the varying complexity of the tasks rather than to actual differences in cognitive
skills among the parrots. Therefore, option A is the most logical conclusion.

Why Other Choices Are Incorrect:

B) are useful for identifying puzzles that the parrots can solve but not for identifying
puzzles they find challenging.

● The passage doesn't suggest that the results are useful for identifying either
type of puzzle. It focuses on the problem with comparing problem-solving
abilities based on puzzles of different complexities.

C) should not be taken as indicative of the problem-solving abilities of any bird species
other than Psittacus erithacus.

● The focus of the passage is on the study's methodology and how it affects the
interpretation of the African grey parrots' abilities, not on the generalizability to
other bird species.

D) reveal more about the parrots' problem-solving abilities when engaging with
human-made objects than when solving natural problems.
www.strategictestprep.com ♟️ 1-833-300-PREP (7737) ♟️[email protected] 195
● The passage doesn't compare human-made objects to natural problems; it
focuses on the different complexities of the puzzles used in the study.

8. B
The study found that chameleon hatchlings, despite being primarily solitary animals
with limited social interactions, also display the eye-tracking behavior seen in social
mammals. This suggests that the behavior may not be strictly related to socialization
or communal living. Therefore, option B most logically completes the text by
emphasizing the need for caution in attributing this behavior solely to social factors.

Why Other Choices Are Incorrect:

A) eye-tracking behavior in mammals is a learned trait influenced by the social


environment.

● The study actually shows that even in a non-social species (chameleons), the
behavior is present, indicating that it might not be a learned trait influenced by
socialization.

C) the behavior of tracking moving objects with their eyes will likely diminish as
chameleons mature.

● The text does not provide information on how this behavior changes as the
chameleons mature, making this option speculative and unsupported.

D) this tendency is a universal trait among all vertebrates, regardless of their lifestyle.

● While the study extends our knowledge to include a certain species of reptile, it
does not provide enough evidence to conclude that this is a universal trait
among all vertebrates.

9. B

www.strategictestprep.com ♟️ 1-833-300-PREP (7737) ♟️[email protected] 196


The passage states that "The Book of Healing" is a comprehensive scientific and
philosophical encyclopedia written by Persian polymath Avicenna. It covers various
scientific fields and serves as a record of the knowledge available at that time.
However, the passage also notes that some chapters contain concepts borrowed from
Greek philosophers like Aristotle and Galen. The logical conclusion, then, is that while
Avicenna was a Persian scholar, he was influenced by Greek thought when compiling
his work. Therefore, option B most logically completes the text.

Why Other Choices Are Incorrect:

A) "The Book of Healing" is essentially a translation of Greek scientific works into


Persian.

● The passage notes that the book is a comprehensive encyclopedia covering a


range of subjects, suggesting that it is not merely a translation of Greek works.

C) the book should be credited to Greek philosophers as it mainly contains their ideas
and concepts.

● This statement is too strong; the passage only says that some chapters seem to
borrow from Greek philosophers, not that the entire book consists mainly of
Greek ideas.

D) Avicenna likely had limited knowledge of Persian scientific traditions when


compiling "The Book of Healing."

● The passage does not provide information to suggest that Avicenna had limited
knowledge of Persian scientific traditions. He was a Persian polymath, and the
book serves as a record of the knowledge of that time, which suggests that he
was well-versed in various fields, likely including Persian traditions.

10. C

● The text suggests that computational models in astrophysics often rely on


simplified assumptions, implying that these models could be flawed or
incomplete.

www.strategictestprep.com ♟️ 1-833-300-PREP (7737) ♟️[email protected] 197


● Dr. Sara Ruiz argues for the importance of recognizing this limitation and
indicates that for the models to be "truly robust," a particular approach should
be taken.

Option C is correct because it addresses this concern directly. Testing the models
against a variety of scenarios, especially those that challenge the assumptions, would
provide a more robust and reliable result, aligning with Dr. Ruiz's argument.

Why the other choices are incorrect:

● A) suggests ignoring the limitations, which contradicts Dr. Ruiz's argument that
acknowledging and dealing with these limitations is crucial.

● B) focuses only on refining algorithms, but the text suggests that the issue lies
in the simplified assumptions, not necessarily the algorithms themselves.

● D) suggests limiting the use of models to directly observable phenomena, which


ignores the initial point that computational models are used precisely because
some phenomena cannot be directly observed.

11. C

The statement immediately before the blank indicates that David Anthony was on
target, so the blank must refer to evidence supporting his theory. What was Anthony's
theory? That Inda-European languages were spread by the Yamnaya, horse-herding
nomads who lived on the Eurasian steppes more than 5,000 years ago. Note that to
identify this information, you must essentially back up to the beginning of the
passage-the phrase this possibility in the last (second) sentence refers to an idea
described in the previous (first) sentence, and there is no way to make sense out of the
passage without knowing what that idea is. The correct answer must support it with
information indicating that the Yamnaya moved throughout the regions, plural, where
Inda-European languages became established. (A) focuses on Germany, which is only
one area, so it can be eliminated. (B) is off-topic-the Yamnaya's genetic roots are
irrelevant to the argument. (D) does not fit because the correct answer must involve
the Yamnaya's migration and its eventual effects. Even though this option mentions the
Yamnaya's language, it does not mention how its influence was spread. Only (C)
contains that information: if "the Yamnaya's genes appeared throughout Europe and

www.strategictestprep.com ♟️ 1-833-300-PREP (7737) ♟️[email protected] 198


Asia," these people must have been present in many regions, hence the spread of their
language.

12. D

The first two examples in the list describe situations in which things run backwards, so
logically, the final item in the list must describe a normal occurrence that is reversed.
Objects interacting unpredictably; planets remaining in motion; and particles of
different weights moving at varying speeds are not examples of a phenomenon
running backwards. Only apples rising back to a tree, rather than falling from one,
illustrates the required idea. (D) is thus correct.

13. Answer: B) the evolution of the cheetah's speed was not solely in response to the
agility of its prey.

Explanation:

The passage presents a theory from some zoologists that the evolution of the
cheetah's speed might have been a response to increased agility of its prey. Yet, it
counters this idea by pointing out that fossil records do not support this theory, as
there was no noticeable increase in the agility of antelope or gazelles when cheetahs
evolved their speed.

A) "cheetahs evolved their speed to escape from predators, not to chase prey" - This
introduces a new theory not presented or alluded to in the passage.

B) "the evolution of the cheetah's speed was not solely in response to the agility of its
prey" - This choice logically follows from the information that there's no fossil evidence
of increased agility in the cheetah's prey during the time the cheetah evolved its speed.

C) "antelope and gazelles have always been slower than the cheetah" - This might be
true, but the passage is discussing agility, not speed, making this option not directly
relevant.

D) "the agility of prey is the sole factor determining a predator's speed" - This is
contradictory to the main argument of the passage, which challenges this exact notion
regarding cheetahs.

14. Answer: C) enhance the credibility and depth of their arguments.


www.strategictestprep.com ♟️ 1-833-300-PREP (7737) ♟️[email protected] 199
Explanation:

The passage highlights the importance of addressing counterarguments in effective


academic writing. It implies that by doing so, authors are not merely listing supportive
research but also addressing and potentially countering opposing views.

A) "can make their essays lengthier and more detailed" - While incorporating more
research might make essays longer, the primary aim of considering counterarguments
isn't necessarily length or detail.

B) "ensure that their arguments remain uncontested" - This is inaccurate because the
mere presence of counterarguments means there is some contestation. The point is to
address and potentially counter these opposing views.

C) "enhance the credibility and depth of their arguments" - This is consistent with the
idea that addressing counterarguments can make an author's stance more robust and
more thoroughly examined.

D) "show readers that they are well-read in their field" - While addressing a range of
research might suggest this, the primary emphasis in the passage is on the strength
and thoroughness of the argument, not on showcasing reading breadth.

15. Answer: A) are universally recognized in the academic realm.

Explanation:

The passage highlights the fact that many faculty members at top-tier universities
around the world have affiliations with Ivy League institutions, suggesting a global
recognition of the quality of these institutions. Given this context, the skills acquired
from Ivy League institutions are likely to be broadly recognized in academia.

A) "are universally recognized in the academic realm." - This option directly aligns with
the idea that faculty members at top-tier global universities (outside the Ivy League)
value the training received at Ivy League institutions. The phrase "universally
recognized" suggests a global appreciation, which fits the context.

B) "lead to lucrative offers from corporate sectors." - While individuals from Ivy League
institutions might receive lucrative job offers, the passage specifically discusses their
representation in academia, not the corporate world.
www.strategictestprep.com ♟️ 1-833-300-PREP (7737) ♟️[email protected] 200
C) "are specific only to the American education system." - This choice is counter to the
information provided. If the skills were specific only to the American system, they
wouldn't be as valued in top-tier universities around the world.

D) "discourage individuals from seeking opportunities outside academia." - The


passage does not suggest that Ivy League affiliations discourage other opportunities; it
simply notes a significant representation in global academia.

16. Answer: D) these mimicking non-toxic amphibians benefit from a deceptive


warning signal without bearing the metabolic cost of producing toxins.

Explanation:

The passage presents the concept that certain colors and patterns in amphibians serve
as a warning to predators about the presence of toxins. Dr. Lucy Green's observation
that non-toxic amphibians mimic these warning patterns suggests they are capitalizing
on the avoidance behavior of predators without actually producing toxins.

A) "non-toxic amphibians have evolved to develop toxins but haven't been successful."
- This is not suggested by the passage. The mimicry of the patterns doesn't indicate an
unsuccessful attempt to produce toxins.

B) "the colors on the skin of amphibians are a result of their dietary habits." - This is
not addressed in the passage, which discusses the relationship between color patterns
and defense mechanisms, not dietary habits.

C) "predators often make mistakes in distinguishing between toxic and non-toxic


amphibians." - This might be inferred, but it isn't the primary point the observation is
making.

D) "these mimicking non-toxic amphibians benefit from a deceptive warning signal


without bearing the metabolic cost of producing toxins." - This is the most logical
conclusion. If non-toxic amphibians are mimicking the warning colors of toxic
amphibians, they are likely gaining the protective benefits without the associated costs.

17. Answer: C) risk downplaying the breadth of Catherine’s impact on French culture.

Explanation:

www.strategictestprep.com ♟️ 1-833-300-PREP (7737) ♟️[email protected] 201


The passage emphasizes Catherine de' Medici's lesser-known role as a patron of the
arts, suggesting she had a wider influence than just in politics. The logical completion
would be a statement underscoring her impact in another domain, such as culture or
arts.

A) "may underestimate the Renaissance era's influence on French politics." - This


option shifts the focus to the Renaissance era's impact on politics rather than
Catherine's influence in areas beyond politics.

B) "might not be aware of the Medici family’s prominence in Italy." - This distracts from
the main point by introducing the broader Medici family’s prominence in Italy, which is
not the main focus of the passage.

C) "risk downplaying the breadth of Catherine’s impact on French culture." - This


directly aligns with the passage's point: Catherine's influence extended beyond politics
to the cultural sphere, which included her patronage of the arts.

D) "often focus on the controversies surrounding her reign." - While this could be a
possible result of focusing solely on her political endeavors, the passage doesn't allude
to any controversies, making this choice less directly related to the passage's main
point.

18. Answer: B) honeybees may sometimes outcompete or interfere with the pollination
habits of wild bees.

Explanation:

The passage indicates that when honeybees are absent, pollination by wild bees
increases. This suggests a possible competition or interference when honeybees are
present.

A) "wild bees are more effective pollinators than honeybees." - The passage doesn't
specifically state that wild bees are more effective, only that their pollination increases
in the absence of honeybees.

B) "honeybees may sometimes outcompete or interfere with the pollination habits of


wild bees." - This choice is supported by the observation that in the absence of
honeybees, wild bee pollination increases. It implies a relationship where honeybees
may outcompete or otherwise inhibit wild bees when they are present.
www.strategictestprep.com ♟️ 1-833-300-PREP (7737) ♟️[email protected] 202
C) "the absence of wild bees would lead to a decrease in the pollination rate by
honeybees." - This is not supported by the passage. The research discussed only what
happens in the absence of honeybees, not the other way around.

D) "wild bees and honeybees share the exact same habitats and pollinate the same
types of flowers." - While the passage does discuss both bees' roles in pollination, it
doesn't make a claim about them sharing the exact same habitats or preferences.

19. Answer: A) the ancient Luristan civilization had some form of contact or exchange
with distant civilizations.

Explanation:

The passage describes ancient scrolls found in the Luristan caves that give insights
into the customs and beliefs of an ancient civilization. The scrolls not only mention a
celestial event that other neighboring civilizations recorded but also reference rituals
that resemble practices from faraway civilizations.

A) "the ancient Luristan civilization had some form of contact or exchange with distant
civilizations." - This option logically fits the context since the scrolls mention rituals
that are similar to those of civilizations located far away, suggesting there might have
been some form of interaction or exchange.

B) "the Luristan caves were inhabited by multiple civilizations over different periods." -
The passage only speaks about one civilization's scrolls being found in the Luristan
caves. There's no evidence provided that multiple civilizations inhabited these caves.

C) "the celestial event mentioned in the scrolls was witnessed by civilizations across
the globe." - While the celestial event is mentioned in relation to other neighboring
civilizations, there's no indication that it was witnessed globally.

D) "Old Luri was a language that was spoken not just in Luristan but also in distant
lands." - The passage states that Old Luri predates most known languages in the area,
but there's no evidence suggesting it was spoken in distant lands.

20. Answer: A) might indicate disparities in comprehension abilities among the


students when in reality, familiarity with the content could be a confounding variable.

Explanation:
www.strategictestprep.com ♟️ 1-833-300-PREP (7737) ♟️[email protected] 203
The main issue presented in the passage is that the study did not control for the
familiarity of students with the content of the reading passages.

A) "might indicate disparities in comprehension abilities among the students when in


reality, familiarity with the content could be a confounding variable." - This choice is
correct. If students are more familiar with certain stories, they might find them easier to
understand, leading to a skewed perception of their reading comprehension skills.

B) "provide deep insights into the cognitive reading abilities of students exposed to
well-known tales as opposed to unfamiliar ones." - This choice assumes that the study
aimed to compare familiar tales with unfamiliar ones, which is not what the passage
stated.

C) "cannot be applied to understand the comprehension abilities of students in higher


grade levels." - This might be true, but the main problem highlighted in the passage
relates to the familiarity of content, not the grade level of the students.

D) "suggest that students are better equipped to understand stories from their own
cultural background than those from other cultures." - This makes an assumption that
is not supported by the information given in the passage. The primary issue was the
familiarity with the content, not necessarily cultural familiarity.

21. Answer: A) these Western musicians learned African rhythms from intermediaries
who had direct exposure to traditional African music.

Explanation:

The passage mentions that many Western musicians started using traditional African
rhythms in their compositions, even though they hadn't been to Africa. The passage
also notes that these rhythms are too intricate for someone to spontaneously
reproduce without some exposure.

A) "these Western musicians learned African rhythms from intermediaries who had
direct exposure to traditional African music." - This choice aligns with the provided
information. If the musicians hadn't been to Africa but were using the rhythms, they
probably learned them from someone who had.

B) "Western music of the early twentieth century was evolving to be more rhythmically
complex, making the inclusion of African rhythms a natural progression." - This
www.strategictestprep.com ♟️ 1-833-300-PREP (7737) ♟️[email protected] 204
statement is speculative and doesn't necessarily explain how Western musicians came
to know these specific African rhythms.

C) "African rhythms were not as influential on Western music as previously believed." -


This choice contradicts the passage which clearly states that many Western musicians
began incorporating African rhythms.

D) "many of these Western musicians likely had a natural affinity for African rhythms,
leading them to reproduce these sounds intuitively." - The passage points out the
complexity of African rhythms, suggesting that it's improbable for musicians to
spontaneously reproduce them without prior exposure.

22. Answer: B) Dariel's artworks with the blue pigment were influenced or imported
from neighboring regions where the pigment was available earlier.

Explanation:

The central puzzle here is how Dariel artworks from before the 6th century could use a
blue pigment that wasn't synthesized locally until the 5th century, especially given that
Dariel’s prominence in the arts began in the 6th century.

A) "the artists of Dariel were ahead of their time, innovating in pigment synthesis
before the rest of the region." - This doesn't reconcile the discrepancy. The pigment
was synthesized in the 5th century, but the city's prominence in the arts started in the
6th century. Being ahead of their time doesn't clarify this gap.

B) "Dariel's artworks with the blue pigment were influenced or imported from
neighboring regions where the pigment was available earlier." - This option provides a
logical solution: if the pigment was synthesized elsewhere earlier, Dariel could have
obtained it through trade or influence, even before its arts scene became prominent.

C) "Dariel started synthesizing the blue pigment after recognizing its popularity in
neighboring cities during the 6th century CE." - This conflicts with the information
given, as the pigment was synthesized in the 5th century.

D) "the ancient records pertaining to Dariel’s prominence are based solely on the city's
innovations in pigment synthesis." - This doesn't address the key discrepancy about
the timing of the pigment's synthesis versus the city's rise to prominence in the arts.

www.strategictestprep.com ♟️ 1-833-300-PREP (7737) ♟️[email protected] 205


23. Answer: A) Green sea turtles' native enzymes are essential for their increased
feeding rate at higher temperatures.

Explanation:

A) "Green sea turtles' native enzymes are essential for their increased feeding rate at
higher temperatures." - Correct. The passage states that after the native enzymes were
replaced with those from another turtle species, the modified Green sea turtles did not
exhibit an increased feeding rate at 25° Celsius. This suggests that the native enzymes
are crucial for this behavior.

B) "All sea turtles increase their feeding rate at higher temperatures." - Incorrect. The
research was specifically on the Green sea turtle, and there's no information given
about all sea turtles.

C) "Enzymes from other turtle species impair the feeding ability of the Green sea
turtles." - Incorrect. The passage states that the modified turtles showed no change in
feeding rate, not an impairment.

D) "Green sea turtles can only feed in water temperatures above 20° Celsius." -
Incorrect. The research observed turtles' behavior at 20° Celsius and 25° Celsius, but
there's no evidence to suggest they can only feed above 20° Celsius.

24. Answer: D) countries implementing sustainable practices without consistent


adherence or monitoring.

Explanation:

A) "an increase in marine species diversity due to sustainable fishing." - Incorrect. The
concern of the environmental groups is about monitoring and adherence, not about the
positive outcomes of the act.

B) "nations ignoring other sources of marine pollution aside from fishing." - Incorrect.
The text focuses on the Act's approach to sustainable fishing practices and not on
other sources of pollution.

C) "the rise of new sustainable fishing technologies." - Incorrect. The primary concern
expressed is about the monitoring of practices, not the development of new
technologies.
www.strategictestprep.com ♟️ 1-833-300-PREP (7737) ♟️[email protected] 206
D) "countries implementing sustainable practices without consistent adherence or
monitoring." - Correct. Since the Act does not mandate the monitoring of the
sustainable practices, the logical concern is that countries might not consistently
adhere to these practices after their implementation.

25. Answer: B) the baobab tree naturally spread to Australia without direct human
involvement.

Explanation:

A) "the baobab tree's presence in Australia is due to Indigenous Australians


introducing it from Africa." - Incorrect. The genetic divergence occurred 200,000 years
ago, while humans only arrived in Australia around 65,000 years ago, making this
timeline implausible.

B) "the baobab tree naturally spread to Australia without direct human involvement." -
Correct. Since the Australian variety diverged from the African counterpart around
200,000 years ago and humans only reached Australia 65,000 years ago, it suggests
that the tree's presence in Australia was not due to human introduction.

C) "the African baobab varieties are more genetically pure than their Australian
counterparts." - Incorrect. The text only discusses the divergence of the two varieties
and doesn't address the genetic purity of either one.

D) "Indigenous Australians have selectively bred the baobab tree to enhance its water
storage properties." - Incorrect. The text doesn't provide any information about
selective breeding practices related to the baobab tree by Indigenous Australians.

26. Answer: A) may not be an exclusive representation of the god Ra.

Explanation:

A) "may not be an exclusive representation of the god Ra." - Correct. Since the solar
disk is associated with other deities besides Ra, the presence of a solar disk with a
male figure does not conclusively identify the figure as Ra.

B) "is evidence of the diminishing importance of Ra in Egyptian society." - Incorrect. The


passage does not provide any information about the changing importance of Ra in

www.strategictestprep.com ♟️ 1-833-300-PREP (7737) ♟️[email protected] 207


Egyptian society. The focus is solely on the association of the solar disk with various
deities.

C) "confirms the dominance of Ra in ancient Egyptian art and culture." - Incorrect. The
new research suggests that the solar disk's association is broader than just with Ra.
Therefore, the presence of a solar disk does not confirm Ra's dominance.

D) "is unlikely to have any significant meaning in ancient Egyptian mythology." -


Incorrect. The solar disk's depiction indicates its importance in Egyptian mythology,
given its association with various deities. The question is which deity, not whether it
has significance.

27. Answer: B) the narratives in Wagner's operas may pose a challenge for listeners
unfamiliar with Germanic myths.

Explanation:

A) "Beethoven's Symphony No. 9 is more frequently performed worldwide than any of


Wagner's operas." - Incorrect. The passage does not provide information about the
frequency of performances of either composer's works, so this cannot be inferred.

B) "the narratives in Wagner's operas may pose a challenge for listeners unfamiliar
with Germanic myths." - Correct. This is the logical conclusion based on the
information given. While Beethoven's Symphony No. 9 is celebrated for its universal
themes, Wagner's operas delve deep into Germanic myths, which suggests that
listeners unfamiliar with these tales might find them challenging.

C) "Beethoven's works are superior to Wagner's in terms of musical complexity and


innovation." - Incorrect. The passage does not make a judgment regarding the musical
complexity or innovation of either composer's works.

D) "Wagner's operas focus exclusively on themes derived from Germanic mythology." -


Incorrect. The passage states that Wagner's operas "delve deep into Germanic myths,"
but it doesn't specify that they focus exclusively on these themes.

28. Answer: A) Mesopotamia and the Indus Valley were likely in some form of contact
or trade around 2000 B.C.E.

Explanation:
www.strategictestprep.com ♟️ 1-833-300-PREP (7737) ♟️[email protected] 208
A) "Mesopotamia and the Indus Valley were likely in some form of contact or trade
around 2000 B.C.E." - Correct. Given that the geometric patterns were unique to the
Indus Valley and then later appeared in Mesopotamia, it is logical to infer that the two
regions had some form of contact or trade that led to the transfer of this design.

B) "Mesopotamia borrowed these geometric patterns from other civilizations before


adapting them from the Indus Valley." - Incorrect. The passage states that these
patterns were unique to the Indus Valley before appearing in Mesopotamia, which
contradicts the idea that Mesopotamia borrowed them from another civilization prior to
the Indus Valley.

C) "the Indus Valley Civilization was primarily influenced by Mesopotamian culture


during its peak." - Incorrect. The information provided suggests that Mesopotamia may
have been influenced by the Indus Valley (due to the appearance of the patterns), not
the other way around.

D) "geometric patterns on seals were the primary mode of communication between


ancient civilizations." - Incorrect. While the seals indicate some form of interaction or
influence, the passage does not suggest that they were the primary mode of
communication between these civilizations.

29. Answer: D) face challenges in assembling a matching group that can act as an
effective control in their studies.

Explanation:

A) "should rely on patient self-reporting for the most accurate results." - Incorrect. The
text does not mention anything about self-reporting as a method of gathering data.

B) "are forced to study only the immediate effects of the drug and not the long-term
consequences." - Incorrect. The main issue highlighted in the text is the challenge of
finding a suitable control group, not the duration of the study.

C) "can select a group of patients who are markedly healthier than those taking the
drug." - Incorrect. The text specifies the need for a control group that is similar in
health conditions, not one that is markedly healthier.

D) "face challenges in assembling a matching group that can act as an effective control
in their studies." - Correct. The text underscores the difficulty of finding a control group
www.strategictestprep.com ♟️ 1-833-300-PREP (7737) ♟️[email protected] 209
due to the many variables and interactions with other medications, hence this choice
aptly captures the essence of the challenge faced by researchers.

www.strategictestprep.com ♟️ 1-833-300-PREP (7737) ♟️[email protected] 210


VIII. Expression of Ideas
i. Transition Questions 🏃💨
Transition questions are the most common type of Writing question, with each test
containing around 8 to 10 such items (around 15-20% of the test).

Support Contrast Causation

EXCEPTIONS

Reinforcers:

www.strategictestprep.com ♟️ 1-833-300-PREP (7737) ♟️[email protected] 211


Strategies

Strategy #1: Read the sentence before the transition and the sentence the transition is
in to determine a relationship

Strategy #2: Categorize your answer choices then pick an answer that matches the
relationship

Strategy #3 (shortcut): Cross off answer choices from the same category
(because they function the same way and you cannot have two multiple choice
answers that are correct)

***The most important thing to understand is that wherever the transition word is
physically located in the sentence, it is linking to the previous sentence.

Case 1 Example:
Sentence 1. Transition, sentence 2. Sentence 3.

In this case, if the transition word is in the second sentence, it is related to the first
sentence (the sentence before it).

Case 2 Example:
Sentence 1. Sentence 2. Sentence 3, transition.

In this case, if the transition word is in the third sentence, it is related to the second
sentence (the sentence before it). Note that the transition word does not need to be
at the very beginning of the sentence. It can be at the middle or end of the sentence
as well.

www.strategictestprep.com ♟️ 1-833-300-PREP (7737) ♟️[email protected] 212


Examples

Example #1
Although T.S. Eliot devoted several years to writing "The Waste Land," it sold only
about 330 copies in the six months following its publication in 1922. _____Eliot was
forced to seek other sources of income. In the mid-1920s, he took a job as the director
of a new publishing firm called Faber & Faber.

Which choice completes the text with the most logical transition?

A) Alternatively,

B) Consequently,

C) In fact,

D) Moreover,

Example #2

Conditions in the interior of Antartica are inhospitable to many forms of life: sub-zero
temperatures, high winds, and extreme dryness make it impossible for most animals to
survive. _____ the Antarctic Peninsula and the surrounding islands have relatively mild
temperatures and liquid water, allowing many species to thrive there.

Which choice completes the text with the most logical transition?
A) Therefore,
B) For instance,
C) Indeed,
D) In contrast,

www.strategictestprep.com ♟️ 1-833-300-PREP (7737) ♟️[email protected] 213


Example #3
Despite the sub-zero temperatures, high winds, and extreme dryness that characterize
the Antarctic : interior, a small number of hardy species dwell there. ____ certain
mosses, lichens, and microscopic protozoa have adapted to the harsh : conditions and
are able to thrive.

Which choice completes the text with the most logical transition?

A) In addition,
B) Specifically,
C) Accordingly,
D) Nevertheless

Example #4
Healthy arctic marine mammals have a thick layer of fat beneath the skin. Tropical
marine mammals, _ _ have very few fat reserves in their bodies. As a result, many of
them dwell primarily in the warm waters that surround coral reefs.

Which choice completes the text with the most logical transition?

A) therefore,
B) likewise,
C) however,
D) for instance

www.strategictestprep.com ♟️ 1-833-300-PREP (7737) ♟️[email protected] 214


Example #5
Tropical marine mammals have few fat reserves in their bodies and dwell primarily in
the warm waters that surround coral reefs. In contrast, healthy arctic marine mammals
have a thick layer of fat beneath their skin, _._ _ they are able to tolerate much colder
waters.

Which choice completes the text with the most logical transition?
A) so
B) for
C) yet
D) whereas

Example Transition Question Answers

1. B

2. D

3. B

4. C

5. A

www.strategictestprep.com ♟️ 1-833-300-PREP (7737) ♟️[email protected] 215


Extra Practice

www.strategictestprep.com ♟️ 1-833-300-PREP (7737) ♟️[email protected] 216


www.strategictestprep.com ♟️ 1-833-300-PREP (7737) ♟️[email protected] 217
www.strategictestprep.com ♟️ 1-833-300-PREP (7737) ♟️[email protected] 218
www.strategictestprep.com ♟️ 1-833-300-PREP (7737) ♟️[email protected] 219
A) No change
B) Therefore,
C) However,
D) Thus,

www.strategictestprep.com ♟️ 1-833-300-PREP (7737) ♟️[email protected] 220


www.strategictestprep.com ♟️ 1-833-300-PREP (7737) ♟️[email protected] 221
www.strategictestprep.com ♟️ 1-833-300-PREP (7737) ♟️[email protected] 222
Extra Practice Answers
25. A
28. D
28. A
29. A
2. A
16. D
12. D
25. C
34. B
31. B
2. B

www.strategictestprep.com ♟️ 1-833-300-PREP (7737) ♟️[email protected] 223


Bonus Practice Questions
1. Many feared the invention of the atomic bomb due to its capabilities of mass
destruction. ________ physicist Edward Teller argued that the hydrogen bomb was
necessary to the defense of the United States and the cause of world peace, a claim
that was controversial at the time.

Which choice completes the text with the most logical transition?
A. Indeed,
B. Nevertheless,
C. Moreover,
D. Consequently,

2. In the early 1500s, the Spanish crown began sending envoys to the New World to
discover new sources of wealth. ______ these conquistadors, as they were known,
encountered all manner of strange plants, animals, and people during their
explorations.

Which choice completes the text with the most logical transition?
A. On the other hand,
B. However,
C. For instance,
D. Nevertheless,

3. In December 1941, Francis Murphy was living in what must have seemed like the
ideal city for a young artist: Florence. She was studying firsthand the unique
brushstrokes of Italy’s Renaissance masters and beginning to make a name for herself
as a painter. ______ Murphy longed to return to her childhood home of Ireland; only
there, she believed, could her art truly thrive.

Which choice completes the text with the most logical transition?
A. Still,
B. Therefore,
C. Indeed,
D. Furthermore,

www.strategictestprep.com ♟️ 1-833-300-PREP (7737) ♟️[email protected] 224


4. In the summer of 1952, the US government utilized public taxpayer dollars to
conduct a series of nuclear weapon tests in the Nevada desert. ______ the testing
program, codenamed Operation Upshot-Knothole, was surrounded by secrecy and
security, with the public kept largely in the dark about the nature and scale of the tests.

Which choice completes the text with the most logical transition?
A. However,
B. Subsequently,
C. Moreover,
D. Therefore,

5. In the early 20th century, the American economy was dominated by large
corporations. ______ this led to the rise of powerful labor unions, which sought to
protect the rights of workers in a system that many felt was unfairly tilted in favor of
the powerful few.

Which choice completes the text with the most logical transition?
A. In addition,
B. Nevertheless,
C. Meanwhile,
D. Consequently,

6. In the early 1950s, the US government began to worry that the Soviet Union was
developing a nuclear missile capability. ______ the US responded by launching a crash
program to develop its own intercontinental ballistic missiles, which would be able to
reach targets in the Soviet Union.
Which choice completes the text with the most logical transition?
A. Nevertheless,
B. Moreover,
C. Thus,
D. However,

www.strategictestprep.com ♟️ 1-833-300-PREP (7737) ♟️[email protected] 225


7. In the early twentieth century, the US government sought to protect the country’s
remaining wilderness by designating certain areas as national parks. ______ they
deemed certain wildlife species as "endangered" and prohibited hunters from killing
them.
Which choice completes the text with the most logical transition?
A. Still,
B. Moreover,
C. Therefore,
D. Nevertheless,

8. In the late nineteenth century, the textile industry in England was transformed by
the first automated looms, which produced finished cloth from raw material with
minimal human labor. ______ by the 1920s, the same type of weaving machine was
being used in mills all over the world, from Russia to India.
Which choice completes the text with the most logical transition?
A. Additionally,
B. For instance,
C. Moreover,
D. Eventually,

9. Even before the introduction of the 3D printer, it was possible to create objects using
computer-controlled machines, meaning the technology was actually two decades old
by the time the 3D printers first became available. ______ it wasn’t until businesses
and individuals began to acquire 3D printers in their own homes that the technology
saw a major, widespread uptake.
Which choice completes the text with the most logical transition?
A. However,
B. Still,
C. Therefore,
D. Similarly,

www.strategictestprep.com ♟️ 1-833-300-PREP (7737) ♟️[email protected] 226


10. In 2017, former Mississippi Governor Phil Bryant signed a law protecting
Confederate symbols—including monuments, flags, and place names—from removal.
______many civil rights groups protested the law, arguing that such symbols serve to
glorify a shameful period in US history and should be taken down.

Which choice completes the text with the most logical transition?
A. Eventually,
B. Therefore,
C. Likewise,
D. Consequently,

11. Few political figures in the history of the United States have so consistently
opposed the expansion of presidential power as Senator Charles Curtis. _____ Curtis
was explicit in his belief that the President should not possess more power than
Congress when he famously declared that the US Constitution “knows no distinction
between powers granted to a President” and those “granted to Congress.”

Which choice completes the text with the most logical transition?
A. Thus,
B. Presently,
C. However,
D. Indeed,

12. In a province known for its picturesque snow-covered peaks, local officials are
leading the charge to reduce carbon emissions and combat global climate change.
______ there is a shift away from carbon-based energy that has emphasized the
importance of finding alternative energy sources that can provide renewable energy
with minimal environmental impacts.
Which choice completes the text with the most logical transition?
A. Therefore,
B. Furthermore,
C. In fact,

www.strategictestprep.com ♟️ 1-833-300-PREP (7737) ♟️[email protected] 227


D. In spite of this,

13. Computer scientists in the 1950s initially viewed robotics as a futuristic, lofty goal.
______ the rise in digital computing more than a decade later vastly increased the
potential of robots by allowing them to be programmed to complete complex tasks.

Which choice completes the text with the most logical transition?
A. In effect,
B. In theory,
C. Nonetheless,
D. In fact,

14. The US Securities and Exchange Commission (SEC) enforces laws designed to
promote fairness and protect investors from fraud. ______ the SEC requires publicly
traded companies to disclose financial information to potential investors, so that these
investors can make informed decisions about their investments.

Which choice completes the text with the most logical transition?
A. In turn,
B. Ultimately,
C. However,
D. For instance,

15. The US government encouraged oil drilling in the Arctic National Wildlife Refuge
(ANWR) beginning in the 1980s. ______, no oil was ever extracted and the US
government has since (partially) reversed its position, re-designating much of the
ANWR as a protected wilderness area.

Which choice completes the text with the most logical transition?
A. Though,
B. Notably,

www.strategictestprep.com ♟️ 1-833-300-PREP (7737) ♟️[email protected] 228


C. Moreover,
D. Nevertheless,

16. In the 1920s, the self-described Soviet visionary Gustav Klutsis created design
works encouraging revolutionary zeal, heralding the dawn of a new social order.
______ Klutsis’s geometric style, as evidenced in his design collages, illuminated what
were to be the fundamental elements of the modern, proletarian experience.

Which choice completes the text with the most logical transition?
A. Significantly,
B. Moreover,
C. Nonetheless,
D. Meanwhile,

17. During the 1800s, hunting and habitat loss put grizzly bears on the endangered
species list. ______, conservation efforts such as the Endangered Species Act have
helped keep the wild grizzly bear population from dwindling further.

Which choice completes the text with the most logical transition?
A. Next,
B. Therefore,
C. Consequently,
D. Nevertheless,

18. A visioning exercise is a powerful tool for helping an individual or group to identify
and articulate their dreams for the future. ______ this framework provides structure and
guidance, allowing participants to concretely imagine the outcome of their plans,
instead of simply talking in generalities.

Which choice completes the text with the most logical transition?
A. Nonetheless,
B. Similarly,

www.strategictestprep.com ♟️ 1-833-300-PREP (7737) ♟️[email protected] 229


C. Therefore,
D. For example,

19. In 1785, French scientist Charles Coulomb developed a law detailing the force
between two stationary charged objects. He found that the force between the
charges was directly proportional to the product of their charges and inversely
proportional to the square of the distance between their centers. _______ this
observation led to the development of Coulomb's Law, a foundational concept in
electromagnetism.

Which choice completes the text so that it conforms to the conventions of Standard
English?

A) Consequently;

B) However,

C) Though,

D) And,

20. In tropical regions, many plants have evolved broad, flat leaves to maximize
photosynthesis. These expansive surfaces, however, also expose the plants to more
pests and diseases. _______ many of these plants release natural insecticides or
develop tough textures to deter herbivores and pathogens.

Which choice completes the text so that it conforms to the conventions of Standard
English?

A) Resulting,

B) Moreover,

C) Consequently,

D) Although,

21. The Shinkansen, or bullet train, revolutionized rail travel in Japan when it was
introduced in the 1960s. Not only did it reduce the travel time between Tokyo and

www.strategictestprep.com ♟️ 1-833-300-PREP (7737) ♟️[email protected] 230


Osaka by hours, but it also set an impeccable standard for punctuality and safety in
train services worldwide. _______ prior to the Shinkansen’s debut, most
long-distance train journeys in Japan were considerably slower and less predictable
in terms of arrival times.

Which choice completes the text so that it conforms to the conventions of Standard
English?

A) Interestingly enough

B) Coincidentally,

C) Indeed,

D) In contrast,

22. For many, the thought of swimming with sharks evokes fear and trepidation.
_______ when the species in question is the docile whale shark, many tourists
eagerly pay for the experience of diving alongside these gentle giants.

Which choice completes the text so that it conforms to the conventions of Standard
English?

A) Conversely,

B) Similarly,

C) Indeed,

D) Moreover,

23. In the early 20th century, Marie Curie conducted groundbreaking research on
radioactivity. She faced numerous challenges, including limited resources and
skepticism from her peers. However, Curie was persistent. She collaborated with top
scientists of her time and refined her experimental techniques. _______ in 1903, she
became the first woman to win a Nobel Prize in Physics.

www.strategictestprep.com ♟️ 1-833-300-PREP (7737) ♟️[email protected] 231


Which choice completes the text with the most logical transition?

A) Conversely,
B) Ironically,
C) Subsequently,
D) Alternatively,

24. Mariana Trench, located in the western Pacific Ocean, is the deepest part of the
world's oceans. It reaches a depth of about 36,070 feet. This trench has been visited
very few times due to its extreme depths and the challenges associated with those
depths. _______ it's considered one of the least explored parts of Earth.

Which choice completes the text with the most logical transition?

A) Similarly,

B) Therefore,

C) Nevertheless,

D) On the contrary,

25. In 2020, marine biologist Dr. Lila Jenkins and her team discovered a coral reef
system off the coast of Madagascar that had remained hidden until then. The reef
system, covering more than 200 square kilometers, was found to be particularly
resilient to the warming ocean temperatures. _______ Dr. Jenkins believes that the
unique properties of this reef could offer insights into protecting other coral reefs
around the world.

Which choice completes the text with the most logical transition?

A) Incidentally,

B) Surprisingly,

C) As a result,

D) Conversely,

www.strategictestprep.com ♟️ 1-833-300-PREP (7737) ♟️[email protected] 232


26. For many years, the cheetah was believed to be the fastest land animal, reaching
speeds up to 70 miles per hour in short sprints. _______ in a 2022 study conducted by
wildlife biologist Dr. Lara Henley, the pronghorn antelope was recorded running short
distances at speeds slightly surpassing that of the cheetah. Which choice completes
the text with the most logical transition?

A) Simultaneously,

B) Additionally,

C) For instance,

D) Surprisingly,

27. While some people use synthetic pesticides to combat garden pests, these
chemicals can be harmful to beneficial insects and the environment. _______ many
gardeners have started using natural predators, like ladybugs and praying mantises, to
control unwanted insects without the use of chemicals.

Which choice completes the text with the most logical transition?

A) Obviously,

B) For instance,

C) Therefore,

D) Regardless,

28.

While many marine animals have lungs and need to surface for air, others can extract
oxygen directly from the water. Whales and dolphins, _______ , must come to the
surface to breathe.

Choices:

A) for example,

www.strategictestprep.com ♟️ 1-833-300-PREP (7737) ♟️[email protected] 233


B) on the other hand,

C) as a result,

D) in conclusion,

29. The Great Barrier Reef, stretching over 2,300 kilometers off the eastern coast of
Australia, is the world's largest coral reef system. This UNESCO World Heritage site is
composed of over 2,900 individual reefs and is home to an abundance of marine life.
_______ in recent decades, the reef has faced threats from climate change, including
coral bleaching events due to rising sea temperatures.

Which choice completes the text with the most logical transition?

A) In addition,

B) Surprisingly,

C) However,

D) Concurrently,

30. Nutritionist Emily Fields highlights two significant benefits of including spinach in
one's diet. Firstly, spinach is packed with iron, which plays a key role in transporting
oxygen throughout the body. _______ it is a rich source of vitamin K, essential for blood
clotting and bone health.

Which choice completes the text with the most logical transition?

A) Interestingly,

B) Consequently,

C) Moreover,

D) Secondly,

www.strategictestprep.com ♟️ 1-833-300-PREP (7737) ♟️[email protected] 234


31. Water is a limited resource, and in many parts of the world, access to clean water is
a significant challenge. ________, in areas with arid climates, innovative techniques like
fog catching are being employed to harvest moisture from the air.

Which choice completes the text with the most logical transition?

A) Nevertheless,

B) Conversely,

C) Consequently,

D) In spite of this,

32. Renowned architect Tania Vasquez is celebrated for her ability to integrate green
spaces into urban designs. _______ for her project in downtown Manhattan, she
transformed a derelict railway into a lush, elevated park known as "The Skyway
Garden."

Choices:

A) Similarly,

B) On the contrary,

C) As a result,

D) For example,

33. In his award-winning novel "The Lighthouse," Thomas Hart delves into the psyche
of a sailor named William, who is haunted by his past. William retreats to a seaside
town to find solace and work on his memoir. ______ Joanna, a journalist from New York,
comes to the same town to cover a local festival, only to cross paths with William and
discover their intertwined histories.

Which choice completes the text with the most logical transition?

A) Simultaneously,

www.strategictestprep.com ♟️ 1-833-300-PREP (7737) ♟️[email protected] 235


B) In comparison,

C) Ironically,

D) Therefore,

34. Marine life has developed numerous strategies to cope with the dangers of the
open ocean. The mimic octopus (Thaumoctopus mimicus), ______, can alter its shape
and color to imitate more than fifteen different species, including flounders and lionfish,
in order to deter potential predators.

Options:

A) on the other hand,

B) therefore,

C) for instance,

D) in conclusion,

35. In 1969, the Cuyahoga River in Ohio famously caught fire due to high levels of
pollution. This event shocked the nation and increased awareness about the dire state
of many of America's waterways. ______, it played a significant role in the passing of
the Clean Water Act of 1972, which sought to reduce pollutants in U.S. waters.

A) On the contrary,

B) Surprisingly,

C) For instance,

D) Consequently,

36. In her documentary series "Under the Ocean's Surface," marine biologist Dr. Sylvia
Thompson dives deep into the coral reefs of the Caribbean. She captures the
breathtaking colors and varied species that inhabit these marine ecosystems. ______,
she ventures into the mangroves, showcasing the intricate roots and diverse fauna that
make these habitats vital for coastal protection.

www.strategictestprep.com ♟️ 1-833-300-PREP (7737) ♟️[email protected] 236


Which choice completes the text with the most logical transition?

A) Alternatively,

B) Meanwhile,

C) For instance,

D) In another episode,

37. While jazz and classical music have traditionally been seen as distinct genres,
some musicians have ventured to bridge the gap between them. Wynton Marsalis,
______, has composed orchestral works that integrate jazz rhythms and improvisational
elements with classical structures.

Which choice completes the text with the most logical transition?

A) incidentally,

B) on the other hand,

C) moreover,

D) for instance,

38. Machu Picchu, the ancient Incan citadel located in the Andes Mountains, is a
renowned archaeological wonder known for its intricate stonework. The Pyramids of
Giza, _______ are architectural marvels from ancient Egypt, showcasing precise
alignment with celestial bodies and impressive construction techniques.

Which choice completes the text with the most logical transition?

A) interestingly,

B) as a result,

C) on the other hand,

D) similarly,

www.strategictestprep.com ♟️ 1-833-300-PREP (7737) ♟️[email protected] 237


39. Historically, small family-owned farms dominated the agricultural landscape.
These farms primarily focused on subsistence, growing just enough food to feed the
family with a little left over for selling or trading. _______ modern agriculture is marked
by industrial-sized farms producing vast amounts of crops for large-scale distribution.

Which choice completes the text with the most logical transition?

A) Consequently,

B) Similarly,

C) For instance,

D) However,

40. Certain plants, like the resurrection plant, have an extraordinary ability to survive in
arid conditions. When water is scarce, these plants enter a state of dormancy,
appearing completely dead to the naked eye. _______ once they receive even a small
amount of water, they "resurrect" and return to their green, lively state within hours.

Which choice completes the text with the most logical transition?

A) Alternatively,

B) Surprisingly,

C) However,

D) Remarkably,

41. In the mid-1800s, London was the bustling capital of the British Empire. It was a
hub for commerce, culture, and politics, attracting people from all over the world. Many
considered it the epitome of modern civilization with its advanced infrastructure and
cultural institutions. _______ Charles Dickens, in his novels, often portrayed the city's
grimy streets, poverty, and social inequality, highlighting a darker side of life in the
metropolis.

Which choice completes the text with the most logical transition?

www.strategictestprep.com ♟️ 1-833-300-PREP (7737) ♟️[email protected] 238


A) Moreover,

B) In fact,

C) Consequently,

D) Yet,

42. During the Renaissance, various European cities became centers of learning and
culture. Florence, for instance, was the cradle of Renaissance art and thought, housing
brilliant artists like Leonardo da Vinci and Michelangelo. _______ in England, the city of
Stratford-upon-Avon was the home of William Shakespeare, one of the most
significant playwrights of the period.

A) Moreover,

B) Consequently,

C) Similarly,

D) Therefore,

www.strategictestprep.com ♟️ 1-833-300-PREP (7737) ♟️[email protected] 239


Bonus Practice Answers
1.
Choice B is the best answer. “Nevertheless” logically signals a contrast from the prior
sentence. Although people feared the destruction of atomic bomb, Teller recognized
the benefits of using it (world peace, defending the US).

2.
Choice C is the best answer. “For instance” logically signals that the information in this
sentence about the conquistadors—that they encountered strange plants, animals, and
people—provides one example of what the Spanish envoys discovered during their
explorations in the New World.

3.
Choice A is the best answer. “Still” logically signals that the information about Murphy
in this sentence—that she longed to leave Florence and return to Ireland—contrasts
with what one would expect after reading about Murphy’s experiences in Florence in
the previous sentences.

4.
Choice A is the best answer. “However” logically signals that the claim in this
sentence—that the public was kept in the dark about the testing program—contrasts
with the previous information about using public money to fund the venture.

5.
Choice D is the best answer. “Consequently” logically signals that the information in
this sentence—that labor unions rose in response to the dominance of large
corporations—is a result or consequence of the previous information about the
economy.

6.
Choice C is the best answer. “Thus” logically signals that the information in this
sentence—that the US launched a crash program to develop its own intercontinental

www.strategictestprep.com ♟️ 1-833-300-PREP (7737) ♟️[email protected] 240


ballistic missiles—is a result or consequence of the previous information about the US
government’s worries about the Soviet Union’s nuclear capabilities.

7.
Choice B is the best answer. “Moreover” logically signals another supporting detail as
to how the US government is protecting the country's wilderness.

8.
Choice D is the best answer. “Eventually” logically signals that the claim in this
sentence—that the same type of weaving machine was being used in mills all over the
world by the 1920s—is a result or consequence of the information in the previous
sentence about the advent of automated looms at the end of the nineteenth century.

9.
Choice A is the best answer. “However” logically signals that the claim made in the
previous sentence—that computer-controlled machines were two decades old before
the advent of 3D printers but the technology saw its major uptake when businesses
and individuals began to acquire 3D printers for their own homes—contrasts with a
major, widespread uptake.

10.
Choice D is the best answer. “Consequently” logically signals that the claim in this
sentence—that civil rights groups protested the law—is a result or consequence of the
information in the previous sentence (i.e., that the law protected Confederate symbols
from removal).

11.
Choice D is the best answer. “Indeed” logically signals that the claim in this sentence
reinforces the claim made in the previous sentence—that Senator Curtis opposed the
expansion of presidential powers.

www.strategictestprep.com ♟️ 1-833-300-PREP (7737) ♟️[email protected] 241


12.
Choice C is the best answer. “In fact” logically signals that the claim in this
sentence—that the focus has shifted away from carbon-based energy and toward the
importance of finding alternative, renewable energy sources—reinforces the point
made before it about the local officials attempting to reduce carbon emissions.

13.
Choice C is the best answer. “Nonetheless” logically signals that the claim in this
sentence—that digital computing allowed robots to be programmed to complete
complex tasks—is true even though the previously mentioned information indicated
that robotics was a futuristic, lofty goal in the 1950s.

14.
Choice D is the best answer. “For instance” logically signals that the information in this
sentence—about one particular example of how the SEC enforces laws to protect
investors—is meant to provide a specific illustration of the more general information in
the previous sentence about the SEC.

15.
Choice D is the best answer. “Nevertheless” logically signals a contrast with the
previous sentence, claiming that the US government has since partially reversed its
position on the ANWR in spite of the fact that the US government previously
encouraged oil drilling in the area.

16.
Choice A is the best answer. “Significantly” logically signals that the information in this
sentence—that Klutsis’s geometric design collages illuminated the elements of the
proletarian experience—is important and relevant to Klutsis’s self-described goal,
which had been articulated in the previous sentence.

17.

www.strategictestprep.com ♟️ 1-833-300-PREP (7737) ♟️[email protected] 242


Choice D is the best answer. “Nevertheless” logically signals that the information in
this sentence—that the grizzly bear population has been kept from dwindling further,
in part due to conservation efforts—contrasts positively with the expectation from the
previous sentence that hunting and habitat loss had left the grizzly bear population in
danger.

18.
Choice D is the best answer. “For example,” logically signals that the claim in this
sentence—that the visioning exercise provides structure and guidance, and allows
participants to concretely imagine the outcome of their plans—as a specific example
for how visioning exercises can be a powerful tool to identify and articulate their future
dreams.

19. Answer: A) Consequently;

Explanation: The sentence structure indicates a result or effect based on the previous
statement about Charles Coulomb's findings.

A) "Consequently;" – This is the correct choice. "Consequently" signifies that what


follows is a result or consequence of what was mentioned before. The semicolon
properly connects the two independent clauses that are closely related in meaning.

B) "However," – This choice would indicate a contrast or contradiction to the previous


statement, which is not the case here. The following statement is a continuation or
result of the prior statement, not a contradiction.

C) "Though," – This choice also indicates contrast or contradiction, which doesn't fit the
context. The statement that follows doesn't oppose the first statement.

D) "And," – This choice simply adds another statement without indicating the
cause-effect relationship between the two statements.

20. Answer: C) Consequently,

Explanation:

The correct answer is "Consequently," because it provides a cause-and-effect transition


from the information presented in the previous sentences.
www.strategictestprep.com ♟️ 1-833-300-PREP (7737) ♟️[email protected] 243
A) "Resulting," – This choice would be more appropriate if it were "As a result," but
"Resulting," by itself doesn't fit the context of the sentence to indicate a result.

B) "Moreover," – This choice would imply that the information that follows is an
addition to the previous idea, not a direct result of it.

C) "Consequently," – This choice shows that the actions of the plants (releasing
insecticides or developing tough textures) are a direct consequence of their broad
leaves attracting more pests and diseases.

D) "Although," – This choice would be used to introduce a contrasting idea or exception


to the previous statement, but the subsequent information in the sentence isn't
contrasting.

21. Answer: D) In contrast,

Explanation:

The sentence is highlighting the contrast between the speed, punctuality, and safety of
the Shinkansen and the conditions of train travel in Japan prior to its introduction.

A) "Interestingly enough" – This phrase does not provide a clear contrast or


comparison between the two situations, making it a less appropriate choice.

B) "Coincidentally," – The situation before the Shinkansen’s debut is not presented as a


coincidence but rather as a point of comparison.

C) "Indeed," – This word is typically used to emphasize a point or provide further


confirmation. It doesn't offer the contrast needed in this context.

D) "In contrast," – This choice appropriately highlights the difference between the state
of train travel before and after the Shinkansen's introduction.

22. Answer: A) Conversely,

Explanation:
The sentence sets up a contrast between the general fear people have of swimming
with sharks and the specific situation in which people actually want to swim with a
particular type of shark (the whale shark).

www.strategictestprep.com ♟️ 1-833-300-PREP (7737) ♟️[email protected] 244


A) "Conversely," - This is the correct choice. "Conversely" is used to introduce a
statement or idea which reverses one that has just been made or referred to.

B) "Similarly," - This choice would suggest that the second part of the sentence is
presenting a similar idea to the first, but it's actually presenting a contrasting one.

C) "Indeed," - This would be used to emphasize a point or confirm it, but it doesn't set
up a contrast like the sentence requires.

D) "Moreover," - This would add more information in the same vein as the preceding
statement, rather than contrasting it.

23. Answer: C) Subsequently,

Explanation:

● A) Conversely, suggests an action or outcome that contrasts with what has been
previously mentioned. Given that the text is highlighting Curie's success
following her efforts, this transition doesn't fit the context.

● B) Ironically, suggests an unexpected or contrary outcome, but Curie's


achievement of winning a Nobel Prize logically follows her dedicated research
efforts.

● C) Subsequently, is the correct choice. It indicates a logical sequence of events


or the next step in a series of events. In this case, after Curie's persistent
research, she achieved a significant milestone.

● D) Alternatively, suggests a different method or approach, but the sentence is


about Curie's achievement as a result of her efforts, not a different approach she
could have taken.

24. Answer: B) Therefore,

Explanation:

www.strategictestprep.com ♟️ 1-833-300-PREP (7737) ♟️[email protected] 245


● A) Similarly, would indicate a comparison with something that was just
mentioned. But the subsequent information doesn't provide a comparison;
instead, it presents a result or conclusion based on the previous statements.

● B) Therefore, is the correct choice. The information provided earlier in the


sentence (the depth of the trench and the challenges of exploring it) logically
leads to the conclusion that the Mariana Trench is one of the least explored
parts of Earth.

● C) Nevertheless, implies a contrast with the previous statement. The following


information doesn't contrast but rather is a result of the preceding information.

● D) On the contrary, also implies a contradiction or opposition to the previous


statement. The subsequent information is not opposing but is instead a
continuation of the idea presented earlier.

25. Answer: C) As a result,

Explanation:

● A) Incidentally, implies that the following information is shared by chance or as


a side note. It doesn't convey the causality implied in the subsequent statement.

● B) Surprisingly, suggests that the next statement will provide something


unexpected or counter to what was described before. It does not fit the context
where the resilience of the coral provides a reason for the subsequent belief.

● C) As a result, is the correct choice. The resilience of the reef system logically
leads to Dr. Jenkins' belief about the reef's potential insights for protection
strategies.

● D) Conversely, implies a contrast or opposite viewpoint to the preceding


statement. The subsequent statement is not contrasting but is instead a
continuation or inference from the previous details.

26. Answer: D) Surprisingly,

www.strategictestprep.com ♟️ 1-833-300-PREP (7737) ♟️[email protected] 246


Explanation:

● A) Simultaneously, suggests that two actions or events are happening at the


same time, but the context here is about contrasting an old belief with a new
finding, not simultaneous occurrences.

● B) Additionally, is used to add further information or emphasize a point. In this


case, the new information is not additional but rather contrasts the previous
statement about the cheetah.

● C) For instance, is used to give examples. The following information is not an


example of the previous statement but a contrasting discovery.

● D) Surprisingly, is the correct choice. It introduces an unexpected or contrasting


point to the prior information. The previous belief about the cheetah is
contrasted with the surprising new findings about the pronghorn antelope.

27. Correct Answer: C) Therefore,

Answer Explanation: The initial statement presents the potential harm of using
synthetic pesticides. The subsequent statement offers a solution that gardeners have
begun to use in response to that problem. The transition "Therefore," introduces a
conclusion or result based on the previously mentioned information.

Incorrect Choices:

A) "Obviously," is used to state something that should be self-evident based on the


previous information, but the transition to using natural predators isn't "obvious" from
the drawbacks of synthetic pesticides alone.

B) "For instance," provides an instance or illustration of a previously mentioned topic.


However, the use of natural predators isn't just a single example of the drawbacks of
synthetic pesticides; it's presented as a contrasting solution.

D) "Regardless," implies continuing despite something, which doesn't fit the flow of
the sentence since the shift to natural predators is directly because of the drawbacks of
synthetic pesticides.

www.strategictestprep.com ♟️ 1-833-300-PREP (7737) ♟️[email protected] 247


28. Correct Answer: B) on the other hand,

Answer Explanation: The correct answer is "on the other hand," because the sentence
is providing a contrasting example to the marine animals that can extract oxygen
directly from the water.

Incorrect Choices:

A) "for example," is used to provide a specific instance of the point previously made. In
this context, it would imply that whales and dolphins are examples of animals that
extract oxygen from water, which is contrary to the information provided.

C) "as a result," indicates causality or a result from a preceding action or situation. This
doesn't fit here because the fact that whales and dolphins need to surface isn't a direct
result of other marine animals extracting oxygen from water.

D) "in conclusion," is typically used to summarize or provide a final statement on a


topic. It doesn't fit the context because the sentence isn't concluding a broader
discussion about marine animals.

29. Correct Answer: C) However,

Answer Explanation: The correct answer is "however," because the sentence is


introducing a contrasting idea. After describing the grandeur and significance of the
Great Barrier Reef, the transition shifts the tone to discuss the challenges it faces,
making "Nevertheless" an appropriate transition to introduce a contrasting point.

Incorrect Choices:

A) "In addition," is used when providing more of the same kind of information, not
when introducing a contrasting point.

www.strategictestprep.com ♟️ 1-833-300-PREP (7737) ♟️[email protected] 248


B) "Surprisingly," suggests that the subsequent information is unexpected. In this
context, given widespread knowledge about the threats to coral reefs, it's not
particularly surprising.

D) "Concurrently," means happening at the same time. While it can indicate a form of
contrast, it's not as strong or clear as "Nevertheless" in this context.

30. Correct Answer: D) Secondly,

Answer Explanation: The correct answer is "Secondly," because the passage is


presenting a sequence of reasons or benefits of including spinach in one's diet. The
transition "Firstly" indicates that a subsequent point in the sequence, "Secondly," will
follow.

Incorrect Choices:

A) "Interestingly," can be used to introduce a point that might be surprising or not


commonly known, but it doesn't provide a sequence.

B) "Consequently," indicates a result or outcome, which is not the context here.

C) "Moreover," introduces an additional point but doesn't maintain the sequence


established by "Firstly."

31. Correct Answer: C) Consequently

Answer Explanation: The correct answer is "Consequently," because the passage sets
up a context about the challenge of accessing clean water and then highlights what
those have to do as a result of this challenge to get the water they need.

Incorrect Choices:

A) "Nevertheless," suggests a contrast or an opposing idea to the previous statement,


which isn't the case in this context.

www.strategictestprep.com ♟️ 1-833-300-PREP (7737) ♟️[email protected] 249


B) "Conversely," implies a direct opposite or contrast, which isn't the relationship
between the two statements.

D) "In spite of this," indicates that the subsequent statement will go against the prior
expectation set by the previous sentence. However, the following statement is not
contradictory but illustrative.

Tip: If you notice three contrast words, eliminate them! If one was correct, they would
all be correct!

32. Correct Answer: D) For example,

Answer Explanation: The correct answer is "For example," because the subsequent
information provides a specific instance or example of Tania Vasquez's celebrated
ability to integrate green spaces into urban designs.

Incorrect Choices:

A) "Similarly," implies that the subsequent information is going to be about another


architect or project similar to Vasquez's, but it's about one of her specific projects.

B) "On the contrary," suggests a contrasting or opposing idea is coming next. This is
not the case, as the information provided supports Vasquez's celebrated ability.

C) "As a result," suggests a cause-and-effect relationship, implying that because of her


celebrated ability, the subsequent event occurred. This doesn't capture the illustrative
nature of the following information.

33. Correct Answer: A) Simultaneously,

Answer Explanation:

The correct answer is "Simultaneously," as it introduces two events or situations that


are occurring around the same time. In this context, while William is in the seaside
town, Joanna also arrives there for a different reason.

Incorrect Choices:

www.strategictestprep.com ♟️ 1-833-300-PREP (7737) ♟️[email protected] 250


B) "In comparison," is used to draw a contrast between two situations or things. Here,
the situations of William and Joanna are not contrasting, but rather happening in
parallel.

C) "Ironically," indicates an unexpected twist or event that is opposite to what one


might expect. While it could be seen as unexpected that Joanna and William's
histories are intertwined, the lead-in to this revelation isn't necessarily ironic in the
given context.

D) "Therefore," is used to indicate a cause-and-effect relationship. The presence of


Joanna in the town isn't a direct result or consequence of William's actions or presence,
so this transition doesn't fit.

34. Correct Answer: C) for instance,

Answer Explanation:

The correct answer is "for instance," as it provides an example of the "strategies"


marine life has developed, as mentioned in the previous sentence.

Incorrect Choices:

A) "on the other hand," is used to contrast two different situations or points. In this
context, the behavior of the mimic octopus isn't contrasting with the strategies of other
marine life but rather is an example of such strategies.

B) "therefore," is used to indicate a result or consequence. The mimic octopus's ability


to change its shape and color isn't a result of marine life's strategies; it's an illustration
of them.

D) "in conclusion," typically wraps up or summarizes a set of statements or arguments.


Here, it's introducing an example rather than summarizing, so this choice doesn't fit the
context.

35. Correct Answer: D) Consequently,

www.strategictestprep.com ♟️ 1-833-300-PREP (7737) ♟️[email protected] 251


Answer Explanation: The statement introduces the 1969 Cuyahoga River fire and its
impact on the nation's awareness of water pollution. The next sentence describes a
direct result of that increased awareness, which is the passing of the Clean Water Act.
The term "Consequently" effectively communicates this cause-and-effect relationship.

Incorrect Choices:

A) "On the contrary," would suggest an opposing or contrasting idea, which is not the
case here.

B) "Surprisingly," indicates that the information that follows is unexpected. There's no


element of surprise in the relation between the event and the act's passing.

C) "For instance," is used to provide an example of a previously mentioned general


idea. The information about the Clean Water Act is not just an instance of the
Cuyahoga River fire but a direct result of it.

36. Correct Answer: D) In another episode,

Answer Explanation: The sentence structure presents a sequence of events: first, Dr.
Sylvia Thompson's exploration of the coral reefs, and then her venture into the
mangroves. "In another episode," provides a logical progression indicating that her
exploration of the mangroves is part of a different segment of her documentary series.

Incorrect Choices:

A) "Alternatively," suggests an opposite or different approach or method, which is not


the sentiment being conveyed.

B) "Meanwhile," indicates actions occurring at the same time. The context here is
sequential, not simultaneous.

C) "For instance," is used to provide an example or specific case. This choice doesn't
align with the narrative flow that moves from one subject of the documentary series to
another.

www.strategictestprep.com ♟️ 1-833-300-PREP (7737) ♟️[email protected] 252


37. Correct Answer: D) for instance,

Answer Explanation: The sentence is introducing Wynton Marsalis as an example of


someone who has combined the elements of jazz and classical music. Therefore, "for
instance" is the most appropriate choice as it introduces an example that supports the
previous statement.

Incorrect Choices:

A) "incidentally," introduces a side note or an additional piece of information that might


not be directly related to the main point.

B) "on the other hand," introduces a contrasting idea or an alternative perspective,


which is not the context here.

C) "moreover," adds additional similar information to the previous statement, but in this
context, we are looking for a specific example to illustrate the point.

38. Correct Answer: D) similarly,

Answer Explanation: The sentence is drawing a parallel between Machu Picchu and
the Pyramids of Giza, noting that both are marvels from ancient civilizations renowned
for their architectural achievements. "Similarly" is the most fitting transition to
introduce an analogy or likeness between the two.

Incorrect Choices:

A) "interestingly," is used to introduce a surprising or novel idea. In this context, the


comparison between the two architectural wonders is not particularly surprising.

B) "as a result," indicates causality or a sequence of events, which isn't the relationship
being presented between Machu Picchu and the Pyramids of Giza.

C) "on the other hand," introduces a contrasting idea, but the sentence is presenting a
similarity, not a contrast, between the two sites.

www.strategictestprep.com ♟️ 1-833-300-PREP (7737) ♟️[email protected] 253


39. Correct Answer: D) However,

Answer Explanation: The passage sets up a contrast between historical small-scale,


family-owned farms and the modern trend of large, industrial-sized farms. The word
"However" correctly introduces the idea that, despite the historical context, a different
trend is seen in modern times.

Incorrect Choices:

A) "Consequently," suggests a cause-and-effect relationship, implying that modern


industrial-sized farms are a direct result of the historical dominance of small
family-owned farms. This isn't the relationship being described in the passage.

B) "Similarly," is used to draw parallels or likenesses between two ideas or situations.


The passage, however, is setting up a contrast, not a similarity.

C) "For instance," is used to give an example or illustration of a prior statement. Given


that the passage is contrasting old and new farming practices, this transition doesn't fit
the context.

40. Answer: D) Remarkably,

Explanation: The statement introduces an exceptional trait of the resurrection plant,


which is its ability to survive extreme drought conditions. The transition should
introduce the result or outcome of this trait, which is the plant's rapid return to a lively
state upon receiving water.

A) "Alternatively," implies an alternative or different method or strategy, which isn't


what the context demands.

B) "Surprisingly," while it does introduce something unexpected, doesn't fully capture


the wonder of the plant's ability.

C) "However," indicates a contrast, and there isn't a contradiction between the two
parts of the sentence.

D) "Remarkably," correctly emphasizes the extraordinary nature of the plant's behavior,


fitting seamlessly with the context of the sentence.

www.strategictestprep.com ♟️ 1-833-300-PREP (7737) ♟️[email protected] 254


41. Answer: D) Yet,

Explanation: The initial part of the passage paints a picture of London as a shining
example of progress and modernity. However, the ending of the passage suggests a
contrasting view, indicating that despite the general perception, there were significant
problems in the city.

A) "Moreover," would add additional information in the same vein as the preceding
statements, which doesn't fit here since the ending is presenting a contrasting view.

B) "In fact," typically introduces information that emphasizes or supports the preceding
statement. In this case, the following information contrasts the initial statements,
making this option unfitting.

C) "Consequently," indicates a result or effect based on the preceding information.


Dickens' portrayal isn't a consequence of London's status but rather a contrast to it.

D) "Yet," introduces a contrast, suggesting that even though London had many positive
attributes, there was another, less favorable, side to it.

42. Answer: C) Similarly,

Explanation: The sentence is discussing how different cities in Europe, during the
Renaissance, were significant for different reasons. The transition word needs to
convey that the situation in Stratford-upon-Avon with William Shakespeare is similar
in cultural importance to what Florence represented with its artists.

A) "Moreover" is typically used to add more information to a similar idea, but here we
are presenting two distinct places.

B) "Consequently" is used to show a result or outcome, which is not the case here.

C) "Similarly" is the correct choice, as it indicates that Stratford-upon-Avon, like


Florence, was also significant during the Renaissance, but for a different reason.

D) "Therefore" is used to indicate a conclusion or a result, which doesn't fit in this


context where two parallel examples are being presented.

www.strategictestprep.com ♟️ 1-833-300-PREP (7737) ♟️[email protected] 255


ii. Note-Taking Questions ✍️
These questions come at the very end of the module. You will be able to pinpoint them
based on their distinct bullet-point style. They are very easy.

There are typically two different types of note-taking questions:

Type I - Comparisons 🤝 and Contrasts🙅


These questions ask you to identify a statement indicating that two people/things are
either similar or different. Depending on the emphasis, correct answers include words
such as like, unlike, or whereas.

Type II - Familiar 🙋‍♀️ and Unfamiliar🫣


These questions ask you to identify a statement that would be used to present a
person, work of art, event, etc. to an audience that either has or does not have previous
knowledge of it.

Strategies

Strategy #1: ❌Do NOT read the notes. ✅ Read the question and pick an answer!
The answer will virtually always be contained in the question itself. The correct choice
will simply rephrase it-you do not need to look at the bullet points at all.

www.strategictestprep.com ♟️ 1-833-300-PREP (7737) ♟️[email protected] 256


Examples
Example #1 - Working Without the Notes

Let’s look at an example so you can see how you can answer it without even reading
the bulleted points.

The student wants to present a similarity between dinosaurs and modern birds. Which
choice most effectively uses relevant information from the notes to achieve this goal?

What type of note-taking question is this? (Circle One)

Type I Type II

Ok, now let’s find the right answer that gives the student what they want!

A) Dinosaurs' mouths contained teeth; however, birds evolved toothless beaks over
millions of years.

B) Mei long, a duck-sized bipedal dinosaur from the Cretaceous era, was found
preserved in volcanic ash.

C) Birds' bones are hollow, a trait that was shared by many dinosaur species.

D) Birds are descended from the theropod group of dinosaurs, which contained
Tyrannosaurus rex.

www.strategictestprep.com ♟️ 1-833-300-PREP (7737) ♟️[email protected] 257


Example #2 - Ignore the Notes

While researching a topic, a student has taken the following notes.

● Carrie Mae Weems (born 1953) is a photographer and installation artist whose
works employ text, fabric, audio, digital images and video.

● She has received major awards from the MacArthur Foundation and the
American Academy in Rome.

● Her 2021 installation in Chicago featured photos, video, text, and furniture.

● Visitors were encouraged to browse and sit at desks in a recreation of a historic


classroom.

● The same year, her "Cyclorama" exhibit at the New York City Armory included
video projections, shadow puppets, and a voice-over narration.

The student wants to emphasize the variety of materials employed by Weems to an


audience unfamiliar with her work. Which choice most effectively uses relevant
information from the notes to achieve this goal?

A) Carrie Mae Weems's artwork has been recognized by both the MacArthur
Foundation and the American Academy in Rome.

B) In 2021, Weems staged an installation in Chicago in which visitors were encouraged


to participate directly in a recreation of a historic classroom.

C) Carrie Mae Weems, who is considered among the most influential contemporary
American artists, employs a diverse set of media: her shows include elements ranging
from photos to furniture to shadow puppets.

D) Carrie Mae Weems's 2021 show in New York City, unlike her show in Chicago the
same year, included shadow puppets as well as a voice-over narration.

www.strategictestprep.com ♟️ 1-833-300-PREP (7737) ♟️[email protected] 258


Example #3 - Ignore the Notes

While researching a topic, a student has taken the following notes:

● Wolves produce a range of vocalizations, including growls, whines, and barks.

● To show submission, a wolf pulls its tail between its legs.

● A raised tail indicates dominance.

● Wolves have powerful scent receptors and can perceive odors a mile or more
away.

● Observers may mistake some of wolves' play behaviors for aggression.

The student wants to present the importance of body language in wolves'


communication. Which choice most effectively uses relevant information from the
notes to achieve this goal?

A) Wolves display a variety of physical and vocal signals when interacting with one
another.

B) The position of a wolf’s tail and ears can reveal its status relative to other animals in
the pack.

C) Some wolf behaviors may appear aggressive to observers but are actually types of
play.

D) Wolves are extremely effective hunters because they can smell prey from far

Answers on the following page.

www.strategictestprep.com ♟️ 1-833-300-PREP (7737) ♟️[email protected] 259


Answers
Example #1: C
Example #2: C
Example #3: B

Extra Practice
1. While researching a topic, a student has taken the following notes:

● Civil rights activists Alice Paul and Lucy Burns formed the National Women’s
Party in 1916.
● The party organized votes for women in the United States and was active in
Washington, DC.
● Paul specialized in pushing for an amendment to the US Constitution to
guarantee the right to vote for women.
● Burns designed the NWP’s protest tactics, which included picketing the White
House from 1917 to 1919.

The student wants to emphasize a contrast between the primary objectives of each of
the activists. Which choice most effectively uses relevant information from the notes to
accomplish this goal?

A. Alice Paul and Lucy Burns established the National Women's Party to promote
voting rights for women, focusing on pushing for an amendment to the US Constitution
and protesting the White House, respectively.
B. Alice Paul and Lucy Burns wanted to form the National Women’s Party to advocate
for women’s voting rights and created protest tactics.
C. Alice Paul and Lucy Burns were united in their goal to organize votes for women in
the US, although Paul was focused on the amendment while Burns was on the
protests. D. Alice Paul and Lucy Burns both worked to promote voting rights for
women, with Paul specialized in pushing for an amendment and Burns in protesting
the White House.

2. While researching a topic, a student has taken the following notes:

www.strategictestprep.com ♟️ 1-833-300-PREP (7737) ♟️[email protected] 260


● House sparrows live in many different urban habitats.
● Habitat modification by humans can drastically affect the distribution of house
sparrows. In 2017, ecologists at the University of Barcelona studied the
urbanization of house sparrow populations.
● The researchers categorized different urban environments based on the amount
of vegetation and building height.
● They found that house sparrows inhabit both densely built-up areas and areas
with high vegetation, but prefer moderate-scale urbanization.

The student wants to emphasize the conclusion of the research study. Which choice
most effectively uses relevant information from the notes to accomplish this goal?

A. Researchers studied the urbanization of house sparrow populations and found that
moderate-scale urbanization is preferable.
B. The University of Barcelona conducted a study of house sparrow populations in
2017 and found that house sparrows prefer areas with moderate-scale urbanization.
C. Urban environments were categorized based on vegetation and building height and
house sparrows inhabiting each habitat were studied.
D. House sparrows live in many different habitats and habitat modification by humans
can drastically influence their distribution.

3. While researching a topic, a student has taken the following notes:

● Clostridium difficile spores are Gram-positive, rod-shaped bacteria.


● They are anaerobic, able to exist and function in the absence of oxygen.
● The bacteria form spores when nutrients are scarce.

www.strategictestprep.com ♟️ 1-833-300-PREP (7737) ♟️[email protected] 261


● The spores can survive extreme temperatures, pH levels, and solvent exposure.

The student wants to add a fact to emphasize the bacteria’s hardiness. Which choice
most effectively uses relevant information from the notes to accomplish this goal?
A. Clostridium difficile spores have the ability to survive without oxygen.
B. Clostridium difficile forms spores when environmental conditions become
unfavorable.
C. Clostridium difficile spores can survive extreme temperatures, pH levels, and solvent
exposure.
D. Clostridium difficile are Gram-positive, rod-shaped bacteria that are anaerobic.

4. While researching a topic, a student has taken the following notes:


● Alpine marmots are large, burrowing rodents that live in the European Alps.
● The species is vulnerable to changes in climate, and warmer temperatures can
shorten the marmots' hibernation period.
● Researchers have observed that the mammal adjusts its hibernation period in
correlation to climate and snow depth.

The student wants to emphasize the marmot’s response to climate change. Which
choice most effectively uses relevant information from the notes to accomplish this
goal?
A. Alpine marmots are vulnerable to climate changes due to their short hibernation
period.
B. Alpine marmots are large, burrowing rodents and react to climate change by
adjusting their hibernation period in relation to the temperature and snow depth.
C. The Alpine marmots in the European Alps can be affected by changes in the climate,
so researchers are closely monitoring their hibernation period.
D. Researchers have studied the Alpine marmots’ hibernation period and noticed it is
closely correlated to climate and snow depth.

5. While researching a topic, a student has taken the following notes:


● In 2014, scientists discovered an unusual coral reef in the deep sea near Hawaii.

www.strategictestprep.com ♟️ 1-833-300-PREP (7737) ♟️[email protected] 262


● The reef was made up of dead coral, likely from an ancient reef that had died
thousands of years ago.
● Living coral, however, was growing on the bedrock of the ancient reef.
● The species of coral growing on the bedrock was different from the one that had
died thousands of years ago.

The student wants to emphasize an interesting discovery from the study. Which choice
most effectively uses relevant information from the notes to accomplish this goal?

A. Scientists discovered an unusual coral reef near Hawaii in 2014; it consisted of a


dead ancient reef and living coral growing on the bedrock.
B. The reef near Hawaii was made up of dead ancient coral, but scientists also found
living coral species growing on the bedrock.
C. The deep sea coral reef discovered near Hawaii is unusual because it consists of
dead coral from thousands of years ago and living coral from a different species.
D. In 2014, scientists found a coral reef near Hawaii that was made up of dead coral,
likely from an ancient reef, and living coral growing on the bedrock.

6. While researching a topic, a student has taken the following notes:


● The Inuit people traditionally lived in the Arctic region.
● During the winter, their clothing was designed to help them withstand extreme
cold temperatures.
● Inuits used animal skins and fur to make clothing like parkas, pants, and mittens
to keep them warm and dry.
● Boots were also made from animal skin and fur and were designed to keep feet
warm by keeping out snow and moisture.

The student wants to emphasize the type of material Inuits used for their clothing and
footwear. Which choice most effectively uses relevant information from the notes to
accomplish this goal?
A. Inuits’ traditional clothing and footwear were designed to help them withstand
extreme cold temperatures in the Arctic region.
B. Inuits made clothing such as parkas, pants, and mittens, as well as boots.
C. Inuit people used animal skins and fur to make clothing like parkas and mittens and
boots to keep them warm and dry in the Arctic region.

www.strategictestprep.com ♟️ 1-833-300-PREP (7737) ♟️[email protected] 263


D. The Inuits’ traditional clothing and footwear used materials that kept out snow and
moisture.

7. While researching a topic, a student has taken the following notes:


● The Grand Teton National Park is located in Wyoming.
● It covers 310,000 acres and is home to several mountain peaks, including Grand
Teton itself.
● The park is a popular destination for hikers and other outdoor enthusiasts.
● It is also home to numerous animals, including elk, moose, bison, and grizzly
bears.

The student wants to emphasize the diversity of life found in the park. Which choice
most effectively uses relevant information from the notes to accomplish this goal?
A. The Grand Teton National Park covers 310,000 acres and is a popular destination
for outdoor enthusiasts.
B. Hikers and other outdoor enthusiasts often visit the Grand Teton National Park,
which is located in Wyoming, to experience its stunning mountain peaks.
C. The Grand Teton National Park, home to an array of creatures including elk, moose,
bison, and grizzly bears, offers hikers and outdoor enthusiasts a diverse and beautiful
natural setting.
D. The Grand Teton National Park is located in Wyoming and is home to elk, moose,
bison, and grizzly bears.

8. While researching a topic, a student has taken the following notes:


● Cory Booker is a US senator from New Jersey.
● He was first elected in 2013.
● In 2017, Booker proposed a bill called the Marijuana Justice Act.
● If passed, the bill would end the federal criminalization of marijuana and
expunge convictions related to the drug.

The student wants to emphasize the timeline of events surrounding Senator Booker’s
bill. Which choice most effectively uses relevant information from the notes to
accomplish this goal?

www.strategictestprep.com ♟️ 1-833-300-PREP (7737) ♟️[email protected] 264


A. Cory Booker, a US senator from New Jersey, proposed the Marijuana Justice Act in
2017.
B. In 2017, the Marijuana Justice Act, a bill proposed by US senator Cory Booker, was
introduced in the Senate.
C. In 2013, Cory Booker was elected to the US Senate and four years later proposed
the Marijuana Justice Act.
D. The Marijuana Justice Act, proposed by US senator Cory Booker, would end the
federal criminalization of marijuana.

9. While researching a topic, a student has taken the following notes:


● Joseph Priestley was a scientist and theologian who was born in 1733.
● He wrote a book called The History and Present State of Electricity.
● He was the first person to recognize oxygen, which he referred to as
"dephlogisticated air.

The student wants to highlight an accomplishment of Joseph Priestley. Assuming the


student’s readers are already familiar with Priestley, which choice most effectively uses
relevant information from the notes to accomplish this goal?
A. Joseph Priestley, who was born in 1733, wrote a book about electricity, and he was
the first to discover oxygen.
B. A 1733 scientist and theologian called Joseph Priestley wrote a book about
electricity and referred to oxygen as "dephlogisticated air."
C. A book called The History and Present State of Electricity was written by Joseph
Priestley, who was the first to recognize oxygen.
D. In 1733, Joseph Priestley wrote a book called The History and Present State of
Electricity.

10. While researching a topic, a student has taken the following notes:
● Prostratin is a natural compound found in eucalyptus trees.
● In lab studies, it has been shown to activate a protein that can inhibit HIV
replication.
● It may be effective as a treatment for HIV infections.

www.strategictestprep.com ♟️ 1-833-300-PREP (7737) ♟️[email protected] 265


The student wants to present Prostratin and its potential as a treatment for HIV
infections. Which choice most effectively uses relevant information from the notes to
accomplish this goal?
A. Eucalyptus trees contain Prostratin, a compound that may be useful in HIV
treatments.
B. Prostratin has been found to be an effective protein in the treatment of HIV.
C. Lab studies have shown that Prostratin, a compound from eucalyptus trees, can help
prevent HIV replication.
D. Eucalyptus trees contain a naturally occurring compound with potential for treating
HIV.

11. While researching a topic, a student has taken the following notes:
● The burning of fossil fuels is the largest source of air pollution.
● The burning of biofuels, such as wood or animal dung, is also a source of air
pollution.
● The use of renewable energy sources, such as solar or wind, can reduce air
pollution.

The student wants to explain the different sources of air pollution. Which choice most
effectively uses relevant information from the notes to accomplish this goal?
A. Burning fossil fuels, biofuels, and renewable energy sources can all lead to air
pollution.
B. Fossil fuels and biofuels are sources of air pollution, while renewable energy
sources can help reduce it.
C. The use of renewable energy sources, such as wind, is a major source of air
pollution.
D. Air pollution is caused by burning fossil fuels, biofuels, and renewable energy
sources.

12. While researching a topic, a student has taken the following notes:
● The Aleutian Islands are located in the northern Pacific Ocean and form a bridge
between the United States and Russia.
● The area has a diverse range of wildlife, including fin whales and bald eagles.
● The waters of the Aleutians have a significant impact on the climate of the
North Pacific region.

www.strategictestprep.com ♟️ 1-833-300-PREP (7737) ♟️[email protected] 266


The student wants to summarize the importance of the Aleutian Islands. Which choice
most effectively uses relevant information from the notes to accomplish this goal?
A. The Aleutian Islands form a bridge between the United States and Russia and are
home to a variety of animals, such as fin whales and bald eagles.
B. The remote Aleutian Islands provide habitat for species such as fin whales and bald
eagles.
C. The Aleutian Islands are located in the northern Pacific Ocean, providing a
connection between the United States and Russia.
D. The Aleutian Islands are home to a range of wildlife, like fin whales and bald eagles,
and have a significant influence on the climate of the North Pacific.

13. While researching a topic, a student has taken the following notes:
● The French Impressionists were a group of painters who sought to capture the
subjectivity of the observer in their work.
● Pierre-Auguste Renoir’s On the Shaded Terrace depicts a family seated on a
terrace, shown from the point of view of cascading light and shadow.
● In Monet’s The Artist’s Garden at Vétheuil, heavy shadows form a contrast to
the bright flowers and trees.

The student wants to compare the two paintings. Which choice most effectively uses
relevant information from the notes to accomplish this goal?
A. Renoir and Monet belonged to the same style of painting, the French Impressionists.
B. Renoir’s and Monet’s paintings both represent the subjectivity of the observer.
C. Both Renoir and Monet created paintings featuring cascading light and shadow.
D. Renoir’s On the Shaded Terrace and Monet’s The Artist’s Garden at Vétheuil contain
elements of shadow and light.

14. While researching a topic, a student has taken the following notes:
● The Civil War began in 1861 and ended in 1865.
● Nearly 3 million soldiers participated in the war.
● Between 650,000 and 850,000 soldiers died in the conflict, making it the
deadliest war in American history.

www.strategictestprep.com ♟️ 1-833-300-PREP (7737) ♟️[email protected] 267


The student wants to emphasize the scale of the war’s casualties. Which choice most
effectively uses relevant information from the notes to accomplish this goal?
A. The Civil War lasted four years, beginning in 1861 and ending in 1865.
B. By the time the Civil War ended, between 650,000 and 850,000 soldiers had died.
C. The Civil War began in 1861 and saw nearly 3 million soldiers take part.
D. The Civil War was the deadliest war in American history, with casualties estimated
between 650,000 and 850,000.

15. While researching a topic, a student has taken the following notes:
● Lake Erie forms part of the US-Canada border.
● It is the shallowest of the Great Lakes and is known for its warm water.
● It has remained free of most harmful algal blooms since the announcement of
the Great Lake Water Quality Agreement in 1989.

The student wants to emphasize the significance of the 1989 announcement. Which
choice most effectively uses relevant information from the notes to accomplish this
goal?
A. Lake Erie is the shallowest of the Great Lakes and has warmer water than the other
lakes.
B. In 1989, the Great Lake Water Quality Agreement was announced, preventing most
harmful algal blooms in Lake Erie.
C. The Great Lake Water Quality Agreement of 1989 has been very effective in
keeping Lake Erie clean.
D. Lake Erie, which lies along the US-Canada border, is known for its warm, clean
water.

16. While studying the ancient city of Pompeii, an archaeologist has taken the
following notes:
• Pompeii was an ancient Roman city near modern Naples.

• Dr. Marco Bellini undertook a research project in Pompeii in 2024.

• He was particularly interested in a newly discovered residential area.

www.strategictestprep.com ♟️ 1-833-300-PREP (7737) ♟️[email protected] 268


• Using advanced imaging techniques, Dr. Bellini found distinct murals hidden beneath
layers of ash.

• The murals depicted daily life in Pompeii and were different from previously
discovered artworks.

The archaeologist wants to summarize Dr. Bellini's research in Pompeii. Which choice
most effectively uses relevant information from the notes to accomplish this goal?

A) Dr. Bellini used advanced imaging techniques in 2024 to study a newly discovered
residential area in the ancient Roman city of Pompeii.

B) Pompeii, an ancient Roman city, has been the subject of many research projects,
including Dr. Bellini's 2024 study.

C) Dr. Bellini's 2024 research in Pompeii uncovered daily life murals in a newly
discovered residential area, distinct from previously found artworks.

D) In 2024, Dr. Bellini was interested in murals that depicted daily life in Pompeii and
were hidden beneath ash layers.

17. While researching a topic, a student has taken the following notes:
• The Renaissance was a cultural movement that began in Italy during the Late Middle
Ages.

• Leonardo da Vinci, an Italian polymath of the Renaissance, was known for his works
in art, science, and anatomy.

• Michelangelo, another leading figure from the period, was primarily recognized for his
sculpture and painting.

• Da Vinci often merged science and art, as seen in his detailed anatomical sketches
that were both scientifically accurate and artistically refined.

• Michelangelo’s work in the Sistine Chapel is considered one of the greatest


achievements in Western art.

www.strategictestprep.com ♟️ 1-833-300-PREP (7737) ♟️[email protected] 269


The student wants to compare the contributions of da Vinci and Michelangelo during
the Renaissance. Which choice most effectively uses relevant information from the
notes to accomplish this goal?

A) Both da Vinci and Michelangelo were pivotal figures during the Renaissance, with
da Vinci known for his scientific works and Michelangelo for his Western art
achievements.

B) Leonardo da Vinci combined art and science in his anatomical sketches, while
Michelangelo’s main contribution was his work in the Sistine Chapel.

C) Although da Vinci and Michelangelo were both recognized figures during the
Renaissance, da Vinci was known more for his interdisciplinary works while
Michelangelo is remembered primarily for his art.

D) Da Vinci's detailed anatomical sketches highlight his scientific pursuits during the
Renaissance, while Michelangelo’s Sistine Chapel showcases his unparalleled artistry.

18. While researching a topic, a student has taken the following notes:

• The Amazon Rainforest spans multiple countries.

• The forest is about 5.5 million square kilometers in size.

• The Sahara Desert is primarily located in North Africa.

• The desert stretches across 11 countries.

• It covers approximately 9.2 million square kilometers.

The student wants to compare the sizes of the Amazon Rainforest and the Sahara
Desert. Which choice most effectively uses relevant information from the notes to
accomplish this goal?

A) The Sahara Desert, which spans 11 countries, is significantly larger in size than
many other natural regions.

www.strategictestprep.com ♟️ 1-833-300-PREP (7737) ♟️[email protected] 270


B) The Amazon Rainforest is about 5.5 million square kilometers, making it
considerably smaller than the Sahara Desert, which covers about 9.2 million square
kilometers.

C) The Amazon Rainforest and the Sahara Desert both span multiple countries, with
the forest located primarily in South America and the desert in North Africa.

D) Both the Sahara Desert and the Amazon Rainforest are significant natural
landmarks, with one located in Africa and the other in South America.

19. While researching a topic, a student has taken the following notes:

• Gabriela Garcia is a renowned sculptor based in Madrid.

• She often uses a technique she coined as "lumisculpt," which refers to sculptures that
incorporate light.

• "Lumisculpt" is derived from the words "luminous" and "sculpture."

• One of her famous pieces, "Aurora," features a marble statue with a light source
inside, creating a soft glow.

• Another piece, "Daybreak," portrays a bronze bird with LED lights as eyes.

The student wants to provide a definition and example of "lumisculpt." Which choice
most effectively uses relevant information from the notes to accomplish this goal?

A) Gabriela Garcia, a Madrid-based artist, invented "lumisculpt" and created "Aurora."

B) Gabriela Garcia uses the term "lumisculpt," derived from "luminous" and "sculpture,"
to describe her artworks that include light, such as the marble statue "Aurora" that
radiates a soft glow.

C) "Daybreak," portraying a bronze bird with LED lights as eyes, is an example of


Gabriela Garcia's unique style.

D) The sculptor Gabriela Garcia, known for "lumisculpt," has artworks that are popular
in Madrid.

www.strategictestprep.com ♟️ 1-833-300-PREP (7737) ♟️[email protected] 271


20. While researching a topic, a student has taken the following notes:

• Maria Tallchief was a prominent figure in 20th-century ballet.

• She was the first Native American to become a prima ballerina.

• Tallchief's groundbreaking performances with the New York City Ballet included
roles in Firebird and Nutcracker.

• In 1996, she received the Kennedy Center Honors for her lifetime contributions to
American culture.

• Many today remember her for bringing a blend of athleticism and artistry to her
dance.

The student wants to emphasize the groundbreaking nature of Tallchief's career.


Which choice most effectively uses relevant information from the notes to accomplish
this goal?

A) Tallchief was a significant figure in 20th-century ballet with performances in


Firebird and Nutcracker.

B) In 1996, Tallchief was honored with the Kennedy Center Honors for her significant
contributions to American culture.

C) Many remember Tallchief for her unique blend of athleticism and artistry in her
dance.

D) Maria Tallchief was the first Native American to achieve the status of prima
ballerina, showcasing her groundbreaking career in ballet.

21. While researching a topic, a student has taken the following notes:

• Oceanographers often use large vessels for deep-sea exploration.

• These vessels have limitations due to their size and can't access many narrow or
shallow areas.

www.strategictestprep.com ♟️ 1-833-300-PREP (7737) ♟️[email protected] 272


• Drones, small remote-controlled vehicles, can reach areas that are hard for larger
vessels to access.

‘• Drones can be deployed from the shore or larger vessels.

• Recently, oceanographic research has increasingly incorporated drones due to their


flexibility.

The student wants to emphasize an advantage of using drones for oceanographic


research. Which choice most effectively uses relevant information from the notes to
accomplish this goal?

A) Drones, as an extension of research, have been increasingly used in recent


oceanographic studies.

B) Large oceanographic vessels, despite their extensive capabilities, cannot access


many narrow or shallow parts of the ocean.

C) Drones, being small and remote-controlled, can navigate narrow and shallow areas
of the ocean that are inaccessible to larger oceanographic vessels.

D) While both drones and large vessels are used in oceanography, they are deployed
in different manners.

22. While researching a topic, a student has taken the following notes:

• Haruki Murakami is a renowned Japanese author known for blending magical realism
with deep psychological insight.

• Born in Kyoto, Japan, he now resides in Tokyo.

• One of his most acclaimed works is "Norwegian Wood," published in 1987.

• "Norwegian Wood" is a coming-of-age novel that delves into love, loss, and
nostalgia against the backdrop of 1960s Tokyo.

The student wants to introduce "Norwegian Wood" and its significance in Murakami's
oeuvre. Which choice most effectively uses relevant information from the notes to
accomplish this goal?

www.strategictestprep.com ♟️ 1-833-300-PREP (7737) ♟️[email protected] 273


A) Haruki Murakami, an author from Kyoto, Japan, is recognized for his unique blend of
magical realism. He wrote the novel "Norwegian Wood" in 1987.

B) "Norwegian Wood" was written by Haruki Murakami in 1987. It is a story about love
and loss in 1960s Tokyo and showcases Murakami's blend of magical realism and
psychological insight.

C) A coming-of-age novel set in 1960s Tokyo, "Norwegian Wood" (1987) is one of


Haruki Murakami's most esteemed works, reflecting his trademark style of blending
magical realism with deep psychological introspection.

D) Haruki Murakami's "Norwegian Wood" explores the themes of love, loss, and
nostalgia. He is a Japanese author currently residing in Tokyo.

23. While researching a topic, a student has taken the following notes:

• Wellington is the capital of New Zealand.

• The city’s population is 212,700.

• Wellington contains 4.32 percent of New Zealand’s population.

• Canberra is the capital of Australia.

• The city’s population is 431,380.

• Canberra contains 1.68 percent of Australia’s population.

Which choice most effectively uses information from the given sentences to emphasize
the relative percentages of the two capitals’ populations in their respective countries?

A) Wellington, with a population of 212,700, is the capital of New Zealand, while


Canberra, with a population of 431,380, is the capital of Australia.

B) While both Wellington and Canberra serve as capitals, they have populations of
212,700 and 431,380, respectively.

www.strategictestprep.com ♟️ 1-833-300-PREP (7737) ♟️[email protected] 274


C) Despite Canberra (population 431,380) having a larger population than Wellington
(population 212,700), Wellington accounts for a larger percentage of its country’s total
population.

D) Comparing New Zealand and Australia, 212,700 represents 4.32 percent of New
Zealand’s population, and 431,380 is 1.68 percent of Australia’s.

24. While researching a topic, a student has taken the following notes:

• The Hanging Gardens of Babylon were one of the Seven Wonders of the Ancient
World.

• They were constructed in ancient Mesopotamia, which is present-day Iraq.

• The gardens were described as a series of terraces stacked one above the other. •
Historians believe the gardens were built by King Nebuchadnezzar II for his homesick
wife.

• A system of aqueducts and archimedes screws was believed to have been used to
water the gardens.

The student wants to explain the uniqueness and technological achievement of the
Hanging Gardens. Which choice most effectively uses relevant information from the
notes to accomplish this goal?

A) The Hanging Gardens of Babylon, located in ancient Mesopotamia, were a wonder


that showcased the beauty of terraced gardens.

B) King Nebuchadnezzar II possibly built the Hanging Gardens of Babylon in


Mesopotamia for his wife.

C) Historians believe a complex system of aqueducts and archimedes screws was used
in the Hanging Gardens, highlighting the technological marvels of ancient
Mesopotamia.

D) The Hanging Gardens were one of the Seven Wonders of the Ancient World and
were located in present-day Iraq.

25. While researching a topic, a student has taken the following notes:

www.strategictestprep.com ♟️ 1-833-300-PREP (7737) ♟️[email protected] 275


• The Amazon Rainforest covers parts of nine countries in South America.

• It is often referred to as the "lungs of the Earth" because it produces 20% of the
world's oxygen.

• Dr. Helena Martins and her team conducted a study on the impact of deforestation on
the Amazon’s capacity to produce oxygen.

• Using satellite imaging, they assessed the health and density of trees in various
sections of the rainforest.

• Their findings suggest that regions with the most deforestation showed a 15%
decline in oxygen production.

The student wants to emphasize the significance of new-age technology in studying


environmental changes. Which choice most effectively uses relevant information from
the notes to accomplish this goal?

A) Dr. Helena Martins and her team found a 15% decline in oxygen production in
heavily deforested parts of the Amazon.

B) The Amazon Rainforest, covering parts of nine countries, is vital for the world's
oxygen supply.

C) Using satellite imaging, Dr. Helena Martins and her team were able to assess the
impact of deforestation on the Amazon’s oxygen production.

D) The Amazon Rainforest, often referred to as the "lungs of the Earth," is under threat
from deforestation.

26. While researching a topic, a student has taken the following notes:

• The Trans-Siberian Railway is the world's longest railway line.

• It was built between 1891 and 1916.

• The railway line spans a distance from Moscow in European Russia to Vladivostok on
the Pacific Ocean.

www.strategictestprep.com ♟️ 1-833-300-PREP (7737) ♟️[email protected] 276


• It is approximately 9,289 kilometers (5,772 miles) long.

The student wants to emphasize the impressive length of the Trans-Siberian Railway.
Which choice most effectively uses relevant information from the notes to accomplish
this goal?

A) The Trans-Siberian Railway, constructed between 1891 and 1916, is an iconic


railway line connecting Moscow to Vladivostok.

B) Spanning a distance of approximately 9,289 kilometers (5,772 miles), the


Trans-Siberian Railway is the world's longest railway line.

C) The Trans-Siberian Railway starts in European Russia's Moscow and ends in


Vladivostok by the Pacific Ocean.

D) As a significant achievement in railway construction, the Trans-Siberian Railway


was completed in 1916.

27.

While researching a topic, a student has taken the following notes:

• The Great Barrier Reef, located off the coast of Australia, is the world's largest coral
reef system.

• Over the past few decades, the reef has suffered significant damage due to various
environmental factors.

• Marine biologist Dr. Lisa Harmon conducted a study in 2022 to investigate the main
factors leading to coral bleaching.

• Dr. Harmon collected data using underwater drones and satellite imagery.

• After extensive research, Dr. Harmon identified rising sea temperatures as the
primary cause of coral bleaching on the reef.

The student wants to highlight the central focus of Dr. Harmon's study. Which choice
most effectively uses relevant information from the notes to accomplish this goal?

www.strategictestprep.com ♟️ 1-833-300-PREP (7737) ♟️[email protected] 277


A) Dr. Lisa Harmon, a marine biologist, used underwater drones and satellite imagery
to study the Great Barrier Reef in 2022.

B) The world's largest coral reef system, the Great Barrier Reef, has been damaged
significantly over the past few decades.

C) Dr. Lisa Harmon's study in 2022 was aimed at determining the primary causes of
coral bleaching in the Great Barrier Reef.

D) After her study on the Great Barrier Reef, Dr. Harmon concluded that rising sea
temperatures are the main culprit behind coral bleaching.

28. While researching a topic, a student has taken the following notes:

• Cheetahs can accelerate from 0 to 60 mph in just a few seconds.

• Scientists have been intrigued by the biomechanics that enable such swift
acceleration.

• Biologist Dr. Elena Roberts used high-speed cameras to capture the movement of
cheetahs in the wild.

• Her footage revealed the crucial role of the cheetah's tail in maintaining balance
during rapid acceleration.

• Dr. Roberts' findings suggest the tail acts as a rudder, allowing for precise changes
in direction at high speeds.

The student wants to highlight the technique Dr. Roberts used for her study. Which
choice most effectively uses relevant information from the notes to accomplish this
goal?

A) Cheetahs can achieve rapid acceleration, and Dr. Elena Roberts discovered the tail's
role in this by studying cheetahs in their natural habitat.

B) To understand the swift acceleration of cheetahs, Dr. Elena Roberts employed


high-speed cameras to capture the biomechanics of their movement in the wild.

www.strategictestprep.com ♟️ 1-833-300-PREP (7737) ♟️[email protected] 278


C) Dr. Elena Roberts' research indicates that the tail of a cheetah functions like a
rudder during high-speed chases.

D) While cheetahs can reach up to 60 mph in mere seconds, the biomechanics behind
this ability were analyzed by Dr. Elena Roberts through observation.

29. While researching a topic, a student has taken the following notes:

• Maria Altmann was an Austrian-American Jewish refugee.

• In the early 2000s, she undertook a legal battle to reclaim art that was taken from
her family by the Nazis.

• One of the paintings, "Portrait of Adele Bloch-Bauer I," was painted by Gustav Klimt.

• The U.S. Supreme Court ruled in her favor in the case Republic of Austria v. Altmann
in 2004.

• The painting was returned to her and later sold for $135 million. The student wants
to emphasize the significance of the painting in question. Which choice most effectively
uses relevant information from the notes to accomplish this goal?

A) Maria Altmann was an Austrian-American Jewish refugee who reclaimed a painting


that the Nazis had taken from her family.

B) The painting "Portrait of Adele Bloch-Bauer I" by Gustav Klimt was at the center of
Maria Altmann's legal battle and was later sold for $135 million.

C) Maria Altmann won her case in the U.S. Supreme Court in 2004, leading to the
return of several artworks taken by the Nazis.

D) In Republic of Austria v. Altmann, Maria Altmann took on a nation in the U.S.


Supreme Court and emerged victorious.

30. While researching a topic, a student has taken the following notes:

• Small family farms face difficulties in competing with large-scale commercial farming
operations.

www.strategictestprep.com ♟️ 1-833-300-PREP (7737) ♟️[email protected] 279


• Environmental scientist Dr. Lana Fields initiated a study on sustainable agricultural
methods used by small-scale farmers.

• Fields and her research team surveyed 200 small family farms, investigating their
techniques for soil health, water usage, and crop rotation.

• Out of the surveyed farms, only 38 utilized at least two methods of sustainable
agriculture, and merely 15 fully integrated all three techniques in their farming
practices.

The student wants to emphasize the study’s methodology. Which choice most
effectively uses relevant information from the notes to accomplish this goal?

A) Small family farms face challenges, but those that integrate sustainable techniques
stand a better chance against large commercial operations.

B) Dr. Lana Fields is an environmental scientist who advocates for sustainable farming
practices.

C) To understand the adoption of sustainable agricultural methods by small family


farms, Dr. Lana Fields surveyed 200 such farms, focusing on soil health, water usage,
and crop rotation techniques.

D) The study revealed that only a limited number of surveyed family farms fully utilize
sustainable agricultural techniques.

31. While researching a topic, a student has taken the following notes:

• The Maori are the indigenous Polynesian people of New Zealand.

• Their culture is influenced by Polynesian traditions and their unique history on the
islands of New Zealand.

• Ria Paki is a Maori historian and advocate for indigenous rights.

• She established the Aotearoa Cultural Center in Wellington, New Zealand, in 1995.

• Atawhai Ruru is another Maori historian, known for her paintings that tell Maori
legends.

www.strategictestprep.com ♟️ 1-833-300-PREP (7737) ♟️[email protected] 280


• She opened the Te Rangi Art Gallery in Christchurch, New Zealand, in 1995.

The student wants to highlight the efforts and timing of Paki's and Ruru's contributions
to Maori culture. Which choice most effectively uses relevant information from the
notes to accomplish this goal?

A) The Maori, indigenous to New Zealand, have advocates like Ria Paki and artists like
Atawhai Ruru who depict their rich culture and history.

B) Ria Paki and Atawhai Ruru have played significant roles in preserving and
showcasing Maori culture in New Zealand.

C) Since 1995, Ria Paki and Atawhai Ruru have established cultural centers and
galleries, respectively, dedicated to Maori culture and history.

D) Maori culture, deeply rooted in Polynesian traditions, has been preserved and
promoted by historians and artists in New Zealand.

32. While researching a topic, a student has taken the following notes:

• The monarch butterfly is known for its lengthy migration from North America to
Central Mexico.

• Dr. Maria Gonzales and Dr. Andrew Smith of the University of Monterrey explored
the impact of climate change on the migration patterns of these butterflies.

• Gonzales and Smith captured and tagged thousands of butterflies and tracked their
migratory paths over three years.

• Their data indicated that warmer temperatures caused the butterflies to travel longer
distances.

• Climate change might be altering traditional monarch migration routes.

The student wants to summarize the primary purpose of Gonzales and Smith's study.
Which choice most effectively uses relevant information from the notes to accomplish
this goal?

www.strategictestprep.com ♟️ 1-833-300-PREP (7737) ♟️[email protected] 281


A) Dr. Maria Gonzales and Dr. Andrew Smith of the University of Monterrey aimed to
understand the effect of climate change on the migration patterns of monarch
butterflies.

B) Over three years, Dr. Maria Gonzales and Dr. Andrew Smith captured, tagged, and
tracked thousands of monarch butterflies.

C) Monarch butterflies, which are known for their lengthy migration, may now be
traveling longer distances due to warmer temperatures.

D) The University of Monterrey has shown interest in the monarch butterfly's migration
due to the efforts of Dr. Maria Gonzales and Dr. Andrew Smith.

33. While researching a topic, a student has taken the following notes:

• Akiko Suzuki is a renowned Japanese chef.

• In 2015, she opened the restaurant Mizu in Tokyo, which specializes in traditional
Japanese seafood dishes.

• The restaurant's signature dish is "Sora Maki," a sushi roll with unique combinations
of fish and vegetables.

• In 2020, Suzuki launched the restaurant Umi in Osaka, which also specializes in
traditional Japanese seafood.

• Umi's highlighted dish is "Kaze Tempura," a deep-fried seafood platter with a special
dipping sauce.

The student wants to emphasize a common focus between the two restaurants owned
by Akiko Suzuki. Which choice most effectively uses relevant information from the
notes to accomplish this goal?

A) Akiko Suzuki, having opened Mizu in Tokyo, later introduced another restaurant,
Umi, in Osaka.

B) While Mizu's signature dish is "Sora Maki," Umi is famous for its "Kaze Tempura."

www.strategictestprep.com ♟️ 1-833-300-PREP (7737) ♟️[email protected] 282


C) Both Mizu in Tokyo and Umi in Osaka, owned by Akiko Suzuki, specialize in
traditional Japanese seafood dishes.

D) Umi, located in Osaka, was launched by renowned chef Akiko Suzuki five years after
she opened Mizu in Tokyo.

34. While researching a topic, a student has taken the following notes:

• Corals play a crucial role in marine ecosystems, providing habitats and serving as a
food source for many marine organisms.

• Dr. Maria Soto from the University of Florida sought to understand the impact of
rising sea temperatures on coral health.

• Dr. Soto's team collected samples from corals in five different reef locations and
monitored them for signs of stress.

• Increased sea temperatures were found to lead to coral bleaching, a condition where
corals expel the symbiotic algae living in their tissues.

• Bleached corals are more vulnerable to diseases and may not reproduce efficiently.

The student wants to emphasize the purpose of Dr. Maria Soto's research. Which
choice most effectively uses relevant information from the notes to accomplish this
goal?

A) Dr. Maria Soto was interested in the role that corals play in marine ecosystems,
especially as habitats and food sources for marine organisms.

B) Rising sea temperatures can lead to coral bleaching, making corals more susceptible
to diseases, as discovered by Dr. Maria Soto's research.

C) Dr. Maria Soto from the University of Florida aimed to understand the impact of
rising sea temperatures on the health of corals.

D) After studying corals from five different reef locations, Dr. Maria Soto found that
increased sea temperatures result in coral bleaching.

35. While researching a topic, a student has taken the following notes:

www.strategictestprep.com ♟️ 1-833-300-PREP (7737) ♟️[email protected] 283


• Black holes are regions of spacetime where gravity is so strong that nothing can
escape from it, not even light.

• There are different types of black holes: stellar, supermassive, and primordial.

• Stellar black holes are formed when massive stars collapse.

• Supermassive black holes, often found at the centers of galaxies, can have masses
greater than a million suns combined.

• Primordial black holes are theorized to have formed shortly after the Big Bang and
are smaller than stellar black holes.

The student wants to emphasize the difference in origin between two types of black
holes. Which choice most effectively uses relevant information from the notes to
accomplish this goal?

A) Black holes are fascinating regions of spacetime from which nothing can escape.

B) Stellar black holes and supermassive black holes can be found in different areas of
a galaxy.

C) Stellar black holes form from the collapse of massive stars, whereas primordial
black holes are theorized to have formed shortly after the Big Bang.

D) Supermassive black holes can have masses greater than a million suns, making
them intriguing subjects of study.

36. While researching a topic, a student has taken the following notes:

• Giorgia Lupi is an Italian information designer who is renowned for her data
visualization art.

• Her 2016 piece, "Bruises: The Data We Don't See," is a hand-drawn data
visualization.

• This artwork used data about her friend's son's illness to map emotions, treatments,
and symptoms over time.

www.strategictestprep.com ♟️ 1-833-300-PREP (7737) ♟️[email protected] 284


• The piece juxtaposes personal experiences with medical data, conveying an
emotional journey.

The student wants to describe "Bruises: The Data We Don't See" to an audience
unfamiliar with Giorgia Lupi. Which choice most effectively uses relevant information
from the notes to accomplish this goal?

A) Giorgia Lupi, an Italian artist known for her data art, created "Bruises: The Data We
Don't See" in 2016.

B) "Bruises: The Data We Don't See" is a hand-drawn data visualization piece by


Giorgia Lupi, mapping the emotions, treatments, and symptoms related to an illness.

C) The 2016 artwork by Giorgia Lupi titled "Bruises: The Data We Don't See"
visualizes medical data alongside personal experiences.

D) In "Bruises: The Data We Don't See," Giorgia Lupi juxtaposed her friend's son's
medical journey with emotional experiences, hand-drawing the data to show
treatments, emotions, and symptoms over time.

37. While researching a topic, a student has taken the following notes:

• The Lascaux caves in France are renowned for their Paleolithic cave paintings.

• The paintings date back around 17,000 years.

• Lidar technology is a method that measures distances using laser light.

• Archaeologist Marie Dupont used Lidar technology to examine the nooks and
crannies of the Lascaux caves.

• Dupont's examination led to the discovery of previously unnoticed smaller sketches


adjacent to the main paintings.

The student wants to introduce Dupont's groundbreaking findings in the Lascaux


caves. Which choice most effectively uses relevant information from the notes to
accomplish this goal?

www.strategictestprep.com ♟️ 1-833-300-PREP (7737) ♟️[email protected] 285


A) Marie Dupont utilized Lidar technology to study the intricate details of the Lascaux
cave paintings in France.

B) The Lascaux caves, known for their ancient paintings, were further studied using
lasers.

C) With Lidar technology, archaeologist Marie Dupont uncovered previously hidden


sketches in the famous Lascaux caves.

D) Dating back 17,000 years, the Lascaux caves in France were recently scanned using
laser technology by Dupont.

38. While researching a topic, a student has taken the following notes:

• OLED (Organic Light Emitting Diodes) screens are popular in modern televisions and
smartphones.

• Traditional LCD (Liquid Crystal Display) screens are illuminated by a backlight, which
can result in less perfect black tones.

• OLED screens emit their own light when an electric current passes, eliminating the
need for a backlight.

• As a result of this self-illumination, OLED screens can produce perfect blacks,


leading to higher contrast ratios.

The student wants to highlight the unique feature of OLED screens that sets them
apart from traditional LCD screens. Which choice most effectively uses relevant
information from the notes to accomplish this goal?

A) OLED screens, found in modern televisions and smartphones, do not rely on a


backlight as traditional LCD screens do.

B) The absence of a backlight in OLED screens results in the emission of their own
light when an electric current passes.

C) OLED screens can produce perfect blacks and higher contrast ratios due to their
ability to self-illuminate, eliminating the need for a backlight.

www.strategictestprep.com ♟️ 1-833-300-PREP (7737) ♟️[email protected] 286


D) Modern televisions often incorporate OLED technology, which is different from the
traditional LCD screen technology.

39. While researching a topic, a student has taken the following notes:

• The Silk Road was an ancient network of trade routes that connected the East to the
West.

• Marco Polo, the Venetian explorer, documented his travels along the Silk Road in the
13th century.

• The Silk Road facilitated not only the exchange of goods but also the spread of
knowledge, religion, and culture.

• Historian Angela Wu theorizes that the exchange of agricultural techniques along the
Silk Road played a significant role in global food diversification.

• This theory is often referred to as the "Agricultural Exchange Theory."

• Wu's studies indicate that Polo's writings also alluded to the exchange of farming
methods.

The student wants to present the Agricultural Exchange Theory to an audience


unfamiliar with the Silk Road. Which choice most effectively uses relevant information
from the notes to accomplish this goal?

A) Angela Wu, a historian, believes that the Silk Road was important for the exchange
of goods, especially agricultural products.

B) Marco Polo's documentation of his travels along the Silk Road in the 13th century
highlighted the importance of trade in ancient times.

C) The Agricultural Exchange Theory posits that the Silk Road facilitated the exchange
of agricultural techniques, significantly contributing to global food diversification.

D) Trade routes like the Silk Road played a crucial role in the exchange of knowledge,
religion, and culture throughout history.

40. While researching a topic, a student has taken the following notes:

www.strategictestprep.com ♟️ 1-833-300-PREP (7737) ♟️[email protected] 287


• In the 16th century, Leonardo da Vinci sketched designs for a machine known as the
"flying machine."

• The sketches reveal a contraption that mimicked the flapping wings of birds.

• Aviation engineer Maria Renata studied da Vinci's designs to understand their


feasibility.

• She used modern materials to construct a replica based on da Vinci's sketches.

• Her tests showed the machine, while innovative, couldn't achieve sustained flight
with the power sources available in the 16th century.

• However, with some modifications and current technology, it could be made to hover
for short durations.

The student wants to highlight the feasibility of da Vinci's "flying machine" design
using modern technology. Which choice most effectively uses relevant information
from the notes to accomplish this goal?

A) Leonardo da Vinci's "flying machine" design, sketched in the 16th century, couldn't
achieve sustained flight during his time.

B) Maria Renata used modern materials to construct a replica of da Vinci's "flying


machine" and demonstrated that with modifications and current technology, it could
hover for short durations.

C) Da Vinci's "flying machine" was based on the flapping wings of birds, and while
innovative, it was not capable of actual flight in the 16th century.

D) Aviation engineer Maria Renata's study of da Vinci's designs led her to construct a
replica, revealing the innovative nature of da Vinci's vision.

41. While researching a topic, a student has taken the following notes:

• The Museum of Modern Art (MoMA) was established in New York City in 1929.

• The museum is known for its collection of modern and contemporary art, including
works of painting, sculpture, photography, and film.

www.strategictestprep.com ♟️ 1-833-300-PREP (7737) ♟️[email protected] 288


• In 2020, art critic Jenna Rosenthal published the book "Gazing Beyond Frames:
Women Artists in MoMA."

• The book delves into the contributions of female artists whose works have been
displayed at MoMA throughout the years.

• One of the highlighted artists in the book is Frida Kahlo.

The student wants to introduce Jenna Rosenthal’s book to an audience already familiar
with the Museum of Modern Art. Which choice most effectively uses relevant
information from the notes to accomplish this goal?

A) Jenna Rosenthal’s "Gazing Beyond Frames: Women Artists in MoMA" examines the
contributions of female artists whose works are showcased at the Museum of Modern
Art.

B) MoMA, the subject of Jenna Rosenthal’s book title, was established in 1929 and is
famous for its vast collection of modern and contemporary art.

C) "Gazing Beyond Frames: Women Artists in MoMA" focuses on the Museum of


Modern Art, which since 1929 has held a significant collection of art, including
paintings, sculptures, and photographs.

D) Frida Kahlo, whose art is exhibited at MoMA, is one of the women artists
extensively discussed by Jenna Rosenthal in her book.

42. While researching a topic, a student has taken the following notes:

• The cheetah (acinonyx jubatus) is renowned for its speed and agility.

• It's the fastest land animal, reaching speeds of up to 70 mph.

• Cheetahs primarily hunt small to medium-sized ungulates, like the springbok or


Thomson's gazelle.

• Unlike many other large cats, cheetahs do not ambush their prey from a concealed
position.

www.strategictestprep.com ♟️ 1-833-300-PREP (7737) ♟️[email protected] 289


• Instead, they use their incredible speed in short bursts to chase and overtake their
prey in the open.

• Another method cheetahs use is to push the prey off balance with their dewclaw.

The student wants to emphasize a similarity between the two hunting methods of the
cheetah. Which choice most effectively uses relevant information from the notes to
accomplish this goal?

A) The cheetah, unlike other large cats, relies on its unparalleled speed rather than
concealment to hunt its prey.

B) Neither of the cheetah's hunting methods involves ambushing its prey from a
concealed position.

C) Cheetahs, in addition to chasing their prey in the open, also use their dewclaw to
trip them.

D) Being the fastest land animal, the cheetah primarily chases its prey rather than
using ambush tactics.

Answers - Extra Practice


1.
Choice C is the best answer. The sentence highlights the contrasting objectives of Alice
Paul and Lucy Burns—Paul pushed for an amendment to the US Constitution, whereas
Burns designed the NWP’s protest tactics—by using the phrase “although Paul was
focused on the amendment while Burns was on the protests.

2.
www.strategictestprep.com ♟️ 1-833-300-PREP (7737) ♟️[email protected] 290
Choice B is the best answer. By noting that the University of Barcelona conducted a
study and emphasizing the researchers’ conclusion that house sparrows prefer areas
with moderate-scale urbanization, this sentence emphasizes the conclusion of the
research study. Please note that choice A is incorrect, as the question asks specifically
about the research study and this does not address that specifically, but is more of a
general statement.

3.
Choice C is the best answer. The sentence emphasizes the hardiness of the bacteria by
noting that their spores can survive extreme conditions.

4.
Choice B is the best answer. The sentence emphasizes the marmot’s response to
climate change by noting the animal’s adjustment of the hibernation period in
correlation to climate and snow depth.

5.
Choice C is the best answer. The sentence emphasizes an interesting discovery from
the study, noting that the deep sea coral reef near Hawaii is “unusual” because it
consists of dead coral from thousands of years ago and living coral from a different
species. Choice A is incorrect. While the sentence describes the reef, it doesn’t
emphasize an interesting discovery from the study. Choice B is incorrect. While the
sentence notes that living coral was found on the reef, it doesn’t emphasize an
interesting discovery from the study. Choice D is incorrect. The sentence describes the
reef; it doesn’t emphasize an interesting discovery from the study.

6.
Choice C is the best answer. The sentence emphasizes the type of material Inuits used
for their clothing and footwear; it specifically mentions animal skins and fur. Choice A is
incorrect. While the sentence mentions the material used for Inuits’ clothing and
footwear, it does not explicitly indicate what this material is. Choice B is incorrect.
While the sentence mentions the material used for Inuits’ clothing and footwear as
well as what that material was used to make, it does not explicitly indicate what the

www.strategictestprep.com ♟️ 1-833-300-PREP (7737) ♟️[email protected] 291


material is. Choice D is incorrect. The sentence mentions the material used to make
Inuits’ traditional clothing and footwear, but it doesn’t mention what the material is.

7.
Choice C is the best answer. The sentence emphasizes the diversity of life in the Grand
Teton National Park with the phrase “an array of..” and noting that the park is home to
elk, moose, bison, and grizzly bears. Choice A is incorrect. The sentence describes the
Grand Teton National Park’s size and popularity; it doesn’t emphasize the park’s
biodiversity. Choice B is incorrect. While the sentence mentions the park’s mountain
peaks, it doesn’t emphasize its biodiversity. Choice D is incorrect. The sentence
mentions the park’s wildlife; it doesn’t emphasize its biodiversity.

8.
Choice C is the best answer. The sentence emphasizes the timeline of events
surrounding Senator Booker’s bill, noting he was elected to the US Senate four years
prior to proposing his bill. Choice A is incorrect. While the sentence mentions when
Senator Booker proposed his bill, it doesn’t emphasize the timeline of events. Choice B
is incorrect. While the sentence mentions Senator Booker and his bill, it doesn’t
mention when he was elected or emphasize a timeline of events. Choice D is incorrect.
While the sentence mentions the bill Senator Booker proposed, it doesn’t mention
when he proposed it or emphasize a timeline of events.

9.
Choice D is the best answer. The sentence highlights Joseph Priestley’s
accomplishment of writing a book about electricity. The other three answer choices
describe or introduce Priestly in some way, as if the reader isn’t familiar with him, so
they are incorrect.

10.

www.strategictestprep.com ♟️ 1-833-300-PREP (7737) ♟️[email protected] 292


Choice C is the best answer. The sentence presents Prostratin and its potential as a
treatment for HIV infections, describing the compound as being found in eucalyptus
trees and noting that lab studies have shown that it can help prevent HIV replication.

11.
Choice B is the best answer. The sentence explains the different sources of air
pollution, noting that fossil fuels and biofuels are sources of air pollution, while
renewable energy sources can help reduce it.

12.
Choice D is the best answer. The sentence summarizes the importance of the Aleutian
Islands, noting their role as a home to various wildlife and their “significant” influence
on the climate of the North Pacific region.

13.
Choice D is the best answer. The sentence compares the two paintings by noting that
both contain shadow and light elements. The other answer choices are incorrect
because they compare the two artists in more general capacities, not specifically to the
two paintings.

14.
Choice D is the best answer. The sentence emphasizes the scale of the war’s
casualties, noting that the Civil War was the “deadliest war” in American history and
that casualties were estimated between 650,000 and 850,000. Although choice B
mentions the casualties as well, it does not emphasize the scale of them.

15.
Choice B is the best answer. The sentence emphasizes the significance of the 1989
announcement, noting that it has prevented most harmful algal blooms in Lake Erie.

www.strategictestprep.com ♟️ 1-833-300-PREP (7737) ♟️[email protected] 293


16. Answer: C) Dr. Bellini's 2024 research in Pompeii uncovered daily life murals in a
newly discovered residential area, distinct from previously found artworks.

Explanation:

A) This choice introduces Dr. Bellini's use of advanced imaging techniques and the
newly discovered residential area in Pompeii. However, it does not mention the
significant finding of the distinct murals that depicted daily life.

B) This option gives a general context about Pompeii and mentions Dr. Bellini's 2024
study, but it doesn't provide specifics about the newly discovered murals or their
significance.

C) This choice is the most comprehensive. It describes the timing of Dr. Bellini's
research, the discovery of murals depicting daily life in a new residential area, and
importantly highlights how these murals are distinct from previously discovered
artworks. It effectively presents the essence of Dr. Bellini's research using relevant
information from the notes.

D) While this option touches upon the murals and their depiction of daily life, it doesn't
clarify that these murals were a new and distinct discovery or that they were found in a
newly discovered residential area.

17. Answer: C) Although da Vinci and Michelangelo were both recognized figures
during the Renaissance, da Vinci was known more for his interdisciplinary works while
Michelangelo is remembered primarily for his art.

Explanation:

● A) This choice provides an overview of the two figures but doesn’t draw a clear
distinction between their respective contributions to the Renaissance.

● B) While this option does differentiate between da Vinci's and Michelangelo's


contributions, it overly simplifies Michelangelo's role by only mentioning the
Sistine Chapel.

www.strategictestprep.com ♟️ 1-833-300-PREP (7737) ♟️[email protected] 294


● C) This choice effectively contrasts the interdisciplinary nature of da Vinci's
contributions with Michelangelo's primarily artistic achievements, which aligns
well with the notes provided.

● D) This choice, similar to option B, overly emphasizes da Vinci's anatomical


sketches and Michelangelo’s Sistine Chapel work, without sufficiently capturing
the broader contrast in their contributions to the Renaissance.

18. Answer: B) The Amazon Rainforest is about 5.5 million square kilometers, making
it considerably smaller than the Sahara Desert, which covers about 9.2 million square
kilometers.

Explanation:

A) This choice highlights the vastness of the Sahara Desert but does not provide a
direct comparison with the size of the Amazon Rainforest.

B) This option directly compares the sizes of the Amazon Rainforest and the Sahara
Desert, which is consistent with the goal of the question.

C) This choice provides geographic context about both the rainforest and the desert
but doesn't directly compare their sizes.

D) This option states a general fact about the significance and location of both the
desert and the rainforest but doesn't focus on their sizes.

19. Answer: B) Gabriela Garcia uses the term "lumisculpt," derived from "luminous"
and "sculpture," to describe her artworks that include light, such as the marble statue
"Aurora" that radiates a soft glow.

Explanation:

A) This option briefly mentions the "lumisculpt" technique and one of Gabriela Garcia's
artworks but does not define the term.

B) This choice provides both a definition of "lumisculpt" (derived from "luminous" and
"sculpture") and a specific example ("Aurora"), making it the most comprehensive
answer to the student's goal.
www.strategictestprep.com ♟️ 1-833-300-PREP (7737) ♟️[email protected] 295
C) While this option offers an example of a "lumisculpt" piece by Gabriela Garcia, it
doesn't provide a clear definition of the term.

D) This choice vaguely refers to Gabriela Garcia's unique style but does not clearly
define "lumisculpt" or provide a detailed example.

20. Answer: D) Maria Tallchief was the first Native American to achieve the status of
prima ballerina, showcasing her groundbreaking career in ballet.

Explanation:

A) This choice mentions some of Tallchief's significant roles but doesn't specifically
emphasize the groundbreaking aspect of her career.

B) While this statement recognizes her contributions to American culture with an


award, it doesn't hone in on the trailblazing nature of her ballet career.

C) This provides a general commendation of her dancing style without focusing on


what made her career revolutionary.

D) By highlighting that Tallchief was the first Native American to become a prima
ballerina, this choice effectively emphasizes the pioneering nature of her career, making
it the best answer.

21. Answer: C) Drones, being small and remote-controlled, can navigate narrow and
shallow areas of the ocean that are inaccessible to larger oceanographic vessels.

Explanation:

A) This choice mentions the increased use of drones but doesn't specify the
reason for their growing popularity or their advantage over large vessels.

B) This option focuses on the limitation of large vessels and does not contrast
that with the advantage of drones directly.

C) This statement highlights the specific advantage of drones: their ability to


access narrow and shallow areas which large vessels cannot. It directly
addresses the goal of the question.

www.strategictestprep.com ♟️ 1-833-300-PREP (7737) ♟️[email protected] 296


D) This option provides a general observation about the deployment methods
without emphasizing the specific advantages of drones.

22. Answer: C) A coming-of-age novel set in 1960s Tokyo, "Norwegian Wood" (1987)
is one of Haruki Murakami's most esteemed works, reflecting his trademark style of
blending magical realism with deep psychological introspection.

Explanation:

● A) This option introduces Murakami and his novel but omits the significance and
depth of "Norwegian Wood" within Murakami's broader literary style.

● B) This choice offers some context about the novel and its setting. It introduces
the novel and Murakami's literary style but doesn't emphasize the novel's
acclaim or significance in his oeuvre.

● C) This statement is comprehensive. It describes the novel's setting, era, and


themes, notes its acclaim and importance among Murakami's works, and
highlights Murakami's distinct literary style. It effectively introduces both
"Norwegian Wood" and its significance in Murakami's collection of works.

● D) This choice touches upon the novel's themes and provides some background
about the author, but it does not mention the novel's acclaim or its
representation of Murakami's broader literary style.

23. Answer: C) Despite Canberra (population 431,380) having a larger population than
Wellington (population 212,700), Wellington accounts for a larger percentage of its
country’s total population.

Explanation:

● A) This choice states the populations of the two capitals but does not provide
any direct comparison or insight into their relative significance within their
respective countries.

● B) Again, this choice mentions the population figures but does not emphasize
the relative percentages of the two capitals’ populations with respect to their
national populations.

www.strategictestprep.com ♟️ 1-833-300-PREP (7737) ♟️[email protected] 297


● C) This choice directly contrasts the populations of the two cities, highlighting
that, even though Canberra has a larger absolute population, Wellington
comprises a larger proportion of its national population. This emphasizes the
relative significance effectively.

● D) While this choice gives the percentages that each city’s population
represents within its country, it is presented in a more convoluted manner,
making it less direct than choice C.

24. Answer: C) Historians believe a complex system of aqueducts and archimedes


screws was used in the Hanging Gardens, highlighting the technological marvels of
ancient Mesopotamia.

Explanation:

● A) This option highlights the beauty and design of the gardens but does not
delve into the technological achievements associated with it.

● B) This choice provides a reason for the construction of the gardens but does not
explain what made them a technological or unique achievement.

● C) This choice directly emphasizes the technological innovation and marvel of


the Hanging Gardens by mentioning the complex system of aqueducts and
archimedes screws. This gives readers an idea of why the gardens were so
significant in terms of engineering and design.

● D) While stating that the Hanging Gardens were one of the Seven Wonders
does highlight their significance, the mention of their location in present-day
Iraq does not provide insight into their technological achievement.

25. Answer: C) Using satellite imaging, Dr. Helena Martins and her team were able to
assess the impact of deforestation on the Amazon’s oxygen production.

Explanation:

● A) This option provides specific results from Dr. Martins' study but does not
emphasize the role of technology in their findings.

www.strategictestprep.com ♟️ 1-833-300-PREP (7737) ♟️[email protected] 298


● B) This choice gives general information about the Amazon Rainforest's
significance but does not touch on the technological methods used in studying
it.

● C) This option directly emphasizes the role of new-age technology (satellite


imaging) in assessing environmental changes in the Amazon Rainforest, which
aligns with the student's objective.

● D) While this choice provides context regarding the importance of the Amazon,
it does not highlight the technological methods used in the study.

26. Answer: B) Spanning a distance of approximately 9,289 kilometers (5,772 miles),


the Trans-Siberian Railway is the world's longest railway line.

Explanation:

● A) This option provides a historical timeline of the railway's construction and its
starting and end points but does not directly emphasize its length.

● B) This choice directly emphasizes the distance covered by the railway,


highlighting the impressive length of 9,289 kilometers, and also mentions its
status as the world's longest railway line, making it the best answer.

● C) This choice provides the starting and ending points of the railway but does
not emphasize or even mention its distance.

● D) This option focuses on the year of completion and its significance in railway
construction but does not mention or emphasize the railway's length.

27. Answer: C) Dr. Lisa Harmon's study in 2022 was aimed at determining the primary
causes of coral bleaching in the Great Barrier Reef.

Explanation:

www.strategictestprep.com ♟️ 1-833-300-PREP (7737) ♟️[email protected] 299


A) This option provides information on the methods Dr. Harmon used in her
study but does not directly state the main purpose of the research.

B) This choice describes the state of the Great Barrier Reef but doesn't specify
the aim of Dr. Harmon's research.

C) This choice directly states the focus of the research, emphasizing Dr.
Harmon's goal to identify the primary causes of coral bleaching in the reef. It
captures the central objective of the research, making it the best answer.

D) This option discloses the findings of Dr. Harmon's research but doesn't clarify
the initial aim or purpose of the study.

28. Answer: B) To understand the swift acceleration of cheetahs, Dr. Elena Roberts
employed high-speed cameras to capture the biomechanics of their movement in the
wild.

Explanation:

A) This option touches upon the context of Dr. Roberts' study but doesn't
specifically highlight the technique she used (high-speed cameras).

B) This choice clearly states the method (high-speed cameras) used by Dr.
Roberts to study the acceleration of cheetahs, making it the most appropriate
answer.

C) This option outlines a conclusion from Dr. Roberts' research but doesn't
emphasize the method she used for the study.

D) Although this choice provides context about the rapid acceleration of


cheetahs and mentions Dr. Roberts' analysis, it doesn't specify the use of
high-speed cameras, which is central to the question's focus.

29. Answer: B) The painting "Portrait of Adele Bloch-Bauer I" by Gustav Klimt was at
the center of Maria Altmann's legal battle and was later sold for $135 million.

Explanation:

www.strategictestprep.com ♟️ 1-833-300-PREP (7737) ♟️[email protected] 300


A) While this option introduces Maria Altmann and her objective to reclaim a painting,
it doesn't highlight the significance or value of the painting itself.

B) This is the correct choice. It directly emphasizes the significance of the painting in
question, naming the artwork and its artist, and noting its substantial sale price.

C) This choice discusses the legal success of Maria Altmann but doesn't directly
highlight the importance or value of the painting "Portrait of Adele Bloch-Bauer I."

D) This option emphasizes the legal challenge faced by Maria Altmann and her
eventual success but doesn't underscore the significance of the painting itself.

30. Answer: C) To understand the adoption of sustainable agricultural methods by


small family farms, Dr. Lana Fields surveyed 200 such farms, focusing on soil health,
water usage, and crop rotation techniques.

Explanation:

● A) This option provides a generalized observation on small family farms and


their advantages using sustainable techniques but doesn't focus on the
methodology of the study.

● B) This choice speaks about Dr. Lana Fields' advocacy but doesn't delve into the
methodology she used for the study.

● C) This is the correct choice. It directly highlights the methodology, detailing


how Dr. Fields set out to understand the adoption of sustainable techniques by
small family farms.

● D) This option provides results from the study but does not give specific details
on how the study was conducted or what the methodology was.

31. Answer: C) Since 1995, Ria Paki and Atawhai Ruru have established cultural
centers and galleries, respectively, dedicated to Maori culture and history.

Explanation:

● A) While this option mentions both Ria Paki and Atawhai Ruru, it does not
emphasize the timing or specific contributions of their efforts.

www.strategictestprep.com ♟️ 1-833-300-PREP (7737) ♟️[email protected] 301


● B) This choice acknowledges the significance of Paki's and Ruru's roles but
doesn't specify the timing or the exact nature of their contributions.

● C) This is the correct answer. It directly emphasizes the timing (since 1995) of
their contributions and what those contributions were: establishing a cultural
center and a gallery.

● D) This option provides general information about Maori culture but does not
address the specific contributions of Paki and Ruru.

32. Answer: A) Dr. Maria Gonzales and Dr. Andrew Smith of the University of
Monterrey aimed to understand the effect of climate change on the migration patterns
of monarch butterflies.

Explanation:

A) This is the correct answer. It succinctly states the main purpose of the study
conducted by Gonzales and Smith, which is to understand the effect of climate change
on monarch butterfly migration.

B) This option provides details about the methodology used by the researchers
(capturing, tagging, and tracking) but doesn't directly state the overarching aim of the
study.

C) This choice offers a conclusion or finding from the study (that warmer temperatures
might be causing butterflies to travel longer distances) but does not present the
primary objective of the research.

D) This answer focuses on the University of Monterrey's interest in the topic, but it
doesn't convey the specific goal of the study conducted by Gonzales and Smith.

33. Answer: C) Both Mizu in Tokyo and Umi in Osaka, owned by Akiko Suzuki, specialize
in traditional Japanese seafood dishes.

Explanation:
www.strategictestprep.com ♟️ 1-833-300-PREP (7737) ♟️[email protected] 302
A) This choice simply provides information about the opening of the two restaurants
but doesn't emphasize the thematic similarity between them.

B) This option highlights the signature dishes of the two restaurants, which might
suggest a difference rather than a common focus.

C) This is the correct answer. It directly captures the common focus of both restaurants:
they specialize in traditional Japanese seafood dishes.

D) While this choice provides a timeline of the openings of the two restaurants, it
doesn't emphasize their common focus on traditional Japanese seafood dishes.

34. Answer: C) Dr. Maria Soto from the University of Florida aimed to understand the
impact of rising sea temperatures on the health of corals.

Explanation:

A) This choice touches upon the importance of corals in marine ecosystems but does
not directly address the purpose of Dr. Soto's research, which was focused on the
impact of rising sea temperatures on coral health.

B) This choice describes one of the findings from Dr. Soto's research rather than the
aim or purpose behind the research.

C) This is the correct answer. It clearly captures the primary objective of Dr. Soto's
research: to understand the effects of rising sea temperatures on coral health.

D) Similar to option B, this choice provides a conclusion or result of the study but
doesn't directly address the primary aim or purpose of the research.

35. Answer: C) Stellar black holes form from the collapse of massive stars, whereas
primordial black holes are theorized to have formed shortly after the Big Bang.

Explanation:

A) This choice is a generalized statement about black holes and doesn't highlight any
difference between the types of black holes.

www.strategictestprep.com ♟️ 1-833-300-PREP (7737) ♟️[email protected] 303


B) This choice mentions where stellar and supermassive black holes can be found but
doesn't really contrast their origins.

C) This is the correct answer as it emphasizes the differing origins of stellar and
primordial black holes.

D) This statement provides information only about supermassive black holes and
doesn't draw any distinction between origins.

36. Answer: D) In "Bruises: The Data We Don't See," Giorgia Lupi juxtaposed her
friend's son's medical journey with emotional experiences, hand-drawing the data to
show treatments, emotions, and symptoms over time.

Explanation:

● A) This choice introduces Giorgia Lupi and the title of her work but fails to
provide a detailed description of the artwork.

● B) This choice describes the artwork but omits the context behind the data (that
it pertains to her friend's son's illness).

● C) This statement offers a general description but doesn't provide the specificity
and depth about the content and style of the artwork.

● D) This is the correct answer because it effectively captures the essence of


"Bruises: The Data We Don't See." It provides context (her friend's son's medical
journey), the artwork's style (hand-drawn), and its content (treatments,
emotions, and symptoms).

37. Answer: C) With Lidar technology, archaeologist Marie Dupont uncovered


previously hidden sketches in the famous Lascaux caves.

Explanation:

www.strategictestprep.com ♟️ 1-833-300-PREP (7737) ♟️[email protected] 304


● A) While this choice mentions the method used by Dupont and the location, it
doesn't specify the significant outcome of her study: the discovery of new
sketches.

● B) This option is too vague. It doesn't specify who conducted the study or what
the findings were, only that lasers were used for a study.

● C) This is the correct answer. It combines the methodology (Lidar technology),


the researcher (Marie Dupont), the significant location (Lascaux caves), and the
significant outcome (discovery of previously hidden sketches).

● D) This choice provides context about the age of the caves and mentions
Dupont's use of laser technology, but it doesn't convey the groundbreaking
nature of her discovery.

38. Answer: C) OLED screens can produce perfect blacks and higher contrast ratios due
to their ability to self-illuminate, eliminating the need for a backlight.

Explanation:

● A) While this choice highlights a difference between OLED and LCD screens, it
does not adequately emphasize the advantage or result of the absence of a
backlight in OLED screens.

● B) This option gives a detail about how OLEDs work but doesn't fully illustrate
the advantage that this feature offers over traditional LCDs.

● C) This is the correct answer. It directly links the unique feature of OLEDs
(self-illumination) with the benefit it brings (perfect blacks and higher contrast
ratios), offering a clear advantage over LCDs.

● D) This choice gives context about the prevalence of OLED technology but
doesn't specifically outline its advantage over LCD screens.

39. Answer: C) The Agricultural Exchange Theory posits that the Silk Road facilitated
the exchange of agricultural techniques, significantly contributing to global food
diversification.

www.strategictestprep.com ♟️ 1-833-300-PREP (7737) ♟️[email protected] 305


Explanation:

● A) This option mentions Angela Wu and the importance of the Silk Road for
exchanging goods, but it doesn't clearly explain the Agricultural Exchange
Theory or its implications for global food diversification.

● B) While Marco Polo's documentation is a valuable resource about the Silk


Road, this choice doesn't touch upon the Agricultural Exchange Theory or its
significance.

● C) This is the correct answer. It directly explains the Agricultural Exchange


Theory and its relevance to the exchange of farming techniques and the broader
impact on global food varieties.

● D) This choice speaks to the broader importance of the Silk Road in history but
doesn't focus on the Agricultural Exchange Theory or the specific topic of
agricultural techniques.

40. Answer: B) Maria Renata used modern materials to construct a replica of da Vinci's
"flying machine" and demonstrated that with modifications and current technology, it
could hover for short durations.

Explanation:

● B) This choice directly addresses the feasibility of da Vinci's design in the


context of modern technology. It uses relevant details to establish that while the
original design wasn't flight-ready during da Vinci's time, with contemporary
materials and some adjustments, it could achieve hovering for brief periods.

● A) This option solely focuses on the design's limitations in the 16th century,
without mentioning its potential feasibility with modern technology.

● C) This choice, similar to A, stresses the design's incapability during da Vinci's


era and its inspiration from birds but does not touch upon its potential in the
current day with advancements in technology.

www.strategictestprep.com ♟️ 1-833-300-PREP (7737) ♟️[email protected] 306


● D) While this answer provides a nod to da Vinci's innovative approach, it doesn't
specifically address the design's feasibility using contemporary technology.

41. Answer: A) Jenna Rosenthal’s "Gazing Beyond Frames: Women Artists in MoMA"
examines the contributions of female artists whose works are showcased at the
Museum of Modern Art.

Explanation:

A) This option is the best choice because it succinctly introduces the theme and
focus of Jenna Rosenthal’s book, targeting an audience already aware of MoMA.

B) This choice redirects the emphasis towards the inception of MoMA and its
collection, instead of highlighting the core theme of Jenna Rosenthal’s book.

C) While this answer touches upon Jenna Rosenthal’s book, it adds unnecessary
details about MoMA's establishment and its art collection, which might be
redundant for an audience already acquainted with the museum.

D) This option is overly specific as it mentions only Frida Kahlo. It doesn't


provide a broad introduction to the primary theme of Jenna Rosenthal’s book.

42. Answer: B) Neither of the cheetah's hunting methods involves ambushing its prey
from a concealed position.

Explanation:

A) This choice focuses on the difference between cheetahs and other large cats,
without drawing a direct parallel between the two methods cheetahs use.

B) This is the correct choice as it directly points out the similarity between the
two hunting methods: neither involves an ambush from a concealed position.

C) This option describes both methods but doesn't directly emphasize the
similarity between them. Instead, it just presents them as separate facts.

D) This choice emphasizes the cheetah's speed without making a clear


comparison between its two hunting methods.

www.strategictestprep.com ♟️ 1-833-300-PREP (7737) ♟️[email protected] 307


IX. Standard English Convention Questions 👩‍🏫
Standard English Convention Questions = Your Grammar Questions! You will find these
questions in each module, starting around #15 - #21. These are quick, easy points if
you know all the rules!

Please note that there is a separate key to these lessons with notes and answers!

Lesson #1: Complete or Incomplete Sentence?

Before we get into all the grammar rules, it is important that you are able to
identify if a sentence is complete or not. You need these three things to have a
complete sentence:

1. _____________________

2.______________________

3._______________________

Identify the subject, verb, and complete thought in the following sentence:

They ran down to the water.

Subject:

Verb:

Complete Thought:

www.strategictestprep.com ♟️ 1-833-300-PREP (7737) ♟️[email protected] 308


Practice: Is It a Sentence?

For each statement below, circle "Sentence" if it can stand alone as an independent
sentence and "Fragment" if it cannot. Once you have read the statement carefully,
spend no more than a couple of seconds selecting your answer. Try to complete the
full exercise in under two minutes.

1. Louis Armstrong was one of the greatest jazz musicians of the twentieth century.

Sentence Fragment

2. He was one of the greatest jazz musicians of the twentieth century.

Sentence Fragment

3. Louis Armstrong, who was one of the greatest jazz musicians of the twentieth
century.

Sentence Fragment

4. Who was one of the greatest jazz musicians of the twentieth century.

Sentence Fragment

5. Louis Armstrong, who was one of the greatest jazz musicians of the twentieth
century, was a vocalist as well as a trumpet player.

Sentence Fragment

6. Today, he is considered one of the greatest jazz musicians of the twentieth century.

Sentence Fragment

www.strategictestprep.com ♟️ 1-833-300-PREP (7737) ♟️[email protected] 309


7. He is, however, considered one of the greatest jazz musicians of the twentieth
century.

Sentence Fragment

8. He is now considered one of the greatest jazz musicians of the twentieth century.

Sentence Fragment

9. Because of his virtuosic trumpet skills, Louis Armstrong is considered one of the
greatest jazz musicians of the twentieth century.

Sentence Fragment

10. Although he was one of the most virtuosic trumpet players of his generation.

Sentence Fragment

11. Many people considering Louis Armstrong the greatest jazz musician of all time.

Sentence Fragment

12. Many of them consider him the greatest jazz musician of all time.

Sentence Fragment

13. Many consider him the greatest jazz musician of all time.

Sentence Fragment

www.strategictestprep.com ♟️ 1-833-300-PREP (7737) ♟️[email protected] 310


14. Many of whom consider him the greatest jazz musician of all time.

Sentence Fragment

15. Having shown an unusual gift for music early in his childhood, Louis Armstrong,
who was born in New Orleans on August 4, 1901.

Sentence Fragment

16. Having shown an unusual gift for music early in his childhood, Louis Armstrong,
who was born in New Orleans on August 4, 1901, went on to become one of the
greatest jazz musicians of the twentieth century.

Sentence Fragment

Check your answers on the next page.

www.strategictestprep.com ♟️ 1-833-300-PREP (7737) ♟️[email protected] 311


Answers to Is it a Sentence? Exercise

1. Sentence 9. Sentence

2. Sentence 10.Fragment

3. Fragment 11.Fragment

4. Fragment 12.Sentence

5. Sentence 13.Sentence

6. Sentence 14.Fragment

7. Sentence 15.Fragment

8. Sentence 16.Sentence

www.strategictestprep.com ♟️ 1-833-300-PREP (7737) ♟️[email protected] 312


Lesson #2: Know Your Verb Tenses
It is important to identify and understand the difference between verb tenses and how
they are used. Please complete the table below by filling in the example column.

Verb Tense When Used Example

Present Happening at that


moment, right now

Past Happened at one moment


in time but is over

Future Yet to happen

Past Perfect Ongoing in the past, but


does not continue in the
present

Present Perfect Ongoing in the past, but


continues in the present

Practice: Match up the phrase to the correct tense by drawing a line to connect
them. Then, check your answers on the next page.

Now,

Yesterday, she had been

Since she was a little girl, she is

For the past twenty minutes, she was

Next Year, she will be

she has been

www.strategictestprep.com ♟️ 1-833-300-PREP (7737) ♟️[email protected] 313


Answers to Matching Exercise

www.strategictestprep.com ♟️ 1-833-300-PREP (7737) ♟️[email protected] 314


Lesson #3: Subject/Verb Agreement

How will you know you are on a subject/verb agreement question? Well, you will have
verb tenses as your answer choices!

The important thing is to be able to pinpoint what is doing the action word in the
sentence (subject). Everything between the subject and the verb should be crossed
out so you can easily see what is supposed to be the correct tense!

Example 1:

The box of chocolates are on the table.

Example 2:

The commissioner, along with his 20 staff members, run a tight campaign against the
incumbent.

www.strategictestprep.com ♟️ 1-833-300-PREP (7737) ♟️[email protected] 315


Practice: Try these Subject/Verb Agreement Questions then Check Your Answers on
the following page!

Example 1

Example 2

Although the Concorde was retired in 2003, a plane capable of flying halfway around
the world in a mere four hours could soon exist. For engineers, the elimination of sonic
booms _ _ one of the biggest challenges involved in building the new craft. Some of
them claim, however, that they have found a way of reducing the amount of noise the
planes make.

A) is
B) are
C) have been
D) were

www.strategictestprep.com ♟️ 1-833-300-PREP (7737) ♟️[email protected] 316


Example 3

Example 4

www.strategictestprep.com ♟️ 1-833-300-PREP (7737) ♟️[email protected] 317


Example 5

For several weeks each year, more than 200 artists from 60 countries gather in Santa
Fe to offer handmade masterworks. Every July, one of the world's largest folk-art
festivals _____ together artists from every corner of the globe for a vast and colorful
international bazaar.

A) brings

B) bring

C) have brought

D) bringing

Example 6

www.strategictestprep.com ♟️ 1-833-300-PREP (7737) ♟️[email protected] 318


Subject/Verb Agreement Answers

1. B (the subject is ‘forms’)


2. A (the subject is ‘elimination’)
3. A (the subject is ‘trend’)
4. C (the subject is ‘works’)
5. A (the subject is ‘one’)
6. A (the subject shifts from focusing on ailments to ‘absenteeism.’ It is the
absenteeism that is costly for employers.)

www.strategictestprep.com ♟️ 1-833-300-PREP (7737) ♟️[email protected] 319


Lesson #4: Parallelism

In parallelism, everything matches up. Here are some instances where parallelism
comes into play:

● Listing 3 things and they are all worded in the same way
● All verbs are in the same tense
● Comparing two things: Need an Apples to Apples, not Oranges to Apples
comparison
● Narration stays the same throughout (1st person, 3rd person, etc)

Example 1:

He ran to the store, bought a cup of coffee, and had been jogging to work.

Example 2:

Steph Curry’s jumpshot is more accurate than Lebron James.

Example 3:

There was a table along the first wall, a mirror on the second wall, and the third wall
had an exit door.

www.strategictestprep.com ♟️ 1-833-300-PREP (7737) ♟️[email protected] 320


Practice: Try these Parallelism Questions then Check Your Answers on the
Following Page!

Example 1

Example 2

Healthy marine environments result from a precise balance of factors and are therefore
vulnerable to threats from a variety of sources. For example,________ can all lead to
the creation of ocean waters low in oxygen and inhospitable to marine life.

Which choice completes the text so that it conforms to the conventions of Standard
English?

A) changes in wind circulation, runoff from sewage, and accumulating fertilizers

B) changing wind circulation, runoff from sewage, and accumulating fertilizers

C) changing wind circulation, having runoff from sewage, and to accumulate


fertilizers

D) changes in wind circulation patterns, runoff from sewage, and accumulation of


fertilizers

www.strategictestprep.com ♟️ 1-833-300-PREP (7737) ♟️[email protected] 321


Example 3

Example 4

www.strategictestprep.com ♟️ 1-833-300-PREP (7737) ♟️[email protected] 322


Example 5

Unlike some animals, most lizards have a highly developed sense of vision. As a result,
they are able to use clear body language and ____ their colors in order to
communicate.

Which choice completes the text so that it conforms to the conventions of Standard
English?

A) alter
B) altering
C) they alter
D) altered

Example 6

www.strategictestprep.com ♟️ 1-833-300-PREP (7737) ♟️[email protected] 323


Example 7

Example 8

Example 9

Kite-flying has a long history in Japan: according to legend, the first kites flew nearly
1,400 years ago. Since that time, kite-flying ____ a delightful tradition.

A) had remained
B) has remained
C) remained
D) remains

www.strategictestprep.com ♟️ 1-833-300-PREP (7737) ♟️[email protected] 324


Parallelism Practice Answers

1. B (the verb matches with ‘serving’ and ‘showing’)


2. D (the only option where all three things listed start with the same type of word
- in this case, they all start with a noun so they match up).
3. C (compares one training period to one training period)
Why the other answers are wrong:
● A - compares one training period to multiple training periods (not
parallel)
● B - too wordy and redundant. We already know we are comparing and
don’t need to say ‘compared with’
● D - we would be comparing a training period to physicians (not parallel)
4. D (comparing the moves to the moves)
5. A (‘alter’ must match with the verb ‘use’)
6. C (‘scare’ must match with the verb ‘damage’ because the focus shifts to what
the rodents are doing)
7. B (‘included’ must match with the verb ‘dominated’)
8. D (‘Today,’ needs a present tense verb ‘is’)
9. B (‘Since that time,’ needs a present perfect tense verb ‘has remained’ since it is
ongoing in the past and continuing in the present)

www.strategictestprep.com ♟️ 1-833-300-PREP (7737) ♟️[email protected] 325


Lesson #5: Comma Placement

With comma placement questions, the key is to be proactive instead of reactive. If you
read all of the answer choices first and then try to determine which is right, it can get
confusing.

Strategy: Read the sentence out loud and listen to where you pause. That is where the
commas need to go.

Practice the Strategy: Determine where commas are needed in the underlined
portion by using the strategy outlined above.

Example #1:

The advantages of growing up in a bilingual home can start as early as six months of
age. A study conducted by York University's Institute of Health found that infants who
are regularly exposed to more than one language show better attentional control than
infants being raised in monolingual environments. The findings suggest that early
exposure to multiple languages could set the stage for lifelong cognitive benefits.

Example #2

Pterosaurs, the first flying vertebrates had wings made of a membrane that ended in
over- elongated fingers. These reptiles filled the skies between 66 and 220 million
years ago, their bodies ranging in size from less than a foot to the length of a small
airplane.

Example #3

The first recorded mention of the Agojie-the all-female military regiment in the
kingdom of Dahomey-dates to 1729. However, it is possible that the unit was formed
even earlier, toward the beginning of Dahomey's existence when a troupe consisting of
women who were already experienced elephant-hunters was created.

www.strategictestprep.com ♟️ 1-833-300-PREP (7737) ♟️[email protected] 326


Answers to Practice the Strategy Exercise:

Example #1: no commas needed - you can read right through it without pausing

The advantages of growing up in a bilingual home can start as early as six months of
age. A study conducted by York University's Institute of Health found that infants who
are regularly exposed to more than one language show better attentional control than
infants being raised in monolingual environments. The findings suggest that early
exposure to multiple languages could set the stage for lifelong cognitive benefits.

Example #2: need a comma after vertebrates - “the first flying vertebrates” is extra
information describing more about Pterosaurs. Thus, it needs to go between two
commas.

Pterosaurs, the first flying vertebrates, had wings made of a membrane that ended in
over- elongated fingers. These reptiles filled the skies between 66 and 220 million
years ago, their bodies ranging in size from less than a foot to the length of a small
airplane.

Example #3: comma after existence - you need to pause after existence

The first recorded mention of the Agojie-the all-female military regiment in the
kingdom of Dahomey-dates to 1729. However, it is possible that the unit was formed
even earlier, toward the beginning of Dahomey's existence, when a troupe consisting
of women who were already experienced elephant-hunters was created.

www.strategictestprep.com ♟️ 1-833-300-PREP (7737) ♟️[email protected] 327


Order Need Commas? Example

Description then NO Professional tennis player


subject Coco Gauff won the US Open
last year.

Subject then YES Coco Gauff, a professional


description tennis player, won the US
Open last year.

Please note that this is an important comma rule tested frequently.

Exercise: For each of the following, determine whether you need commas or not. If
you do need commas, place them in their respective spots.

1. President of the United States Jimmy Carter had the lowest rating for an
incumbent going into an election.

Commas No Commas

2. Beyonce Knowles lead singer of Destiny’s Child emerged with one of the most
successful solo careers ever.

Commas No Commas

www.strategictestprep.com ♟️ 1-833-300-PREP (7737) ♟️[email protected] 328


Answers:
1. No Commas
2. Commas - Beyonce Knowles, lead singer of Destiny’s Child, emerged with one
of the most successful solo careers ever.

Comma Placement Practice: Try these questions, then check your answers on the
following page.

Example 1

Jamaica Kincaid (born May 25, 1949) is a novelist, essayist, and gardener. Born Elaine
Potter Richardson in Antigua, she came to the United States at the age of 17 to work
as an au pair in Westchester : County, New York. She eventually won a scholarship to
Franconia College in New Hampshire but returned to New York City to write. In 1985,
she published ____a semi autobiographical story of a young girl growing up in Antigua
that won acclaim for the honesty of its depiction of familial relationships.

Which choice completes the text so that it conforms to the conventions of Standard
English?
A) The novel, Annie John,
B) The novel Annie John,
C) The novel Annie John
D) The novel: Annie John,

Example 2

Ada Lovelace and her ______ were two of the most influential figures in history
computer science. After Babbage sketched out his ideas for an "analytical engine,"
Lovelace demonstrated that the machine might be able to carry out variety of complex
tasks.

Which choice completes the text so that it conforms to the conventions of Standard
English?

A) acquaintance , Charles Babbage,


B) acquaintance Charles Babbage
C) acquaintance Charles Babbage,
D) acquaintance, Charles Babbage

www.strategictestprep.com ♟️ 1-833-300-PREP (7737) ♟️[email protected] 329


Example 3

Example 4

www.strategictestprep.com ♟️ 1-833-300-PREP (7737) ♟️[email protected] 330


Example 5

Although there was no contact between Japan and Britain during the prehistoric era,
there are surprising parallels between them. In both places, inhabitants built stone
circles, crafted elaborate pots, and used flaked stone tools. Furthermore, the _ _ bear
striking similarities to the circle at Stonehenge.

Which choice completes the text so that it conforms to the conventions of Standard
English?

A) astonishing Japanese stone circles at Oyu,


B) astonishing, Japanese stone circles at Oyu
C) astonishing Japanese, stone circles at Oyu
D) astonishing Japanese stone circles at Oyu

In the popular imagination, ants are often depicted as brave soldiers or dutiful factory
workers. According to ____ however, this portrayal is a human fiction.

Example 6

Wallace Fowlie, one of Stendhal's most sympathetic critics, claimed that the writer's
genius was due in part to the way he blurred the line between his own interior life and
those of his characters. Stendhal was, __________ "literally inhabited by his
creations,"discovering himself as he discovered them.

A) Fowlie argues,
B) Fowlie argues
C) Fowlie argues-
D) Fowlie, argues

www.strategictestprep.com ♟️ 1-833-300-PREP (7737) ♟️[email protected] 331


Example 7

www.strategictestprep.com ♟️ 1-833-300-PREP (7737) ♟️[email protected] 332


Answers to Comma Placement Practice
Example 1: B
Example 2: B (to put Charles Babbage between two commas would mean his name is
non-essential. If we took his name out, we would be missing context because we
wouldn’t know who Ada Lovelace’s acquaintance was. Thus, no commas are needed
here).
Example 3: D
Example 4: C
Example 5: D
Example 6: A
Example 7: C

www.strategictestprep.com ♟️ 1-833-300-PREP (7737) ♟️[email protected] 333


Lesson #6: Lead-Ins and Their Subjects

A “lead-in” is an introduction to the sentence that is leading up to the subject and what
the sentence is about. The lead-in points directly to the subject and the subject must
come directly after the introduction comma.

Example:

Running down the stairs, the little girl almost slipped and fell.

Lead-in:

Subject:

Basically, ask yourself, “who or what could be running down the stairs?” Make sure the
next word makes sense as the subject!

Exercise: Circle the word that would make sense to come directly after the lead-in
as the subject.

1. Arguably the most influential artwork of the Renaissance period, __________

Michelangelo the David the David’s shape

2. As the country with the most megacities in the world, ___________

China’s population people in China China

3. Sneaking down the chimney in the middle of the night, ___________

Santa Santa’s excitement the presents

www.strategictestprep.com ♟️ 1-833-300-PREP (7737) ♟️[email protected] 334


Answers:
1. The david
2. China
3. Santa

Lead-in/subject Practice: Try these questions, then check your answers on the
following page.

Example 1

www.strategictestprep.com ♟️ 1-833-300-PREP (7737) ♟️[email protected] 335


Example 2

Example 3

www.strategictestprep.com ♟️ 1-833-300-PREP (7737) ♟️[email protected] 336


Example 4

www.strategictestprep.com ♟️ 1-833-300-PREP (7737) ♟️[email protected] 337


Answers to Lead-In Practice
1. B
2. A
3. C
4. D

www.strategictestprep.com ♟️ 1-833-300-PREP (7737) ♟️[email protected] 338


Lesson #7: Punctuation

There are four punctuation rules you must know. If you know these rules, your success
rate will be high on these questions!

Rule #1 - The Colon

_________________: __________________

Rule #2 - The Semicolon

_________________; __________________

*The semicolon can also be used to __________________________________.

Ex.

Rule #3 - The Comma Rule

_____________, _____________ *
*unless the second part starts with a FANBOYS (For, And, Nor, But, Or, Yet, So)

Rule #4 - Dashes/Parentheses/Commas

Either two commas, two dashes, or two parentheses should be used to separate
non-essential information, you cannot mix and match.

www.strategictestprep.com ♟️ 1-833-300-PREP (7737) ♟️[email protected] 339


Rule #4 Example 1

The Tower of London, which was begun by William the Conqueror in 1078, is one of
the largest and most imposing fortifications in England.

Correct Incorrect

Rule #4 Example 2

The Tower of London - which was begun by William the Conqueror in 1078 - is one of
the largest and most imposing fortifications in England.

Correct Incorrect

Rule #4 Example 2

The Tower of London, (which was begun by William the Conqueror in 1078), is one of
the largest and most imposing fortifications in England.

Correct Incorrect

Rule #4 Example 3

The Tower of London - which was begun by William the Conqueror in 1078, is one of
the largest and most imposing fortifications in England.

Correct Incorrect

www.strategictestprep.com ♟️ 1-833-300-PREP (7737) ♟️[email protected] 340


Rule #4 Exercise Answers

1. Correct

2. Correct

3. Incorrect

4. Incorrect

Strategy: Make sure to use process of elimination! Weed out the wrong answers to get
to the correct answer.

Punctuation Practice Questions: Try these below and then check your answers on
the following page.

Example 1
Judith Jamison decided on a career in dance only after three semesters of studying
psychology at Fisk _____ her education at the Philadelphia Dance Academy. In 1964,
she was spotted by choreographer Agnes de Mille, who invited her to appear in a
performance with the American Ballet Theater.

A) University, she completed


B) University; and she completed
C) University, completing
D) University; completing

www.strategictestprep.com ♟️ 1-833-300-PREP (7737) ♟️[email protected] 341


Example 2

Wrangell-St. Elias National Park, the largest national park in the United States,
represents everything compelling about Alaska. It is _ _ than Belgium. It showcases
towering mountains such as Mount St. Elias, which stands more 18,000 feet tall, as
well as glaciers.

A) immense; larger, in fact,


B) immense, larger-in fact
C) immense-larger, in fact,
D) immense-larger, in fact-

Example 3

The tower of London was constructed as a prison in the eleventh century (the year__
but over the centuries it has been used as everything from a treasury to an armory.

Which choice completes the text so that it conforms to the conventions of Standard
English?

A) (1078 to be exact);
B) 1078, to be exact,
C) 1078 to be exact),
D) 1078 to be exact-

Example 4

In surveys, a significant majority of people usually agree that cycling is a desirable


means of transit, especially in densely populated areas. Practically speaking, however,
there is often a large gap between desired and actual _ _ for example, only 20% of
short- distance trips in residential environments are made by bicycle.

Which choice completes the text so that it conforms to the conventions of Standard
English?
www.strategictestprep.com ♟️ 1-833-300-PREP (7737) ♟️[email protected] 342
A) numbers in Germany
B) numbers: in Germany
C) numbers, in Germany,
D) numbers in Germany,

Example 5

Facing extreme conditions such as starvation and stress, some bacteria enter a
dormant state in which life processes stop. Biologists have discovered how these
microbes assess environmental conditions before reviving _ _ carries implications for
evaluating life on Earth as well as other planets.

Which choice completes the text so that it conforms to the conventions of Standard
English?

A) themselves, this process


B) themselves this process:
C) themselves this process
D) themselves-a process that

Example 6

The production of ceramics in China, which began during the Neolithic period, has
continued until the present ____ range from bricks and tiles, to hand-built pots and jars
fired in kilns, to the refined porcelain wares made originally for the : imperial court and
later for export.

Which choice completes the text so that it conforms to the conventions of Standard
English?

A) day, although construction materials


B) day, construction materials
C) day: construction materials
D) day; construction materials

www.strategictestprep.com ♟️ 1-833-300-PREP (7737) ♟️[email protected] 343


Answers to Punctuation
1. C
2. C
3. C
4. B
5. D
6. D (the second part is not explaining the first part, so a colon won’t work here).

www.strategictestprep.com ♟️ 1-833-300-PREP (7737) ♟️[email protected] 344


Lesson #8: Punctuation with a Transition

"Strong" Transitions
Certain transitions-formally known as conjunctive adverbs- are considered "strong"
enough to begin a sentence. Common examples include however, therefore, thus,
consequently, moreover, and nevertheless. When used to begin a clause, these
transitions must follow a period or semicolon, never a comma.

Incorrect: The tomato is often treated as a vegetable, however, it is actually a fruit.

Correct: The tomato is often treated as a vegetable. However, it is actually a fruit.

Correct: The tomato is often treated as a vegetable; however, it is actually a fruit.

Two Step Process to Solving Transitions with Punctuation Questions

Step #1: Disregard the transition word and determine if you have two complete
sentences. Then eliminate any punctuation marks that will not work.

Two Complete Sentences: Need a ; or : or .


Not Both Complete: Need a , or -

Step #2: Determine if the contrast is between the 1st and 2nd sentence or the 2nd and
3rd sentence. The transition word should always be placed in the last sentence of the
two where the contrast occurs.

Contrast Between 1st and 2nd sentence: Place contrast word in 2nd sentence

Contrast Between 2nd and 3rd sentence: Place contrast word in 3rd sentence

www.strategictestprep.com ♟️ 1-833-300-PREP (7737) ♟️[email protected] 345


Example 1

Most theories claim the Moon formed out of the debris of a collision between the Earth
and a Mars-sized object known as Theia, which came together in orbit over a period of
months or years. A new simulation advances a different _ _ may have formed in a
matter of hours, when material from the Earth and Theia was launched into orbit
immediately after the impact.

A) theory, though the Moon


B) theory, though: the Moon
C) theory; though, the Moon
D) theory though the Moon

Example 2

Patients who participate in clinical trials hope to gain access to experimental


treatments that would not otherwise be available to them. Every clinical drug trial
conducted today randomly assigns patients to one of two _______ receives the actual
medication.

A) groups, however. Only one of which


B) groups; however, only one of which
C) groups, however, only one of them
D) groups; however, only one of them
I

www.strategictestprep.com ♟️ 1-833-300-PREP (7737) ♟️[email protected] 346


Example 3

Example 4

www.strategictestprep.com ♟️ 1-833-300-PREP (7737) ♟️[email protected] 347


Answers to Punctuation with Transitions
1. B
2. D
3. C (you do not have two complete sentences here, so you need a comma instead)
4. A

www.strategictestprep.com ♟️ 1-833-300-PREP (7737) ♟️[email protected] 348


Lesson #9: Apostrophes

There vs. Their vs. They’re

It’s vs. Its vs. Its’

Brother’s vs. Brothers’

Is the phrase mother’s uncles’ grammatically correct?

Example 1

Because the lemur, a small animal native to Madagascar, shares some traits with other
primates,_____ frequently mistaken for an ancestor of modern monkeys and apes.

A) its

B) it’s

C) its’

D) their

www.strategictestprep.com ♟️ 1-833-300-PREP (7737) ♟️[email protected] 349


Example 2

Example 3
Throughout his 1986 novel-in-verse, The Golden Gate, Vikram Seth maintains a
stubborn resistance to traditional structure. The narrator does not hesitate to interrupt
the ____ in order to explain a shift in the plot or to comment on the structure of the
book.

A) stories events

B) story's events

C) stories' event's

D) story's events'

www.strategictestprep.com ♟️ 1-833-300-PREP (7737) ♟️[email protected] 350


Answers to Apostrophes Examples
1. B
2. D
3. B

www.strategictestprep.com ♟️ 1-833-300-PREP (7737) ♟️[email protected] 351


Lesson #10: Additional Grammar Concepts

Affect vs. Effect

Example 1

Example 2

Some traditional assumptions about how to treat jellyfish stings have recently been
called into question: rinsing the ________areas with seawater for example only
spreads the stings to a larger area.

A) affected
B) effected
C) affecting
D) effecting

www.strategictestprep.com ♟️ 1-833-300-PREP (7737) ♟️[email protected] 352


Answers to Affect vs. Effect
1. A
2. A

Who vs. Whom

Many runners, even those _ _ train regularly, have a tendency to stick to an


established distance. As a result, they never develop a clear sense of their athletic
potential.

A) whom
B) who
C) which
D) they
www.strategictestprep.com ♟️ 1-833-300-PREP (7737) ♟️[email protected] 353
Answers to Who vs. Whom
1. A
2. B

www.strategictestprep.com ♟️ 1-833-300-PREP (7737) ♟️[email protected] 354


Standard English Convention Practice Questions
1. Eliot’s lines in The Waste Land, however, are not totally without rhythm;
rather, by repeating certain words and phrases and by staggering line length,
he creates a ______ that simultaneously expresses the poem’s despair and
grounds the reader in the poem’s language.

Which choice completes the text so that it conforms to the conventions of Standard
English?
A. cadence,
B. cadence
C. cadence;
D. cadence:

2. In the United States and worldwide, high school __________ have been
gradually improving over the last several years, which is a relief to those who
were worried that the educational system is breaking.
A) student’s scores
B) students’ scores’
C) students score’s
D) students’ scores

3. The global digital divide, which describes the uneven spread of internet
access and technology between different countries, is a source of significant

www.strategictestprep.com ♟️ 1-833-300-PREP (7737) ♟️[email protected] 355


concern. Organizations such as the nonprofit Per Scholas have helped to
close the gap by providing people from underserved communities ______
with the technology and skills necessary to find jobs in the digital economy.

Which choice completes the text so that it conforms to the conventions of Standard
English?
A. them
B. themselves
C. their
D. they

4. In her 2014 novel Americanah, Nigerian author Chimamanda Ngozi Adichie


examines the lives of immigrants in a manner that has been likened to the
work of James Baldwin and Toni Morrison. At the center of the novel are
main characters Ifemelu and Obinze, who explore both their African roots as
well as their newfound freedom as Americans in the United States. Through
the characters’ experiences, Adichie presents a nuanced portrait of the
challenges that come with leaving one’s home country to pursue the
American ______ .

Which choice completes the text so that it conforms to the conventions of Standard
English?
A. characters’ experiences
B. characters experience’s
C. characters experiences
D. characters’ experiences’

5. When Toni Morrison was awarded the Nobel Prize in Literature in 1993, she
became the first African American woman to receive this honor. A prolific and
versatile writer, Morrison wrote nonfiction collections, novels, essays, ______
and perhaps most famously, she wrote the libretto for Margaret Garner, an
opera.

www.strategictestprep.com ♟️ 1-833-300-PREP (7737) ♟️[email protected] 356


Which choice completes the text so that it conforms to the conventions of Standard
English?
A. and poetry;
B. and poetry,
C. and, poetry
D. and, poetry,

6. In 1988, the World Health Organization declared that the polio virus had
been eradicated from the western hemisphere, a major victory in public
health history. This success was largely due to the introduction of the oral
polio vaccine, which was easy to administer and ______ no needles.

Which choice completes the text so that it conforms to the conventions of Standard
English?
A. necessitated
B. necessitating
C. needed
D. needs

7. Over the past decade, advances in sequencing technology have


revolutionized genetic research and unlocked unprecedented opportunities in
gene therapy: ______ researchers have now discovered new ways to introduce
corrective genetic information into cells, allowing them to precisely target
and eliminate genetic diseases.

Which choice completes the text so that it conforms to the conventions of Standard
English?
A. for instance
B. for example
C. for instance:
D. for example,

www.strategictestprep.com ♟️ 1-833-300-PREP (7737) ♟️[email protected] 357


8. The technique of “partial credit scoring” involves giving test takers partial
points for their responses to a question if they can demonstrate ______ or a
basic level of understanding.

Which choice completes the text so that it conforms to the conventions of Standard
English?
A. comprehension,
B. comprehension;
C. comprehension.
D. comprehension

9. Determining the age of objects at an archeological site is an important prior


step to understanding them, as what is learned can provide insight into the
______ of the people who created them.

Which choice completes the text so that it conforms to the conventions of Standard
English?
A. cultured
B. culture's
C. culture
D. cultures'

www.strategictestprep.com ♟️ 1-833-300-PREP (7737) ♟️[email protected] 358


10. Ronald Joffrey and Gerald Arpino founded the Joffrey School of Ballet in
1953. In the years since, it _____into one of the world's most prestigious
training centers for classical dance.

Which choice completes the text so that it conforms to the conventions of Standard
English?

A. evolves
B. would evolve
C. has evolved
D. is evolving

11. The invention of the printing press in the 15th century revolutionized the
way people spread and shared information, giving rise to a new form of
literature known _____ the novel.

Which choice completes the text so that it conforms to the conventions of Standard
English?
A. for
B. to
C. by
D. as

12.The Caribbean monk seal, once considered one of the most abundant marine
mammals in the region, has been extinct since the 1950s, its population
decimated by overfishing and ______.

Which choice completes the text so that it conforms to the conventions of Standard
English?
A. hunted

www.strategictestprep.com ♟️ 1-833-300-PREP (7737) ♟️[email protected] 359


B. hunt
C. by being hunted
D. hunting

13. The discovery of ___________has led to a deeper understanding of our


universe and how planetary systems form.

Which choice completes the text so that it conforms to the conventions of Standard
English?

A. exoplanets, or planets that orbit stars other than the sun,


B. exoplanets or planets that orbit stars other than the sun,
C. exoplanets or planets that orbit stars other than the sun
D. exoplanets, or planets, that orbit stars other than the sun,

14. Researchers studying “boolean operators,” which expand or restrict the


results of an online search engine, were shocked to realize that the influence
of the operators _____ less than anticipated.

Which choice completes the text so that it conforms to the conventions of Standard
English?

A. are
B. was
C. were
D. have been

www.strategictestprep.com ♟️ 1-833-300-PREP (7737) ♟️[email protected] 360


15. The term “dark matter” was coined in the 1930s to describe an invisible,
mysterious substance that astronomers believe _________for around 27
percent of the universe’s mass.

Which choice completes the text so that it conforms to the conventions of Standard
English?
A. account
B. accounts
C. were accounting
D. are accounting

16. The recent report from the United Nations, which warns of the dangers of
climate change, ______ the need for governments to take immediate action to
protect their citizens from the effects of global warming.

Which choice completes the text so that it conforms to the conventions of Standard
English?
A. have underscored
B. underscores
C. underscoring
D. have been underscoring

17. The new exhibits at the museum ______blank visitors a unique opportunity
to explore a variety of artifacts from different cultures and time periods.

Which choice completes the text so that it conforms to the conventions of Standard
English?

www.strategictestprep.com ♟️ 1-833-300-PREP (7737) ♟️[email protected] 361


A. give
B. given
C. gives
D. giving

18. Written in 2015, the article “Cosmic Rays and the Search for Dark Matter”
______ the discoveries of a team of scientists that studied cosmic rays and
their potential to reveal the nature of dark matter.

Which choice completes the text so that it conforms to the conventions of Standard
English?
A. detail
B. detailing
C. detailed
D. has detailed

19. The ongoing supply chain issues caused by the pandemic ______ the world
economy, affecting industries in a wide range of sectors.

Which choice completes the text so that it conforms to the conventions of Standard
English?
A. is devastate
B. has devastated
C. devastated
D. devastates

20. While some have been disappointed with the novel’s sluggish pacing and
lack of character development, most critics have agreed that the author’s
______ for the story is remarkable.

www.strategictestprep.com ♟️ 1-833-300-PREP (7737) ♟️[email protected] 362


Which choice completes the text so that it conforms to the conventions of Standard
English?
A. vividness
B. vividity
C. vivacity
D. vivaciousness

21. Richard James was often called upon to mediate disputes between
management and _______ his career as a union negotiator made him the
perfect candidate to settle workplace disagreements.
Which choice completes the text so that it conforms to the conventions of Standard
English?
A. labor,
B. labor
C. labor:
D. labor-

22. The novel The Catcher in the Rye, written by J. D. Salinger, has become a
classic of American literature for _____ depiction of adolescent angst and
alienation.

Which choice completes the text so that it conforms to the conventions of Standard
English?
A. it’s
B. its
C. its’
D. it

www.strategictestprep.com ♟️ 1-833-300-PREP (7737) ♟️[email protected] 363


23. Many ___________ are overpriced due to supply chain issues and microchip
shortages.
Which choice completes the text so that it conforms to the conventions of Standard
English?
A. dealerships cars
B. dealership’s cars
C. dealerships’ cars
D. dealerships’ cars’

24. Researchers investigating the behavior of ants cautioned ______ ants can
follow complex paths even without direct visual contact.

Which choice completes the text so that it conforms to the conventions of Standard
English?
A. that
B. before
C. why
D. with

25. Less than two hours away from New York City lies _____ a town where the
landscape and rural character are a stark contrast to the hustle and bustle of
the big city.

Which choice completes the text so that it conforms to the conventions of Standard
English?

www.strategictestprep.com ♟️ 1-833-300-PREP (7737) ♟️[email protected] 364


A. Highmount
B. Highmount.
C. Highmount,
D. Highmount;

26. The popularity of both electric cars and hybrid vehicles ______grown steadily
over the past decade.
Which choice completes the text so that it conforms to the conventions of Standard
English?
A. has
B. have
C. had
D. was

27. Beginning its journey in the deep ocean off of Antarctica, the Weddell Sea
Trench flows for almost 3,000 miles along the South American coast,
extending as far north as Brazil before ______ into the Atlantic Ocean.

Which choice completes the text so that it conforms to the conventions of Standard
English?
A. it empties
B. emptying
C. the emptying of it
D. they empty

www.strategictestprep.com ♟️ 1-833-300-PREP (7737) ♟️[email protected] 365


28. With the impending pandemic’s effects on the economy, individuals and
businesses ______ more likely to face financial difficulties.

Which choice completes the text so that it conforms to the conventions of Standard
English?
A. has been
B. are
C. had been
D. was

29. Inhabitants of North America’s arid regions have developed strategies for
tapping underground sources of ________ to ensure that their access to the
precious resource remains secure.

A. water regulating it
B. water; regulating it
C. water. Regulating it
D. water, regulating it

30. Over the past 10 years, climate change ________ the health of the ocean, as
coral reefs have seen a 75% reduction.
A) Has effected
B) Effects
C) Has affected

www.strategictestprep.com ♟️ 1-833-300-PREP (7737) ♟️[email protected] 366


D) Affects

31.He rode to school on a bus full of students ______ he did not know very well.
A) Who
B) Whom
C) Which
D) That

32.This assignment is extra credit _____ however, we still need to hand it in.
A) only
B) only,
C) only:
D) only;

33. Eliot’s lines in The Waste Land, however, are not totally without
________rather, by repeating certain words and phrases and by staggering
line length, he creates a cadence that simultaneously expresses the poem’s
despair and grounds the reader in the poem’s language.

Which choice completes the text so that it conforms to the conventions of Standard
English?
A. rhythm
B. rhythm,
C. rhythm:
D. rhythm;

www.strategictestprep.com ♟️ 1-833-300-PREP (7737) ♟️[email protected] 367


34. Tom Ford was the creative director for Gucci for ten years and was known for
seductive campaigns that boosted sales for the fashion giant. Named in 2003 as
one of the greatest achievements in fashion by a US designer, ________ led to
the Gucci Group’s position as the top fashion brand in the world.

A) Ford created the signature Gucci floral pattern in 2001; it


B) Ford’s creation of the signature Gucci floral pattern
C) in 2001 Ford created the signature Gucci floral pattern that
D) the signature Gucci floral pattern was created by Ford in 2001 and

35. Healthy marine environments result from a precise balance of factors and are
therefore vulnerable to threats from a variety of sources. For example, changes
in wind circulation, runoff from sewage, and _______ can all lead to the creation
of ocean waters low in oxygen and inhospitable to marine life.

Which choice completes the text so that it conforms to the conventions of Standard
English?

A) to accumulate fertilizers

B) they accumulate fertilizers

C) accumulating fertilizers

D) accumulation of fertilizers

36. In urban areas, noise pollution is a major concern. As city planners aim to
reduce the disturbance, many are looking at green solutions. In Stockholm,
for example, officials have begun planting more trees in heavily trafficked
areas. Trees not only absorb carbon dioxide, but _______ also act as natural
sound barriers.

www.strategictestprep.com ♟️ 1-833-300-PREP (7737) ♟️[email protected] 368


Which choice completes the text so that it conforms to the conventions of Standard
English?

A) they

B) one

C) you

D) its

37. In the early 20th century, Virginia Woolf established herself as a prominent
figure in the literary world. She was not only a prolific writer but also a critic
who believed in the fluidity of gender and _______ many traditional norms of
her time were restrictive.

Which choice completes the text so that it conforms to the conventions of Standard
English?

A) argued,

B) argued:

C) argued;

D) argued

38. Canadian astronaut Chris Hadfield became an internet sensation for his
rendition of David Bowie's "Space Oddity," filmed aboard the International
Space Station. _______ performance not only showcased his musical talents
but also helped promote interest in space exploration among younger
generations.

www.strategictestprep.com ♟️ 1-833-300-PREP (7737) ♟️[email protected] 369


Which choice completes the text so that it conforms to the conventions of Standard
English?

A) He’s

B) His

C) Him

D) He’d

39. In 2015, renowned physicist Dr. Emily Thornton released a groundbreaking


paper on quantum mechanics. The paper, which _______ "a turning point in
modern physics," delved deep into the intricacies of particle behavior at the
quantum level.

Which choice completes the text so that it conforms to the conventions of Standard
English?

A) journal editor Mark Hale, described


B) journal editor, Mark Hale described,
C) journal editor Mark Hale described
D) journal editor, Mark Hale, described,

40. In 1977, the Voyager 1 space probe was launched by NASA with the primary
mission of studying the outer Solar System. Onboard the probe, scientists
included the Voyager Golden Record, which contains sounds and images
chosen to portray the diversity of life on Earth. Some astronomers _______
this as an attempt to communicate with extraterrestrial life, offering a
snapshot of humanity's cultural and scientific achievements.

Which choice completes the text so that it conforms to the conventions of Standard
English?

A) viewing
B) view

www.strategictestprep.com ♟️ 1-833-300-PREP (7737) ♟️[email protected] 370


C) have viewed
D) to view

41. Question: In the depths of the Amazon rainforest, the mimic octopus, known
for its ability to take on the appearance of other sea creatures, changes its
skin pattern and texture. This unique camouflage not only confuses predators
but also allows the octopus _______ more closely with its environment.
Which choice completes the text so that it conforms to the conventions of
Standard English?

A) to blend

B) blending

C) it blends

D) that blended

42. The novel "To Kill a Mockingbird" by Harper Lee addresses deep-seated
issues of race and injustice in the American South. Through the character of
Atticus Finch, readers learn that understanding and empathy _______ more
effective means of addressing prejudice than confrontation. Which choice
completes the text so that it conforms to the conventions of Standard
English?

A) is

B) are

C) has been

D) being

43. In the early 1900s, Marie Curie achieved significant recognition for her
research on radioactivity. Her contributions to science were not limited to this
discovery _______ as a trailblazer for women in the scientific community,
Curie set an example for generations to come.

www.strategictestprep.com ♟️ 1-833-300-PREP (7737) ♟️[email protected] 371


Which choice completes the text so that it conforms to the conventions of Standard
English?

A) alone, however:

B) alone. However,

C) alone however

D) alone, however.

44. In 1888, the Kodak camera was launched, introducing the concept of
snapshot photography to the masses. This invention allowed ordinary
people, not just professionals, to capture moments from their daily lives.
When color film was developed in the 1930s, _______ broadening the scope
of photography even further by allowing the vibrant hues of life to be
immortalized.

Which choice completes the text so that it conforms to the conventions of Standard
English?

A) it revolutionized the world of photography,

B) the world of photography was revolutionized,

C) photography's world revolutionized,

D) having a revolution in the photography world,

45. In the 1960s, environmental activist Rachel Carson authored "Silent Spring,"
a groundbreaking book detailing the dangers of pesticide use. Her
meticulous research and clear writing style made the book an instant classic.
Carson's impact on environmental awareness _______ felt today, with many
policies and guidelines being traced back to her advocacy.

Which choice completes the text so that it conforms to the conventions of Standard
English?

A) were

www.strategictestprep.com ♟️ 1-833-300-PREP (7737) ♟️[email protected] 372


B) is

C) have been

D) are being

46. In her memoir, chef Elaine Wong recounted her culinary journey, which
began in the kitchens of her grandmother in Beijing. Wong learned the
intricate art of dumpling-making and the delicate balance of flavors in
traditional Chinese cuisine there. As she migrated to the West, she fused
these foundational techniques with Western culinary traditions, creating a
unique style of her own, _______ her memoir doesn't delve deeply into the
business challenges she faced when opening her first restaurant in New York.

Which choice completes the text so that it conforms to the conventions of Standard
English?

A) so

B) thus

C) and

D) yet

47. During summer, coastal birds have a varied diet based on the availability of
food in their environment. While these birds typically prefer to feed on small
fish and marine insects, changes in ocean currents can lead to a decrease in
these food sources, _______ in the birds having to scavenge along the
shoreline for alternative sustenance.

Which choice completes the text so that it conforms to the conventions of Standard
English?

A) results

B) to result

C) resulting

www.strategictestprep.com ♟️ 1-833-300-PREP (7737) ♟️[email protected] 373


D) resulted

48. While researching the patterns of monarch butterfly migration, biologist


Carlos Mendez noted that specific milkweed plants play a pivotal role in the
butterflies' reproductive process. If these butterflies were exposed to a
different variety of milkweed, _______ Mendez sought to understand through
his experiments.

Which choice completes the text so that it conforms to the conventions of Standard
English?

A) will the monarchs adapt?

B) the monarchs will adapt.

C) the monarchs adapt?

D) will the monarchs have adapted?

49. In the early 20th century, Marie Curie's groundbreaking research on


radioactivity led to numerous scientific advances. Today, Curie _______
regarded as one of the most influential scientists of her era.

Which choice completes the text so that it conforms to the conventions of Standard
English?

A) had been

B) will be

C) was

D) is

50. To combat deforestation and its adverse effects, environmentalists have


urged governments worldwide to invest in afforestation projects. These
projects not only promote the planting of new trees but also _______ a
habitat for various wildlife species, which can lead to increased biodiversity
in the region.

www.strategictestprep.com ♟️ 1-833-300-PREP (7737) ♟️[email protected] 374


Which choice completes the text so that it conforms to the conventions of Standard
English?

A) providing

B) provides

C) provide

D) provided

51. Despite its rich taste and creamy texture, _______ it is high in calories and
can contribute to weight gain if consumed in large quantities.

Which choice completes the text so that it conforms to the conventions of Standard
English?

A) there are drawbacks to eating too much chocolate:

B) the drawbacks of eating too much chocolate are:

C) chocolate's two major concerns are that

D) chocolate has a few concerns:

52. The aquaporin proteins, which play a crucial role in regulating water
transport within cells, were discovered in 1992. A molecular biologist
_______ was awarded the Nobel Prize in Chemistry in 2003 for his pioneering
work on these proteins.

Which choice completes the text so that it conforms to the conventions of Standard
English?

A) having been named Peter Agre

B) named Peter Agre

C) that is named Peter Agre

www.strategictestprep.com ♟️ 1-833-300-PREP (7737) ♟️[email protected] 375


D) being Peter Agre,

53. To adapt to high altitudes where oxygen is sparse, the red blood cells in
Andean geese _______ an increased ability to transport oxygen to muscles.
This adaptation enables these birds to fly over the Andes Mountains without
experiencing fatigue.

Which choice completes the text so that it conforms to the conventions of Standard
English?

A) demonstrate

B) to demonstrate

C) demonstrating

D) have demonstrated

54. Established in 1951 to foster cooperation and trade in Western Europe, the
European Coal and Steel Community was the precursor to the modern
European Union. By the time the Maastricht Treaty was signed in 1992, the
number of participating countries _______ considerably from the original six.

Which choice completes the text so that it conforms to the conventions of Standard
English?

A) grows
B) had grown
C) will grow
D) is growing

55. A small feline native to the high Andes, the Andean mountain cat is one of
the least known and rarest of all wild cats. Due to its secretive nature and
remote habitat, this cat has been difficult to study in the wild. Its thick fur
allows it to survive in cold, harsh environments, providing insulation against
the freezing temperatures. In addition to its natural adaptability, the Andean
mountain cat's diet mainly consists of mountain viscachas, _______ is a type
of rodent found in the Andes.

www.strategictestprep.com ♟️ 1-833-300-PREP (7737) ♟️[email protected] 376


Which choice completes the text so that it conforms to the conventions of Standard
English?

A) which

B) that

C) who

D) where

56. Nature and its intricate relationships are recurring themes in Barbara
Kingsolver's works. In "Prodigal Summer", for instance, the character of
Deanna embodies the wild spirit of the Appalachian woods where she
resides, while in "Flight Behavior", the protagonist Dellarobia ______ a deep
bond with the monarch butterflies that alight on her family's property.

Which choice completes the text so that it conforms to the conventions of Standard
English?

A) had been forming


B) forms
C) was formed
D) forming

57. Architect Isabelle Larkin designed an innovative bridge by intertwining


metal beams, which not only provided structural strength but also aesthetic
appeal. She achieved this pattern by twisting each beam around the ______,
creating a helical form that became iconic for the city.

Which choice completes the text so that it conforms to the conventions of Standard
English?

A) beam's axis's.

B) beams' axises.

C) beam's axis.

www.strategictestprep.com ♟️ 1-833-300-PREP (7737) ♟️[email protected] 377


D) beams axes'.

58. In a deep exploration of the works of poet Emily Dickinson, literature


professor Daniel Mortimer emphasized that many of Dickinson's poems
present the theme of mortality as "a shadow lurking behind joy's ______
given that Dickinson often penned her verses during a time of personal grief
and the wider context of the American Civil War.

Which choice completes the text so that it conforms to the conventions of Standard
English?

A) facade”; thus,

B) facade,”

C) facade” and,

D) facade.”

59. In the 1800s, the city of Paris underwent a major transformation under the
direction of Baron Haussmann, a process often referred to as
Haussmannization. While the vast renovations improved sanitation and
traffic flow, many criticized the project for displacing thousands of residents.
Haussmann's redesign, _______ it led to the Parisian boulevards and iconic
architecture we know today, also erased much of the city's medieval past.

Which choice completes the text so that it conforms to the conventions of Standard
English?

A) although,

B) although

C) but;

D) but,

60. In the dense forests of the Amazon, one can observe the peculiar behavior of
the lanternfly. The insect's luminescent glow, visible mainly during nighttime,

www.strategictestprep.com ♟️ 1-833-300-PREP (7737) ♟️[email protected] 378


attracts various predators. Yet, the patterns on the lanternfly’s wings serve to
deter these potential threats. Bright spots on the ________ evolutionary
strategy.

Which choice completes the text so that it conforms to the conventions of Standard
English?

A) forest's floor mirror the fly's

B) forests floor mirror the flies

C) forest's floor mirror the flies'

D) forests' floor mirror the fly’s

61. In the dense rainforests of South America, bioluminescent fungi attract


insects to aid in their spore distribution. Flashes of light emitted from the
_______ to nocturnal creatures navigating the forest floor.

Which choice completes the text so that it conforms to the conventions of Standard
English?

A) fungis stems signal the path

B) fungi’s stems signals the path

C) fungi’s stems signal the path

D) fungis’ stems signals the path

62. Famed astrobiologist Sara Kepler has extensively studied the potential for
life on exoplanets. In her groundbreaking 2025 work, Kepler delves deep into
the conditions required for life on planets outside our solar system. Her
publication titled _______ details the discoveries of microbial ecosystems on
planets in the Proxima Centauri star system.

Which choice completes the text so that it conforms to the conventions of Standard
English?

www.strategictestprep.com ♟️ 1-833-300-PREP (7737) ♟️[email protected] 379


A) Beyond the Stars—

B) Beyond the Stars,

C) Beyond the Stars

D) Beyond the Stars:

63. For centuries, inhabitants of the Polynesian islands ________ the stars, ocean
currents, and bird migrations to expertly navigate the vast Pacific Ocean in
canoes without the use of maps or compasses. This indigenous knowledge
was passed down through generations and remains a testament to their
impressive maritime skills.

Which choice completes the text so that it conforms to the conventions of Standard
English?

A) relying on

B) rely on

C) have relied on

D) relied on

64. While many musicians today opt for digital methods of recording and
production, _______ still ;prefer the warmth and authenticity of analog
recording techniques, using vintage equipment from the 1970s and 1980s.

Which choice completes the text so that it conforms to the conventions of Standard
English?

A) many

B) some of them

C) others

D) the others

www.strategictestprep.com ♟️ 1-833-300-PREP (7737) ♟️[email protected] 380


65. In the 1920s, an aspiring writer named Ernest Hemingway was searching for
a distinctive style that would set his works apart. His friend and mentor,
Gertrude Stein, _______ the idea of using concise, straightforward language
to capture the essence of his narratives without unnecessary
embellishments. This suggestion greatly influenced Hemingway's unique and
impactful writing style.

Which choice completes the text so that it conforms to the conventions of Standard
English?

A) is introducing

B) introduces

C) had introduced

D) introduced

66. Emily Dickinson, the enigmatic 19th-century poet, lived much of her life in
seclusion, rarely leaving her home in Amherst, Massachusetts. Despite her
reclusive nature, Dickinson maintained friendships through correspondence,
______ over 1,800 poems, most of which were published posthumously.

Which choice completes the text so that it conforms to the conventions of Standard
English?

A) writing

B) she wrote

C) written

D) and wrote

67. When examining the works of French composer Claude Debussy, ______
have often overlooked his profound inspiration from East Asian musical
scales and rhythms.

www.strategictestprep.com ♟️ 1-833-300-PREP (7737) ♟️[email protected] 381


Which choice completes the text so that it conforms to the conventions of Standard
English?

A) many musicologists have emphasized Debussy's break from European classical


traditions but

B) Debussy's break from European classical traditions has been the emphasis for many
musicologists, who

C) there are many musicologists who have emphasized Debussy's break from
European classical traditions, but they

D) the emphasis for many musicologists has been Debussy's break from European
classical traditions; they

68. In analyzing the paintings of French Impressionist Claude Monet, ______ tend
to overlook his deep appreciation for Japanese woodblock prints, which
profoundly influenced his approach to depicting nature.

Which choice completes the text so that it conforms to the conventions of Standard
English?

A) many art historians focus on Monet’s innovative use of color and light but

B) focusing on Monet’s innovative use of color and light has been the primary concern
for many art historians, who

C) there are numerous art historians who focus on Monet’s innovative use of color and
light, but they

D) the concentration of numerous art historians is on Monet’s innovative use of color


and light; they

69. Samantha Kane, a prominent historian focused on medieval European


societies, has written several groundbreaking books during her career. Her
first book, The Rise of Knights, in 1995, offered an in-depth look at the
chivalric code. Subsequent works include Castle Secrets ______ The Tapestry
Tales, in 2002, which explored the role of women in medieval artwork.

www.strategictestprep.com ♟️ 1-833-300-PREP (7737) ♟️[email protected] 382


Which choice completes the text so that it conforms to the conventions of Standard
English?

A) on 1998; followed by

B) in 1998; followed by

C) in 1998, and

D) on 1998, and then

70. The city of Venice, Italy, is renowned for its canals, which serve as streets for
boats and gondolas. One of the most famous waterways, the Grand Canal,
winds its way through the heart of the city—intersecting smaller canals and
passing by historic landmarks and palazzos. A truly Venetian experience
involves taking a gondola ride on this canal, especially during the evening
when the buildings ______ sparkle with the reflections of the water.

Which choice completes the text so that it conforms to the conventions of Standard
English?

A) on either side,

B) on either side—

C) on either side:

D) on either side

71. In a study on the effects of music on productivity, researcher Elena Martinez


found that participants' concentration levels shifted based on the type of
music played. The genres of music presented ______ some participants
listened to classical music, while others heard jazz or rock.

Which choice completes the text so that it conforms to the conventions of Standard
English?

A) varied:

www.strategictestprep.com ♟️ 1-833-300-PREP (7737) ♟️[email protected] 383


B) varied

C) were varying,

D) varied;

72. Canadian poet Rupi Kaur's influential works include "Milk and Honey," a
collection of poems and prose about survival and the experience of violence,
abuse, love, and loss; "The Sun and Her Flowers," which delves into themes
of growth and healing, honoring one’s roots, and ______ "Home Body," her
introspective take on the physical and emotional spaces we inhabit.

Which choice completes the text so that it conforms to the conventions of Standard
English?

A) rising to embrace one's journey;

B) rising: to embrace one's journey,

C) rising to embrace one's journey,

D) rising; to embrace one's journey

73. Regarded as one of the pioneers of environmental activism, Rachel Carson's


groundbreaking book, "Silent Spring," which was published in 1962, _______
the dangerous effects of pesticides on the environment and human health.

Which choice completes the text so that it conforms to the conventions of Standard
English?

A) highlighting
B) had highlighted
C) highlights
D) highlighted

74. Following a city-wide campaign to promote cycling as an eco-friendly mode


of transportation, the number of daily cyclists increased by over seventy
_______ improvements such as dedicated bike lanes and safer crossings are

www.strategictestprep.com ♟️ 1-833-300-PREP (7737) ♟️[email protected] 384


credited for this uptick, some fear a potential surge in traffic-related
incidents.

Which choice completes the text so that it conforms to the conventions of Standard
English?

A) percent, while
B) percent and while
C) percent. While
D) percent while

75. American composer Aaron Copland's "Appalachian Spring" (1944), a ballet


score for Martha Graham's dance company, _______ the evolution of
American classical music with its unique blend of folk tunes and innovative
harmonies.

Which choice completes the text so that it conforms to the conventions of Standard
English?

A) has reflected

B) are reflecting

C) have reflected

D) reflect

76. Environmentalist Jane Goodall is best known for her groundbreaking studies
on chimpanzees in Tanzania's Gombe Stream National Park. Her observations
revolutionized our understanding of primate behavior and cognition. Goodall,
_______ many believed that chimps were simple creatures, has consistently
shown through her work that they have complex social structures and
emotions similar to humans.

Which choice completes the text so that it conforms to the conventions of Standard
English?

A) whereas

www.strategictestprep.com ♟️ 1-833-300-PREP (7737) ♟️[email protected] 385


B) where

C) while

D) whom

77. From a distance, the intricate sculptures of Italian artist Marco Bianchi
appear to be carved from delicate marble. However, when inspected closely,
these art pieces show themselves to be _______ twist, fold, and detail crafted
from recycled paper and eco-friendly adhesives.

Which choice completes the text so that it conforms to the conventions of Standard
English?

A) entirely paper-based, each

B) entirely paper-based; each

C) entirely paper-based, with each

D) entirely paper-based. Each

78. In a hydroponic system, plants grow without soil, deriving their essential
nutrients directly from a water solution. As the plants mature, their roots
________ deeper into the nutrient-rich water to absorb more minerals and
vitamins.

Which choice completes the text so that it conforms to the conventions of Standard
English?

A) will dive

B) dive

C) had dived

D) are diving

www.strategictestprep.com ♟️ 1-833-300-PREP (7737) ♟️[email protected] 386


79. Even as more people opt for plant-based diets, fast-food chains have
responded with a surge in meatless menu options. A report published last
year ______ that these new offerings led to a significant increase in the
number of vegetarian patrons frequenting these establishments.

Which choice completes the text so that it conforms to the conventions of Standard
English?

A) indicates

B) indicating

C) which indicates

D) had indicated

80. Renowned physicist Marie Curie and her husband Pierre conducted
groundbreaking research on radioactivity, a phenomenon that ______ the
emission of energy from unstable atomic nuclei.

Which choice completes the text so that it conforms to the conventions of Standard
English?

A) has described

B) describing

C) describes

D) describe

81. Dietary preferences among birds are incredibly diverse. For instance, the
African gray parrot (Psittacus erithacus) primarily feasts on nuts and seeds
______ the peregrine falcon (Falco peregrinus) predominantly hunts other
birds while in flight.

Which choice completes the text so that it conforms to the conventions of Standard
English?

www.strategictestprep.com ♟️ 1-833-300-PREP (7737) ♟️[email protected] 387


A) moreover

B) while

C) however

D) in fact,

82. The Galápagos Islands, known for their unique biodiversity, are home to a
variety of plant and animal species. The blue-footed booby, a bird native to
the islands, has a diet primarily composed of fish. When hunting, the bird
dives swiftly into the water, using its sharp beak ______ the fish before
bringing it to the surface.

Which choice completes the text so that it conforms to the conventions of Standard
English?

A) catching, and securing,

B) to catch and secure

C) for catching and for securing

D) catch and then secure

83. During the Renaissance, several European cities became hubs for art and
culture. Among the renowned artists of this period ______ Leonardo da Vinci,
Michelangelo, and Raphael, all of whom contributed significantly to the
world of art.

Which choice completes the text so that it conforms to the conventions of Standard
English?

A) is

B) were

C) has been

www.strategictestprep.com ♟️ 1-833-300-PREP (7737) ♟️[email protected] 388


D) being

84. Paleontologists had previously believed that the Tyrannosaurus rex was
primarily a scavenger, based on the evidence available. However, after the
discovery of a Triceratops fossil with T. rex tooth marks in Montana, ______
this giant predator might have been an active hunter.

Which choice completes the text so that it conforms to the conventions of Standard
English?

A) researcher Dr. Emily Hart has proposed that

B) the proposition of researcher Dr. Emily Hart is that

C) that, researcher Dr. Emily Hart proposes,

D) the proposal from researcher Dr. Emily Hart is suggesting that

85. The Gobekli Tepe, located in modern-day Turkey, is often referred to as the
world's oldest temple. Predating Stonehenge by more than 6,000 years,
archaeologist Klaus Schmidt discovered the site in 1994 and was astonished
by its intricate _______ inside were animals, symbols, and intricate patterns
which gave insights into prehistoric religious practices.

Which choice completes the text so that it conforms to the conventions of Standard
English?

A) carvings. Etched

B) carvings, etched

C) carvings and etched

D) carvings etched

86. In the culinary world, many dishes are attributed to renowned chefs, even
though they were actually conceived and perfected by their less-known
assistants or _______ whose creative contributions often go unrecognized.

www.strategictestprep.com ♟️ 1-833-300-PREP (7737) ♟️[email protected] 389


Which choice completes the text so that it conforms to the conventions of Standard
English?

A) sous-chefs

B) sous chef’s

C) sous chefs

D) sous-chef’s

87. Like many marine animals, the humpback whale (Megaptera novaeangliae)
relies on a layer of blubber to keep warm in cold oceanic waters. During their
annual migration to warmer waters, they _______ by relying on this stored
fat, rarely eating during the journey.

Which choice completes the text so that it conforms to the conventions of Standard
English?

A) had sustained

B) sustain

C) would sustain

D) sustains

88. Despite facing numerous challenges early on, Katherine Johnson showed a
proclivity for mathematics from a young age. Not only did she excel
academically, Johnson _______ the trajectory analysis for the first American in
space, John Glenn, and was dubbed a "human computer" for her accurate
calculations.

Which choice completes the text so that it conforms to the conventions of Standard
English?

A) managed—impressively—to calculate

B) managed—impressively to calculate

www.strategictestprep.com ♟️ 1-833-300-PREP (7737) ♟️[email protected] 390


C) managed—impressively, to calculate

D) managed—impressively. To calculate

89. Throughout his literary career, Japanese author Haruki Murakami has drawn
readers into surreal worlds, often blending the mundane with the mystical.
Notably, the cat—a recurring symbol in many cultures—_______ a prominent
and enigmatic figure in several of his novels.

Which choice completes the text so that it conforms to the conventions of Standard
English?

A) remain

B) has remained

C) were

D) remains

90. Contemporary composer Lin Yang often draws from traditional Chinese
music in her work. In 2017, she released a piece that intricately wove
guzheng strings into a symphonic orchestra backdrop _______ in 2019, she
blended bamboo flute melodies with modern electronic beats, generating a
harmonious fusion of old and new.

Which choice completes the text so that it conforms to the conventions of Standard
English?

A) ; next,

B) then

C) and

D) . While

91. Astronomer Caroline Herschel, born in 1750, was initially overshadowed by


her brother Sir William Herschel, known for his discovery of the planet

www.strategictestprep.com ♟️ 1-833-300-PREP (7737) ♟️[email protected] 391


Uranus. However, over time, Caroline herself achieved recognition in the
world of astronomy. _______ and she became the first woman to be awarded
a Gold Medal from the Royal Astronomical Society.

Which choice completes the text so that it conforms to the conventions of Standard
English?

A) Discovering several comets during her career;

B) Several comets were discovered by her during her career,

C) During her career, discovering several comets,

D) During her career, she discovered several comets,

92. The Hubble Space Telescope has provided invaluable insights into the
universe's vast expanse. Through its lenses, astronomers have observed
distant galaxies, nebulous gas clouds, bright _______ promoting a deeper
understanding of space; and inspiring generations of stargazers.

Which choice completes the text so that it conforms to the conventions of Standard
English?

A) stars, supernovae, and black holes,

B) stars, supernovae, and black holes;

C) stars; supernovae, and black holes,

D) stars supernovae, and black holes,

93. Paleontologist Sara Mitchell specializes in the study of ancient aquatic life,
focusing on creatures like the ichthyosaur and the plesiosaur. Mitchell's work
is not solely theoretical, _______ she frequently conducts excavations at
various dig sites, working alongside teams of other experts to uncover and
document fossilized remains.

Which choice completes the text so that it conforms to the conventions of Standard
English?

www.strategictestprep.com ♟️ 1-833-300-PREP (7737) ♟️[email protected] 392


A) though, in practice,

B) though in practice,

C) though: in practice,

D) though; in practice

94. In the acclaimed documentary "Whispers of the Past," filmmaker Alex


Hernandez delves deep into historical records, diaries, and personal letters to
illuminate the _______ who settled in the Appalachian Mountains during the
18th century.

Which choice completes the text so that it conforms to the conventions of Standard
English?

A) experiences of the pioneer’s

B) experiences’ of the pioneers

C) experiences of the pioneers

D) experience’s of the pioneers’

95. In her celebrated novel "Whispers of the Wind," Clara Bellwood entwined
narratives _______ love, loss, and the intricacies of human relationships, using
a coastal town as the backdrop to her characters' evolving dynamics.

Which choice completes the text so that it conforms to the conventions of Standard
English?

A) about

B) about,

C) about—

D) about:

www.strategictestprep.com ♟️ 1-833-300-PREP (7737) ♟️[email protected] 393


96. Australian chef Kate McBain and nutritionist Anna Lisle, both known for
their dedication to healthy, wholesome meals, realized the market's need for
quick and nutritious recipes. In response, they co-founded Nourish Kitchen, a
culinary _______ focuses on providing easy-to-follow recipes that champion
fresh, local ingredients and promote a balanced lifestyle.

Which choice completes the text so that it conforms to the conventions of Standard
English?

A) establishment

B) establishment which

C) establishment, that

D) establishment that

97. Research conducted by Dr. Maria Gomez at the University of Barcelona


revealed that many marine creatures, including the widely known seahorse
Hippocampus erectus, have a unique pigment called __________ responsible
for the vibrant colors they display. This pigment helps them camouflage in
their diverse underwater habitats.

Which choice completes the text so that it conforms to the conventions of Standard
English?

A) xanthochrome, (XC)

B) xanthochrome (XC)

C) xanthochrome, (XC),

D) xanthochrome (XC,)

98. During a deep-sea expedition in 2015, marine biologist Dr. Lara Atkinson
made a startling discovery. She came across a rare bioluminescent creature
known as a "glow squid" in the Mariana Trench. This finding _______ a
breakthrough in understanding the adaptations of deep-sea creatures to
extreme environments.

www.strategictestprep.com ♟️ 1-833-300-PREP (7737) ♟️[email protected] 394


Which choice completes the text so that it conforms to the conventions of Standard
English?

A) proves to be

B) has

C) marked

D) thus

99. Utilizing research from marine biologists, environmentalist Alex Monroe and
her team developed a filtration system for harbors _______ the rapid growth
of toxic algae, a phenomenon that has been affecting marine life drastically.

Which choice completes the text so that it conforms to the conventions of Standard
English?

A) combating

B) to combat

C) combated

D) and combat

100. In 2018, botanist Maya Patel initiated a reforestation project in the


Amazon rainforest, planting over 10,000 native tree saplings across a
deforested region. As a result of this project, _______.

Which choice completes the text so that it conforms to the conventions of Standard
English?

A) the habitat restoration for numerous endangered species was achieved through the
saplings.

B) the saplings have achieved habitat restoration for numerous endangered species.

www.strategictestprep.com ♟️ 1-833-300-PREP (7737) ♟️[email protected] 395


C) the saplings’ achievement in habitat restoration for numerous endangered species
has been realized.

D) habitat restoration for numerous endangered species has been achieved by the
saplings.

Standard English Convention Answers


1.
Choice B is the best answer. The convention being tested is the use of punctuation
after noun phrases. No punctuation is needed because the noun phrase (“a cadence”) is
a restrictive appositive, meaning that it provides essential identifying information about
the noun phrase before it (“a rhythm”).

2.
The correct answer is D. Since many students are scoring in high school, we need a
plural possessive to denote the fact that multiple students own the scores. The word
scores needs no apostrophe, as we are just talking about a quantity of scores in the
plural sense.

3.
Choice B is the best answer. The convention being tested is pronoun-antecedent
agreement. The reflexive pronoun “themselves” agrees in number and person with the
pronoun antecedent “people,” indicating that it is the people themselves who are being
provided with the necessary technology and skills.

www.strategictestprep.com ♟️ 1-833-300-PREP (7737) ♟️[email protected] 396


4.
Choice A is the best answer. The convention being tested is the use of the apostrophe
to denote proper possession. It is a plural possessive, as the characters own the
experiences. Thus, the apostrophe must come after the s in characters.

5.
Choice B is the best answer. The convention being tested is punctuation use between
independent clauses. An independent clause is a complete sentence, and this choice
correctly uses a comma to mark the boundary between the independent clause “Toni
Morrison…honor” and the independent clause that follows it, “A prolific…opera.” The
comma correctly signals that the two clauses are related and should remain together.
The participial phrase beginning with “and” modifies the subject of the preceding
sentence, “Toni Morrison.”

6.
Choice C is the best answer. The convention being tested is verb tense. The past tense
verb “needed” is the correct choice because it is used to indicate that the vaccine’s lack
of a needle requirement is a past event. The other choices are either in the wrong tense
or are not verbs.

7.
Choice D is the best answer. The convention being tested is the use of punctuation to
mark boundaries between supplements and clauses. The comma after “example” is
used to separate the independent clause (“advance…gene therapy”) from the
supplementary adverb phrase “for example.” A comma used in this way is used to
separate two clauses of equal weight. In this case, the comma after “example”
separates the clause discussing the advances in sequencing technology from the
clause discussing the discoveries made possible by these advances.

8.
Choice A is the best answer. There are not two complete sentences, so that involves
elimination of the period and the semicolon. However, you do need to pause before the
FANBOYS (or), which is why you need a comma.

www.strategictestprep.com ♟️ 1-833-300-PREP (7737) ♟️[email protected] 397


9.
Choice C is the best answer. The convention being tested is the use of possessive
apostrophes. In this case, no possessive is necessary, because “culture” is a noun being
used to refer to the abstract concept of a people’s values, beliefs, and behaviors rather
than a specific possession owned by a people.

10.
Choice C is the best answer. “In the years since” points to present perfect tense, as it is
ongoing in the past and continuing in the present, so the best option is to pick “has
evolved.”

11.
Choice D is the best answer. The convention being tested is the use of prepositions to
mark relationships between words in a sentence. The correct preposition to pair with
the word “known” is “as,” which is used to introduce the phrase “the novel,” indicating
that the novel is the form of literature being referenced.

12.
Choice D is the best answer. The convention being tested is verb tense. The verb
“hunting” is in the present tense and agrees with the verb in the preceding clause (“has
been extinct”).

13.

www.strategictestprep.com ♟️ 1-833-300-PREP (7737) ♟️[email protected] 398


Choice A is the best answer. The convention tested here is the use of commas. “Or
planets that orbit stars other than the sun” is a clause describing more about the
subject exoplanets. This extra information must be separated with a pair of commas.

14.
Choice B is the best answer. The convention being tested is subject-verb agreement.
The singular verb “was” agrees in number with the singular subject “the influence”

15.
Choice B is the best answer. The convention being tested is subject/verb agreement.
The present tense form of “accounts” is linked to the singular subject, substance.

16.
Choice B is the best answer. The convention being tested is the use of verbs to express
tense in a sentence. In this choice, the present simple tense verb “underscores” is
logically consistent with the other verbs in the sentence, which are all in the present
simple tense (e.g., “warns” and “take”). The present simple tense correctly indicates
that the action is occurring in the present (the report is underscoring) and is ongoing
(the need exists).

17.
Choice A is the best answer. The convention being tested is the use of verbs to express
tense in a sentence. The verb “give” is plural present tense, which appropriately
matches the subject, “exhibits.”

18.
Choice C is the best answer. The convention being tested is verb tense. The simple
past tense verb “detailed” is consistent with the other past tense verbs (e.g., “studied”)

www.strategictestprep.com ♟️ 1-833-300-PREP (7737) ♟️[email protected] 399


used to describe the article’s content. Further, the simple past tense verb correctly
indicates that the action of detailing the discoveries has already taken place in the past
(2015).

19.
Choice B is the best answer. The convention being tested is verb tense. The verb “has
devastated” is in the present perfect tense, which indicates an action that began in the
past and continues to the present. This is consistent with the fact that the pandemic
began in the past and continues to have an effect on the world economy (“ongoing”).

20.
Choice A is the best answer. The convention being tested is vocabulary. The word
“vividness” is the correct term to describe the author’s ability to produce a clear and
distinct description of the story. The other choices are not actual words.

21.
Choice C is the best answer. The convention being tested is punctuation. Since there
are two independent clauses, a colon is needed to separate the information.
Furthermore, the right hand side of the colon typically expands on/explains the left
hand side, which is precisely what is happening in this example.

22.
Choice B is the best answer. The convention being tested is the possessive form of a
noun. The possessive form of “it” is “its.”

www.strategictestprep.com ♟️ 1-833-300-PREP (7737) ♟️[email protected] 400


23.
Choice C is the best answer. The convention being tested is the use of the possessive
form. We need a plural possessive, as “many” indicates we are talking about multiple
dealerships that possess cars. Thus, we need to state “dealerships’ cars” and place the
apostrophe outside of the ‘s’.

24.
Choice A is the best answer. The convention being tested is the use of subordinating
conjunctions to introduce subordinate clauses. Subordinating conjunctions are used to
indicate that the clause that follows is less important than information conveyed in the
main clause. In this sentence, “that” is correctly used to introduce the subordinate
clause.

25.
Choice C is the best answer. The convention being tested is the general use of
punctuation in English. In this case, you cannot use a semicolon or period, as the
second part is not a complete sentence. Because you need a pause after Highmount,
you must pick C and not A.

26.
Choice A is the best answer. The convention being tested here is subject/verb
agreement. “Has” is a singular tense to compliment the subject “popularity”. The
simple present-tense verb “has” accurately indicates that the action of “growing” has
been ongoing throughout the past decade and continues in the present.

27.

www.strategictestprep.com ♟️ 1-833-300-PREP (7737) ♟️[email protected] 401


Answer Explanation
Choice B is the best answer. The convention being tested here is parallelism. The form
of the verb “emptying” directly matches with “extending” and is the most efficient way
to convey the point the sentence is making.

28.
Choice B is the best answer. The convention being tested is subject-verb agreement.
The plural verb “are” correctly agrees with the plural subject “individuals and
businesses.”

29.
Choice D is the best answer. The convention being tested is punctuation and the
appropriate punctuation mark is a comma. Choices B and C are incorrect because they
require a complete sentence on both sides. Choice A is incorrect because a pause is
required after “water.”

30.
Choice C is the correct answer. Affect is a verb and an action word is needed here.
Furthermore, “over the past 10 years” denotes that it is ongoing, so “has affected” is
the proper answer.

31.
Choice B is the correct answer. Students are an object in the sentence, so to describe
more about them requires the word whom. If students were the subject, then we

www.strategictestprep.com ♟️ 1-833-300-PREP (7737) ♟️[email protected] 402


would use the word who. However, “he” is the subject in this sentence because he is
doing the actions “rode” and “know.” Which and that are incorrect answers because
students are people, not objects.

32.
Choice D is the correct answer. You should use a semicolon to replace a period
between related sentences when the second sentence starts with either a conjunctive
adverb or a transitional expression. In this case, we have two complete sentences that
are related and the second sentence starts with the transition word “however.”

33.
Choice D is the correct answer. You should use a semicolon to replace a period
between related sentences when the second sentence starts with either a conjunctive
adverb or a transitional expression. In this case, we have two complete sentences that
are related and the second sentence starts with the transition word “however.”

34.
The correct answer is B. Since the sentence begins with “named in 2003 as one of the
greatest achievements in fashion,” the subject that is introduced immediately following
the comma must be an achievement. Since Ford, in 2001, and the signature Gucci floral
pattern are not achievements, we must choose B, as “Ford’s creation of the signature
pattern..” is an achievement.

35. Answer: D) accumulation of fertilizers

Explanation:

A) "to accumulate fertilizers" - The infinitive "to accumulate" doesn't provide a


complete thought in the context of the sentence. It suggests an intention or purpose,
which doesn't fit the preceding list of threats.

www.strategictestprep.com ♟️ 1-833-300-PREP (7737) ♟️[email protected] 403


B) "they accumulate fertilizers" - The pronoun "they" introduces ambiguity. It's unclear
what "they" refers to, making this choice grammatically awkward in the sentence's
context.

C) "accumulating fertilizers" - This present participle suggests an ongoing action but


doesn't fit well within the structure of the list of threats provided.

D) "accumulation of fertilizers" - This choice provides a clear and direct noun


("accumulation") that aligns with the other threats mentioned ("changes" and "runoff").
The prepositional phrase "of fertilizers" then specifies the type of accumulation being
referred to. This option seamlessly fits into the list format of the sentence and is
grammatically consistent with the preceding items.

36. Answer: A) they

Explanation: The question requires a pronoun that refers back to "trees." Among the
choices provided:

A) "they" is the correct choice because it is a plural pronoun that matches the plural
noun "trees."

B) "one" is typically used to refer back to an individual in a general sense and doesn't
fit contextually when referring to a group like "trees."

C) "you" is typically used for direct address and doesn't fit contextually in this scenario.

D) "its" is a singular possessive pronoun and does not match the plural noun "trees."

37. Answer: D) argued

Explanation:

A) argued, - "many traditional norms of her time were restrictive" is a continuation of


the thought started with "argued," so the comma makes the sentence structure
incorrect.

B) argued: - A colon is used to elaborate on or provide a list related to the preceding


clause. In this case, the information after "argued" is not an elaboration or list, but

www.strategictestprep.com ♟️ 1-833-300-PREP (7737) ♟️[email protected] 404


rather a continuation of Virginia Woolf's perspective. Therefore, the colon is not
appropriate here.

C) argued; - A semicolon typically separates two independent clauses that are closely
related in theme. While "Virginia Woolf established herself as a prominent figure in the
literary world" and "many traditional norms of her time were restrictive" are
thematically related, the latter clause is a direct result of her argument and is not
independent. Hence, the semicolon is not the appropriate punctuation here.

D) argued - This is the correct choice because it smoothly connects Woolf's belief to
the ensuing statement without unnecessary punctuation, creating a cohesive sentence.

38. Answer: B) His

Explanation: The sentence is discussing the performance of Chris Hadfield. The correct
possessive pronoun to refer to something that belongs to him is "his."

A) "He’s" is a contraction of "he is" or "he has." This does not fit the context of the
sentence as the sentence does not require a verb form after the blank.

B) "His" is the correct possessive pronoun to refer to something belonging to Chris


Hadfield.

C) "Him" is an object pronoun and is not appropriate here since we are referring to
Chris Hadfield's performance, which requires a possessive form.

D) "He’d" is a contraction of "he would" or "he had." This does not fit the context as it
introduces an unnecessary verb form.

39. Answer: C) journal editor Mark Hale described

Explanation: The sentence provides a statement by journal editor Mark Hale about Dr.
Emily Thornton's paper on quantum mechanics.

A) "journal editor Mark Hale, described" – The comma after "Hale" is misplaced and
breaks up the subject ("journal editor Mark Hale") from its verb ("described").

www.strategictestprep.com ♟️ 1-833-300-PREP (7737) ♟️[email protected] 405


B) "journal editor, Mark Hale described," – The first comma after "editor" is unnecessary
and incorrectly separates "journal editor" from "Mark Hale." The second comma after
"described" is also misplaced.

C) "journal editor Mark Hale described" – This is the correct choice. It attributes the
description of the paper as "a turning point in modern physics" to journal editor Mark
Hale without adding unnecessary commas.

D) "journal editor, Mark Hale, described," – This choice uses unnecessary commas
around "Mark Hale" and after "described."

40. Answer: B) view

Explanation: The sentence requires a verb that correctly and directly links the action of
the "astronomers" to their perspective on the Voyager Golden Record.

A) "viewing" – The gerund form does not form a complete verb phrase that correctly
attributes the perspective to the astronomers in the context of this sentence.

B) "view" – This is the correct choice. It clearly attributes the perspective about the
Voyager Golden Record to the astronomers, making the sentence grammatically
accurate.

C) "have viewed" – This form suggests an action that started in the past and has
relevance to or continues into the present. While it's not entirely wrong, the present
simple "view" is more straightforward and suitable for the context.

D) "to view" – This infinitive form doesn't appropriately complete the thought in this
context. It would fit better in a sentence structure like "Astronomers are likely to view,"
but not in the given sentence.

41. Answer: A) to blend

Explanation:

A) to blend : The phrasing "allows the octopus to blend more closely with its
environment" is the most grammatically and semantically correct way to complete the
sentence. The word "allows" is followed by "to" plus a verb to express permission or
possibility, making "to blend" the appropriate choice.

www.strategictestprep.com ♟️ 1-833-300-PREP (7737) ♟️[email protected] 406


B) blending : While "blending" is a gerund and can be used as a noun, the structure
"allows the octopus blending" is grammatically incorrect. The proper structure would
require a preposition or an auxiliary verb, which is not present in this choice.

C) it blends : Using "it blends" creates redundancy in the sentence, as we already have
the subject "the octopus" mentioned. Additionally, the phrase "allows the octopus it
blends" is not grammatically correct in Standard English.

D) that blended : The past tense "blended" is inconsistent with the rest of the
sentence, which is in the present tense. Furthermore, the structure "allows the octopus
that blended" is not grammatically correct in this context.

42. Answer: B) are

Explanation: The subject "understanding and empathy" is plural, so the verb form
should be plural as well.

A) "is" – This is a singular verb form and does not match the plural subject
"understanding and empathy."

B) "are" – This is the correct choice. The plural verb "are" correctly agrees in number
with the plural subject "understanding and empathy."

C) "has been" – This choice uses the present perfect tense and is not the right fit for the
context of the sentence, which states a general truth about the novel's themes.

D) "being" – This form is not appropriate for the context of the sentence, as it doesn't
provide a clear verb to match with the subject.

43. Answer: B) alone. However,

Explanation: The sentence requires a distinct break to transition from the mention of
Marie Curie's recognition for research to her broader influence as a role model for
women in science.

www.strategictestprep.com ♟️ 1-833-300-PREP (7737) ♟️[email protected] 407


A) "alone, however:" – A colon is not suitable here since it is typically used to introduce
a list, an explanation, or a definition, and none of those functions are needed in this
context.

B) "alone. However," – This is the correct choice. Using a full stop followed by
"However," provides the necessary pause and establishes the intended contrast
between Curie's research achievements and her broader impact.

C) "alone however" – This choice lacks the necessary punctuation to create the
intended contrast.

D) "alone, however." – While this choice provides a contrast with "however," ending
the sentence here feels abrupt and incomplete.

44. Answer: A) it revolutionized the world of photography,

Explanation: The sentence requires a clear and direct continuation that shows the
impact of the development of color film on the realm of photography.

A) "it revolutionized the world of photography," – This choice is correct. It concisely and
directly communicates the transformative impact of color film on photography.

B) "the world of photography was revolutionized," – While this choice technically


conveys the impact of color film, it uses a passive voice, making it less direct than
option A.

C) "photography's world revolutionized," – This choice awkwardly phrases the idea,


and it's missing a verb to indicate who or what caused the revolution.

D) "having a revolution in the photography world," – This option is wordy and does not
provide a clear cause-and-effect relationship between the development of color film
and its impact.

45. Answer: B) is

Explanation: The sentence is describing Carson's enduring impact on environmental


awareness, suggesting that her influence is still relevant and felt in the present. The
present simple tense is suitable for expressing a present state or condition.

www.strategictestprep.com ♟️ 1-833-300-PREP (7737) ♟️[email protected] 408


A) "were" – This is the simple past tense, which indicates a specific action or state that
occurred in the past and is completed. The context, however, suggests an ongoing
impact, making this choice unsuitable.

B) "is" – This is correct. The present simple tense "is" conveys that Carson's impact is
currently felt and ongoing.

C) "have been" – This is the present perfect tense, which describes an action or state
that began in the past and continues into the present. However, the phrase "felt today"
already indicates the present impact, so using "have been" is redundant in this context.

D) "are being" – This present continuous tense implies an ongoing action, but it doesn't
align correctly with the singular subject "Carson's impact." Additionally, the phrase
"are being felt" is awkward and less direct than simply stating "is felt."

46. Answer: D) yet

Explanation:

The sentence is presenting a contrast: while Wong's memoir recounts her culinary
journey, it doesn't thoroughly explore the business challenges she encountered. The
word "yet" is used to introduce this contrasting idea, and it should be followed by a
comma to correctly separate the two clauses.

A) "so" – This indicates a cause-effect relationship, which isn't the relationship being
described between the two parts of the sentence..

B) "thus" – This indicates a cause-effect relationship, which isn't the relationship being
described between the two parts of the sentence.

C) "and" – This suggests that the two parts of the sentence are simply additive, not
contrasting. It doesn't convey the idea that while one thing is true, another (somewhat
opposing) thing is also true.

D) "yet," – This is the correct choice. "Yet" introduces the contrast, and the comma
correctly separates the two contrasting ideas in the sentence.

47. Answer: C) resulting

www.strategictestprep.com ♟️ 1-833-300-PREP (7737) ♟️[email protected] 409


Explanation:

The correct answer is "resulting" because it provides a cause-and-effect transition from


the information presented about the decrease in typical food sources due to changes in
ocean currents.

A) "results" – This choice would create an awkward sentence structure: "decrease in


these food sources, results in the birds..." which doesn't flow well.

B) "to result" – This choice would not provide a logical continuation. The sentence
would read: "decrease in these food sources, to result in the birds..." which is
grammatically incorrect and disrupts the flow.

C) "resulting" – This choice indicates the consequence or result of the changes in ocean
currents on the birds' behavior. The sentence would read: "decrease in these food
sources, resulting in the birds..." which is a smooth continuation and logically connects
the preceding idea.

D) "resulted" – Like the previous example, this choice implies a completed action,
whereas "resulting" indicates an ongoing effect of the changes in ocean currents.

48. Answer: A) will the monarchs adapt?

Explanation:

The correct answer is "will the monarchs adapt?" because the context sets up a
hypothetical scenario and poses a question about the potential outcome.

A) "will the monarchs adapt?" – This choice correctly frames a direct question
regarding the butterflies' potential reaction to a different variety of milkweed.

B) "the monarchs will adapt." – This choice is structured as a statement, but given the
preceding context, a question form is more appropriate to reflect Carlos Mendez's
inquiry or uncertainty.

C) "the monarchs adapt?" – While this is phrased as a question, the use of the present
tense "adapt" without an auxiliary verb makes the structure feel incomplete and
inconsistent with the hypothetical nature of the scenario.

www.strategictestprep.com ♟️ 1-833-300-PREP (7737) ♟️[email protected] 410


D) "will the monarchs have adapted?" – This choice uses the future perfect tense,
which isn't quite appropriate for the situation. The context doesn't imply an action
completed in the future before another action but rather a direct potential outcome to
the hypothetical scenario.

49. Answer: D) is

Explanation:

The statement is referring to a current and continuing opinion or recognition of Marie


Curie's significance in the history of science, which necessitates the use of the present
tense.

A) "had been" – This uses the past perfect tense, suggesting that Curie was once
considered one of the most influential scientists but might not be considered so
anymore. This choice does not align with the notion that this recognition is still
currently held.

B) "will be" – This is incorrect because it implies a future recognition of Curie as one of
the most influential scientists, not a present-day belief.

C) "was" – This past tense suggests that there was a specific past time when Curie
was considered one of the most influential scientists, but does not imply that she is
still regarded so today.

D) "is" – This choice correctly communicates that, in the present day, Marie Curie is
recognized as one of the most influential scientists of her era.

50. Answer: C) provide

Explanation:

This question requires determining the correct verb form based on the structure and
subject-verb agreement.

A) "providing" – This gerund form of the verb does not fit properly in the sentence
structure as there's no auxiliary verb before it to form a continuous tense.

www.strategictestprep.com ♟️ 1-833-300-PREP (7737) ♟️[email protected] 411


B) "provides" – This is the third-person singular form of the verb. The subject in this
part of the sentence, "projects," is plural, so we need a plural verb form.

C) "provide" – This is the correct choice. It's the base form of the verb and agrees in
number with the plural subject "projects." Thus, the projects "provide" a habitat.

D) "provided" – This is the past form of the verb. The context of the sentence is
present, so this doesn't align with the intended meaning.

51. Answer: A) there are drawbacks to eating too much chocolate:

Explanation:

The sentence intends to juxtapose the positive attributes of chocolate with its potential
negatives. The best choice would offer a clear transition to introduce the issues with
consuming chocolate in excess.

A) "there are drawbacks to eating too much chocolate:" – This option directly
addresses the point being made, introducing the drawbacks of excessive chocolate
consumption in a clear and concise manner.

B) "the drawbacks of eating too much chocolate are:" – This option, though technically
correct, is less direct and sounds more passive than option A.

C) "chocolate's two major concerns are that" – This choice is awkward in phrasing and
implies that there are exactly two issues, which isn't supported by the subsequent
clause.

D) "chocolate has a few concerns:" – This option is vague and doesn't specify that the
concerns are related to eating it in excess.

52. Answer: B) named Peter Agre

Explanation:

The sentence is conveying the idea that a particular molecular biologist, Peter Agre,
made a discovery and was awarded for it. The correct choice should clearly and
concisely attribute the discovery to Peter Agre.

www.strategictestprep.com ♟️ 1-833-300-PREP (7737) ♟️[email protected] 412


A) "having been named Peter Agre" – This option is overly verbose and unnecessarily
passive.

B) "named Peter Agre" – This is the correct format. It succinctly and directly attributes
the discovery to Peter Agre without any unnecessary words or passive construction.

C) "that is named Peter Agre" – This option is overly verbose and unnecessarily
passive.

D) "being Peter Agre," – The word "being" is unnecessary here and creates a clunky
sentence structure.

53. Answer: A) demonstrate

Explanation:

The sentence is discussing a characteristic of the red blood cells in Andean geese in
relation to high altitudes. The correct choice should describe this characteristic in a
manner that fits coherently within the given sentence structure.

A) "demonstrate" – This is the correct format. It directly and succinctly indicates the
characteristic of the red blood cells in Andean geese: they "demonstrate" an increased
ability.

B) "to demonstrate" – Using the infinitive "to demonstrate" disrupts the flow of the
sentence and doesn't align grammatically with the subject "red blood cells."

C) "demonstrating" – This gerund form creates a continuation of the prior idea but isn't
as direct or fitting as the simple present tense "demonstrate."

D) "have demonstrated" – This present perfect tense implies a completed action in the
past leading up to the present. However, the sentence is referring to a current, ongoing
characteristic of the red blood cells, making this choice not the best fit.

54. Answer: B) had grown

Explanation:

www.strategictestprep.com ♟️ 1-833-300-PREP (7737) ♟️[email protected] 413


The action described pertains to the growth in the number of countries participating,
and this action happened before a specific point in the past, "by the time the Maastricht
Treaty was signed in 1992." Hence, the correct verb tense to capture this relationship is
the past perfect tense.

A) "grows" – This is the simple present tense and indicates an ongoing or general
action. It doesn't align with the past context given in the sentence.

B) "had grown" – The past perfect tense indicates a past action that happened before
another past action or point in time. In this context, the growth in the number of
countries had already taken place by the time the Maastricht Treaty was signed in
1992. This choice is correct.

C) "will grow" – This is the future tense, suggesting an action that will happen in the
future. It doesn't fit the past context of the sentence.

D) "is growing" – This is the present continuous tense, which denotes an action
currently taking place. It doesn't fit the past context of the sentence.

55. Answer: A) which

Explanation:

The sentence is giving additional information about the "mountain viscachas",


describing it as a type of rodent found in the Andes.

A) "which" – This is the correct choice. It introduces a non-restrictive relative clause


that provides additional information about the mountain viscachas. This clause can be
removed without changing the main meaning of the sentence.

B) "that" – "That" is typically used in restrictive relative clauses. A restrictive clause


identifies which specific thing or things we're talking about. In this context, we're not
specifying or restricting which mountain viscachas we're referring to, so "that" is
inappropriate.

C) "who" – "Who" is used to refer to people, not animals or objects, so this is not the
correct choice.

www.strategictestprep.com ♟️ 1-833-300-PREP (7737) ♟️[email protected] 414


D) "where" – "Where" is used to refer to places, and in this context, we need a word
that refers to the mountain viscachas. Thus, "where" is not appropriate.

56. Answer: B) forms

Explanation:

The context of the sentence gives a general description of the themes and characters in
Barbara Kingsolver’s novels. Typically, when providing general descriptions of a
character's traits or actions in literature, we use the present tense.

A) "had been forming" – This is the past perfect continuous tense. It suggests an
ongoing action in the past that was completed before another past action or event.
Given the context, this tense doesn't fit.

B) "forms" – This is the simple present tense and is the correct choice. It offers a
general statement about the relationship Dellarobia has with the monarch butterflies
in the book "Flight Behavior".

C) "was formed" – This is the passive voice in the past tense. It suggests that the bond
was formed by someone or something else, which isn't what the sentence is implying.

D) "forming" – This is the present participle and would be more appropriate if it was
used with a helping verb (e.g., "is forming"). On its own, it doesn't fit the structure of
the sentence.

57. Answer: C) beam's axis.

Explanation:

The context suggests that each beam is twisted around its own axis.

A) "beam’s axis’s." – The use of a double possessive here is unnecessary and not
standard in this context.

B) "beams' axises." – This suggests that there are multiple beams, but the sentence
structure indicates each beam individually. Also, "axises" is not the correct plural form
of "axis".

www.strategictestprep.com ♟️ 1-833-300-PREP (7737) ♟️[email protected] 415


C) "beam’s axis." – This is the correct choice. It indicates the axis of an individual beam,
which aligns with the context given.

D) "beams axes'." – This choice incorrectly combines the plural form "beams" with
another possessive form "axes'." The context doesn't indicate multiple axes.

58. Answer: B) facade,”

Explanation:

The context of the sentence introduces an observation made by literature professor


Daniel Mortimer about the works of Emily Dickinson and subsequently provides
supporting details for that observation.

A) "facade”; thus," – The semicolon isn't appropriate here, as it typically separates two
independent clauses. The following portion of the sentence ("given that...") is not an
independent clause but rather an extension of the previous thought.

B) "facade,”" – This is the correct choice. The comma following the quotation mark
indicates a continuation of the sentence, allowing the statement to be further
elaborated upon or clarified.

C) "facade” and," – The word "and" is not needed in this context. The subsequent
portion of the sentence is not an additional point but rather an explanation of the
professor's observation.

D) "facade.” " – The period after the quotation mark prematurely ends the sentence.
The remaining portion of the sentence ("given that...") would then begin awkwardly
without a clear connection to the previous statement.

59. Answer: B) although

Explanation: In this sentence structure, the word "although" is used to introduce a


contrasting idea within the main clause. The dependent clause "it led to the Parisian
boulevards and iconic architecture we know today" provides a contrast within the main
clause without the need for an additional comma after "although."

Now, for the incorrect choices:

www.strategictestprep.com ♟️ 1-833-300-PREP (7737) ♟️[email protected] 416


A) although, Explanation: This choice incorrectly places a comma after "although,"
which interrupts the flow of the contrast being introduced within the main clause.

C) but; Explanation: The use of "but" followed by a semicolon is not appropriate in this
context. The semicolon typically separates two independent clauses, and "but" is not
needed in conjunction with a semicolon.

D) but, Explanation: The word "but" introduces a contrast, but it's typically placed
between the clauses it's contrasting. In this case, using "although" is more seamless
because it introduces a contrast within the main clause without breaking the flow.

60. Answer: A) forest's floor mirror the fly's

Explanation:

The sentence requires a possessive form for both "forest" and "fly" to indicate
ownership or association. Let's evaluate each option:

A) "forest's floor mirror the fly's" - This is the correct choice. It uses the possessive
form for both "forest" and "fly" to indicate that the floor of the forest mirrors the
strategy of the individual lanternfly.

B) "forests floor mirror the flies" - This choice incorrectly uses the plural "forests"
instead of the possessive "forest’s." Also, "flies" is not in its possessive form, which is
needed to convey that the evolutionary strategy belongs to the flies.

C) "forest's floor mirror the flies'" - While "forest’s" is used correctly to show
possession, "flies’" implies a possession related to multiple flies, while the context
seems to describe the strategy of a singular lanternfly.

D) "forests' floor mirror the fly’s" - This choice uses the possessive form "forests’"
which implies there are multiple forests, but the context refers to a singular "dense
forest of the Amazon." The possessive "fly’s" is used correctly.

61. Answer: C) fungi’s stems signal the path

www.strategictestprep.com ♟️ 1-833-300-PREP (7737) ♟️[email protected] 417


Explanation:

The sentence needs to express that the emitted light comes from the stems of the
bioluminescent fungi.

A) "fungis stems signal the path" - "Fungis" is not the correct plural or possessive form
of "fungus."

B) "fungi’s stems signals the path" - The possessive "fungi’s" is correct, but the verb
"signals" should be "signal" to agree with the plural subject "stems."

C) "fungi’s stems signal the path" - This is the correct choice. The possessive "fungi’s"
indicates that it's the stems of the fungi that emit the light. The plural verb "signal"
correctly agrees with the plural subject "stems."

D) "fungis’ stems signals the path" - "Fungis’" is not a correct form in English.
Additionally, the verb "signals" doesn't agree in number with the plural subject
"stems."

62. C) Beyond the Stars

Explanation: The title of the publication, "Beyond the Stars," fits seamlessly into the
sentence without the need for additional punctuation. The text naturally flows into the
explanation of what the publication covers.

Now, for the incorrect choices:

A) Beyond the Stars— Explanation: An em dash (—) is used to create a strong break in
a sentence or to set off parenthetical information. In this case, the details about the
discoveries on planets in the Proxima Centauri star system are not parenthetical or a
sudden break from the rest of the sentence. Thus, the em dash is not appropriate.

B) Beyond the Stars, Explanation: While a comma is often used after book or
publication titles in certain contexts (like in a list or before additional information about
the book), it's not needed here because the following details directly relate to the title
and describe its content.

D) Beyond the Stars: Explanation: A colon is typically used to introduce a list, an


explanation, or an example of what is mentioned before the colon. While the details

www.strategictestprep.com ♟️ 1-833-300-PREP (7737) ♟️[email protected] 418


about the microbial ecosystems can be seen as an explanation of the publication's
content, the current sentence structure doesn't require a colon because the details flow
naturally without it.

63. Answer: D) relied on

Explanation:

The sentence discusses an action (relying on stars, ocean currents, and bird migrations)
that started in the past and continued for an extended period but doesn't necessarily
imply its continuation to the present in the same way. The simple past tense is the
most appropriate choice for this context.

A) "relying on" would suggest a continuous action, but it doesn't fit with the past
context "For centuries" provided by the sentence.

B) "rely on" is the simple present tense and indicates a habitual action. However, it
doesn't fit well with the past context given in the sentence.

C) "have relied on" is the present perfect tense, which describes an action that began in
the past and continues into the present. While not completely wrong, it doesn't suit
the context as well as the simple past does.

D) "relied on" is the correct answer. The simple past tense is most appropriate here,
given the context that suggests a longstanding past action.

64. Answer: C) others

Explanation:

The sentence contrasts two approaches to musical recording: digital methods and
analog methods. The word "others" is used to indicate this contrast.

A) "many" doesn't effectively contrast with "many" from the first part of the sentence,
making it redundant.

B) "some of them" is unnecessarily wordy and doesn't set up a clear contrast.

www.strategictestprep.com ♟️ 1-833-300-PREP (7737) ♟️[email protected] 419


C) "others" is the correct choice because it provides a clear contrast to "many musicians
today" from the first part of the sentence.

D) "the others" implies a specific group distinct from "many musicians," which isn't
clear from the context.

65. Answer: D) introduced

Explanation:

The passage recounts historical events that happened in the past, specifically in the
1920s. Therefore, the verb tense should be consistent with other verbs used in the
context.

A) "is introducing" is in the present continuous tense, which does not fit the past
context established in the passage.

B) "introduces" is in the simple present tense, which is inconsistent with the past tense
verbs used elsewhere in the passage.

C) "had introduced" is in the past perfect tense, used to indicate an action that took
place before another past action. In this context, however, there's no need to indicate a
sequence of past actions, as Stein's introduction of the idea and Hemingway's search
happened around the same time.

D) "introduced" is in the simple past tense, consistent with the other verbs used in the
passage such as "was searching."

66. Answer: A) writing

Explanation:

The sentence aims to describe the activities and accomplishments of Emily Dickinson.
Her act of writing poems is presented as a continuing aspect of her life, making it
parallel to her act of maintaining friendships through correspondence.

A) "writing" establishes a parallelism between Dickinson's activities of maintaining


friendships and producing poems. The use of the present participle "writing"
seamlessly continues the description of her actions.

www.strategictestprep.com ♟️ 1-833-300-PREP (7737) ♟️[email protected] 420


B) "she wrote" interrupts the flow of the sentence and is redundant since the subject
"Emily Dickinson" has already been established at the beginning of the sentence.

C) "written" lacks an auxiliary verb and thus does not fit grammatically in the sentence.

D) "and wrote" is grammatically correct but creates a list-like structure that sounds
less fluid than the continuous action implied by "writing."

67. Answer: A) many musicologists have emphasized Debussy's break from


European classical traditions but

Explanation:

The structure of this sentence mirrors the previous question. It presents a point about
what is commonly recognized about Debussy's work and contrasts it with what has
been overlooked.

A) "many musicologists have emphasized Debussy's break from European classical


traditions but" offers a concise introduction to the main observation and uses "but" to
set up a clear counterpoint.

B) "Debussy's break from European classical traditions has been the emphasis for
many musicologists, who" makes the sentence structure more intricate with the
relative pronoun "who."

C) "there are many musicologists who have emphasized Debussy's break from
European classical traditions, but they" is a lengthier way to make the point and adds
unnecessary words.

D) "the emphasis for many musicologists has been Debussy's break from European
classical traditions; they" employs a semicolon which, while separating two related
ideas, doesn't provide as immediate and lucid a contrast as the conjunction "but."

68. Answer: A) many art historians focus on Monet’s innovative use of color and
light but

Explanation:

www.strategictestprep.com ♟️ 1-833-300-PREP (7737) ♟️[email protected] 421


The sentence wants to establish a contrast between two perspectives on Monet's
work: his use of color and light, and his appreciation for Japanese woodblock prints.

A) "many art historians focus on Monet’s innovative use of color and light but"
succinctly conveys the primary observation and then uses "but" to introduce the
contrasting element.

B) "focusing on Monet’s innovative use of color and light has been the primary concern
for many art historians, who" introduces an unnecessarily complex structure and the
relative pronoun "who" complicates the sentence's flow.

C) "there are numerous art historians who focus on Monet’s innovative use of color and
light, but they" is a wordier choice and doesn't convey the point as directly as option A.

D) "the concentration of numerous art historians is on Monet’s innovative use of color


and light; they" uses a semicolon which, while dividing two related ideas, doesn't
provide the direct and immediate contrast that the conjunction "but" offers.

69. Answer: B) in 1998; followed by

Explanation:

The sentence is trying to present a sequence of Samantha Kane's works in


chronological order.

A) "on 1998; followed by" incorrectly uses "on" instead of "in" for the year.

B) "in 1998; followed by" is correct. It uses "in" appropriately for the year and uses the
semicolon followed by the phrase "followed by" to show a clear sequence of her
works.

C) "in 1998, and" connects the two works but does not provide a clear sequence or
indicate that one work followed the other.

D) "on 1998, and then" again incorrectly uses "on" instead of "in" for the year.

70. Answer: D) "on either side"

Answer Explanation:

www.strategictestprep.com ♟️ 1-833-300-PREP (7737) ♟️[email protected] 422


A) "on either side," - Incorrect. There's no need for a comma after "side" because the
phrase "on either side" directly modifies "buildings" without introducing an
independent clause or additional information that would require separation.

B) "on either side—" - Incorrect. An em dash typically sets off a parenthetical thought
or emphasizes the conclusion of a sentence. In this context, the phrase "on either side"
doesn't introduce a break or additional thought that needs such emphasis.

C) "on either side:" - Incorrect. A colon is used to introduce a list, a quote, or a direct
statement. The phrase that follows "on either side" is neither a list, a direct statement,
nor a quote.

D) "on either side" - Correct. The phrase "on either side" is a prepositional phrase that
functions as an adverbial modifier, describing where the buildings sparkle. No
additional punctuation is necessary to separate it from the verb "sparkle."

71. Answer: D) varied;

Explanation:

A) varied: - The colon is improperly used here. A colon is typically used to introduce a
list, a quote, or an expansion on the first clause. Since the second clause is an
independent clause and not a straightforward list, quote, or an expansion, a colon is
not the appropriate punctuation.

B) varied - This choice is incorrect because it results in a run-on sentence. Without


proper punctuation, like a semicolon, to separate the two independent clauses, the
sentence is grammatically incorrect.

C) were varying, - This choice is incorrect for two reasons. First, "were varying" implies
ongoing action, which does not accurately reflect the completed action of presenting
varied genres of music in the past. Second, using a comma after "were varying" would
still result in a comma splice, since it would be incorrectly joining two independent
clauses with just a comma.

D) varied; - The semicolon is used correctly in this sentence to connect two


independent clauses. The first clause is "The genres of music presented varied" and
www.strategictestprep.com ♟️ 1-833-300-PREP (7737) ♟️[email protected] 423
the second clause "some participants listened to classical music, while others heard
jazz or rock" can stand alone as a complete sentence. The semicolon here functions to
closely link the two clauses, showing that the second clause is a continuation or an
illustration of the first.

72. Answer: A) rising to embrace one's journey;

Explanation:

The question lists Rupi Kaur's works and provides brief descriptors for each.

A) "rising to embrace one's journey;" - This is the correct choice. The semicolon at the
end indicates that another item on the list is coming, which is "Home Body."

B) "rising: to embrace one's journey," - A colon is typically used to introduce a list,


explanation, or clarification. Its use here is inappropriate because it does not fit the
structure of the sentence.

C) "rising to embrace one's journey," - This choice is close, but the lack of a semicolon
at the end suggests that there isn't another item following on the list, which is
incorrect since "Home Body" is mentioned right after.

D) "rising; to embrace one's journey" - The use of a semicolon between "rising" and "to
embrace one's journey" is inappropriate. Semicolons typically separate two
independent clauses or are used in lists where the items contain commas. This is not
the case here.

73. Answer:
D) highlighted

Explanation:
The sentence is discussing a past event, the publication of Rachel Carson's book and
its content. Thus, we need a verb in the past tense to describe what the book did.

A) "highlighting" - This continuous form is not appropriate here because it doesn't


clearly indicate a past action.
B) "had highlighted" - This past perfect form is used to describe an action that
happened before another past action. In this context, there's no other past action
mentioned to which this action would be prior, making it an unsuitable choice.
www.strategictestprep.com ♟️ 1-833-300-PREP (7737) ♟️[email protected] 424
C) "highlights" - This is the present tense and suggests that the book currently
highlights something. Since the book was published in the past and the sentence is
describing its content at that time, this choice is not appropriate.
D) "highlighted" - This is the correct choice. It uses the simple past tense to describe
what "Silent Spring" did when it was published.

74. Answer:
C) percent. While

Explanation:

A) percent, while - This choice creates a comma splice, as it improperly uses a comma
to join two independent clauses which could stand alone as separate sentences.

B) percent and while - This choice is incorrect because the word "and" typically joins
two related but not contrasting ideas, and the use of "while" immediately after "and" is
awkward and not grammatically correct in this context.

C) percent. While - This is the correct choice because "the number of daily cyclists
increased by over seventy percent" is a complete thought and stands as a full
sentence. The following clause "While improvements such as dedicated bike lanes and
safer crossings are credited for this uptick, some fear a potential surge in traffic-related
incidents." is also a complete sentence that introduces a contrast to the first idea. The
full stop (period) at the end of the first sentence is the appropriate punctuation to
separate these two independent clauses, and "While" at the beginning of the next
sentence indicates the contrast.

D) percent while - This choice is incorrect because it runs two independent clauses
together without proper punctuation, creating a run-on sentence. "While" is a
subordinating conjunction that should be used to start a dependent clause, but here it
introduces what should be a separate sentence and thus requires proper punctuation
before it.

75. Answer: A) has reflected

Explanation: The sentence highlights the influence of Aaron Copland's work on the
evolution of American classical music. To determine the correct verb form, we must
consider the subject and the tense.

www.strategictestprep.com ♟️ 1-833-300-PREP (7737) ♟️[email protected] 425


The subject is singular: "Appalachian Spring." Hence, the verb should be in its singular
form.

A) "has reflected" - This choice is in the present perfect tense and singular form,
indicating that the composition's influence began in the past and continues into the
present. It agrees with the singular subject.

B) "are reflecting" - This choice is in the present continuous tense and plural form. The
verb "are" does not align with the singular subject.

C) "have reflected" - This choice is in the present perfect tense but in plural form. The
verb "have" doesn't match with the singular subject.

D) "reflect" - While this choice is in the simple present tense and could technically
work, the present perfect tense more accurately indicates the ongoing influence from
the past to the present.

76. Answer: C) while

Explanation: The sentence is contrasting the general belief about chimpanzees with
the discoveries and beliefs of Jane Goodall. We need a word that introduces this
contrast.

A) "whereas" - This word can be used to introduce a contrast, but in this context, the
placement is awkward, making the sentence structure unclear.

B) "where" - This is typically used to introduce a place or location, and is not


appropriate for contrasting ideas in this context.

C) "while" - This word introduces a contrast and is appropriately used to compare two
opposing ideas or beliefs. In this context, it clearly contrasts the prevailing belief about
chimps with Goodall's findings.

D) "whom" - This word is used to refer to an object of a verb or preposition and is not
appropriate for contrasting ideas.

77. Answer: C) entirely paper-based, with each

Explanation:

www.strategictestprep.com ♟️ 1-833-300-PREP (7737) ♟️[email protected] 426


A) entirely paper-based, each
This option is incorrect because it suggests that what follows the comma is a list or a
continuation of the same sentence. However, "each twist, fold, and detail crafted from
recycled paper and eco-friendly adhesives" is not a list that directly follows the
adjective "paper-based." It's additional information that requires a different sentence
structure or punctuation to be grammatically correct.

B) entirely paper-based; each


This option is not quite right either. Although a semicolon can correctly link two closely
related independent clauses, the phrase following it in this option, "each twist, fold,
and detail crafted from recycled paper and eco-friendly adhesives," does not stand as
an independent clause because it lacks a main verb. The semicolon here would imply
that what follows could be a complete sentence on its own, which it is not in this
context.

C) entirely paper-based, with each


This option is correct. It uses a comma followed by the phrase "with each" to continue
the description of how the art pieces "show themselves." This structure adds
information about the sculptures in a way that clearly links it to the initial clause. It
avoids the creation of a sentence fragment while still maintaining a smooth flow from
the description of the sculptures' appearance to the details of their construction.

D) entirely paper-based. Each


This option incorrectly treats "Each twist, fold, and detail crafted from recycled paper
and eco-friendly adhesives" as if it were a complete sentence. Without a main verb,
this phrase is a fragment. The initial impulse to choose this option might come from the
contrast indicated by "However" at the beginning of the sentence, which often leads to
a new sentence following it. However, because the second part isn't a stand-alone
sentence, this option results in a grammatical error.

78. Answer: B) dive

Explanation: This sentence discusses a general action or behavior associated with


plants in a hydroponic system.

A) "will dive" - This suggests a future action. However, the context describes a general
trait of the plants in hydroponic systems, so the simple present tense is more
appropriate.

www.strategictestprep.com ♟️ 1-833-300-PREP (7737) ♟️[email protected] 427


B) "dive" - This is the correct choice. Using the simple present tense "dive" describes
the general behavior of the plants in the hydroponic system as they grow.

C) "had dived" - The past perfect tense is not appropriate here because the context is
not discussing something that took place before another past action.

D) "are diving" - This choice, in the present continuous tense, suggests an action
currently in progress. While it might be technically correct, the simple present tense
"dive" is a more straightforward way to describe a general or repeated action.

79. Answer: A) indicates

Explanation: The sentence requires a verb that describes the action or result of the
published report.

A) "indicates" - This is the correct choice. The verb in the simple present tense,
"indicates," clearly expresses what the report does in relation to the given context.

B) "indicating" - This gerund form would suggest an ongoing action but does not fit
properly within the structure of the sentence.

C) "which indicates" - The addition of "which" is unnecessary and makes the sentence
more convoluted than it needs to be.

D) "had indicated" - The past perfect tense is not the right fit for the context. We're not
emphasizing an action that happened before another past action, so this form is
inappropriate.

80. Answer: C) describes

Explanation: The sentence requires a verb that explains what the term "radioactivity"
means.

A) "has described" - The present perfect tense is not appropriate in this context
because the definition of "radioactivity" is a static fact and not an action that has
continued up to the present.

B) "describing" - This present participle doesn't provide a complete verb for the main
clause, leading to a sentence fragment.

www.strategictestprep.com ♟️ 1-833-300-PREP (7737) ♟️[email protected] 428


C) "describes" - This is the correct choice. The simple present tense verb "describes"
aptly explains the meaning of the term "radioactivity."

D) "describe" - The verb needs to match in number with its subject ("radioactivity"),
which is singular. Therefore, "describe" (plural form) isn't the correct choice.

81. Answer: B) while

Explanation:

A) moreover - "Moreover" is used to add information that supports or expands upon


the previous point. In this sentence, however, the intention is to contrast the diets of
two different birds, not to add supporting information.

B) while - This choice correctly indicates that the action of the second bird is occurring
simultaneously or in contrast with the first. The sentence is drawing a direct
comparison between the dietary habits of two different birds, and "while" serves to
highlight the different preferences in a single coherent statement.

C) however - "However" indicates a contrast or a turn in the discussion, which is


appropriate here, but it typically is used to start a new sentence or after a semicolon; it
doesn't fit as well to continue the flow of a comparison within a single sentence.

D) in fact, - "In fact," is used to emphasize a point or to present a piece of evidence that
supports what has previously been stated. It doesn't fit here because the sentence
aims to compare two different dietary habits, not to provide additional evidence or an
emphatic statement about the first part.

82. Answer: B) to catch and secure

Explanation:

A) "catching, and securing," - The commas around "and securing" are unnecessary and
disrupt the natural flow of the sentence.

B) "to catch and secure" - This option is clear, concise, and smoothly integrates into the
sentence, indicating the purpose of the bird using its beak.

www.strategictestprep.com ♟️ 1-833-300-PREP (7737) ♟️[email protected] 429


C) "for catching and for securing" - This option is redundant. The use of "for" before
both "catching" and "securing" is repetitive and makes the sentence more cumbersome.

D) "catch and then secure" - The use of "then" suggests a sequence of events, implying
the bird first catches the fish and after some time, secures it. This isn't the natural or
immediate process described.

83. Answer: B) were

Explanation:

A) "is" - This choice is singular, but the context indicates that there are multiple artists
being discussed, so a plural verb is needed.

B) "were" - This is the correct choice because it matches in number with the plural
subject "Among the renowned artists of this period."

C) "has been" - This choice uses the present perfect tense, which is not suitable for
discussing a specific historical period in the past.

D) "being" - This form does not fit grammatically in the sentence and does not provide
a correct verb tense for discussing historical figures.

84. Answer: A) researcher Dr. Emily Hart has proposed that

Explanation:

A) "researcher Dr. Emily Hart has proposed that" - This choice flows logically and
offers a direct statement that Dr. Emily Hart has presented an alternative viewpoint
based on the new evidence. It integrates seamlessly into the context.

B) "the proposition of researcher Dr. Emily Hart is that" - This choice is more wordy and
doesn't integrate as naturally with the context. The phrasing "is that" feels repetitive
after the mention of the prior belief about the T. rex.

C) "that, researcher Dr. Emily Hart proposes," - This choice disrupts the flow of the
sentence by inserting "that" before the mention of the researcher's proposal. The
logical progression from the prior belief to the new proposal is interrupted.

www.strategictestprep.com ♟️ 1-833-300-PREP (7737) ♟️[email protected] 430


D) "the proposal from researcher Dr. Emily Hart is suggesting that" - This choice is
verbose and redundant. Using both "proposal" and "suggesting" in the same statement
makes the sentence cumbersome.

85. Answer: A) carvings. Etched

Explanation:

A) "carvings. Etched" - This option successfully separates the type of artistic work on
the site (carvings) from the description of their content. The use of a period offers a
clear end to one statement and the beginning of a new related one.

B) "carvings, etched" - This option attempts to connect the type of artistic work directly
to its description, but the comma is unnecessary and disrupts the flow of the sentence.

C) "carvings and etched" - This option is grammatically incorrect. The conjunction


"and" suggests that "carvings" and "etched" are parallel items, but they're not in this
context.

D) "carvings etched" - This choice omits necessary punctuation to distinguish between


the type of artistic work and its description.

86. Answer: A) sous-chefs

Explanation:

A) "sous-chefs" - This choice correctly uses the hyphenated plural form "sous-chefs" to
refer to multiple chefs who are second in command in a kitchen.

B) "sous chef’s" - This choice is incorrect because it uses an apostrophe, which


indicates possession and not plurality.

C) "sous chefs" - While it correctly refers to multiple chefs, it misses the hyphen, which
should be present in the term "sous-chef".

D) "sous-chef’s" - This choice incorrectly uses an apostrophe to indicate possession


rather than plurality.

87. Answer: B) sustain

www.strategictestprep.com ♟️ 1-833-300-PREP (7737) ♟️[email protected] 431


Explanation:

A) had sustained - This choice incorrectly uses the past perfect tense, which is used to
indicate that an action was completed at some point in the past before another action.
Since the sentence is discussing a regular occurrence during the whales' migrations
and is not referring to a completed action in the past, the past perfect tense is not
appropriate here.

B) sustain - This choice correctly uses the present tense to indicate a habitual action.
Humpback whales "sustain" themselves on their blubber during their annual
migrations, a behavior that happens regularly each year.

C) would sustain - "Would sustain" could be used for habitual past actions, but in this
context, there is no indication that the sentence is referring to past habits that are no
longer occurring. The sentence appears to describe the ongoing, regular behavior of
the whales.

D) sustains - This choice uses the present tense, but it is singular while the subject,
"humpback whale," is acting as a plural noun in this context because it refers to all
humpback whales as a group. Therefore, the verb should also be plural, "sustain,"
rather than the singular "sustains."

88. Answer: A) managed—impressively—to calculate

Explanation:

A) "managed—impressively—to calculate" - This choice uses two em dashes correctly


to emphasize the word "impressively", which underscores the significance of Johnson's
achievement. The dashes set apart this extra information without breaking the flow of
the main idea.

B) "managed—impressively to calculate" - The use of only one em dash feels


incomplete and disrupts the flow of the sentence.

C) "managed—impressively, to calculate" - As with the previous example, introducing


a comma after "impressively" is redundant since the em dash already serves to
separate and emphasize the term.

www.strategictestprep.com ♟️ 1-833-300-PREP (7737) ♟️[email protected] 432


D) "managed—impressively. To calculate" - Beginning a new sentence with "To
calculate" is fragmented and doesn't convey the full sense of Johnson's achievement in
one cohesive thought.

89. Answer: D) remains

Explanation:

A) "remain" - This is the base form of the verb and does not fit with the singular
subject "the cat."

B) "has remained" - This present perfect tense implies an action that began in the past
and continues into the present. While this isn't entirely incorrect, it's not the most
concise or clear way to indicate the cat's role in Murakami's works.

C) "were" - This past tense plural form doesn't match the singular subject "the cat."

D) "remains" - This is the correct choice. It's present tense and matches the singular
subject "the cat." The verb indicates that the cat continues to be a prominent figure in
Murakami's works.

90. Answer: A) ; next,

Explanation:

A) ; next, - This choice is appropriate because it uses a semicolon to link two


independent clauses that are closely related, and "next" to indicate the sequence of
events. The semicolon is proper here because it denotes that what follows is directly
related to the first clause but still represents a separate thought. The word "next"
introduces the subsequent action, which occurred at a different time, and the comma
after "next" is correctly placed to separate the transitional word from the clause that
follows.

B) then - Without a semicolon or period before it, "then" does not properly connect the
two independent clauses. If "then" were preceded by a period, making it ". Then," it
would correctly start a new sentence, but standing alone, it is insufficient.

C) and - While "and" is a conjunction that can connect two independent clauses, it is
less effective in this sentence because it would not emphasize the chronological order

www.strategictestprep.com ♟️ 1-833-300-PREP (7737) ♟️[email protected] 433


of events. "And" would simply imply that two actions are part of one continuous
thought or process, which doesn't highlight the sequence as effectively as "; next,".

D) . While - Starting a new sentence with "While" implies simultaneity or contrast,


neither of which is applicable in this context. "While" would suggest that the actions
were occurring at the same time or in contrast to each other, but the sentence is
actually conveying a sequence of distinct events over time.

91. Answer: D) During her career, she discovered several comets,

Explanation:

A) "Discovering several comets during her career;" - Using a semicolon ";" here doesn't
provide a smooth connection to the subsequent statement. The following statement is
not an independent clause.

B) "Several comets were discovered by her during her career," - This passive
construction is less direct and less engaging than the active voice, making the sentence
sound a bit awkward.

C) "During her career, discovering several comets," - This phrasing is not grammatically
sound. The participle "discovering" does not form a complete thought when paired
with the rest of the sentence.

D) "During her career, she discovered several comets," - This is the correct choice. It
clearly and actively describes Caroline Herschel's achievements in her career before
introducing the fact about her award.

92. Answer: A) stars, supernovae, and black holes,

Explanation:

A) "stars, supernovae, and black holes," - This is the correct choice. It uses commas
appropriately to list the objects observed through the Hubble Space Telescope and
provides a smooth transition to the following list of its impacts.

B) "stars, supernovae, and black holes;" - The semicolon is not the appropriate
punctuation here. The list of objects is not an independent clause.

www.strategictestprep.com ♟️ 1-833-300-PREP (7737) ♟️[email protected] 434


C) "stars; supernovae, and black holes," - The semicolon is not used correctly here.
"Supernovae" and "black holes" are items in the list, not independent clauses.

D) "stars supernovae, and black holes," - This choice omits the necessary comma
between "stars" and "supernovae," making the list unclear and less readable.

93. Answer: D) though; in practice

Answer Explanation:

A) "though, in practice," - The comma after "though" is not strong enough to link two
independent clauses.

B) "though in practice," - This choice lacks necessary punctuation to separate the two
clauses.

C) "though: in practice," - A colon is generally not used after conjunctions like "though."
It is more commonly used to introduce a list, a quote, or an explanation that directly
follows the preceding clause.

D) “though; in practice” - A semicolon is used here to link two independent but related
clauses. The first part of the sentence, "Paleontologist Sara Mitchell specializes in the
study of ancient aquatic life, focusing on creatures like the ichthyosaur and the
plesiosaur," is an independent clause that could stand alone as a sentence. The second
part, "she frequently conducts excavations at various dig sites, working alongside
teams of other experts to uncover and document fossilized remains," is also an
independent clause.

The word "though" is a conjunction that introduces a contrast between the two
clauses. The semicolon before "though" is correct because it is used to connect two
related independent clauses. The comma after "though" is appropriate to separate the
introductory word from the rest of the sentence.

94. Answer: C) experiences of the pioneers

Explanation:

A) "experiences of the pioneer’s" - The use of "pioneer’s" is incorrect. The singular


possessive form is not required here since we are referring to multiple pioneers.

www.strategictestprep.com ♟️ 1-833-300-PREP (7737) ♟️[email protected] 435


B) "experiences’ of the pioneers" - The use of "experiences’" is incorrect. The
possessive form is unnecessary since "of the pioneers" already indicates the subjects of
those experiences.

C) "experiences of the pioneers" - This choice is correct. "Experiences" is the correct


plural form, and "of the pioneers" correctly indicates the subjects of those experiences
without resorting to an unnecessary possessive form.

D) "experience’s of the pioneers’" - The use of both "experience’s" and "pioneers’" is


incorrect. The sentence uses the possessive form twice, making it redundant and
grammatically incorrect.

95. Answer: A) about

Explanation:

A) "about" - This is the correct choice. The preposition "about" appropriately introduces
the narratives Clara Bellwood explored in her novel.

B) "about," - The comma after "about" is not needed. It interrupts the flow of the
sentence and does not follow the conventions of Standard English in this context.

C) "about—" - An em dash is not appropriate here. Em dashes are often used to denote
a sudden break or shift in thought or to set off a parenthetical element. Neither is the
case in this context.

D) "about:" - A colon is used to introduce a list or an explanation. Here, the narratives


(love, loss, and the intricacies of human relationships) are not being presented as a list
separate from the rest of the sentence, making the colon inappropriate.

96. Answer: D) establishment that

Explanation:

A) "establishment" - The absence of "that" or any other connector after


"establishment" makes the sentence feel fragmented and doesn't provide a smooth
connection between "establishment" and "focuses."

www.strategictestprep.com ♟️ 1-833-300-PREP (7737) ♟️[email protected] 436


B) "establishment which" - While "which" can often be used as a relative pronoun like
"that," in restrictive relative clauses (clauses that are essential to the meaning of the
sentence), "that" is more commonly used in American English. Thus, "which" feels
slightly out of place here without the preceding comma.

C) "establishment, that" - The comma before "that" is unnecessary in this context and
disrupts the flow of the sentence. It's not standard usage to separate "that" with a
comma in restrictive clauses.

D) "establishment that" - This is the correct choice. The word "that" serves as a relative
pronoun introducing the restrictive relative clause "focuses on providing easy-to-follow
recipes that champion fresh, local ingredients and promote a balanced lifestyle." It
connects the main clause and the relative clause seamlessly.

97. Answer: B) xanthochrome (XC)

Explanation:

A) "xanthochrome, (XC)" - The comma after "xanthochrome" is unnecessary and


disrupts the flow of the sentence. Moreover, there's no need to place a comma before
the parenthesis.

B) "xanthochrome (XC)" - This is the correct choice. It clearly presents the specific
name of the pigment ("xanthochrome") and its shorthand notation "(XC)" without any
unnecessary punctuation.

C) "xanthochrome, (XC)," - This choice incorrectly adds a comma both before and after
the parentheses. Commas are not required around parentheses when the parenthetical
is closely related to the word it follows, as is the case with an abbreviation.

D) "xanthochrome (XC,)" - The comma inside the parenthesis and just after "XC" is
misplaced and unnecessary.

98. Answer: C) marked

Explanation:

www.strategictestprep.com ♟️ 1-833-300-PREP (7737) ♟️[email protected] 437


A) "proves to be" - This choice is awkward and redundant in the sentence structure. It's
not necessary to say "proves to be a breakthrough" when "marked a breakthrough" is
more direct and appropriate for the past event described.

B) "has" - This choice is grammatically incorrect and doesn't fit the sentence's past
tense context. "This finding has a breakthrough" doesn't convey a clear meaning.

C) "marked" - This is the correct choice. It effectively communicates that the discovery
was a significant event (or "marked a breakthrough") in the field of study. The past
tense aligns with the context of the sentence, indicating an event that occurred in the
past.

D) "thus" - This choice is inappropriate for this context. It is a conjunctive adverb, and
its inclusion would make the sentence read incorrectly as "This finding thus a
breakthrough," which doesn't make grammatical sense.

99. Answer: B) to combat

Explanation:

A) "combating" - This choice could suggest that the act of developing the filtration
system is simultaneously the act of combating toxic algae. It's slightly ambiguous and
doesn't provide the clearest causal relationship between the development of the
system and its purpose.

B) "to combat" - This choice clearly indicates the purpose or intent behind Alex Monroe
and her team's action. They developed a filtration system with the objective "to
combat" the rapid growth of toxic algae. It effectively communicates the
cause-and-effect relationship.

C) "combated" - This choice suggests that after they developed the filtration system,
they then combated the rapid growth of toxic algae. It doesn't clearly convey the
intended cause-and-effect relationship that the filtration system was developed for the
purpose of combating toxic algae.

D) "and combat" - This choice implies that "developing a filtration system" and
"combatting toxic algae" are two separate actions, whereas the intended meaning is
that the filtration system was developed with the intention to combat the toxic algae.

www.strategictestprep.com ♟️ 1-833-300-PREP (7737) ♟️[email protected] 438


100. Answer: B) the saplings have achieved habitat restoration for numerous
endangered species.

Explanation:

A) "the habitat restoration for numerous endangered species was achieved through the
saplings." - This option is passive and lacks the directness in explaining the cause and
effect relationship between the saplings and the restoration.

B) "the saplings have achieved habitat restoration for numerous endangered species." -
This choice is concise, direct, and effectively communicates the positive outcome of the
reforestation project initiated by Patel.

C) "the saplings’ achievement in habitat restoration for numerous endangered species


has been realized." - This option is wordy and less straightforward than option B.

D) "habitat restoration for numerous endangered species has been achieved by the
saplings." - While this choice does state the outcome of the project, it uses a passive
construction that makes it less direct than option B.

www.strategictestprep.com ♟️ 1-833-300-PREP (7737) ♟️[email protected] 439


www.strategictestprep.com ♟️ 1-833-300-PREP (7737) ♟️[email protected] 440
✨How can you make the most out of this workbook? ✨
👉For walkthrough videos, strategies, quizzes, and extra resources that support
each lesson in this workbook, please sign up for our English Self-Paced Course!

👉For individualized support, please check out our Information Pack to learn
more about our private tutoring options and Request a Consultation!

✨Looking for additional resources to guide you in your Digital SAT prep journey? ✨
👉Practice wherever, whenever you want with the Preptly: Digital SAT Prep
App! Join our 2,000+ users and start improving your score today!
📲 Get Preptly in the App Store or in Google Play today!
👉Looking to improve your math score as well? 🤨 We have a new Self-Paced
Digital SAT Math Course! (Available in 2024)

👉Check out our YouTube Channel, which provides free videos that teach you
how to take the Digital SAT like a pro!

👉Follow us for up-to-date tips, tricks, strategies, and more!

www.strategictestprep.com ♟️ 1-833-300-PREP (7737) ♟️[email protected] 441


© 2023 by Strategic Test Prep.
All Rights Reserved.

www.strategictestprep.com ♟️ 1-833-300-PREP (7737) ♟️[email protected] 442

You might also like